SBI PO Exam - Solved Model Question Paper

SBI PO Exam - Solved Model Question Paper

Reasoning Ability: Directions (Q. 1 - 5): Read the information carefully and answer the following questions: If A + B means A is the father of B. If A × B means A is the sister of B If A $ B means A is the wife of B. If A % B means A is the mother of B. If A ÷ B means A is the son of B. 1. What should come in place of question mark to establish that J is brother of T in the expression? J ÷ P % H ? T % L 1) × 2) ÷ 3) $ 4) Either ÷ or × 5) Either + or ÷ 2. Which of the given expressions indicates that M is daughter of D? 1) L % R $ D + T × M 2) L + R $ D + M × T 3) L % R % D + T ÷ M 4) D + L $ R + M × T 5) L $ D ÷ R % M ÷ T 3. Which of the following options is true if the expression 'I + T % J × L ÷ K' is definitely true? 1) L is daughter of T 2) K is son-in-law of I 3) I is grandmother of L 4) T is father of J 5) J is brother of L 4. Which of the following expressions is true if Y is son of X is definitely false? 1) W % L × T × Y ÷ X 2) W + L × T × Y ÷ X 3) X + L × T × Y ÷ W 4) W $ X + L + Y + T 5) W % X + T × Y ÷ L 5. What should come in place of question mark to establish that T is sister-in-law of Q in the expression. R % T × P ? Q + V 1) ÷ 2) % 3) × 4) $ 5) Either $ or × Directions (Q. 6 - 10): Study the following information and answer the questions given below: Eight people - E, F, G, H, J, K, L and M - are sitting around a circular table, facing the centre. Each of them is of a different profession - Chartered Accountant, Columnist, Doctor, Engineer, Financial Analyst, Lawyer, Professor and Scientist, but not necessarily in the same order. F is sitting second to the left of K. The Scientist is an immediate neighbour of K. There are only three people between the Scientist and E. Only one person sits between the Engineer and E. The Columnist is on the immediate right of the Engineer. M is second to the right of K. H is the Scientist. G and J are immediate neighbours of each other. Neither G nor J is an Engineer. The Financial Analyst is on the immediate left of F. The Lawyer is second to the right of the Columnist. The Professor is an immediate neighbour of the Engineer. G is second to the right of the Chartered Accountant. 6. Who is sitting second to the right of E? 1) The Lawyer 2) G 3) The Engineer 4) F 5) K 7. Who among the following is the Professor? 1) F 2) L 3) M 4) K 5) J 8. Four of the following five are alike in a certain way based on the given arrangement and hence form a group. Which of the following does not belong to that group? 1) Chartered Accountant - H 2) M - Doctor 3) J - Engineer 4) Financial Analyst - L 5) Lawyer - K 9. What is the position of L with respect to the Scientist? 1) Third to the left 2) Second to the right 3) Second to the left 4) Third to the right 5) Immediate right 10. Which of the following statements is true according to the given arrangement? 1) The Lawyer is second to the left of the Doctor. 2) E is an immediate neighbour of the Financial Analyst. 3) H sits exactly between F and the Financial Analyst. 4) Only four people sit between the Columnist and F. 5) All of the given statements are true. Directions (Q. 11 - 15): In each of the questions below, two/three statements are given followed by conclusions/ group of conclusions numbered I and II. You have to assume all the statements to be true even if they seem to be at variance with the commonly known facts and then decide which of the given two conclusions logically follows from the information given in the statements. Give answer 1) if only conclusion I follows. 2) if only conclusion II follows. 3) if either conclusion I or conclusion II follows. 4) if neither conclusion I nor conclusion II follows. 5) if both conclusion I and II follow. (11 - 12): Statements: Some squares are circles. No circle is a triangle. No line is a square. 11. Conclusions: I. All squares can never be triangles. II. Some lines are circles. Ans: 1 12. Conclusions: I. No triangle is a suqare. II. No line is a circle. Ans: 4 (13 - 14): Statements: All songs are poems. All poems are rhymes. No rhyme is paragraph. 13. Conclusions: I. No Song is a paragraph. II. No poem is a paragraph. Ans: 5 14. Conclusions: I. All rhymes are poems. II. All Songs are rhymes. Ans: 2 15. Statements: Some dews are drops. All drops are stones. Conclusions: I. At least some dews are stones. II. At least some stones are drops. Ans: 5 Directions (Q. 16 - 18): Each of the questions below consists of a question and two statements numbered I and II given below it. You have to decide whether the data given in the statements are sufficient to answer the questions. Read both the statements and give answer 1) if the data in statement I alone are sufficient to answer the question, while the data in statement II alone are not sufficient to answer the question. 2) if the data in statement II alone are sufficient to answer the question, while the data in statement I alone are not sufficient to answer the question. 3) if the data either in statement I alone or statement II alone are sufficient to answer the question. 4) if the data in statement I and II together are not sufficient to answer the question. 5) if the data in both statements I and II together are necessary to answer the question. 16. Seventeen people are standing in a straight line facing south. What is Bhavna's position from the left end of the line? I. Sandeep is standing second to the left of Sheetal. Only five people stand between Sheetal and the one who is standing at the extreme right end of the line. Four people stand between Sandeep and Bhavna. II. Anita is standing fourth to the left of Sheetal. Less than three people are standing between Bhavna and Anita. Ans: 5 17. Five letters - A, E, G, N and R - are arranged from left to right according to certain conditions. Which letter is placed third? I. G is placed second to the right of A. E is on the immediate right of G. There are only two letters between R and G. II. N is exactly between A and G. Neither A nor G is at the extreme and of the arrangement. Ans: 3 18. Six people - S, T, U, V, W and X - are sitting around a circular table facing the centre. What is T's position with respect to X? I. Only two people sit between U and W. X is second to the left of W. V and T are immediate neighbours of each other. II. T is to the immediate right of V. There are only two people between T and S. X is an immediate neighbour of S but not of V. Ans: 2 Directions (Q.19 - 20): Read the following information carefully and answer the given questions. The convenience of online shopping is what I like best about it. Where else can you shop even at midnight wearing your night suit? You do not have to wait in a line or wait till the shop assistant is ready to help you with your purchases. It is a much better experience as compared to going to a retail store. - A consumer's view. 19. Which of the following can be a strong argument in favour of retail store owners? 1) Online shopping portals offer a great deal of discounts which retail stores offer only during the sale season. 2) One can compare a variety of products online which cannot be done at retail stores. 3) Many online shopping portals offer the 'cash on delivery' feature which is for those who are sceptical about online payments. 4) Many consumers prefer shopping at retail stores which are nearer to their houses. 5) In online shopping the customer may be deceived as he cannot touch the product he is paying for. 20. Which of the following can be inferred from the given information? (An inference is something that is not directly stated but can be inferred from the given information) 1) One can shop online only at night. 2) Those who are not comfortable using computers can never enjoy the experience of online shopping. 3) All retail stores provide shopping assistants to each and every customer. 4) The consumer whose view is presented has shopped at retail stores as well as online. 5) The consumer whose view is presented does not have any retail stores in her vicinity. 21. Read the following information carefully and answer the given question: Many manufacturing companies are now shifting base to the rural areas of the country as there is a scarcity of space in urban areas. Analysts say that this shift will not have a huge impact on the prices of the products manufactured by these companies as only about 30% consumers live in urban areas. Which of the following may be a consequence of the given information? 1) The prices of such products will decrease drastically in the urban areas. 2) People living in urban areas will not be allowed to work in such manufacturing companies. 3) These manufacturing companies had set-ups in the urban areas before shifting base. 4) Those who had already migrated to the urban areas will not shift back to rural areas. 5) The number of people migrating from rural to urban areas in search of jobs may reduce. 22. Read the following information carefully and answer the given question: 'Pets are not allowed in the park premises.' - a notice put up at the park entrance by the authority that is responsible for maintenance of the park. Which of the following can be an assumption according to the given information?(An assumption is something that is supposed or taken for granted) 1) At least some people who visit the park have pets. 2) This is the only park which does not allow pets. 3) People who ignored this notice were fined. 4) There are more than one entrance to the park. 5) Many people have now stopped visiting the park. Directions (Q.23-25): Read the following information carefully and answer the given questions: Despite repeated announcements thatmobile phones were not allowed in the examination hall, three students were caught with their mobile phones. (A) Mobile phones nowadays have a lot of features and it is easy to cheat with their help. (B) The invigilator must immediately confiscate the mobile phones and ask the students to leave the exam hall immediately. (C) Mobile phones are very expensive and leaving them in bags outside the exam hall is not safe. (D) There have been incidents where students who left the exam hall early stole the mobile phones kept in the bags of the students who were writing the exam. (E) The school authorities must ask the students to leave their phones in the custody of the invigilator before the exam in order to avoid thefts of mobile phones. (F) None of the other students were carrying their phones in the exam hall. 23. Which of the following among (A), (B), (C) and (D) may be a strong argument in favour of the three students who were caught with their mobile phones? 1) Only (A) 2) Both (A) and (B) 3) Both (C) and (D) 4) Only (C) 5) Both (B) and (D) 24. Which of the following among (A), (B), (E) and (F) may be the reason behind the school making such announcements before the exam? 1) Only (B) 2) Both (B) and (E) 3) Only (F) 4) Only (A) 5) Both (E) and (F) 25. Which of the following among (A), (B), (D) and (F) can be in immediate course of action for the invigilator? 1) Only (B) 2) Both (A) and (D) 3) Only (A) 4) Both (D) and (F) 5) Only (F) Directions (Q. 26-28) Study the information carefully to answer these questions. In a team, there are 240 members (males and females). Two-thirds of them are males. Fifteen per cent of males are graduates. Remaining males are non-graduates. Three-fourths of the females are graduates. Remaining females are non-graduates. 26. What is the difference between the number of females who are non-graduates and the number of males who are graduates? 1) 2 2) 24 3) 4 4) 116 5) 36 27. What is the sum of the number of females who are graduates and the number of males who are non-graduates? 1) 184 2) 96 3) 156 4) 84 5) 196 28. What is the ratio of the total number of males to the number of females who are non-graduates? 1) 6 : 1 2) 8 : 1 3) 8 : 3 4) 5 : 2 5) 7 : 2 General Awareness, Marketing & Computers: 29. 'C' in CPU denotes 1) Central 2) Common 3) Convenient 4) Computer 5) Circuitry 30. A joystick is primarily used to/for 1) Print Text 2) Computer gaming 3) Enter text 4) Draw pictures 5) Control sound on the screen 31. Which is not a storage device? 1) CD 2) DVD 3) Floppy disk 4) Printer 5) Hard disk 32. Which of the following uses a handheld operating system? 1) A Supercomputer 2) A Personal computer 3) A Laptop 4) A Mainframe 5) A PDA 33. To display the contents of a folder in Windows Explorer you should 1) click on it 2) collapse it 3) name it 4) give it a password 5) rename it 34. The CPU comprises of Control, Memory, and ------ units. 1) Micro processor 2) Arithmetic/Logic 3) Output 4) ROM 5) Input 35. ................ is the most important/powerful computer in a typical network. 1) Desktop 2) Network client 3) Network server 4) Network station 5) Network switch 36. A (n) ................ appearing on a web page opens another document when clicked. 1) Anchor 2) URL 3) Hyperlink 4) reference 5) heading 37. Which of the following refers to the rectangular area for displaying information and running programmes? 1) Desktop 2) Dialog box 3) Menu 4) Window 5) Icon 38. ................ is a Windows utility programme that locates and eliminates unnecessary fragments and rearranges files and unused disk space to optimise operations. 1) Backup 2) Disk Cleanup 3) Disk Defragmenter 4) Restore 5) Disk Restorer 39. Which of the following refers to too much electricity and may cause a voltage surge? 1) Anomaly 2) Shock 3) Spike 4) Virus 5) Splash 40. The software that is used to create text-based documents are referred to as 1) DBMS 2) Suites 3) Spreadsheets 4) Presentation software 5) Word processors 41. ................ devices convert human-understandable data and programmes into a form that the computer can process. 1) Printing 2) Output 3) Solid state 4) Monitor 5) Input 42. Effective Selling Skills depend on the 1) Number of Languages known to the DSA 2) Data on marketing staff 3) Information regarding IT market 4) Knowledge of related markets 5) Ability to talk fast 43. A Direct Selling Agent (DSA) is required to be adept in 1) Surrogate marketing 2) Training skills 3) Communication skills 4) Market Research 5) OTC Marketing 44. Leads can be best sourced from 1) Foreign Customers 2) Yellow paages 3) Dictionary 4) List of vendors 5) Local supply chains 45. A successful marketing person requires one of the following qualities: 1) Empathy 2) Sympathy 3) Insistence 4) Aggressiveness 5) Pride 46. Innovation in marketing is the same as 1) Abbreviation 2) Communication 3) Creativity 4) Aspiration 5) Research work 47. Market segmentation can be resorted to by dividing the target group as per 1) Income levels of customers 2) age of the employees 3) Needs of the salespersons 4) Marketing skills of the employees 5) Size of the organisation 48. Post-sales activities include 1) Sales presentation 2) Customer's feedback 3) Customer identification 4) Customer's apathy 5) Product design 49. The 'USP' of a product denotes the 1) Usefulness of the product 2) Drawbacks of a Product 3) Main Functions 4) Number of allied products available 5) High selling features of a product 50. The competitive position of a company can be improved by 1) Increasing the selling price 2) Reducing the margin 3) Ignoring competitors 4) Increasing the cost price 5) Understanding and fulfilling customers' needs 51. Value-added services means 1) Low-cost products 2) High-cost products 3) At-par services 4) Additional services for the same cost 5) Giving discounts 52. The target market for Debit Cards is 1) All existing account-holders 2) All agriculturists 3) All DSAs 4) All vendors 5) All Outsourced agents 53. A good Brand can be built up by way of 1) Customer Grievances 2) Break-down of IT support 3) Old age 4) Large number of products 5) Consistent offering of good services English Language: Directions (Q. 54-58): Rearrange the following six sentences (A), (B), (C), (D), (E) and (F) in the proper sequence to form a meaningful paragraph and then answer the questions given below. (A) The group desired to enhance the learning experience in schools with an interactive digital medium that could be used within and outside the class-room. (B) Then the teacher can act on the downloaded data rather than collect it from each and every student and thereby save his time and effort. (C) Edutor, decided the group of engineers, all alumni of the Indian Institute of Technology, when they founded Edutor Technologies in August 2009. (D) They can even take tests and submit them digitally using the same tablets and the teachers in turn can download, the tests using the company's cloud services. (E) With this desire they created a solution that digitises school text books and other learning material so that students no longer need to carry as many books to school and back as before, but can access their study material on their touch-screen tablets. (F) Amechanic works on motors and an accountant has his computer. Likewise, if a student has to work on a machine of device, what should it be called? 54. Which of the following sentences should be the FIRST after rearrangement? 1) F 2) D 3) A 4) C 5) E 55. Which of the following sentences should be the THIRD after rearrangement? 1) A 2) B 3) D 4) E 5) F 56. Which of the following sentences should be the SIXTH (LAST) after rearrangement? 1) A 2) F 3) E 4) B 5) D 57. Which of the following sentences should be the FOURTH after rearrangement? 1) A 2) F 3) E 4) B 5) C 58. Which of the following sentences should be the FIFTH after rearrangement? 1) A 2) D 3) C 4) E 5) F Directions (Q. 59-68): In the following passage there are blanks, each of which has been numbered. These numbers are printed below the passage and against each, five words/ phrases are suggested, one of which fits the blank appropriately. Find out the appropriate word/ phrase in each case. There is a considerable amount of research about the factors that make a company innovate. So is it possible to create an environment (59) to innovation? This is a particularly pertinent (60) for India today. Massive problems in health, education etc (61) be solved using a conventional Approach but (62) creative and innovative solutions that can ensure radical change and (63). There are several factors in India's(64). Few countries have the rich diversity that India or its large, young population (65). While these (66) innovation policy interventions certain additional steps are also required. These include (67) investment in research and development by (68) the government and the private sector, easy transfer of technolgy from the academic world etc. To fulfil its promise of being prosperious and to be at the forefront, India must be innovative. 59. 1) stimuli 2) conducive 3) incentive 4) facilitated 5) impetus 60. 1) objective 2) controversy 3) doubt 4) question 5) inference 61. 1) cannot 2) possibly 3) should 4) never 5) must 62. 1) necesary 2) apply 3) need 4) consider 5) requires 63. 1) quantity 2) advantages 3) increase 4) chaos 5) growth 64. 1) challenges 2) praises 3) favour 4) leverage 5) esteem 65. 1) blessed 2) enjoys 3) endows 4) prevails 5) occurs 66. 1) aid 2) jeopardise 3) promotes 4) endure 5) cater 67. 1) acute 2) utilising 3) restricting 4) inspiring 5) increased 68. 1) both 2) besides 3) combining 4) participating 5) also Directions (Q. 69-73): In each of the following questions four words are given, of which two words are most nearly the same or opposite in meaning. Find the two words which are most nearly the same or opposite in meaning and indicate the number of the correct letter combination, by darkening the appropriate oval in your answer sheet. 69. (A) consent (B) nascent (C) emerging (D) Insecure 1) A ?C 2) B ? D 3) B ? C 4) A ? D 5) A ? B 70. (A) elated (B) eccentric (C) explicit (D) abnormal 1) A ?B 2) B ?D 3) A ? C 4) A ?D 5) D ? C 71. (A) abundance (B) incomparable (C) Projection (D) plethora 1) A ?C 2) A ?B 3) C ? D 4) B ?D 5) A ? D 72. (A) purposefully (B) inaccurately (C) inadvertently (D) unchangeably 1) A ?C 2) A ?B 3) B ? C 4) B ?D 5) A ? D 73. (A) germane (B) generate (C) reliable (D) irrelevant 1) B ?D 2) B ?C 3) A ? B 4) C ?D 5) A ? D Section A: GK Questions 1. 24th ASEAN summit venue Answer : Nay Pyi Taw 2. A Scheme named Rajiv Gandhi Scheme for Empowerment of Adolescent Girls is also known as Answer : Sabla 3. A. R. Rahman street is in which country Answer : Canada 4. Capital of Canada Answer : Ottawa 5. Capital of Ethiopia Answer : Addis Ababa 6. Capital of Sweden Answer : Stockholm 7. 150 km missile launched by DRDO recently Answer : Prithvi 8. 2015 Commonwealth Heads of Government meeting will be held in Answer : Malta 9. Amount provided for developing 100 samrt cities Answer : 7060 crores 10. Amul related to which revolution Answer : White Revolution 11. Amul set up Dairy worth 200 crore at which city Answer : Varanasi 12. Army Flag day is observed on Answer : 7th December 13. Ashes trophy played between countries Answer : Australia and England 14. Author of lowland Answer : Jhumpa Lahari 14. Azlan Shah who died recently is a Answer : Hockey Player 15. Accidental prime minister written by ?SANJAY BARU? and was published by Answer : Penguin books limited 16. Capital of Bhutan Answer : Thimphu 17. Capital of Brazil Answer : Brasilia 18. Adult literacy rate in 2011 census Answer : 74.04 % 19. Best ranked IIT among BRICS top universities 2014 Answer : IIT Delhi 20. Bharatiya Mahila Bank Chairman Answer : Usha Ananantha Subramanium 21. Capital of Argentina Answer : Buenos Aires 22. Bharatiya Mahila Bank inaugurated by Answer : Manmohan Singh 23. Book ?My Journey : Transforming into dreams? written by Answer : Usha Ananantha Subramanium APJ Abdul Kalam 24. Budget in which article Answer : Usha Ananantha SubramaniumArticle 112 25. Banks Nationalization Answer : 1969 26. Beiji oil refinery located in Answer : Iraq 27. C.Rangarajan estimation percentage of poverty line Answer : 29.5% 28. Camel is the official animal of which state Answer : Rajasthan 29. Capro Group has won the International Business of the year award is owned by Answer : Lord Swaraj Paul 30. Casual Vacancy written by Answer : J K Rowling 31. CBS full form Answer : Core Baking Solution 32. Chemical and fertilizer cabinet minister Answer : Ananth Kumar 33. Chinese Currency Answer : Yuan 34. Commercial Paper is issued for how many days Answer : A minimum of 7 days and a maximum of up to one year from the date of issue 35. Commercial Papers issued in what multiples Answer : Rs. 5 lakh or multiples thereof 36. Committee on Cauvery water dispute Answer : B. S. Chauhan 37. One currency question like that UK 39. Next FIFA world Cup Host country Answer : Russia 40. Capital of Indonesia 41. Hard money 42. Reverse REPO rate 43. Home minister of India 44. International Woman?s Day 45. MB Shah Committee related about ?? IPL Spot fixing 46. Rupee sign 47. WHO head 48. Who is 14th number of PM 49. UNESCO Headquarter 50. Women Hockey World Cup winner country 51. PV Sandhu related to which field 52. Which fruit is banned in America Answer : Mango 53. Is there any fee for lodging a complaint with the banking ombudsman Answer : No fee is required 54. Full form of RAM Answer : Random Access Memory 55. World Bank President Answer : Jim Yong 56. Who is the Lok Sabha Speaker Answer : Sumitra Mahajan 57. Manufacturer of VELOX Answer : Singapore 58. Reliance produced less amount of gas resulting in government loss in which block? Answer : KG ? D6 58. Sri Lankan cricketer who retired very recently Answer : Mahela Jayawardane 59. Currency of Saudi Arabia Answer : Saudi Riyal 60. Name the new Tiger reserve in Maharashtra Answer : Bor Tiger Reserve 61. Chandi Prasad has contributed in which area Answer : Environment 62. Which song is dedicated by Amir Khan to the legendry cricketer Sachin Tendulkar Answer : Dhoom Machale 63. Mascot of commonwealth games of 2014 Answer : Clyde 64. Which is the Insurance regulator Answer : IRDA? 65. Two questions on mutual funds 66. Two questions from budget 67. World TV Day Answer : 21st November 68. World AIDS day Answer : 1 dec 69. Durand Cup is related to Answer : Football 70. TARC headed by 71. Question on Asset Reconstruction Company 72. Whose sign is present 1000Rs note Answer : RBI Governor 73. BCSBI Functions Section B. Computer knowledge questions 1. MICR full form Answer : Magnetic ink character recognition 2. What is artificial intelligence 3. Computer programming error Answer : bug 4. Two questions asked about MS word 5. 1 question from DBMS 6. 1 full form Internet section like that CSS Answer : Cascaded Style Sheet answer 7. C developed by Answer : Denis Richi 8. First calculating machine 9. Binary conversion 10. Latest super computer name Section C. Reasoning Sections 1. Syllogism (5 Questions) 2. Data sufficiency (Hight, direction......) 3. One very simple puzzle 4. One puzzle about ?..5 people sit in line toward north & 5 people south 5. Coding & decoding Section D. Marketing Aptitude Questions 1. Marketing Mixture 2. Outdoor marketing 3. Tele marketing 4. 80% 20% principle ?? 80 work 20 staff 5. Sales 6. Marketing steps SBI Aptitude Questions SBI Preliminary Exam Question Paper Quantitative Aptitude 1. What percentage of numbers from 1 to 70 have 1 or 9 in the unit?s digit? a) 1 b) 14 c) 20 d) 21 e) None of these 2. (935421 × 625) = ? a) 575648125 b) 584638125 c) 584649125 d) 585628125 e) None of these 3. Reena took a loan of Rs.1200 with simple interest for as many years as the rate of interest. If she paid Rs.432 as interest at the end of the loan period, what was the rate of interest? a) 3.6 b) 6 c) 18 d) Data inadequate e) None of these 4. A leak in the bottom of a tank can empty the full tank in 8 hours. An inlet pipe fills water at ther ate of 6 litres a minute. When the tank is full, the inlet is opened and due to the leak, the tank is empty in 12 hours. How many litres does the cistern hold? a) 7580 b) 7960 c) 8290 d) 8640 e) None of these 5. In what ratio must a grocer mix two varieties of pulses costing Rs.15 and Rs.20 per kg respectively so as to get a mixture worth Rs.16.50 kg? a) 3 : 7 b) 5 : 7 c) 7 : 3 d) 7 : 5 e) None of these 6. What was the day of the week on 28th May 2006? a) Thursday b) Friday c) Saturday d) Sunday e) None of these 7. A man can row upstream at 8 kmph and downstream at 13 kmph. The speed of the stream is a) 2.5 km/hr b) 4.2 km/hr c) 5 km/hr d) 10.5 km/hr e) None of these 8. If the sum of the interior angles of a regular polygon measures upto 1440 degrees, how many sides does the polygon have? a) 10 sides b) 8 sides c) 12 sides d) 9 sides e) None of these 9. Number of solutions of the equation tan x + sec x = 2 cos x, lying in the interval [0, 2?] is a) 0 b) 1 c) 2 d) 3 e) None of these 10. A alone can do a piece of work in 6 days and B alone 8 days. A and B undertook to do it for Rs.3200. With the help of C, they completed the work in 3 days. How much is to be paid to C? a) Rs.375 b) Rs.400 c) Rs.600 d) Rs.800 e) None of these 11. A train travels at an average of 50 miles per hour for 2x1/2 hours and then travels at a speed of 70 miles per hour for 1x1/2 hours. How far the train did travels in the entire 4 hours? a) 120 miles b) 150 miles c) 200 miles d) 230 miles e) None of these 12. A shopkeeper give 12% additional discount on the discounted price, after giving an initial discount of 20% on the labeled price of a radio. If the final sale price of the radio is Rs.704, then what is its labeled price? a) Rs.844.80 b) Rs.929.28 c) Rs.1000 d) Rs.1044.80 e) None of these 13. A person was asked to state his age in years. His reply was, ?Take my age three years hence, multiply it by 3 and then subtract three times my age three years ago and you will know how old I am.? What was the age of the person? a) 14 years b) 18 years c) 20 years d) 32 years e) None of these 14. In how many ways can the letters of the word 'LEADER' be arranged? a) 72 b) 144 c) 360 d) 72 e) None of these 15. Find the greatest number that will divide 43, 91 and 183 so as to leave the same remained in each case. a) 4 b) 7 c) 9 d) 13 e) None of these 16. The average weight of 8 person's increases by 2.5 kg when a new person comes in place of one of them weighing 65 kg. What might be the weight of the new person? a) 76Kg b) 76.5Kg c) 85Kg d) Data inadequate e) None of these 17. In Arum?s opinion, his weight is greater than 65 kg but less than 72 kg. His brother doest not agree with Arum and he thinks that Arum?s weight is greater than 60 kg but less than 70 kg. His mother's view is that his weight cannot be greater than 68 kg. If all are them are correct in their estimation, what is the average of different probable weights of Arum? a) 67 kg b) 68 kg c) 69 Kg. d) Data inadequate e) None of these 18. Product of two co-prime numbers is 117. Their LCM should be a) 1 b) 117 c) equal to their H.C.F. d) Cannot be calculated e) None of these 19. A starts a business with Rs.3500 and after 5 months, B joins with A as his partner. After a year, the profit is divided in the ratio 2 : 3. What is B?s contribution in the capital? a) Rs.7500 b) Rs.8000 c) Rs.8500 d) Rs.9000 e) None of these 20. In how many different ways can the letters of the word 'DETAIL' be arranged in such a way that the vowels occupy only the odd positions? a) 32 b) 48 c) 36 d) 60 e) None of these 21. Two dice are tossed. The probability that the total score is a prime number is: a) 1/6 b) 5/12 c) 1/2 d) 7/9 e) None of these 22. The sum of the ages of 5 children born at the intervals of 3 years each is 50 years. What is the age of the youngest child? a) 4 years b) 8 years c) 10 years d) 12 years e) None of these 23. A shopkeeper professes to sell his goods at cost price but uses a weight of 800 gm instead of kilogram weight. Thus, he make a profit of a) 20% b) 22% c) 25% d) Data inadequate e) None of these 24. The number of common terms to the two sequences 17, 21, 25 ?. 417 and 16, 21, 26 ?. 466 is : a) 19 b) 20 c) 21 d) 84 e) None of these 25. ?50 × ?98 is equal to a) 63.75 b) 65.95 c) 70 d) 70.25 e) None of these SBI Aptitude-English SBI PO Preliminary Exam Model Paper English Language Directions (Qs. 1-10): Read the following passage carefully and answer the questions given after the passage. Certain words/phrases have been printed in bold to help you locate them while answering some of the questions. India is rushing headlong towards economic success and modernization, counting on high-tech industries such as information technology and biotechnology to propel the nation to prosperity. India?s recent announcement that it would no longer produce unlicensed inexpensive generic pharmaceuticals bowed to the realities of the World Trade Organisation while at the same time challenging the domestic drug industry to compete with the multinational firms. Unfortunately, its weak higher education sector constitutes the Achilles heel of this strategy. Its systematic disinvestment in higher education in recent years has yielded neither world - class research nor very many highly trained scholars, scientists nor manager to sustain high - tech development. India?s main competitors especially China but also Singapore, Taiwan and South Korea are investing in large and differentiated higher education system. They are providing access to large numbers of students at the bottom of the academic system while at the same time building some research - based universities that are able to compete with the world?s best institutions. The recent London Times Higher Education Supplement ranking of the world?s top 200 universities included three in China, three in Hong Kong, three in South Korea, one in Taiwan and one in India. These countries are positioning themselves in the knowledge based economies of the coming era. There was a time when countries could achieve economic success with cheap labour and low - tech manufacturing. Low wages still help, but contemporary large - scale development requires a ophisticated and at least partly knowledge based economy. India has chosen that path but will find a major stumbling block in its university system. India has significant advantage in the 21st century knowledge race. It?s education sector is the third largest in the world in student numbers after China and the United States. It uses English as the primary language of higher education and research. It has a long academic tradition. Academic freedom is respected. There are a small number of high quality institutions, departments and centres that can form the basis of quality in higher education. The fact that the States, rather than the Central Government, exercise major responsibility for higher education creates a rather cumbersome structure, but the system allows for a variety of policies and approaches. Yet the weaknesses for outweigh the strengths. India educates approximately 10 per cent of its young people in higher education compared with more than half in the major industrialised countries and 15 per cent in China. Almost all of the world?s academic systems resemble a pyramid, with a small high quality tier at the top and a massive sector at the bottom. India has a tiny top tier. None of its universities occupy a solid position at the top. A few of the best universities have some excellent departments and centres and there are a small number of outstanding undergraduate colleges. The University Grants Commission?s recent major support of five universities to build on their recognised strength is a step forward, recognising a differentiated academic system and fostering excellence. These universities, combined, enrol well under 1 per cent of the student population. 1. Which of the following is/are India?s strength/s in terms of higher education? I. Its system of higher education allows variations. II. Medium of instruction for most higher learning is English. III. It has the paraphernalia, albeit small in the number, to build a high quality education sector. (1) Only II (2) Only I & II (3) Only III (4) Only II & III (5) All I, II & III 2. What does the phrase ?Achilles Heel? mean as used in this passage? (1) Weakness (2) Quickness (3) Low quality (4) Nimbleness (5) Advantage 3. Which of the following are Asian countries, other than India, doing to head towards knowledge based economy? I. Building highly competitive research based universities. II. Investing in diverse higher education system III. Providing access to higher education to a select few students. (1) Only I (2) Only I & II (3) Only II & III (4) Only II (5) All I, II & III 4. Which of the following is/are India?s weakness/es when it comes to higher education? I. Indian universities do not have the requisite teaching faculty to cater to the needs of the higher education. II. Only five Indian universities occupy the top position very strongly, in the academic pyramid, when it comes to higher education. III. India has the least percentage of young population taking to higher education as compared to the rest of the comparable countries. (1) Only I & II (2) Only II (3) Only III (4) Only I & III (5) All I, II & III 5. What did India agree to do at the behest of the World Trade Organisation? (1) It would stop manufacturing all types of pharmaceuticals. (2) It would ask its domestic pharmaceuticals companies to compete with the international ones. (3) It would buy only license drugs from USA (4) It would not manufacture cheap common medicines without a license. (5) None of these Directions (6 - 8) Choose the word/ group of words which is most similar in meaning to the word/group of words printed in bold as used in the passage. 6. FOSTERING (1) safeguarding (2) neglecting (3) sidelining (4) nurturing (5) ignoring 7. PROPEL (1) drive (2) jettison (3) burst (4) acclimatize (5) modify 8. STUMLING BLOCK (1) argument (2) frustration (3) advantage (4) hurdle (5) fallout Directions (9 - 10): Choose the word/ group of words which is most opposite in meaning to the word/group of words printed in bold as used in the passage. 9. CUMBERSOME (1) handy (2) manageable (3) breathtaking (4) awkward (5) difficult 10. RESEMBLE (1) against (2) similar to (3) mirror (4) differ from (5) unfavourable to Directions (Qs. 11-15): Read each sentence to find out whether there is any grammatical error or idiomatic error in it. The error if any, will be in one part of the sentence. The number of that part is the answer. If there is no error, the answer is (5). (Ignore error of punctuation, if any). 11. The bane of Indian hockey today is /(1) lack of interest by the part of the public /(2) which in turn is fuelled by the perception that /(3) it doesn?t pay to take up the sport as a career. /(4) No error /(5) 12. Illegal sand mining has become /(1) a boom business fuelled /(2) by the ever - increasing demand /(3) of the construction industry. /(4) No error /(5) 13. In view of the intense cold wave conditions /(1) prevailing in the state, the government declared /(2) holidays in all the schools /(3) for a period of ten days. /(4) No error (5) 14. As market leaders, /(1) we have always been at /(2) the forefront of creating awareness /(3) between the public. /(4) No error (5) 15. If the IPL has succeeded in drawing /(1) an audience across the country, it is because /(2) cricket has always had a strong foundation /(3) and a dedicated audience. /(4) No error /(5) Directions (Qs. 16 -20) Rearrange the given five sentences (A), (B), (C), (D) and (E) in a proper sequence so as to form a meaningful paragraph and answer the given questions. (A) Therefore, it is important to source a large part of economic growth in agriculture, in rural non -agricultural activities and in productive expansion of the informal sector have high employment elasticities, as well as in an export strategy based on labour intensive export. (B) It is important because it creates more resources and has the potential of creating more space for the involvement of the poor. (C) If the growth is sourced from those sectors of the economy or those activities that have a natural tendency to involve the poor in their expansion, such growth helps poverty eradication. (D) Economic growth is important. (E) But this involvement depends on the sources of growth and nature of the growth. 16. Which of the following should be the FIRST sentence after rearrangement? (1) A (2) B (3) C (4) D (5) E 17. Which of the following should be the SECOND sentence after rearrangement? (1) E (2) D (3) C (4) B (5) A 18. Which of the following should be the THIRD sentence after rearrangement? (1) A (2) B (3) C (4) D (5) E 19. Which of the following should be the FOURTH sentence after rearrangement? (1) E (2) D (3) C (4) B (5) A 20. Which of the following should be the FIFTH sentence after rearrangement? (1) A (2) B (3) C (4) D (5) E Directions (Qs. 21- 30): In the following passage there are blanks, each of which has been numbered. These numbers are printed below the passage andagainst each, five words are suggested, one of which fits the blank appropriately. Find the appropriate word in each case. Around the world, forests are being (21) at a rate of about thirteen million hectares a year and deforestation accounts for an estimated 17% - 20% of all global emissions. In addition, forests and other terrestrial carbon sinks play a (22) role in preventing runaway climate change, soaking up a full 2.6 Gt of atmospheric carbon every year. The destruction of forests, therefore not only emits carbon - a staggering 1.6 Gt a year, which severely (23) forests capacity to absorb emissions from other sources - but also drastically (24) the amount of forest land available to act as a carbon sink in the future.However, the effects of deforestation extend beyond carbon. Rainforests (25) a wide variety of ecosystems services, from regulating rainfall to purifying groundwater and keeping fertile soil from (26); deforestation in one area can seriously damage food production and (27) to clean water in an entire region. The value of global ecosystem services has been estimated at 33 trillion USD each year (almost half of global GDP), but these services have been taken for granted without a mechanism to make the market reflect their value. Rainforests are also a home and (28) of income for a huge number of people in Africa, Asia and South America. (29) this, economic pressures frequently drive both local communities and national governments in the developing world to (30) these forests in ways that are unsuitable, completely stripping vast areas for fuel, timber, mining, or agricultural land. 21.(1) ended (2) destroyed (3) extinct (4) killed (5) wasted 22.(1) tough (2) important (3) vital (4) biggest (5) effective 23.(1) affect (2) diminish (3) increases (4) alternates (5) impairs 24.(1) plagues (2) develops (3) reduces (4) shortens (5) influences 25.(1) sell (2) offers (3) give (4) provide (5) earns 26.(1) transforming (2) decoding (3) erupting (4) draining (5) eroding 27.(1) handiness (2) excess (3) availability (4) access (5) supply 28.(1) beginning (2) source (3) ways (4) reference (5) measure 29.(1) Despite (2) Also (3) Inspite (4) Apart (5) Beside 30.(1) exploit (2) encompass (3) nurture (4) work (5) improve SBI Aptitude-English SBI PO Exam Model Paper English Language Directions (Q. 1-10) Read the following passage carefully and answer the questions given below it. Certain words have been printed in bold to help you to locate them while answering some of the questions. Once a thief named Kalu had planned to loot the king's treasury. At midnight, he went to the palace and began to drill a hole in the side wall of the treasury. The king, who was awake in his bedroom just above the treasury, came out to investigate the whirring sound. He was dressed in a simple nightgown and the thief could not recognize him. He asked Kalu who he was and what he was doing. The latter said, "Sir, I am a thief and intend to loot this treasury. I presume that you are also a thief and have come with the same intentin. No matter, let us both go inside and we shall share the loot equally. "Both entered the treasury and divided all the money and the jewels equally between them. Inside a locker they found three big diamond pieces. As the thief was puzzled as to how to divide the three pieces into two portions, the king siggested. "We have taken away everything else. Let us leave one diamond piece for the poor king and share the rest equally". Kalu agreed and when he took his leave, the king asked for his name and address. As Kalu had taken a vow of telling only the truth, he h ave the correct information. The king took away his share of the loot and hid it in his room. Next morning he asked his Prime Minister to inspect the treasury as he had heard some strange sounds during the previous night. The Prime Minister saw to his horror that all the valuables were missing and only a single diamond was left, perhaps inadvertantly, by the theif. He put the diamond in his oiwn shift pocket as its loss could be ascribed to the thief and nobody would suspect the Prime Minister. The Prime Minister went back to the king. The king particularly enquirerd. "Do you mean that the theif has completely denuded the treasury of its valuables and not a single item has been left?" The Prime Minister confirmed it. The king asked the chief of police to bring in Kalu. When Kalu came he was unable to recfognize the king as his accomplice of the previous night. The king asked him, "Are you the theif who has stolen everything from my treasury leaving nothing back?" Kalu confirmed it but said, "Sir, I did leave one diamond back in the locker as advised by an accomplice of mine and it should st ill be there." The Prime Minister interrupted saying, "Your Majesty, this thief is lying. There is nothing left in the locker." The king asked the police chief to search the pockets of the Prime Minister, from where the missing diamond was recovered. The kind told his courtiers, "Here is a Prime Minister, who is a liar and a thief and here is a thief who is at truthful gentleman." 1. The king came out in the middle of the night in order to ... a) Help Kalu to break into the palace treasury b) Share the loot equally between Kalu and himself c) Find out the source of and reason for the sound he had heard d) Catch the thief who had come to steal his valuables e) None of these 2. Kalu could not recognize the kjing because ... a) The king was wearing clothes like those of an ordinary person b) The king's clothes were covered by a simple nightgown c) Kalu had never seen the king before d) Kalu had not seen the king descending from his bedroom e) None of these 3. Which of the following made the king suspect the Prime Minister? The Prime Minister's statement that ... a) Except for one piece of diamond all other valuables were stolen b) All the valuables without any exception were stolen from the treasury c) The thief was lying when he said h e had left one diamond back in the locker d) The search for the diamond did not yield any favourable result e) None of these 4. Which of the following horrified the Prime Minister? a) The valuables missing from the King's treasury b) A piece of diamond left in the locker c) Certain strange sounds heard by the Prime Minister d) The fact that the king suspected him of stealth e) None of these 5. Choose the word which is most nearly the Same in meaning as the word given in bold as used in the passage. Accomplice a) C-traveller b) Collaborator c) Controller d) Coordinator e) None of these 6.Choose the word which is most nearly the Same in meaning as the word given in bold as used in the passage. Ascribed a) Attributed b) Donated c) Attached d) Withdrew e) None of these 7. Choose the word which is most nearly the Same in meaning as the word given in bold as used in the passage. Denuded a) Uncovered b) Stripped c) Destroyed d) Eiscarded e) None of these 8. Choose the word which is most Opposite in meaning of the word given in bold as used in the passage. Inadvertently a) Knowingly b) Sensibly c) Indifferently d) Unwittingly e) None of these 9. Choose the word which is most Opposite in meaning of the word given in bold as used in the passage. Previous a) New b) Preceding c) Novel d) Subsequent e) None of these 10.They decided to leave the diamond inside the locker because ?. A. They wanted some part of the wealth to be left for the Prime Minister. B. It was difficult for them to carry the 3rd piece of diamond. C. The total number of diamonds being odd, they had to leave out onepiece to facilitate equal distribution. a) None b) All the three c) Only A d) Only B e) Only C Directions (Q. 11-15): Rearrange the following six sentences (A), (B), (C), (D), (E) and (F) in the proper sequence to form a meaningful paragraph and then answer the questions given below. A. The reasons for their happiness was that they had come to the Vithal temple of Pandharpur to see their beloved Lord. B. There were rich, poor men, women, children, blind, handicapped among them. C. They were all in a very happy state of mind, as was apparent from their glowing faces. D. They would worship and seek the Divine blessings while in Pandharpur. E. It was an auspicious day and many people had assembled in the temple. F. Though different there was one commonality among. 11. Which of the following should be the Fourth statement after rearrangement? a) A b) B c) C d) D 12. Which of the following should be the Fifth statement after rearrangement? a) A b) B c) C d) D 13. Which of the following should be the Sixth statement after rearrangement? a) A b) B c) C d) D 14. Which of the following should be the First statement after rearrangement? a) A b) B c) C d) E 15. Which of the following should be the Second statement after rearrangement? a) A b) B c) C d) D Directions (Q. 16-20) Read this sentence to find out whether there is any grammatical mistake/error in it. The error, if any, will be in one part of the sentence. Mark the part with the error as your answer. If there is no error, mark ?No error? as your answer. (Ignore the errors of punctuation if any). 16. Mangal Pandey was well known (a) / because he was involved (b) / in the initial stages (c) / of the Indian rebellion. (d) / No error (e) 17. Most of the Indian populatoins still lives (a) / in its villages and thus the contribution of (b) / agriculture to Indian economy (c) / becomes very important. (d) / No error (e) 18. Catherine?s grandfather always (a) / lost his balance while walking (b) / and would be found fallen (c) / on the road. (d) / No error (e) 19. Her doctor was (a) / annoyed because she (b) / ignore her health (c) / even after being hospitalised twice. (d) No error (e) 20. Raghav was worry (a) / about telling his parents (b) / that he wanted to move out (c) / and live independently. / (d) No error (e) Directions (Q. 91-95) Below the four words are given. One of these four words may be wrongly spelt. Find out the word which is wrongly spelt, if there is any. The number of that word is your answer. If all the words are correctly spelt mark All correct as the answer. 21. Below the four words are given. One of these four words may be wrongly spelt. Find out the word which is wrongly spelt, if there is any. The number of that word is your answer. If all the words are correctly spelt mark All correct as the answer. a) Adventure b) Demonstration c) Environment d) Innosent e) All Correct 22. Below the four words are given. One of these four words may be wrongly spelt. Find out the word which is wrongly spelt, if there is any. The number of that word is your answer. If all the words are correctly spelt mark All correct as the answer. a) Limitasion b) Dependable c) Miniature d) Qualitative e) All Correct 23. Below the four words are given. One of these four words may be wrongly spelt. Find out the word which is wrongly spelt, if there is any. The number of that word is your answer. If all the words are correctly spelt mark All correct as the answer. a) Lucrative b) Ancestral c) Performanse d) Incidentally e) All Correct 24. Below the four words are given. One of these four words may be wrongly spelt. Find out the word which is wrongly spelt, if there is any. The number of that word is your answer. If all the words are correctly spelt mark All correct as the answer. a) Futility b) Separasion c) Embarrassment d) Positively e) All Correct 25. Below the four words are given. One of these four words may be wrongly spelt. Find out the word which is wrongly spelt, if there is any. The number of that word is your answer. If all the words are correctly spelt mark All correct as the answer. a) Tournament b) Enhancement c) Amazingly d) Continuation e) All Correct Directions (Q. 26-30): Rearrange the following six sentences (A), (B), (C), (D), (E) and (F) in the proper sequence to form a meaningful paragraph and then answer the questions given below. A. Besides, they get a lot of exposure to novel things through media. B. Therefore, their mental development did not show any extraordinary signs. C. Children of the present generation appear to be smarter than their earlier counterparts. D. Thus, the environment of present days has brought out these changes. E. This is probably because there are lots of opportunities for their indirect learning. F. Children of yester years did not have these facilities. 26. Which of the following will be the Third sentence after rearrangement? a) A b) B c) C d) D 27. Which of the following will be the First sentence after rearrangement? a) A b) B c) C d) D 28. Which of the following will be the Sixth sentence after rearrangement? a) A b) B c) C d) D 29. Which of the following will be the Second sentence after rearrangement? a) A b) B c) C d) E 30. Which of the following will be the Fifth sentence after rearrangement? a) A b) B c) C d) D Aptitude - General SBI Reasoning ability solved question papers SBI stands for State Bank of India, SBI Central Recruitment and promotion of Department recruit clerical cadre in Associate Banks of State Bank of India SBI latest bank profile, SBI, free solved sample placement papers of clerk, SBI Associate Banks Clerks Recruitment 2012 notification and job details, SBI Dates of Written Examination : 07.10.2012 and 14.10.2012,SBI previous years solved question papers, SBI 2010,2011,2012 Clerks and po officer job written test examination syllabus and selection procedure, SBI PO and Associate banks clerks examination questions papers, SBIAssociate Banks Clerks Recruitment 2012 notification and detailed explanation with answers and solutions, How to crack Sbi written test examination tips and trick,SBI, General Awareness, General English, Quantitative Aptitude, Reasoning Ability, Marketing Aptitude / Computer Knowledge whole solved questions paper, Latest general awareness SBI Associate Bank clerical cadre question papers and selection procedure and test pattern SBI CLERK REASONING ABILITY MODEL PAPER Directions : - Select the related letters / word / number / figure from the given alternatives 1 Editor : Magazine (a) Movie (b) Scene (c) Drama (d) Director 2 Hinduism , Christianity , Islam : Religion (a) Ear , Nose , Eyesight : Vision (b) Plus , Minus , Multiple : division (c) Winter , Spring , Summer : Season (d) Humid , Hot , Tundra : Region 3 WOLF : FLOW :: WARD : ? (a) BROW (b) DRAW (c) CRAW (d) SLAW 4 GRAIN : TIZRM :: BRAIN : ? (a) XRIKL (b) YIZRM(c) OPRST (d) ASQMI 5 CUT : BDTVSU :: TIP : ? (a) UVHJOQ (b) SUHJOQ (c) USJHQO (d) SUJHOQ 6 19 : 59 :: 17 : ? (a) 51 (b) 53 (c) 55 (d) 57 7 14 : 20 :: 16 : ? (a) 23 (b) 48 (c) 10 (d) 42 8 100 : 102 :: 100000 : ? (a) 105 (b) 104 (c) 1003 (d) 1004 DIRECTIONS : (QUESTIONS 9 to 13) select the one which is different from the other three . 9 (a) Mile (b) Centimeter (c) Litre (d) Yard 10 (a) High - Up (b) Past - Present (c) Often - Seldom (d) Fresh - Stale 11 (a) 11 - 127 (b) 9 – 85 (c) 7 – 53 (d) 5 – 29 12 (a) 26 Z (b) 24 X (c) 22 V (d) 20 S 13 (a) 8 , 64 , 112 (b) 36 , 6 , 206 (c) 48 , 4 , 202 (d) 9 , 27 , 263 14 In the following series of numerals , which digit has maximum frequency ? 846734378344563464348 (a) 8 (b) 6 (c) 4 (d) 3 15 If the day after tomorrow is Friday , what day will the third day after tomorrow be (a) Saturday (b) Monday (c) Sunday (d) Friday 16 If the ratio of the area of two squares is 16 : 1 , then the ratio of their perimeter is (a) 4 : 1 (b) 16 : 1 (c) 1 : 3 (d) 3 : 4 17 The shade of 18 ft high pole is 20 ft. . Find the length of shade of 27 ft long pole . (a) 36 ft (b) 30 ft (c) 34 ft (d) 40 ft 18 A scores more runs than B but less than C . D scores more than B but less than A . Who is the lowest scorer ? a) A (b) B (c) C (d) D 19 In the alphabets from A to Z , which is the third letter to the right of the letter which is midway between K & S ? (a) R (b) Q (c) P (d) O 20 If first November falls on Monday , then what day will the 25th November be ? (a) Tuesday (b) Thursday (c) Wednesday (d) Friday 21 The length of room is twice its breadth . If the area of the room is 242 sq meters , then find out its breadth (a) 11 (b) 10 (c) 12 (d) 9 22 If the product of two numbers is 10 & their sum is 7 , then the larger of the two number is - (a) – 2 (b) 2 (c) 5 (d) 3 24 Which letter is used only in one of the given words ? Speak , Reap , Shark (a) S (b) P (c) K (d) H 25 A tortoise covers one kilometer in 4 hours . It takes rest for 20 minutes after very kilometer . How much time does it takes for the tortoise to cover 3.5 kilometers ? (a) 14 hours (b) 13 (c) 15 (d) 12 26 Of which of the following words , which one will be at the 3rd position in the dictionary ? 1. socks 2. Shocks 3. Sharp 4. snooker (a) 4 (b) 3 (c) 2 (d) 1 ANSWER 1 2 3 4 5 c c b b b 6 7 8 9 10 b d a c a 11 12 13 14 15 a d a c c 16 17 18 19 20 a b b a b 21 22 23 24 25 a c d c a SBI Aptitude Questions SBI Associate Banks Clerks Exam,SBT and SBI previous years solved question papers, SBT and SBI Aptitude/Reasoning, General awareness questions with answers,SBI and SBI free solved placement papers, SBT and SBI Associate clerks, Probationary officers,specialists officers recruitment pattern ,Test pattern, Written test solved question papers, All IT Non IT (PSU Companies) and Banks IBPS,PO,CWE,model question papers for practice Exam Held On : 16-01-2011Quantitative Aptitude Questions with answers and detailed explanations Directions (81-90) : What should come in place of the question mark (?) in the following questions ? 81.1 38% of 4500 - 25% of ? = 1640 (1) 260 (2) 270 (3) 280 (Ans) (4) 290 (5) None of these Explanations : 4500 * 38/100 - x * 25/100 = 1640 ? 1710 - 1640 = x/4 ? x/4 = 70 ? x = 280 82.2 (?)2 + 152 - 332 = 97 (1) 33 (2) 32 (3) 34 (4) 30 (5) None of these (Ans) Explanations : x2 + 152 - 332 = 97 ? x2 = 97 + 1089 - 725 ? x2 = 1186 - 225 = 961 ? x = 31 83. 3 (7921 + 178) - 5.5 = v? (1) 1512 (2) 1521 (Ans) (3) 1251 (4) 1531 (5) None of these Explanations : 7921 / 178 - 5.5 = vx 44.5 - 5.5 = = vx ? 39.0 = vx ? x = 1521 84. 4 9/10 + 3/11 + 7/15 = ? (1) 1 217/330 (2) 1 221/330 (3) 1 211/330 (Ans) (4) 1 197/330 (5) None of these Explanations : 9 + 3 + 7 = 297 + 90 + 154 = 541 10 11 15 330 330 = 1 211 330 85. 5 1/4th of 1/2 of 3/4th of 52000 = ? (1) 4875 (Ans) (2) 4857 (3) 4785 (4) 4877 (5) None of these Explanations : 52000 * 3/4 * 1/2 * 1/4 = 4875 86.6 57/67 * 32/171 * 45/128 = ? (1) 15/262 (2) 15/268 (Ans) (3) 15/266 (4) 17/268 (5) None of these Explanations : 57/67 * 32/171 * 45/128 = 15/268 87. 7 283 * 56 + 252 = 20 * ? (1) 805 (Ans) (2) 803 (3) 807 (4) 809 (5) None of these Explanations : 283 * 56 + 252 = 20*x ? 15848 + 252 = 20*x ? x = 16100/20 ? x = 805 88.8 (5863 - v2704) * 0.5 = ? (1) 2955.5 (2) 2905.5 (Ans) (3) 2590.5 (4) 2909.5 (5) None of these Explanations : (5863 - v2704) * 0.5 = 2905.5 89.9 3v42875 - ? = 21 (1) 18 (2) 13 (3) 15 (4) 11 (5) None of these (Ans) Explanations : v42875 - x = 21 35 - x = 21 x = 14 90. 10 25639 - 5252 - 3232 = ? (1) 17255 (2) 17551 (3) 17515 (4) 17155 (Ans) (5) None of these Explanations : 25639 - 5252 - 3232 - 25639 -8484 = 17155 Directions (91-95) : What should come in place of the question mark (?) in the following number series? 91. 11. 1 5 13 29 ? 125 253 (1) 83 (2) 69 (3) 61 (Ans) (4) 65 (5) 81 Explanations : SBI - QA - Qn Ans - 91 92. 12. 45 57 81 117 165 ? (1) 235 (2) 215 (3) 205 (4) 245 (5) 225 (Ans) Explanations : SBI - QA - Qn Ans - 92 93. 13.353 354 351 356 349 ? (1) 348 (2) 358 (Ans) (3) 338 (4) 385 (5) 340 Explanations : SBI - QA - Qn Ans - 93 94. 14. 1/4 1/2 3/4 1 1 1/4 1 1/2 1 3/4 ? (1) 2 (Ans) (2) 4 (3) 6 (4) 1 1/5 (5) 1 2/3 Explanations : SBI - QA - Qn Ans - 94 95. 15. 17 18 26 53 117 ? 458 (1) 342 (2) 142 (3) 257 (4) 262 (5) 242 (Ans) Explanations : SBI - QA - Qn Ans - 95 96. 16.If the numerator of a certain fraction increased by 200% and the denominator is increased by 150% the new fraction thus formed is 9/10. What is the original fraction? (1) 3/4 (Ans) (2) 1/4 (3) 3/5 (4) 2/5 (5) None of these Explanations : Let, the original fraction will be x/y So, x * 300% = 9 y * 250% 10 ? x = 3 y 4 97.17. The average of five numbers is 49. The average of the first and the second numbers is 48 and the average of the fourth and fifth numbers is 28. What is the third number ? (1) 92 (2) 91 (3) 95 (4) Cannot be determined (5) None of these (Ans) Explanations : Sum of five numbers = 5 * 49 = 245 Sum of first and second numbers = 2 * 48 = 96 Sum of fourth and fifth numbers = 2 * 28 = 56 So, the third number = 245 - 152 = 93 98. 18. 28% members of a certain group are married. What is the respective ratio between the number of married members to the number of (1) 7 : 17 (2) 5 : 18 (3) 7 : 18 (Ans) (4) Cannot be determined (5) None of these Explanations : Let, total members are 100 So, married members = 28 (and unmarried members = 100 - 28 = 72) So, respective ratio = 28 : 72 = 7 : 18 99. 19. Mr. Khanna took a loan of Rs. 10,000/- on simple interest for two years at the rate 3 p.c.p.a. The total amount that he will be paying as interest in 2 years is 3% of his monthly salary. What is his monthly salary? (1) Rs.30,000/- (2) Rs.16,000/- (3) Rs.20,000/- (Ans) (4) Rs.12,000/- (5) None of these Explanations : S. I = P. R. T. = 10000 * 2 * 3 = 600 Rs. 100 100 Let, his salary will be x So, x * 3/100 = 600 ? x = 20000 Rs. 100. 20.In how many different ways can the letters of the word 'SECOND' be arranged? (1) 720 (Ans) (2) 120 (3) 5040 (4) 270 (5) None of these Explanations : Total words from Second 6P6 = SBI - QA - Qn Ans - 100 = 720 Directions (101-105) : Study the following table carefully and answer the questions that follow : Number of employees from six different banks located in different cities, M = Males, F = Females City Agra Delhi Mumbai Chennai Patna Kolkata A 553 224 254 456 457 388 114 378 234 120 353 325 B 673 116 346 256 346 456 124 235 241 156 348 174 C 443 500 366 345 124 456 235 388 350 234 399 439 D 534 454 478 285 235 235 255 175 124 165 358 234 E 256 235 256 166 574 599 324 198 124 334 125 235 F 556 357 346 287 589 190 189 256 155 181 278 192 101.21. What is the ratio of number of males to the number of females respectively Bank D from all the cities together? (1) 496 : 387 (Ans) (2) 487 : 356 (3) 422 : 385 (4) 486 : 397 (5) None of these Explanations : Males in Bank D = 534 + 478 + 235 + 255 + 124 + 358 = 1984 and Females in Bank D = 454 + 285 + 235 + 175 + 175 + 165 + 234 = 1548 Respective ratio = 1984 : 1548 = 496 : 387 102.22. What is the ratio of the number of males to the number of females respectively in Patna from Bank A, Bank C and Bank E together? (1) 175 : 173 (2) 177 : 173 (3) 177 : 172 (Ans) (4) 175 : 172 (5) None of these Explanations : Males in Patna from Bank A, C and E = 234 + 350 + 124 = 708 and females in Patna from Bank A, C and E = 120 + 234 + 334 = 688 Respective ratio = 708 : 688 = 177 : 172 103. 23.What is the approximate average of the number of the number of males working in all the banks together in Kolkata? (1) 350 (2) 310 (Ans) (3) 340 (4) 380 (5) 360 Explanations : Average males in all banks in Kolkata = 353 + 348 + 399 + 358 + 125 + 278 6 = 1861 = 3101 = 310 approx 6 104. 24.The number of females in all the banks together in Delhi are approximately what percent of the number of males from all the banks together in the same city? (1) 88 (Ans) (2) 98 (3) 78 (4) 68 (5) 58 Explanations : Females of all banks in Delhi = 456 + 256 + 345 + 285 + 166 + 287 = 1795 and males of all banks in Delhi = 254 + 346 + 366 + 478 + 256 + 346 = 2046 approx % = 1795 * 100 = 87.73 2046 = 88% 105.25. The number of females in Bank B from Agra are what percent of the females in Bank C from the same city? (1) 33.2 (2) 23.2 (Ans) (3) 13.2 (4) 28.2 (5) None of these Explanations : Females in Bank B from Agra = 116 and Females in Bank C from Agra = 500 approx. percentage = 116 * 100 = 23.2% 500 106.26. What is the value of (x) in the following equation? images _SBI_ QA -Qno .106 (1) 17 (2) 19 (3) 16 (4) 14 (5) None of these (Ans) Explanations : (x)0.7 / 36 = 9 / (x)1.3 ? x0.7 = x1.3 = 36 * 9 ? x2 = 36 * 9 ? x = 6 * 3 ? x = 18 107. 27.The average speed of a train is 3 times the average speed of a car. The car covers a distance of 520 kms in 8 hours. How much distance will the train cover in 13 hours? (1) 2553 kms (2) 2585 kms (3) 2355 kms (4) 2535 kms (Ans) (5) None of these Explanations : Speed of the car = 520/8 = 65 km/h and speed of the train = 3 * 65 = 195 km/h Distance covered by train in 13 hours = 195 * 13 = 2535 kms 108. 28.The area of a triangle is half the area of a square. The perimeter of the square is 224 cms. What is the area of the triangle ? (1) 1856 cm2 (2) 1658 cm2 (3) 1558 cm2 (4) 1586 cm2 (5) None of these (Ans) Explanations : Perimeter of the square = 4 * side ? 4 * side = 224 ? side = 56cm Area of the square = (56)2 = 3136 cm2 So, Area of the triangle = 1/2 * 3136 = 1568 (cm)2 109.29.Yesterday Priti typed an essay of 5000 words at the speed of 60 words per minute. Today she typed the same essay faster and her speed was 15% more than yesterday. What is the approximate difference in the time she took to type yesterday and the time she took to type today? (1) 20 minutes (2) 30 minutes (3) 10 minutes (Ans) (4) 40 minutes (5) 1 hour Explanations : Yesterday time taken = 5000/60 = 83.3 minutes Today speed of typing in per minute = 60 * 115/100 and time taken = 500/69 = 723 minutes So, difference in time = 83.3 - 72.3 = 11 minutes 110. 30.How much will be the compound interest to be paid on a principle amount Rs. 85,000/- after 3 years at the rate of 6 p.c.p.a ? (1) Rs. 16623.36/- (2) Rs. 16236.36/- (Ans) (3) Rs. 16326.36/- (4) Rs. 16632.36/- (5) None of these Explanations : SBI - QA - Qn Ans - 110 = 85000 * 53 * 53 * 53 = 2530909 50 50 50 25 = 101236.36 Rs. So, C.I = 101236.36 - 85000 = 16236.36 Direction (111-115) : What approximate value should come in place of the question mark (?) in the following questions ? (Note : You are not expected to calculate the exact value.) 111. 31.635 * 455 ÷ 403 = ? (1) 735 (2) 795 (3) 695 (4) 685 (5) 715 (Ans) Explanations : 635 * 455 ÷ 403 = 635 * 455/403 = 288925/403 = 716.93 112. 32.3v5332 = ? (1) 8 (2) 38 (3) 58 (4) 68 (5) 18 (Ans) Explanations : 3v5332 = 3v2 * 2 * 133 = 17.64 113. 33.105.003 + 307.993 + 215.630 = ? (1) 610 (2) 650 (3) 660 (4) 670 (5) 630 (Ans) Explanations : 105.003 + 307.993 + 215.630 = 628.626 114. 34.5223 ÷ 36 * 0.93 = ? (1) 135 (Ans) (2) 125 (3) 145 (4) 155 (5) 115 Explanations : 5223/36 * 0.93 = 4857.39/36 = 134.92 115. 35. v2228 = ? (1) 57 (2) 47 (Ans) (3) 67 (4) 37 (5) 27 Explanations : v2228 = v2*2*557 = 2*23.6 = 47.2 Directions (116 -120) : Study the following graph carefully and answer the questions that follow : SBI - QA - image -Qn 116-120 I SBI - QA - image -Qn 116-120 II 116. 36.The number of females in the Marketing department are approximately what percent of the total employees in Marketing and Customer Relation Departments together? (1) 26 (2) 36 (3) 6 (4) 46 (5) 16 (Ans) Explanations : The number of females in the marketing department = 2000 * 14/100 = 280 Total employees in marketing and customers relation department 4500 * 18/100 + 4500 * 22/100 = 810 + 990 = 1800 Aggreagcate % = 280 * 100/1800 = 15.5 117.37. The total number of females are what percent of the total number of males in the organization ? (1) 90 (2) 70 (3) 80 (Ans) (4) 60 (5) None of these Explanations : Total females are 2000 and total males are 4500 - 2000 = 2500 Its % = 2000 * 100/2500 = 80% 118. 38.What is the ratio of number of males in HR department to the number of males in Accounts department respectively? (1) 3 : 17 (2) 4 : 15 (3) 2 : 15 (Ans) (4) 2 : 13 (5) None of these Explanations : Males in H.R. department = 4500 * 8/100 - 2000 * 16/100 = 360 - 320 = 40 and males in Accounts department = 4500 * 12/100 - 2000 * 12/100 = 540 - 240 = 300 Its ratio = 40 : 300 = 2 : 15 119.39. What is the total number of males from Design, Customer Relation and HR departments together? (1) 1550 (2) 1510 (Ans) (3) 1540 (4) 1580 (5) None of these Explanations : Total males in Design, Customer Relation and II.R. department SBI - QA - Qn Ans - 119 = (1440 - 560) + (990 - 400) + (360 - 320) = 880 + 590 + 40 = 1510 120.40. What is the respective ratio of number of employees in Administrative department to the number of males in the same department? (1) 9 : 4 (Ans) (2) 8 : 3 (3) 7 : 2 (4) 8 : 5 (5) None of these Explanations : Employees in Administrative department = 4500 * 8/100 = 360 and Total males in Administrative department = 4500 * 8/100 - 2000 * 10/100 = 160 Respective ratio = 360 : 160 = 9 : 4 SBI Written Test Questions SBI latest Bank Examination Questions with answers SBI Reasoning and Quantitative Aptitude Questions with answers SBI General English Questions with answersSBI stands for State Bank of India, SBI Central Recruitment and promotion of Department recruit clerical cadre in Associate Banks of State Bank of India SBI latest bank profile, SBI,free solved sample placement papers of clerk, SBI Associate Banks Clerks Recruitment 2012 notification and job details, SBI Dates of Written Examination : 07.10.2012 and 14.10.2012,SBI previous years solved question papers, SBI 2010,2011,2012 Clerks and po officer job written test examination syllabus and selection procedure, SBI PO and Associate banks clerks examination questions papers, SBIAssociate Banks Clerks Recruitment 2012 notification and detailed explanation with answers and solutions, How to crack Sbi written test examination tips and trick,SBI, General Awareness, General English, Quantitative Aptitude, Reasoning Ability, Marketing Aptitude / Computer Knowledge whole solved questions paper, Latest general awareness SBI Associate Bank clerical cadre question papers and selection procedure and test pattern 1. How many such pairs of letters are there in the word GUARDIAN each of which has as many letters between them in the word as in the English alphabet ? (A) None (B) One (C) Two (D) Three (E) More than three 2. Four of the following five are alike in a certain way and so form a group. Which is the onethat does not belong to that group ? (A) 19 (B) 17 (C) 23 (D) 29 (E) 27 3. How many meaningful English words can be made with the letters TEBI using each letter only once in each word ? (A) None (B) One (C) Two (D) Three (E) More than three 4. In a certain code LONG is written as 5123 and GEAR is written as 3748. How is LANE written in that code ? (A) 5427 (B) 5247 (C) 5847 (D) 5237 (E) None of these 5. ‘BD’ is related to ‘EG’ and ‘MO’ is related to ‘PR’ in the same way as ‘FH’ is related to ………. (A) JM (B) IL (C) JL (D) IK P a g e | 2 (E) None of these . How many such digits are there in the number 58674139 each of which is as far away from the beginning of the number as when the digits within the number are rearranged in descending order ? (A) None (B) One (C) Two (D) Three (E) More than three 7. In a certain code BREAKDOWN is written as BFSCJMVNC. How is ORGANISED written in that code ? (A) PSHBMCDRH (B) BHSPMCDRH (C) BHSPOCDRH (D) BHSPNHRDC (E) None of these 8. In a certain code language ‘pik da pa’ means ‘where are you’; ‘da na ja’ means ‘you may come’ and ‘na ka sa’ means ‘he may go’, which of the following means ‘come’ in that code language ? (A) da (B) ja (C) na (D) Cannot be determined (E) None of these 9. Four of the following five are alike in a certain way and so form a group. Which is the one that does not belong to that group ? (A) Copper (B) Iron (C) Aluminium (D) Zinc (E) Steel 10. What should come next in the following number series ? 9 8 9 8 7 9 8 7 6 9 8 7 6 5 9 8 7 6 5 4 9 8 7 6 5 (A) 3 (B) 4 (C) 2 (D) 1 (E) None of these 11. Which of the following is the middle digit of the second highest among the following five numbers ? 254 319 963 842 697 (A) 5 (B) 1 (C) 6 (D) 4 (E) 9 12. Meeta correctly remembers that her father’s birthday is after 8th July but before 12th July. Her brother correctly remembers that their father’s birthday is after 10th July but before 15th July. On which day of July was definitely their father’s birthday ? (A) 10th (B) 11th (C) 10th or 11th (D) Cannot be determined (E) None of these 13. In a class of 50 students M is eighth from top. H is 20th from bottom. How many students are there between M and H ? (A) 22 (B) 23 (C) 24 (D) Cannot be determined (E) None of these 14. Among A, B, C, D and F each scoring different marks in the annual examination, D scored less than only F among them. B scored more than A and C but less than D. Who among them scored least marks among them ? (A) A (B) C (C) B (D) Data inadequate (E) None of these 15. Four of the following five are alike in a certain way and so form a group. Which is the one that does not belong to that group ? (A) B D F (B) V X Z (C) F I K (D) M O Q (E) L N P Directions—(Q. 16-22) In each question below are three statements followed by two conclusions numbered I and II. You have to take the three given statements to be true even if they seem to be at variance from commonly known facts and then decide which of the given conclusions logically follows from the three statements disregarding commonly known facts. Give answers : (A) If only conclusion I follows. (B) If only conclusion II follows. (C) If either conclusion I or II follows. (D) If neither conclusion I nor II follows. (E) If both conclusions I and II follow. 16. Statements : Some desks are tents. Some tents are rivers. All rivers are ponds. Conclusions : I. Some ponds are tents. II. Some ponds are desks. 17. Statements : All chair are pens. Some pens are knives. All knives are rats. Conclusions : I. Some rats are chairs. II. Some rats are pens. 18. Statements : Some forests are huts. Some huts are walls. Some walls are nets. Conclusions : I. Some nets are forests. II. Some nets are huts. 19. Statements : All tables are windows. All windows are rooms. All rooms are buses. Conclusions : I. Some buses are tables. II. Some rooms are tables. 20. Statements : Some trees are boxes. All boxes are bricks. All bricks are dogs. Conclusions : I. Some dogs are trees. II. Some bricks are trees. 21. Statements : All goats are flowers. No flower is branch. Some branches are roots. Conclusions : I. Some roots are goats. II. No root is goat. 22. Statements : All pots are rings. All bangles are rings. All rings are paints. Conclusions : I. Some paints are pots. II. Some bangles are paints. Directions—(Q. 23-29) Study the following arrangement carefully and answer the questions given below— B # A R 5 8 E % M F 4 J 1 U @ H 2 © 9 T I 6 * W 3 P # K 7 $ Y 23. Which of the following is the twelfth to the left of the twentieth from the left end of the above arrangement ? (A) % (B) W (C) $ (D) J (E) None of these 24. How many such numbers are there in the above arrangement each of which is immediately preceded by a consonant and also immediately followed by a symbol ? (A) None (B) One (C) Two (D) Three (E) More than three 25. How many such symbols are there in the above arrangement each of which is immediately preceded by a letter and also immediately followed by a number ? (A) None (B) One (C) Two (D) Three (E) More than three 26. How many such consonants are there in the above arrangement each of which is immediately preceded by a consonant and also immediately followed by a number ? (A) None (B) One (C) Two (D) Three (E) More than three 27. If all the numbers in the above arrangement are dropped, which of the following will be the eleventh from the right end ? (A) U (B) T (C) F (D) H (E) None of these 28. How many such vowels are there in the above arrangement each of which is either immediately preceded by a symbol or immediately followed by a symbol or both ? (A) None (B) One (C) Two (D) Three (E) Four 29. Four of the following are alike in a certain way based on their positions in the above arrangement and so form a group. Which is the one that does not belong to that group ? (A) 1 @ 4 (B) © T H (C) W P 6 (D) # 7 3 (E) 9 2 I Directions (Q. 30–36)—In each question below is given a group of digits/symbols followed by four combinations of letters lettered (A), (B), (C) and (D). You have to find out which of the combinations correctly represents the group of digits/symbols based on the following letter coding system and mark the letter of that combination as the answer.If none of the letter combinations correctly represents the group of digits/ symbols, mark(E) i.e. ‘None of these’ as the answer. Digit/Symbol :4 % 3 9 $ 1 8 @ © 2 # 5 6 * 7 d Letter Code :P M I T R Q J F H A E U N B G L Conditions : (i) If the first element in the group is a symbol and the last element is a digit, the codes are to be interchanged. (ii) If the first element in the group is a digit and the last element is a symbol both are to be coded as the code for the digit. (iii) If both the first and the last elements are even digits both are to be coded as ‘X’. (iv) If both the first and the last elements are odd digits, both are to be coded as ‘Y’. 30. 4%@93* (A) PMFTIB (B) PMFTIP (C) BMFTIB (D) XMFTIX (E) None of these 31. $1896© (A) RQJTNH (B) HQJTNR (C) RQJTNR (D) YQJTNY (E) None of these 32. 2*#836 (A) YBEJIY (B) ABEJIN (C) NBEJIA (D) XBEJIX (E) None of these 33. 8732@9 (A) TGIAFJ (B) YGIAFY (C) JGIAFT (D) XGIAFX (E) None of these 34. 7#$%35 (A) GERMIU (B) UERMIG (C) GERMIG (D) XERMIX (E) None of these 35. 931%©d (A) TIQMHL (B) LIQMHT (C) LIQMHL (D) TIQMHT (E) None of these 36. 46*389 (A) PNBIJT P a g e | 8 (B) XNBIJX (C) TNBIJP (D) PNBIJP (E) None of these Directions—(Q. 37–43) In the following questions, the symbols @, ©, %, $ and d are used with the following meanings illustrated. ‘P % Q’ means ‘P is greater than Q’. ‘P d Q’ means ‘P is neither greater than nor smaller than Q’. ‘P @ Q’ means ‘P is smaller than Q’. ‘P © Q’ means ‘P is either smaller than or equal to Q’. ‘P $ Q’ means ‘P is either greater than or equal to Q’. In each of the following questions assuming the given statements to be true, find out which of the two conclusions I and II given below them is/are definitely true. Give answers : (A) If only conclusion I is true. (B) If only conclusion II is true. (C) If either conclusion I or conclusion II is true. (D) If neither conclusion I nor conclusion II is true. (E) If both conclusions I and II are true. 37. Statements : M @ J, J © R, R d K Conclusions : I. K d J II. K % J 38. Statements : N $ T, T d H, N @ W Conclusions : I. W % T II. H © N 39. Statements : F @ R, R © V, V $ T Conclusions : I. V % F II. F @ T 40. Statements : W © D, D $ B, B @ H Conclusions : I. H % D II. W @ B 41. Statements : F d T, T $ M, M © R Conclusions : I. R $ F II. M © F 42. Statements : H $ N, N % R, R @ J Conclusions : I. R @ H II. J % H P a g e | 9 43. Statements : V % B, B $ D, D © E Conclusions : I. E d B II. D @ V Directions—(Q. 44–49) Study the following information and answer the questions given below— M, N, P, R, T, W, F and H are sitting around a circle facing at the centre. P is third to the left of M and second to the right of T. N is second to the right of P. R is second to the right of W who is second to the right of M. F is not an immediate neighbour of P. 44. Who is to the immediate right of P ? (A) H (B) F (C) R (D) Data inadequate (E) None of these 45. Who is to the immediate right of H ? (A) R (B) F (C) M (D) Data inadequate (E) None of these 46. Who is to the immediate left of R? (A) P (B) H (C) W (D) T (E) Data inadequate 47. Who is third to the right of H ? (A) T (B) W (C) R (D) F (E) Data inadequate 48. Who is second to the right of F ? (A) M (B) R (C) T (D) Data inadequate (E) None of these 49. In which of the following is the first person sitting in between the second and the P a g e | 10 third person ? (A) NHM (B) PHN (C) TRP (D) TWF (E) None of these Directions—(Q. 50–55) In each of the following questions, two rows of numbers are given. The resultant number in each row is to be worked out separately based on the following rules and the questions below the rows of numbers are to be answered. The operations of numbers progress from the left to the right. Rules : (i) If an odd number is followed by another composite odd number, they are to be added. (ii) If an even number is followed by an odd number they are to be added. (iii) If an even number is followed by a number which is the perfect square, the even number is to be subtracted from the perfect square. (iv) If an odd number is followed by a prime odd number, the first number is to be divided by the second number. (v) If an odd number is followed by an even number the second one is to be subtracted from the first number. If ‘p’ is the resultant of the first row, what will be the resultant of the second row ? (A) 58 (B) 76 (C) 27 (D) 82 (E) None of these 51. 12 64 17 20 m 16 If ‘m’ is the resultant of the first row, what will be the resultant of the second row ? (A) 69 (B) 85 (C) 101 (D) 121 (E) None of these 52. 85 17 35 16 19 r If ‘r’ is the resultant of the first row, what will be the resultant of the second row ? (A) 175 (B) – 5 (C) 75 (D) 210 P a g e | 11 (E) None of these 53. 24 15 3 d 6 15 If ‘d’ is the resultant of the first row, what will be the resultant of the second row ? (A) 37 (B) 8 (C) 22 (D) 29 (E) None of these 54. 28 49 15 h 3 12 If ‘h’ is the resultant of the first row, what will be the resultant of the second row ? (A) 13 (B) 15 (C) 19 (D) 27 (E) None of these 55. 36 15 3 12 3 n If ‘n’ is the resultant of the first row, what will be the resultant of the second row ? (A) 15 / 17 (B) 32 (C) 12 / 17 (D) 36 (E) None of these Directions—(Q. 56–60) Below in each question are given two statements I and II. These statements may be either independent causes or may be effects of independent causes or a common cause. One of these statements may be the effect of the other statement. Read both the statements and decide which of the following answer choice correctly depicts the relationship between these two statements. Mark answers : (A) If statement I is the cause and statement II is its effect. (B) If statement II is the cause and statement I is effect. (C) If both the statements I and II are independent causes. (D) If both the statements I and II are effects of independent causes. (E) If both the statements I and II are effects of some common cause. 56. I. This year, the cut off percentage for admission to junior colleges have increased over the last year. II. This year performance of students in Xth final exam was considerably higher than the previous year. 57. I. The conditions of most of the national highways are very bad. II. Govt. has now sanctioned a huge amount of money to maintain the national highways. 58. I. Many students of the local school have failed in English Language paper in the annual examination. II. Many students of the local school have failed in Mathematics paper in the annual examination. 59. I. Rain and thunder showers bashed the city during the past three days. II. Many people stayed indoor during the past three days. 60. I. There has been a considerable increase in the sale of fat free food articles. II. Now people have become more conscious about their health condition and food habits. Directions—(Q. 61-75) In each of the questions given below which one of the five answer figures on the right should come after the problem figures on the left, if the sequence were continued ? Answers with Hints 1. (D) 2. (E) All the rest are prime numbers. 3. (B) B I T E 4. (A) 5. (D) 6. (C) 7. (B) 8. (B) 9. (E) All the rest are pure metals. 10. (B) 98 987 9876 98765 987654 98765 4 11. (D) 963, 8 4 2, 697, 319, 254 12. (B) 13. (A) 14. (D) 15. (C) 16. (A) 17. (B) 18. (D) 19. (E) 20. (E) 21. (C) 22. (A) 23. (A) 20th from the left end is T and 12th to the left of T is %. 24. (C) H 2 © and K 7 $ 25. (A) 26. (B) M F 4 27. (D) After dropping all the numbers. B # A R E % M F J U @ H © T I * W P # K $ Y 11th from the right end is H. 28. (D) # A, E % and U @ 29. (E) 30. (B) 31. (A) 32. (D) 33. (C) 34. (E) 35. (D) 36. (A) 37. (C) 38. (E) 39. (A) 40. (D) 41. (B) 42. (A) 43. (B) 44. (A) 45. (E) 46. (D) 47. (D) 48. (C) 49. (A) 50. (A) 51. (E) 52. (B) 53. (C) 54. (D) 55. (A) P a g e | 14 56. (D) 57. (A) 58. (E) 59. (A) 60. (B) Directions—(Q. 41–50) Read the following passage carefully and answer the questions given below it. Certain words are printed in bold to help you to locate them while answering some of the questions. The yearly festival was close at hand. The store room was packed with silk fabrics. gold ornaments, clay bowls full of sweet curd and platefuls of sweetmeats. The orders had been placed with shops well in advance. The mother was sending out gifts to everyone. The eldest son, a government servant, lived with his wife and children in far off lands. The second son had left home at an early age. As a merchant he travelled all over the world. The other sons had split up over petty squabbles, and they now lived in homes of their own. The relatives were spread all across the world. They rarely visited. The youngest son, left in the company of a servant, was soon bored and stood at the door all day long, waiting and watching. His mother, thrilled and excited, loaded the presents on trays and plates, covered them with colourful kerchiefs, and sent them off with maids and servants. The neighbours looked on. The day came to an end. All the presents had been sent off. The child came back into the house and dejectedly said to his mother, “Maa, you gave a present to everyone, but you didn’t give me anything !” His mother laughed, “I have given all the gifts away to everyone, now see what’s left for you.” She kissed him on the forehead. The child said in a tearful voice, “Don’t I get a gift ?” “You’ll get it when you go far away.” “But when I am close to you, don’t I get something from your own hands ?” His mother reached out her arms and drew him to her. “This is all I have in my own hands. It is the most precious of all.” 41. Why did the woman's second son travel ? (A) He was restless by nature (B) He did not want to stay at home (C) He was rich and could afford to travel (D) His job was such that he had to travel (E) None of these 42. Why did the woman’s eldest son not attend the festival ? (A) He was not on good terms with his youngest brother who lived at home (B) He had quarrelled with his mother (C) His wife did not allow him to return home (D) His job prevented him from taking leave (E) None of these 43. How did the woman prepare for the festival ? 1. She bought expensive gifts for her children and neighbours. 2. She ordered her servants to prepare sweets and food well in advance. 3. She made sure that her youngest child was looked after so that he wouldn’t be bored. (A) None (B) Only 1 (C) Only 2 (D) Both 1 and 2 (E) All 1, 2 and 3 44. What did the youngest child do while his mother was busy ? 1. He waited for a chance to steal some sweetmeats. 2. He pestered his mother to give him a present. 3. He stood at the door with servants. (A) Only 1 (B) Only 2 (C) Both 1 and 3 (D) Only 3 (E) None of these 45. Which of the following can be said about the woman ? (A) She was a widow who had brought up her children single handedly (B) She was not a good mother since her children had left home at an early age (C) She enjoyed sending her family gifts at festival time (D) She gave expensive presents to show that she was wealthy (E) She rarely visited her grand-children because they all lived abroad 46. What did the boy receive from his mother ? (A) She taught him the value of patience (B) She encouraged him to grow up and live independently like his brothers (C) She showed him the importance of giving expensive gifts (D) She gave him a hug to express her love (E) None of these 47. Which of the following is TRUE in the context of the passage ? (A) The woman usually ignored her youngest son (B) The woman’s eldest son lived abroad (C) The members of the woman’s family did not care about her (D) The woman made all the preparations herself since she did not want to burden the servants (E) The woman sent gifts to her children to ensure that they visited her Directions—(Q. 48–49) Choose the word which is most nearly the SAME in meaning as the word printed in bold as used in the passage. 48. Left (A) Gone (B) Quit (C) Remaining (D) Disappeared (E) Forgot 49. Packed (A) Filled (B) Squeezed (C) Crowd (D) Collected (E) Untidy 50. Choose the word which is most OPPOSITE in meaning of the word dejectedly as used in the passage. (A) Calmly (B) Happily (C) Willingly (D) Fortunately (E) Softly Directions—(Q. 51–55) Read each sentence to find out whether there is any error in it. The error, if any, will be in one part of the sentence. The letter of that part is the answer. If there is no error, the answer is (E). (Ignore errors of punctuation, if any) 51. Many multinational companies (A) / have not been as (B) /successful in India (C) /than we expected. (D) No error (E) 52. He has ruined (A) /his eyesight (B) /by not using (C) /his spectacles regularly. (D) No error (E) 53. Mostly of the (A) /newly recruited officers (B) /have no experience (C) /in the banking sector. (D) No error (E) 54. The resignation of (A) /one of our directors (B) /have caused the price (C) / of shares to fall. (D) No error (E) 55. There are many (A) /ways of which (B) /inflation can (C) /be measured. (D) No error (E) Directions—(Q. 56–60) Which of the phrases (A), (B), (C) and (D) given below should replace the phrase given in bold in the following sentence to make the sentence grammatically meaningful and correct. If the sentence is correct as it is and ‘No correction is required.’ mark (E) as the answer. 56. Each of the loan must be approved by the Branch Manager— (A) Every loan (B) Each one of the loan (C) Any of the loan (D) All of the loan (E) No correction required 57. The issue was taken before the Municipal Corporation meeting last week— (A) Taking place at (B) Taken after (C) Being taken in (D) Taken up at (E) No correction required 58. He has asked for the names of those employees involved in the project. (A) had asked (B) having asked about (C) was asked that (D) is asking (E) no correction required 59. Considerate the traffic, it is better to leave for the airport an hour early— (A) While considering (B) Consideration of (C) Considering (D) Being considerate to (E) No correction required 60. He is a good leader, knowing that to motivate his employees to achieve— (A) That known when (B) Who knows how (C) Which knows how (D) Knowing what (E) No correction required Directions—(Q. 61–65) Rearrange the following six sentences (1), (2), (3), (4), (5) and (6) in the proper sequence to form a meaningful paragraph; then answer the questions given below them. (1) The able bodied men of the tribe gathered to discuss how to climb the mountain. (2) As part of their plundering they kidnapped a baby of one of the families. (3) One day the mountain tribe invaded those living in the valley. (4) “We couldn’t climb the mountain. How could you?”, they asked, “It wasn’t your baby !” she replied. (5) There were two tribes in the Andes–one lived in the valley and the other high up in the mountains. (6) Two days later they noticed the child’s mother coming down the mountain that they hadn’t yet figured out how to climb. 61. Which of the following should be the SECOND sentence after rearrangement ? (A) (1) (B) (2) (C) (3) (D) (4) (E) (5) 62. Which of the following should be the FIFTH sentence after rearrangement ? (A) (6) (B) (5) (C) (4) (D) (3) (E) (2) 63. Which of the following should be the FIRST sentence after rearrangement ? (A) (1) (B) (2) (C) (3) (D) (4) (E) (5) 64. Which of the following should be the SIXTH (LAST) sentence after rearrangement ? (A) (1) (B) (2) (C) (3) (D) (4) (E) (5) 65. Which of the following should be the THIRD sentence after rearrangement ? (A) (1) (B) (2) (C) (3) (D) (4) (E) (5) P a g e | 20 Directions—(Q. 66–70) In each question below a sentence with four words printed in bold type is given. These are lettered (A), (B), (C) and (D). One of these four words printed in bold may be either wrongly spelt or inappropriate in the context of the sentence. Find out the word, which is wrongly spelt or inappropriate, if any. The letter of that word is your answer. If all the words printed in bold are correctly spelt and also appropriate in the context of the sentence mark (E) i.e., all correct as your answer. 66. The income (A) of many people in rural (B) India is not adequate (C) to satisfy (D)their basic needs. All correct (E) 67. He is always (A) prompt (B) in caring (C) out instructions. (D) All correct (E) 68. The revized (A) rates (B) of interest will be effective (C) immediately. (D) All correct (E) 69. Such transactions (A) are quiet (B) expensive (C) and time consuming (D) for customers. All correct (E) 70. The guidelines (A) of the new scheme (B) are expected (C) to be finally (D) soon. All correct (E) Directions—(Q. 71–80) In the following passage there are blanks each of which has been numbered. These numbers are printed below the passage and against each five words/ phrases are suggested one of which fits the blank appropriately. Find out the appropriate word in each case. I used to look …(71)… to the holidays. I was usually …(72)… to my uncle’s house where I …(73)… his children. I did not get paid a salary for …(74)… What I received in return however, was far more …(75)… My uncle was an avid reader. During the time I spent with his family I had an …(76)… to read the vast amount of books and magazines that he possessed. This improved my English to some …(77)… Reading became my new …(78–79)… spending my pocket money on a ticket to the cinema I began to …(80)… books. This has benefited me greatly. 71. (A) forward (B) towards (C) backward (D) up (E) around 72. (A) went (B) sent (C) visited (D) travelled (E) gone 73. (A) cared (B) occupy (C) guarded (D) taught (E) played 74. (A) them (B) whom (C) this (D) now (E) which 75. (A) expensive (B) deserving (C) helping (D) demanding (E) valuable 76. (A) opportunity (B) ability (C) use (D) encouragement (E) achievement 77. (A) distance (B) extent (C) time (D) limits (E) degrees 78. (A) activity (B) hope (C) hobby (D) duty (E) worship 79. (A) despite (B) though (C) by (D) instead of (E) while 80. (A) sell (B) read (C) exchange (D) invest (E) buy Answers: 41. (D) 42. (E) 43. (A) 44. (D) 45. (C) 46. (D) 47. (B) 48. (C) 49. (A) 50. (C) 51. (D) Replace ‘than’ with ‘as’. 52. (E) 53. (A) Change ‘Mostly’ to ‘Most’. 54. (C) Change ‘have’ to ‘has’. 55. (B) Replace ‘of’ with ‘in’. 56. (A) 57. (D) 58. (E) 59. (C) 60. (B) 61. (C) 62. (A) 63. (E) 64. (D) 65. (B) 66. (E) 67. (C) 68. (A) 69. (E) 70. (D) 71. (A) 72. (B) 73. (D) 74. (C) 75. (E) 76. (A) 77. (B) 78. (C) 79. (D) 80. (E Whole-Testpaper SBI-Qunatitiave Aptitude SBI stands for State Bank of India, SBI latest bank profile, SBI,free solved sample placement papers of clerk, SBI Associate Banks Clerks Recruitment 2012 notification and job details, SBI previous years solved question papers, SBI 2010,2011,2012 Clerks and po officer job written test examination syllabus and selection procedure, SBI PO and Associate banks clerks examination questions papers, SBIAssociate Banks Clerks Recruitment 2012 notification and detailed explanation with answers and solutions, How to crack Sbi written test examination tips and trick, SBI, General Awareness, General English, Quantitative Aptitude, Reasoning Ability, Marketing Aptitude / Computer Knowledge whole solved questions paper, Latest general awareness Test Paper - III Quantitative Aptitude Directions Q.1-4 : In the following questions what will come in place of question mark (?) ? 1. 25 + 125 *2 =? A. 375 B. 30 C. 6250 D. 152 E. None of these 2. 150% of 15 + 75% of 75 = ? A. 78.75 B. 75.75 C. 281.25 D. 135 E. None of these 3. (((2)4 )1/2 )? = 256 A. 8 B. 6 C. 4 D. 1 E. None of these 4. -20*40*1/5/ 30/25 = ? A. -20/3 B. 400/3 C. -400/9 D. -192 E. None of these Directions Q. 5-9 : In the following series what will come in place of the question mark (?) ? 5. 12 22 69 272 1365 ? A. 8196 B. 8184 C. 8195 D. 6830 E. None of these 6. 1 ? 27 64 125 A. 14 B. 4 C. 9 D. 8 E. None of these 7. 104 153 189 214 ? A. 239 B. 225 C. 264 D. 235 E. None of these 8. 15 17 32 49 81 130 ? A. 179 B. 211 C. 194 D. 226 E.None of these 9. 15 17 21 29 45 77 ? A. 109 B. 125 C. 141 D. 173 E. None of these Directions Q 10-14 : From the following different committees are to be made as per the requirements given in each question. In how many different ways can it be done ? 10 men and 8 women out of which 5 men are teachers, 3 men are doctors and 2 are teachers, 2 doctors, 2 researchers and 1 social worker 10. A committee of 5 in which 3 men and 2 women are there A. 3360 B. 8568 C. 4284 D. 1680 E. None of these 11. A committee of 4 in which at least 2 women are there. A. 1260 B. 1820 C. 3060 D. 1890 E. None of these 12. A committee of 5 in which 2 men teachers, 2 women teachers and 1 doctor are there A. 75 B. 150 C. 214 D. 20 E. None of these 13. A committee of 7 A. 31824 B. 1200 C. 9600 D. 15912 E. None of these 14.A committee of 3 in which there is no teacher and no doctor A. 100 B. 120 C. 10 D. 12 E. None of these Directions Q. 15-19 : Study the following table to answer the given questions Number of Books of different prices bought in different months Month Price January March May July September November More than Rs. 5000 50 106 2 30 25 75 Rs. 4000-.5000 105 1000 40 105 400 375 Rs. 3000-3999 70 100 80 115 200 240 Rs. 2000- 2999 300 500 100 216 135 300 Rs. 1000-1999 140 370 200 225 175 470 Rs. 500-999 200 700 15 400 75 530 Less than Rs. 500 65 135 111 188 25 65 15. In which month was maximum number of books bought? A. March B. November C. July D. September E. None of these 16. For which price range was maximum number of books bought for the given months together? A. Rs. 500-999 B. Rs. 2000-2999 C. 1000-1999 D. 3000-3999 E. None of these 17. The books of price range Rs. 1000-1999 bought in January is what per cent of that of May? A. 30 B. 70 C. 142.86 D. 60 E. None of these 18. How much is the difference in the number of books bought in September and November ? A. 244 B. 776 C. 1020 D. 1310 E. None of these 19. What is the respective ration of books of Rs. 4000-5000 bought in January to that of March? A. 1 : 10 B. 15 : 100 C. 930 : 2911 D. 21 : 200 E. None of these SBI Reasoning and aptitude Questions for practice,Sbi model questions with answers,SBI free solved sample palcement papers Directions (Q.1-5) These questions are based on the following letter/number/symbol arrangement. Study it carefully to answer the questions. A 8 B 6 # 7 H U % 3 $ F V R 2 I @ 4 1 W E 9 © L 5 1. Which of the following is seventh to the right of fifteenth from the right end ? (1) 4 - Ans (2) (3) 1 (4) w (5) None of these Expl : Seventh to the right of fifteenth from the right end is 4. 2. If all the symbols are dropped from the above arrangement which of the following will be eighth from the left end ? (1) F (2) $ (3) U (4) 3 - Ans (5) None of these Expl : If all the symbols are dropped from the arrangement A 8 B 6 7 H U 3 F V R 2 I 4 1 W E9 L 5 I Eighth from the left So, eighth from the left is 3. 3. How many such symbols are three in the above arrangement, each of which is immediately followed by a consonant and also immediately preceded by a number? (1) None (2) One (3) Two - Ans (4) Three (5) More than three Expl : 3 Number Symbol Consonant 3 $F and 9 © L 4. How many such numbers are there in the above arrangement each of which is immediately followed by a vowel but not immediately preceded by a number ? (1) None (2) One - Ans (3) Two (4) Three (5) More than three Number Symbol Vowel R2I 5. Four of the following five are alike in a certain way on the basis of their positions in the above arrangement and so form a group. Which is the one that does not belong to the group ? (1) B7# (2) 14 (3) F2 R (4) 95 L (5) W©E - Ans Expl : B ---+3---> 7 ---1----> # I ---+3---> 4 ---1----> F ---+3---> 2 ---1----> R 9 ---+3---> 5 ---1----> L W ---+3---> © ---2---->E So W © E does not belong to the group. 6. Four of the following five are alike in a certain way and so form a group. Which is the one that does not belong to the group ? (1) Iron (2) Copper (3) Zinc (4) Aluminium (5) Brass - Ans Expl : Except Brass all are metal 7. Four of the following five are alike in a certain way and so form a group. Which is the one that does not belong to the group ? (1) 31 (2) 39 - Ans (3) 47 (4) 41 (5) 43 Expl : Except 39 all are prime number 8. In a certain code ROSE is written as #43 $ and FIRST is written as 5 *#37. How is store written in that code ? (1) 473$# (2) 473#$ (3) 374 #$ - Ans (4) 347#$ (5) None of these Expl : R O S E and F I R S T # 4 3 $ 5 * # 3 7 Similarly, S T O R E 3 7 4 # $ 9. How many meaningful English words can be made from the letters ADER, using each letter only once in each word ? (1) None (2) One (3) Two (4) Three (5) More than three - Ans Expl : Meaningful words READ, DEAR, EARD and DARE 10. How many such pairs of letters are there in the word DISTINGUISH, each of which have as many letters between them in the word as they have between them in the English alphabet ? (1) None (2) One (3) Two (4) Three (5) More than three - Ans Expl : D I S T I N G U I S H 4 9 19 20 9 14 7 21 9 19 8 Four such pair of letters are formed. Direction (Q.11-15) In each question below are four statements followed by four conclusions numbered I, II , III and IV. You have to take the four given statements to be true even if they seem to be at variance from commonly known facts and then decide which of the given conclusions logically follows from the four given statements disregarding commonly known facts. Then, decide which of the answers (1), (2), (3) , (4) and (5) is the correct answer and indicate it on the answer sheet. 11. Statements : All stones are pearls. Some pearls are shells. Some shells are boxes . No box is container Conclusions : I. Some stones are shells II. No pearl is container. III. No shell is container IV. Some pearls are containers. (1) Only II follows (2) Only II and III follow (3) Only either II or IV follows - Ans (4) Only III follows (5) None of follows 12. Statements : Some rings are chains. All chains are bangles. All bracelets are bangles. Some bangles are pendants. Conclusion : I. Some rings are bangles. II. Some chains are pendants III. Some bracelets are rings IV. No pendant is ring. (1) None of follows (2) Only I follows - Ans (3) Only II and III follow (4) Only IV follows (5) None of the above 13. Statements : Some schools are colleges. Some colleges are hostels. No hostel is office. All offices are institutes. Conclusions : I. No hostel is institute II. Some hostels are schools III. Some hostels are institutes IV. Some offices are college (1) Only I follows (2) Only II and III follow (3) Only IV follows (4) Only either I or III follows - Ans (5) None of the above 14. Statements : Some pins are needles. Some threads are needles. All needles are nails. All nails are hammers. Conclusions : I. Some pins are hammers II. Some threads are nails III. Some pins are threads. IV. No pin is thread (1) Only I, II and either III or IV follows - Ans (2) Only III and IV follow (3) Only I and II follow (4) All follow (5) None of the above Conclusions I. ? II. ? III. ? IV. ? So either III or IV and I, II follows. 15. Statements : Some chairs are rooms. No room is sofa. All sofas are tables. Some tables are desks. Conclusions : I. Some sofas are desks II. No room is table III. Some chairs are tables IV. No desk is room. (1) None follows - Ans (2) Only I follows (3) Only either II or III follows (4) Only III and IV follow (5) All follow Conclusions : I. x II. x III. x IV. x None follows Directions (Q.16-20) Study the following information carefully to answer these questions. A, B, C, D, E, F, G and H are sitting around a circle facing the centre. F is third to the right of C and second to the left of H. D is not an immediate neighbour of C or H. E is to the immediate right of A, who is second to the right of G. 16. Who is second to the left of C ? (1) A - Ans (2) B (3) E (4) D (5) None of these 17. Who is to the immediate right of C ? (1) A (2) B - Ans (3) D (4) B or D (5) None of these 18. Which of the following pairs of persons has first person sitting to the right of the second person ? (1) CB (2) AE (3) FG (4) HA (5) DB - Ans 19. Who sits between G and D ? (1) H (2) D (3) F - Ans (4) E (5) None of these 20. Which of the following is the correct position of B with respect to H ? (1) Only I (2) Only II (3) Only III (4) Both II and III - Ans (5) None of these Expl : Directions (Q.21-25) Each of the questions below consists of a question and two statements numbered I and II are given below it. You have to decide whether the data provided in the statements are sufficient to answer the question. Read both the statements and -------- Give answer (1) if the data in Statement I alone are sufficient to answer the question, while the data in Statement II along are not sufficient to answer the question. Give answer (2) if the data in Statement II alone are sufficient to answer the question, while the data in Statement I alone are not sufficient to answer the question. Give answer (3) if the the data in statement I alone or in Statement II alone are sufficient to answer the question. Give answer (4) if the data in both the Statements I and II are not sufficient to answer the question. Give answer (5) if the data in both the Statements I and II together are necessary to answer the question. 21. who reached the station first among L, M, J, T and R, if no two persons reached together ? I. M reached only after J and T. II. L reached before R. Ans : 4 Expl : From statement I (J and T) > M > (L and R) From statement II, L>R From statements I and II, (J and T) >M>L>R 22. Tower 'P' is in which direction with respect to tower 'Q' I. P is to the West of H, which is to the South of Q. II. F is to the West of Q and to the North of P. Ans : 3 Expl : From statement I, 23. What is the Suneeta's rank from top in the class ? I. In the call of 42 children, Suneeta is 29th from the bottom II. Suneeta is ten ranks below Samir. Ans : 1 Expl : From statement I, The tank of Sunita from top = 42 - 28 = 14th 24. What is the code for 'walks' in the code language ? I. In the code language 'she walks fast' is written as 'he ka to'. II. In the code language 'she learns fast' is written as 'jo ka he'. Ans : 5 Expl : From statement I, She walks fast ---> he ka to From statement II, she learns fast ---> jo ka he :. From statements I and II, walks ----> to 25. How is K related to N? I. N. is brother of M 2who is daughter of K II. F is husband of K Ans : 5 From statement I, Brother Daughter N--------------<-------M----------<--------K :. K may be the mother or father of M. From statement II, Husband F-------------<------K From statement I and II , K is the mother of N. 26.Directions (Q.26-30) In these questions a group of digits is given followed by four combinations of letters and symbols numbered (1), )(2), (3) and (4). Digits are to be coded as per the scheme and conditions given below. You have to find out which of the four combinations correctly represents the group of digits. Serial number of that combination is your answer. If none of the combinations is correct, your answer is (5) ie, 'None of these'. Digit : 5 1 2 8 6 3 9 0 4 7 Letter/Symbol code : H $ T A U % # F R @ Conditions : (i) If first digit is odd and the last digit is even their codes are to be interchanged (ii) If the first as well as the last digit is even both are to be coded as * (iii) if the first digit is even and the last digit is odd both are to be coded as the code for last digit. 26. 471536 (1) *@$H%* - Ans (2) R@$H%U (3) U@$H%U (4) R@$H%R (5) None of these Expl : 4 7 1 5 3 6 * @ $ H % * conditions II follows 27. 697845 (1) U#@ARU (2) U#@ARH (3) H#@ARH - Ans (4) H#@ARU (5) None of these Expl : 6 9 7 8 4 5 H # @ A R H Conditions III follows 28. 590247 (1) @#FTRH (2) H#FTR@ - Ans (3) H#FTRH (4) @#FTR@ (5) None of these Expl : 5 9 0 2 4 7 H # F T R @ 29. 348096 (1) %RAF#U (2) %RAF#% (3) URAF#U (4) *RAF#* (5) None of these - Ans Expl : 3 4 8 0 9 6 U R A F # % 30. 374862 (1) %@RAUT (2) %@RAU% (3) *@RAU* (4) T @ RAU% - Ans (5) None of these Expl : 3 7 4 8 6 2 T @ R A U % Condition (i) follows Directions (Q.31-35) In these questions symbols @, #, $, * and % are used with different meaning as follows . 'A @ B' means 'A is not smaller than B' 'A # B' means 'A is neither smaller than nor equal to B'. 'A$B' means 'A is neither greater than nor smaller than B'. 'A*B' means 'A is not greater than B'. 'A%B' means 'A is neither greater than nor equal to B'. In each of the following questions assuming the given statements to be true, find out which of the two conclusions I and II given below them is/are defenitely true. Give answer (1) if only conclusion I is true Give answer (2) if only conclusion II is true. Give answer (3) if either conclusion I or nor conclusion Give answer (4) if neither conclusion I or nor conclusion II is true Give answer (5) if both conclusions I and II are true. %==> < # ==> > *==> < @==> > $ ==> = 31. Statement N @ W, W # H, H % T Conclusion I. H%N II. T#W Ans : 1 Expl : (1) N @ W ==> N>W W # H ==> W >H H%T ==> H< T :.N> W>H<T 32. Statements : F # R, H%R, L*H Conclusions I. F#L II. R @ L Ans : 1 Expl : F # R ==> F>R H%R ==> H<R L*H ==> L<H :. F>R>H>L 33. Statements : J @ K, K%M, M#T Conclusions : I. K% T II. K@ T Ans : 3 Expl : J @ K ==> J> K K%M ==> K<M M#T ==> M>T :. J>K<M>T Conclusion : I. K% T ==> K<T (False) II. K@ T ==> K>T (False) Either I or II is true. 34. Statements V * W, W$H, H @ I Conclusions I. V*I II. I*W Ans : 2 Expl : V * W ==> V<W W$H ==> W=H H @ I ==> H>I :. V< W=H> I Conclusions I. V* I ==> V<I (False) II. I*W ==> I < W (True) 35. Statements L * P, P%V, V#D Conclusions I. L*V I I. L$D Expl : Directions (Q.36-40) These questions are based on the following information. Study it carefully and answer the questions. Seven members L, H, K, T, F, J and R represent different countries in Olympics viz. USA, China, Korea, France, Russia, Australia and Japan; each one competes for a different sport viz., volleyball, Archery, Rifle Shooting, Tennis, Boxing, Athletics and Football. The order of persons, countries and the games is not necessarily the same. K presents China for Archery. T represents USA but not for Velleyball or Rifle Shooting. The one who represents Japan competes for Boxing. F competes Volleyball but not for Korea. L represents Australia for Athletics. The one who represents Russia competes for Tennis. J does not represent Korea or Japan. R competes for Rifles Shooting. 36. D The one who competes for Rifle shooting, represents which country ? (1) France (2) Korea - Ans (3) Japan (4) USA (5) None of these 37. Which of the following combinations is correct ? (1) J-Tennis-France (2) R-Tennis-Russia (3) R-Tennis-France (4) J-Tennis-Russia - Ans (5) None of these 38. Who represents Japan ? (1) F (2) R (3) J (4) H - Ans (5) None of these 39. For which game does T compete ? (1) Boxing (2) Foot ball - Ans (3) Tennis (4) Cannot be determined (5) None of these 40. F represents which country ? (1) France - Ans (2) Russia (3) Japan (4) Korea (5) None of these Expl: Member Country Game L Australia Athletics H Japan Boxing K China Archery T USA Football F France Volleyball J Russia Tennis R Korea Rifle shooting Placement Paper SBI specialists officers Reasoning questions for practice's model question for practice's aptitude Reasoning marketing professional knowledge questions for practice 1. Each odd digit in the number 5263187 is substituted by the next higher digit and each even digit is substituted by the next higher digit and each even digit is substituted by the previous lower digit and the digits so obtained are rearranged in ascending order, which of the following will be the third digit from the left end after the rearrangement ? (1) 2 (2) 4 (3) 5 (4) 6 (5) None of these Ans : (2) Expl: Given number ---> 5 2 6 3 1 8 7 After rearrangement ---> 6 1 5 4 2 7 8 Arranged in ascending order 1, 2, 4, 5, 6, 7, 8 so, the third digit from left end is 4 2. Town D is towards East of town F. Town B is towards North of town D. Town H is towards South of town B. Towards which directions is town H from town F ? (1) East (2) South - East (3) North - East (4) Data inadequate (5) None of these Ans : (4) Expl : According to question, H is South of B. Therefore the position of H may be between B and D or it may be South of D. So, the position of H cannot determined 3. Among A, B, C, D and E each having different weight, D is heavier than only A and C is lighter than B and E. Who among them is the heaviest ? (1) B (2) E (3) C (4) Data inadequate (5) None of these Ans : (4) Expl : D> A (B, E) >C D is heavier than only A So, (B,E) > C> D> A So, heaviest is either B or E 4. How many such pairs of letters are there in the word SEARCHES each of which has as many letters between them in the word as in the English alphabet ? (1) None (2) One (3) Two (4) Three (5) More than three Ans : (4) Expl : 5. If '÷' means '+', '-' means 'x', 'x' means '÷ ' and '+' means '-' then 15-8 x 6 ÷12+4 = ? (1) 20 (2) 28 (3) 8 and 4/7 (4) 2 and 2/3 (5) None of these Ans : (2) Expl : 15 - 8 x 6 ÷ 6 + 12-4 = ? After changing the sign 15 x 8 ÷ 6 + 12 -4 = ? 6. Ashok started walking towards South, After walking 50 m he took a right turn and walked 30 m. He then took a right turn and walked 100 m. He again took a right turn and walked 30 m and stopped. How far and in which direction was he from the starting point ? (1) 50 m South (2) 150 m North (3) 180 m East (4) 50 m North (5) None of these Ans : (4) 7. How many meaningful English words can be made with the letters DLEI using each letter only once in each word ? (1) None (2) One (3) Two (4) Three (5) More than three Ans : (4) Expl : From starting point he is 50 m far away and direction is North. 8. In a certain code TEMPORAL is written as OLDSMBSP. How is CONSIDER written in that code ? (1) RMNBSFEJ (2) BNMRSFEJ (3) RMNBJEFS (4) TOPDQDCH (5) None of these Ans : (1) Expl : As, 9. In a certain code language 'how many goals scored' is written as '5 3 9 7'; 'many more matches' is written as '9 8 2' and He scored five' is written as '1 6 3'. How is 'goals' written in that code language ? (1) 5 (2) 7 (3) 5 or 7 (4) Data inadequate (5) None of these Ans : (3) Expl : How many goals scored 5 3 9 7 (i) Many more matches 9 8 2 (ii) He scored five 1 6 3 (iii) From (i) and (ii), many (9) From (i) and (iii), scored 3 So, goals's code 5 or 7. 10. Pratap correctly remembers that his mother's birthday is before twenty third of April but after Nineteenth of April, whereas his sister correctly remembers that their mother's birthday is not on or after twenty second of April. On which day in April is definitely their mother's birthday ? (1) Twentieth (2) Twenty - first (3) Twentieth or twenty -first (4) Cannot be determined (5) None of the above Ans : (3) Expl : According to Pratap = 20, 21, 22 According to his sister = not after 21st April So, the birthday will be 20 or 21 April from both statements. Directions (Q.11-15) In each of the questions below are given four statements followed by your conclusions numbered I, II, III and IV. You have to take the given statements to be true even if they seem to be at variance from commonly known facts. Read all the conclusions and then decide which of the given conclusions logically follows from the given statements disregarding commonly known facts. 11. Statements All cups are bottles. Some bottles are jugs. No jug is plate. Some plates are tables. Conclusions I. Some tables are bottles II. Some plates are cups. III. No table is bottle IV. Some jugs are cups (1) Only I follows (2) Only II follows (3) Only III follows (4) Only IV follows (5) Only either I or III follows or Ans : (5) Expl : Conclusions I. X ] II. X ] or III. v ] IV. X Either I or III is true 12. Statements All birds are horses. All horses are tigers. Some tigers are lions. Some lions are monkeys. Conclusions I. Some tigers are horses II. Some monkeys are birds III. Some tigers are birds IV. Some monkeys are horses (1) Only I and III follow (2) Only I, II and III follow (3) Only II, III and IV follow (4) All I, II, III and IV follow (5) None of the above Ans : (1) Expl : Conclusions I. v II. X III. v IV. X So, I and III follows only. 13. Statements Some chairs are handles. All handles are pots. All pots are mats. Some mats are buses. Conclusions I. Some buses are handles. II. Some mats are chairs III. No bus is handle IV. Some mats are handles (1) Only I, II and IV follow (2) Only II, III and IV follow (3) Only either I or III and II follow (4) Only either I or III and IV follow (5) Only either I or III and II and IV follow Ans : (5) Conclusions I. X ] II. v ] or III. v ] IV. v Either I or III and II, IV are true. 14. Statements Some sticks are lamps. Some flowers are lamps. Some lamps are dresses. All dresses are shirts Conclusions I. Some shirts are sticks II. Some shirts are flowers III. Some flowers are sticks IV. Some dresses are sticks (1) None follows (2) Only I follows (3) Only II follows (4) Only III follows (5) Only IV follows Ans : (1) Conclusions I. X II. X III. X IV. X None is true 15. Statements Some benches are walls. All wall are houses. Some houses are jungles. All jungles are roads. Conclusions I. Some roads are benches II. Some jungles are walls III. Some houses are benches IV. Some roads are houses (1) Only I and II follow (2) Only I and III follow (3) Only III and IV follow (4) Only II , III and IV follow (5) None of the above Ans : (3) Conclusions I. X II. X III. v IV. v Only III and IV follow. Directions (Q. 16-20) Study the following information carefully and answer the questions given below: A, B, C, D, E, F, G and H are eight employees of an organization working in three department viz. Personnel Administration and Marketing with not more than three of them in any department. Each of them has a different choice of sports from football, cricket, volleyball badminton, lawn tennis, basketball, hockey and table tennis not necessarily in the same order. D works in Administration and does not like either football or cricket. F works in Personnel with only A who likes table tennis. E and H do not work in the same department as D. C likes hockey and does not work in Marketing. G does not work in Administration and does not like either cricket or badminton. One of those who work in Administration likes football. The one who likes volley ball works in personnel. None of those who work in Administration likes either badminton or lawn tennis. H does not like cricket. 16. Which of the following groups of employees work in Administration department ? (1) EGH (2) AF (3) BCD (4) BGD (5) Data inadequate Ans : (3) 17. In which department does E work ? (1) Personnel (2) Marketing (3) Administration (4) Data inadequate (5) None of these Ans : (2) 18. What is E's favourite sport ? (1) Cricket (2) Badminton (3) Basketball (4) Lawn Tennis (5) None of these Ans : (1) 19. Which of the following combinations of employee department - favourite sport is correct ? (1) E-Administration - Cricket (2) F-Personnel -Lawn Tennis (3) H-Marketing -Lawn Tennis (4) B-Administration -Table Tennis (5) None of these Ans : (5) 20. What is G's favourite sport ? (1) Cricket (2) Badminton (3) Basketball (4) Lawn Tennis (5) None of these Ans : (4) Solutions (16-20) Person A B C D E F G H Department Personnel Administration Administration Administration Marketing Personnel Marketing Marketing Sports Table Tennis Football Hockey Basket ball Cricket Volleyball Lawn Tennis Badminton Direction (Q.21-25) In the following questions, the symbols @, $, *, # and & are used with the following meaning as illustrated below : 'P $ Q' means 'P is not smaller than Q'. 'P @ Q' means 'P is neither smaller than nor equal to Q'. 'P # Q' means 'P is neither greater than nor equal to Q'. 'P & Q' means 'P is neither greater than nor smaller than Q'. 'P * Q' means 'P is not greater than Q'. Now in each of the following questions assuming the given statements to be true, find which of the four conclusions I, II, III and IV given below them is/are definitely true and give your answer accordingly. 21. Statements N & B, B $ W, W#H, H*M Conclusions I. M @ W II. H @ N III. W & N IV. W # N (1) Only I is true (2) Only III is true (3) Only IV is true (4) Only either III or IV is true (5) Only either III or IV and I are true Ans : (5) Expl : N= B, B> W, W <H, H< Q So, N= B> W < H < M I. M > W (True) II. H > N (False) III. W = N] or IV. W< N ] .: N > W 22. Statements R * D, D$J, J#M, M @K Conclusions I. K # J II. D@M III.R#M IV. D @ K (1) None is true (2) Only I is true (3) Only II is true (4) Only III is true (5) Only IV is true Ans : (1) Expl : R< D, D>J, J<M, M>K So R< D>J<M>K I. K < J (False) II. D>M (False) III. R<M (False) IV. D>K (False) So, none is true. 23. Statements H@T, T#F, F&E, E * V Conclusions I. V$ F II. E @ T III. H @ V IV. T # V (1) Only I, II and III are true (2) Only I, II and IV are true (3) Only II, III and IV are true (4) Only I, III and IV are true (5) All I, II, III and IV are true Ans : (2) Expl : H>T, T<F, F=E, E<V So, H > T<F= E<V I. V> F (True) II. E >T (True) III. H>V (False) IV. T<V (True) So, only I, II and IV are true. 24. Statements D#R, R*K, K @ F, F$J Conclusions I. J #R II. J # K III. R # F IV. K @ D (1) Only I, II and III are true (2) Only II, III and IV are true (3) Only I, III and IV are true (4) All I, II, III and IV are true (5) None of the above Ans : (5) Expl : D<R, R<K, K>F, F>J So, D<R <K>F>J I. J < R (False) II. J<K (True) III. R<F (False) IV. K>D (True) So, Only II and IV are true. 25. Statements M$K, K @ N, N * R, R#W Conclusions I. W @ K II. M $ R III. K @ W IV. M @ N (1) Only I and II are true (2) Only I, II and III are true (3) Only III and IV are true (4) Only II, III and IV are true (5) None of the above Ans : (5) Expl : M>K, K>N, N<R, R<W So, M>K>N<R<W I. W > K (False) II. M> R (False) III. K>W (False) IV. M>N (True) So, only IV is true. Direction (Q.26-30) Study the following information carefully and answer the questions given below: Following are the conditions for selecting senior Manager-Credit in bank. The candidate must (i) be a graduate in any discipline with at least 60 percent marks. (ii) have post qualification work experience of at least ten years in the Advances Section of a bank. (iii) be at least 30 years and not more than 40 years as on 01.04.2010. (iv) have secured at least 40 per cent marks in the group discussion (v) have secured at least 50 per cent marks in interview Except (a) at (i) above but has secured at least 50 per cent marks in graduation and at least 60 per cent marks in post graduation and at least 60 per cent marks in post graduation in and discipline the case is to be referred to the General Manager -Advances. (b) at (ii) above but has total post qualification work experience of at least seven years out of which at least three years as Manager-credit in a bank the case is to be referred to Executive Director. In each question, below details of one candidate is given. You have to take one of the following courses of action based on the information provided and the conditions and sub-conditions given above and mark the number of that course of action as your answer. You are not to assume anything other than the information provided in each question. All these cases are given to you as on 01.04.2010. Give answer (1) if the case is to be referred to Executive Director Give answer (2) if the case is to be referred to General Manager-Advances Give answer (3) if the data are inadequate to take a decision. Give answer (4) if the candidate is not to be selected. Give answer (5) if the candidate is to be selected. 26. Prakash Gokhale was born on 4th August 1977. He has secured 65 per cent marks in post graduation and 58 per cent marks in graduation. He has been working for the past ten years in the Advances Department of a bank after completing his post graduation. He has secured 45 per cent marks in the group discussion and 50 per cent marks in the interview. Ans : (2) Expl : Prakash has less than 62% in graduation but more than 60 % in PG. So, his case is to be referred to the GM-advances. 27. Amit Narayan was born on 28th May 1974. He has been working in the Advances Department of a bank for the past eleven years after completing his B.Sc degree with 65 per cent marks. He has secured 55 percent marks in the group discussion and 50 per cent marks in the interview. Ans : (5) Expl : Amit Narayan fulfils all requirement. 28. Shobha Gupta has secured 50 per cent marks in the interview and 40 per cent marks in the group discussion. She has been working for the past eight years out of which four years as Manager-Credit in a bank after completing her B.A. degree with 60 per cent marks. She was born on 12th September, 1978. Ans : (1) Expl : Shobha Gupta does not fulfil condition II but B fulfils. So her case is to be referred to the ED. 29. Rohan Maskare was born on 8th March 1974. He has been working in a bank for the past twelve years after completing his B. Com degree with 70 per cent marks. He has secured 50 per cent marks in both the group discussion and the interview. Ans : (3) Expl : Data insufficient (experience in advances department is not clear) 30. Sudha Mehrota has been working in the Advances Department of a bank for the past twelve years after completing her B.Com degree with 60 per cent marks. She has secured 50 per cent marks in the group discussion and 40 per cent marks in the interview. She was born on 15th February 1972. Ans : (4) Expl : Sudha Mehrotra has less than 50 % marks in interview. So, she is not to be selected. Directions (Q. 31-35) In each questions below is given a statement followed by three courses of action numbered (A), (B) and (C). A course of action is a step or administrative decision to be taken for improvement, follow-up or further action in regard to the problem, policy etc., On the basis of the information given in the statement, you have to assume everything in the statement to be true, then decide which of the suggested courses of action logically follow (s) pursuing. 31. Statement : Many Political activists have decided to stage demonstrations and block traffic movement in the city during peak hours to protest against the steep rise in prices of essential commodities. Courses of action (A) The government should immediately ban all forms of agitations in the country. (B) The police authority of the city should deploy additional forces all over the city to help traffic movement in the city. (C) The state administration should carry out preventive arrests of the known criminals staying in the city. (1) Only (A) (2) Only (B) (3) Only (C) (4) Only (A) and (B) (5) None of these Ans : (2) Expl : Statement followed by course of action B only to solve the traffic problems. 32. S tatement : The School dropout rate in many districts in the state has increased sharply during the last few years as the parents of these children make them work in the fields owned by others to earn enough for them to get at least one meal a day. Courses of action (A) The government should put up a mechanism to provide foodgrains to the poor people in these districts through public distribution system to encourage the parents to send their wards to school. (B) The government should close down some of these schools in the district and deploy the teachers of these schools to nearby schools and also ask remaining students to join these schools. (C) The government should issue arrest warrants for all the parents who force their children to work in fields instead of attending classes. (1) Only (A) (2) Only (B) (3) Only (C) (4) Only (A) and (B) (5) None of these Ans : (1) Expl : Course of Action A is logically follow. 33. Statement : A large private bank decided to retrench one-third of its employees in view of the huge losses incurred by it during the past three quarters. Course of action (A) The government should issue a notification to general public to immediately stop all transactions with the bank. (B) The government should direct the bank to refrain from retrenching its employees. (C) The government should ask the central bank of the country to initiate an enquiry into the bank's activities and submit its report. (1) None (2) Only (A) (3) Only (B) (4) Only (C) (5) Only (A) and (C) Ans : (4) Expl : Course of action C logically follows, because government control banking system with the help of Central Bank. 34. Statement : One aspirant was kill due to stampede while participating in a recruitment drive of police constables. Course of action (A) The officials in charge of recruitment process should immediately be suspended. (B) The team of officials should be asked to find out the circumstances which led to the death of the aspirant and submit its report within a week. (C) The government should ask the home department to stagger the number of aspirants over more number of days to avoid such incidents in future. (1) Only (A) (2) Only (B) (3) Only (C) (4) Only (B) and (C) (5) None of these Ans : (4) , Expl : Course of action B and C logically follow 35. Statement : A heavy un seasonal downpour during the last two days has paralysed the normal life in the state in which five persons were killed but this has provided a huge relief to the problem of acute water crisis in the state. Course of action : (A) The state government should set up a committee to review the alarming situation. (B) The state government should immediately remove all the restrictions on use of potable water in all the major cities in the state. (C) The state government should send relief supplies to all the affected areas in the state. (1) None (2) Only (A) (3) Only (B) and (C) (4) Only C (5) All (A), (B), (C) Ans : (5) All course of action follow 36. Statement : The government has decided to instruct the banks to open new branches in such a way that there is one branch of any of the banks in every village of population 1000 and above or a cluster of villages with population less than 1000 to provide banking service to all the citizens. Which of the following will weaken the step taken by the government ? (1) The private sector banks in India have stepped up their branch expansion activities in rural India. (2) Many government owned banks have surplus manpower in its urban branches. (3) All the banks including those in private sector will follow the government directive (4) Large number of branches of many government owned banks in rural areas are making huge losses every year due to lack of adequate business activities (5) None of the above. Ans : (4) Because only to open a branch is note solution. Government may incur loss, where business activity is not satisfactory either the population is 1000 and above. 37. Cause : The government has recently increased its taxes on petrol and diesel by about 10 per cent. Which of the following can be a possible effect of the above cause? (1) The petroleum companies will reduce the prices of petrol and diesel by about 10 per cent (2) The petroleum companies will increase the prices of petrol and diesel by about 10 per cent. (3) The petroleum companies will increase the prices of petrol and diesel by about 5 per cent. (4) The petrol pumps will stop selling petrol and diesel till the taxes are rolled back by the government (5) None of these Ans : (3) Company may hike the price by 5 to 10% (ideal like will be by 5%) 38. Statement : Most of the Companies in IT and ITES sectors in India have started hiring from engineering college campuses this year and are likely to recruit much more than yearly recruitment of the earlier years. Which of the following substantiates the facts stated in the above statement ? (1) IT and ITES are the only sectors in India which are hiring from engineering college campuses. (2) Government has stepped up recruitment activities after a gap of five years. (3) The IT and ITES companies have now decided to visit the engineering college campuses for tier II cities in India as well (4) Availability of qualified engineers will substantially increase in the near future. (5) None of the above Ans : (3) Companies may go to tier II cities for recruitments 39. Effect : Majority of the employees of the ailing organization opted for voluntary retirement scheme and left the organization with all their retirement benefits within a fortnight of launching the scheme. Which of the following can be a probable cause of the above effect? (1) The company has been making huge losses for the past five years and is unable to pay salary to its employees in time. (2) The management of the company made huge personal gains through unlawful activities (3) One of the competitors of the Company went bankrupt last year. (4) The company owns large tracts of land in the state which will fetch huge sum to its owners (5) None of the above. Ans : (1) Company has been making huge losses for the past 5 years and unable to pay the salary on time. SBI Reasoning-English Qquestions SBI and SBT Test of reasoning solved questions with detailed explanations,SBI previous years solved question papers for reasoning and english language computer and marketing awareness,general awareness,SBT and SBI free solved sample placement papers SBI reasoning questions Qn.1 How many such pairs of letters are there in the word GUARDIAN each of which has as many letters between them in the word as in the English alphabet ? None One Two Three More than three Qn.2 Four of the following five are alike in a certain way and so form a group. Which is the one that does not belong to that group ? 19 17 23 29 27 Answer:27 Explanation:All the rest are prime numbers. Qn.3 How many meaningful English words can be made with the letters TEBI using each letter only once in each word ? None One Answer:One Explanation:B I T E Two Three More than three Qn.4 n a certain code LONG is written as 5123 and GEAR is written as 3748. How is LANE written in that code ? 5427 Answer:5427 Explanation:L ---> 5, O ---> 1, N ---> 2, G ---> 3 G ---> 3, E ---> 7, A ---> 4, R ---> 8Therefore,L ---> 5, A ---> 4, N ---> 2, E ---> 75247 5847 5237 None of these Qn.5 ‘BD’ is related to ‘EG’ and ‘MO’ is related to ‘PR’ in the same way as ‘FH’ is related to ………. JM IL JL IK None of these Qn.6 In a certain code BREAKDOWN is written as BFSCJMVNC. How is ORGANISED written in that code ? PSHBMCDRH BHSPMCDRH BHSPOCDRH BHSPNHRDC None of these Qn.7 How many such digits are there in the number 58674139 each of which is as far away from the beginning of the number as when the digits within the number are rearranged in descending order ? None One Two Three More than three Qn.8 In a certain code language ‘pik da pa’ means ‘where are you’; ‘da na ja’ means ‘you may come’ and ‘na ka sa’ means ‘he may go’, which of the following means ‘come’ in that code language ? da ja na Cannot be determined None of these Qn.9 Four of the following five are alike in a certain way and so form a group. Which is the one that does not belong to that group ? B D F V X Z F I K M O Q L N P Qn.10 What should come next in the following number series ? 9 8 9 8 7 98 7 6 9 8 7 6 5 9 8 7 6 5 4 9 8 7 6 53 4 Answer:4 Explanation:98 987 9876 98765 987654 98765 42 1 None of these Qn.11 Which of the following is the middle digit of the second highest among the following five numbers ? 254 319 963 842 6975 1 6 4 Answer:4 Explanation:963, 8 4 2, 697, 319, 254 9 Qn.12 Meeta correctly remembers that her father’s birthday is after 8th July but before 12th July. Her brother correctly remembers that their father’s birthday is after 10th July but before 15th July. On which day of July was definitely their father’s birthday ? 10th 11th 10th or 11th Cannot be determined None of these Qn.13 In a class of 50 students M is eighth from top. H is 20th from bottom. How many students are there between M and H ? 22 23 24 Cannot be determined None of these Qn.14 Among A, B, C, D and F each scoring different marks in the annual examination, D scored less than only F among them. B scored more than A and C but less than D. Who among them scored least marks among them ? A C B Data Inadequate None of these Qn.15 Four of the following five are alike in a certain way and so form a group. Which is the one that does not belong to that group ? Copper Iron Aluminium Zinc Steel Answer:Steel Explanation:All the rest are pure metals. Qn.16 Directions(Q16-20): In each question below are three statements followed by two conclusions numbered I and II. You have to take the three given statements to be true even if they seem to be at variance from commonly known facts and then decide which of the given conclusions logically follows from the three statements disregarding commonly known facts. Give answers : (A) If only conclusion I follows.(B) If only conclusion II follows.(C) If either conclusion I or II follows.(D) If neither conclusion I nor II follows.(E) If both conclusions I and II follow. Statements: This world is neither good nor evil; each man manufactures a world for himself.Conclusions:I. Some people find this world quite good.II. Some people find this world quite bad. A B C D E Answer:E Explanation:The statement mentions that the world for a man is as he makes it himself. So, some people might find it good and some quite bad. Thus, both I and II follow. Qn.17 Statements: The eligibility for admission to the course is minimum second class Master's degree. However, the candidates who have appeared for the final year examination of Master's degree can also apply. Conclusions:I. All candidates who have yet to get their Master's degree will be there in the list of selected candidates. II. All candidates having obtained second class Master's degree will be there in the list of selected candidates .A B C D Answer:D Explanation:The statement mentions that the candidates who have obtained second class Master's degree or have appeared for the final year examination of Master's degree, can apply for admission. This implies that both types of candidates may be selected on certain grounds. Thus, some candidates of each type and not all candidates of any one type, may be selected. So, neither I nor II follows.E Qn.18 Statements: Any student who does not behave properly while in the school brings bad name to himself and also for the school. Conclusions: I. Such student should be removed from the school .II. Stricter discipline does not improve behaviour of the students. A B C D Answer:D Explanation:Clearly, I cannot be deduced from the statement. Also, nothing about discipline is mentioned in the statement. So, neither I nor II follows. E Qn.19 Statements: A Corporate General Manager asked four managers to either submit their resignations by the next day or face termination orders from service. Three of them had submitted their resignations by that evening. Conclusions:I. The next day, the remaining manager would also resign. II. The General Manager would terminate his services the next day.A B C D E Qn.20 Statements: Only good singers are invited in the conference. No one without sweet voice is a good singer. Conclusions:I. All invited singers in the conference have sweet voice. II. Those singers who do not have sweet voice are not invited in the conference.A B C D E Answer:E Explanation:The statement asserts that a good singer always has a sweet voice and only good singers are invited in the conference. This implies that all those invited in the conference have sweet voice and those who do not have sweet voice are not invited. So, both I and II follow. Qn.21 Directions(Q21-25): Study the following arrangement carefully and answer the questions given below- B # A R 5 8 E % M F 4 J 1 U @ H 2 © 9 T I 6 * W 3 P # K 7 $ Y Which of the following is the twelfth to the left of the twentieth from the left end of the above arrangement ? % Answer:% Explanation:20th from the left end is T and 12th to the left of T is % W $ J None of these Qn.22 How many such numbers are there in the above arrangement each of which is immediately preceded by a consonant and also immediately followed by a symbol ? None One Two Answer:Two Explanation:H 2 © and K 7 $ Three More than three Qn.23 How many such symbols are there in the above arrangement each of which is immediately preceded by a letter and also immediately followed by a number ? None One Two Three More than three Qn.24 How many such consonants are there in the above arrangement each of which is immediately preceded by a consonant and also immediately followed by a number ? None One Answer:One Explanation:M F 4 Two Three More than three Qn.25 If all the numbers in the above arrangement are dropped, which of the following will be the eleventh from the right end ? U T F H Answer:H Explanation:After dropping all the numbers. B # A R E % M F J U @ H © T I * W P # K $ Y11th from the right end is H.None of these Qn.26 Directions(26-30): In each question below is given a group of digits/symbols followed by four combinations of letters lettered (A), (B), (C) and (D). You have to find out which of the combinations correctly represents the group of digits/symbols based on the following letter coding system and mark the letter of that combination as the answer. If none of the letter combinations correctly represents the group of digits/ symbols, mark (E) i.e. ”˜None of these’ as the answer. Digit/Symbol :4 % 3 9 $ 1 8 @ © 2 # 5 6 * 7 d Letter Code :P M I T R Q J F H A E U N B G L Conditions :(i) If the first element in the group is a symbol and the last element is a digit, the codes are to be interchanged.(ii) If the first element in the group is a digit and the last element is a symbol both are to be coded as the code for the digit.(iii) If both the first and the last elements are even digits both are to be coded as ”˜X’.(iv) If both the first and the last elements are odd digits, both are to be coded as ”˜Y’. 4%@93* PMFTIB PMFTIP BMFTIB XMFTIX None of these Qn.27 $1896© RQJTNH HQJTNR RQJTNR YQJTNY None of these Qn.28 2*#836 YBEJIY ABEJIN NBEJIA XBEJIX None of these Qn.29 8732@9 TGIAFJ YGIAFY JGIAFT XGIAFX None of these Qn.30 7#$%35 GERMIU UERMIG GERMIG XERMIX None of these Qn.31 Directions(31-35): In the following questions, the symbols @, ©, %, $ and d are used with the following meanings illustrated. 'P % Q' means 'P is greater than Q'.' P d Q' means 'P is neither greater than nor smaller than Q'. 'P @ Q' means 'P is smaller than Q'. 'P © Q' means 'P is either smaller than or equal to Q'. 'P $ Q' means 'P is either greater than or equal to Q'. In each of the following questions assuming the given statements to be true, find out which of the two conclusions I and II given below them is/are definitely true. Give answers :(A) If only conclusion I is true. (B) If only conclusion II is true. (C) If either conclusion I or conclusion II is true. (D) If neither conclusion I nor conclusion II is true (E) If both conclusions I and II are true. Statements : M @ J, J © R, R d KConclusions :I. K d JII. K % J A B C D E Qn.32 Statements : N $ T, T d H, N @ W Conclusions :I. W % TII. H © NA B C D E Qn.33 Statements : F @ R, R © V, V $ T Conclusions :I. V % FII. F @ TA B C D E Qn.34 Statements : W © D, D $ B, B @ H Conclusions :I. H % DII. W @ BA B C D E Qn.35 Statements : F d T, T $ M, M © R Conclusions :I. R $ FII. M © FA B C D E Qn.36 Directions(36-40): Study the following information and answer the questions given below- M, N, P, R, T, W, F and H are sitting around a circle facing at the centre. P is third to the left of M and second to the right of T. N is second to the right of P. R is second to the right of W who is second to the right of M. F is not an immediate neighbour of P. Who is to the immediate right of P ? H F R Data Inadequate None of these Qn.37 Who is to the immediate right of H ? R F M Data Inadequate None of these Qn.38 Who is to the immediate left of R ? P H W T Data Inadequate Qn.39 Who is third to the right of H ? T W R F Data Inadequate Qn.40 Who is second to the right of F ? M R T Data Inadequate None of these ********************************************************************************** Sbi Qn.51 English Language Directions(51-60): Read the following passage carefully and answer the questions given below it. Certain words are printed in bold to help you to locate them while answering some of the questions. The yearly festival was close at hand. The store room was packed with silk fabrics. gold ornaments, clay bowls full of sweet curd and platefuls of sweetmeats. The orders had been placed with shops well in advance. The mother was sending out gifts to everyone. The eldest son, a government servant, lived with his wife and children in far off lands. The second son had left home at an early age. As a merchant he travelled all over the world. The other sons had split up over petty squabbles, and they now lived in homes of their own. The relatives were spread all across the world. They rarely visited. The youngest son, left in the company of a servant, was soon bored and stood at the door all day long, waiting and watching. His mother, thrilled and excited, loaded the presents on trays and plates, covered them with colourful kerchiefs, and sent them off with maids and servants. The neighbours looked on. The day came to an end. All the presents had been sent off. The child came back into the house and dejectedly said to his mother, 'Maa, you gave a present to everyone, but you didn't give me anything !' His mother laughed, 'I have given all the gifts away to everyone, now see what's left for you.' She kissed him on the forehead. The child said in a tearful voice, 'Don't I get a gift ?' 'You'll get it when you go far away.' 'But when I am close to you, don't I get something from your own hands ?' His mother reached out her arms and drew him to her. 'This is all I have in my own hands. It is the most precious of all.' Why did the woman's second son travel ? He was restless by nature He did not want to stay at home He was rich and could afford to travel His job was such that he had to travel None of these Qn.52 Why did the woman's eldest son not attend the festival ? He was not on good terms with his youngest brother who lived at home He had quarrelled with his mother His wife did not allow him to return home His job prevented him from taking leave None of these Qn.53 How did the woman prepare for the festival ? 1. She bought expensive gifts for her children and neighbours. 2. She ordered her servants to prepare sweets and food well in advance. 3. She made sure that her youngest child was looked after so that he wouldn't be bored. None Only 1 Only 2 Both 1 and 2 All 1, 2 and 3 Qn.54 What did the youngest child do while his mother was busy ? 1. He waited for a chance to steal some sweetmeats. 2. He pestered his mother to give him a present. 3. He stood at the door with servants. Only 1 Only 2 Both 1 and 3 Only 3 None of these Qn.55 Which of the following can be said about the woman ? She was a widow who had brought up her children single handedly She was not a good mother since her children had left home at an early age She enjoyed sending her family gifts at festival time She gave expensive presents to show that she was wealthy She rarely visited her grand-children because they all lived abroad Qn.56 What did the boy receive from his mother ? She taught him the value of patience She encouraged him to grow up and live independently like his brothers She showed him the importance of giving expensive gifts She gave him a hug to express her love None of these Qn.57 Which of the following is TRUE in the context of the passage ? The woman usually ignored her youngest son The woman's eldest son lived abroad The members of the woman's family did not care about her The woman made all the preparations herself since she did not want to burden the servants The woman sent gifts to her children to ensure that they visited her Qn.58 Directions(Q58-59): Choose the word which is most nearly the SAME in meaning as the word printed in bold as used in the passage. Left Gone Quit Remaining Disappeared Forgot Qn.59 Packed Filled Squeezed Crowd Collected Untidy Qn.60 Choose the word which is most OPPOSITE in meaning of the word dejectedly as used in the passage. Calmly Happily Willingly Fortunately Softly Qn.61 Directions(61-65): Read each sentence to find out whether there is any error in it. The error, if any, will be in one part of the sentence. The letter of that part is the answer. If there is no error, the answer is (E). (Ignore errors of punctuation, if any) Many multinational companies (A) / have not been as (B) /successful in India (C) /than we expected. (D) No error (E) A B C D Answer:D Explanation:Replace ‘than’ with ‘as’. E Qn.62 He has ruined (A) /his eyesight (B) /by not using (C) /his spectacles regularly. (D) No error (E) A B C D E Qn.63 Mostly of the (A) /newly recruited officers (B) /have no experience (C) /in the banking sector. (D) No error (E) A Answer:A Explanation:Change ‘Mostly’ to ‘Most’. B C D E Qn.64 The resignation of (A) /one of our directors (B) /have caused the price (C) / of shares to fall. (D) No error (E) A B C Answer:C Explanation:Change ‘have’ to ‘has’. D E Qn.65 There are many (A) /ways of which (B) /inflation can (C) /be measured. (D) No error (E) A B Answer:B Explanation:Replace ‘of’ with ‘in’ C D E Qn.66 Directions(Q66-70): Which of the phrases (A), (B), (C) and (D) given below should replace the phrase given in bold in the following sentence to make the sentence grammatically meaningful and correct. If the sentence is correct as it is and 'No correction is required.' mark (E) as the answer. Each of the loan must be approved by the Branch Manager- Every loan Each one of the loan Any of the loan All of the loan No correction required Qn.67 The issue was taken before the Municipal Corporation meeting last week- Taking place at Taken after Being taken in Taken up at No correction required Qn.68 He has asked for the names of those employees involved in the project. had asked having asked about was asked that is asking no correction required Qn.69 Considerate the traffic, it is better to leave for the airport an hour early- While considering Consideration of Considering Being considerate to No correction required Qn.70 He is a good leader, knowing that to motivate his employees to achieve”” That known when Who knows how Which knows how Knowing what No correction required Qn.71 Directions(71-75): Rearrange the following six sentences (1), (2), (3), (4), (5) and (6) in the proper sequence to form a meaningful paragraph; then answer the questions given below them. (1) The able bodied men of the tribe gathered to discuss how to climb the mountain. (2) As part of their plundering they kidnapped a baby of one of the families. (3) One day the mountain tribe invaded those living in the valley. (4) 'We couldn't climb the mountain. How could you?-, they asked, 'It wasn't your baby !' she replied. (5) There were two tribes in the Andes' one lived in the valley and the other high up in the mountains. (6) Two days later they noticed the child's mother coming down the mountain that they hadn't yet figured out how to climb. Which of the following should be the SECOND sentence after rearrangement ? 1 2 3 4 5 Qn.72 Which of the following should be the FIFTH sentence after rearrangement ? 6 5 4 3 2 Qn.73 Which of the following should be the FIRST sentence after rearrangement ? 1 2 3 4 5 Qn.74 Which of the following should be the SIXTH (LAST) sentence after rearrangement ? 1 2 3 4 5 Qn.75 Which of the following should be the THIRD sentence after rearrangement ? 1 2 3 4 5 Qn.76 Directions(Q76-80): In each question below a sentence with four words printed in bold type is given. These are lettered (A), (B), (C) and (D). One of these four words printed in bold may be either wrongly spelt or inappropriate in the context of the sentence. Find out the word, which is wrongly spelt or inappropriate, if any. The letter of that word is your answer. If all the words printed in bold are correctly spelt and also appropriate in the context of the sentence mark (E) i.e., all correct as your answer. The income (A) of many people in rural (B) India is not adequate (C) to satisfy (D) their basic needs. All correct (E) A B C D E Qn.77 He is always (A) prompt (B) in caring (C) out instructions. (D) All correct (E) A B C D E Qn.78 The revized (A) rates (B) of interest will be effective (C) immediately. (D) All correct (E) A B C D E Qn.79 Such transactions (A) are quiet (B) expensive (C) and time consuming (D) for customers. All correct (E) A B C D E Q n.80 The guidelines (A) of the new scheme (B) are expected (C) to be finally (D) soon. All correct (E) A B C D E Qn.81 Directions(81-90): In the following passage there are blanks each of which has been numbered. These numbers are printed below the passage and against each five words/ phrases are suggested one of which fits the blank appropriately. Find out the appropriate word in each case. I used to look …(81)… to the holidays. I was usually …(82)… to my uncle’s house where I …(83)… his children. I did not get paid a salary for …(84)… What I received in return however, was far more …(85)… My uncle was an avid reader. During the time I spent with his family I had an …(86)… to read the vast amount of books and magazines that he possessed. This improved my English to some …(87)… Reading became my new …(88–89)… spending my pocket money on a ticket to the cinema I began to …(90)… books. This has benefited me greatly. forward towards backward up around Qn.82 I was usually …(82)… to my uncle’s house went sent visited travelled gone Qn.83 where I …(83)… his children. cared occupy guarded taught played Qn.84 I did not get paid a salary for …(84)…. them whom this now which Qn.85 What I received in return however, was far more …(85)…. expensive deserving helping demanding valuable Qn.86 During the time I spent with his family I had an …(86)…....... opportunity ability use encouragement achievement Qn.87 This improved my English to some …(87)…. distance extent time limits degrees Qn.88 Reading became my new …(88)… activity hope hobby duty worship Qn.89 Reading became my new …(88–89)… despite though by instead of while Qn.90 I began to …(90)… books. This has benefited me greatly. sell read exchange invest buy Qn.91 Directions(Q91-100): In the following passage there are blanks each of which has been numbered. These numbers are printed below the passage and against each five words/ phrases are suggested one of which fits the blank appropriately. Find out the appropriate word in each case. Each year Middle class Indian children ...(91)... hundred of crores of rupees in pocket money and ...(92)... a heavy burden parental ...(93)... like adults, these kids have ...(94)... connected with budgeting and saving money. unfortunately, basic money ...(95)... is ...(96)... taught in schools. At home, very few parents ...(97)... money matters with their children. Kids who ...(98)... about money ...(99)... have been found to be way ahead of their peers. Indeed, learning to ...(100)... with money properly fosters discipline, good work habits and self respect. spend steal save give invest Qn.92 ...(92)... a heavy burden parental move take risk put lift Qn.93 a heavy burden parental ...(93)... like adults, promises payments demands attitudes incomes Qn.94 these kids have ...(94)... connected with budgeting and saving money. expenses experience problems guidance necessities Qn.95 basic money ...(95)... is ...(96)... taught in schools. availability inflation economics problem management Qn.96 unfortunately, basic money ...(95)... is ...(96)... taught in schools. carefully rarely generally always thoroughly Qn.97 very few parents ...(97)... money matters with their children. discuss understand teach reveal advise Qn.98 Kids who ...(98)... about money quarrel ask learn waste spend Qn.99 ...(99)... have been found to be way ahead of their peers. slowly early timely lately regularly Qn.100 Indeed, learning to ...(100)... with money properly fosters discipline, good work habits and self respect. decide earn control deal pay SBI Reasoning Questions SBI Associate Banks Clerks Exam Exam Held On : 16-01-2011 ,State Bank of India Clerical Cadre,Probatioanry officers,Associate clerks questions for practice,SBI and SBT Examination solved question papers SBT and SBI largest collection of free solved placement papers SBI Reasoning Ability solved question paper for practice,SBI Model question papers for practice 121. 1 How many meaningful English words can be formed with the letters ADIC using each letter only once in each word? (1) None (2) One (Ans) (3) Two (4) Three (5) More than three 122.2 Four of the following five are alike in a certain way and so form a group, which is the one that does not belong to that group? (1) Violin (2) Harp (3) Guitar (4) Flute (Ans) (5) Sitar 123. 3 If it is possible to mark only one meaningful word with the first, fourth, fifth and tenth letters of the word TELEVISION, which of the fallowing would be the second letter of that word from the right end? If no such word can be made, give 'X' as your answer and if more than one such word can be formed, give your answer as 'Y'. (1) X (2) L (3) N (Ans) (4) E (5) Y Explanation : The First, fourth, fifth and tenth letters of the word TELEVISION are = T, E, V, N useful will be VENT. so, second letter from right is N 124.4 In a certain code TRUMP is written SUTQN. How is FIRED written in that code? (1) GJQEF (2) JGEQF (3) JQFEF (4) JGQFE (5) JGQEF (Ans) 125. 5 In a certain language 'give me more' is coded as '7 3 5', 'she has more' is coded as '9 7 1' and 'she asked me gently' is coded as '6 3 2 1'. Which of the following is the code for 'give' in that language? (1) 5 (Ans) (2) 7 (3) 6 (4) 9 (5) 1 Explanation : give me more ? 7 3 5 she has more ? 9 7 1 she asked me gently = 6 3 2 1 So, give = 5 126. 6 If each of the vowels in the word GOLIATHS is changed to the next letter in the English alphabetical series and each consonant is changed to the previous letter in the English alphabetical series, and then the alphabets so formed are arranged in alphabetical order from left to right, which of the following will be sixth from the left of the new arrangement thus formed? (1) S (2) P (Ans) (3) G (4) J (5) F Explanation : After changing the word GOLIATHS will be F P K J B S G R The new word will be in alphabetical order B F G J K P R S So, sixth letter from left = P 127. 7 In a certain language 'WEAK' is coded as 4$9ß' and REST' is coded as 7$8#' and 'WREK' is ceded as '4$ß7'. How will 'KEWRA' be coded in the same code? (1) 7 4 $ ß # (2) ß 7 $ 8 # (3) # ß 9 7 4 (4) $ ß 7 9 4 (Ans) (5) 4 7 8 ß $ Explanation : WEAK ? 4 $ 9 8, REST ? 7 $ 8 # and WREK ? 4 $ 8 7 So, KEWRA ? 4 $ 9 8 7 128.8 How many such pairs of letters are there in the word FORMATION each of which has as many letters between them in the word (in both forward and backward directions) as they have between them in the English alphabetical order? (1) None (2) One (3) Two (Ans) (4) Three (5) More than three Explanation : FORMATION ? F _ _ _ _ _ _ _ _ _ N, ON Ans Two 129. 9 If the digits in the number 79246358 are arranged in descending order from left to right, what will be the difference between the digits which are third from the right and second from the left in the new arrangement? (1) 1 (2) 2 (3) 3 (4) 4 (Ans) (5) 5 Explanation : In descending order the number 79246358 will be 98765432 Difference between 8 and 4 = 8 - 4 = 4 130. 10'Music' is related to 'Notes' in the same way as 'Language' is related to _____ (1) Sentences (2) Combination (3) Grammar (4) Alphabets (Ans) (5) Syntax Explanation : Music ? Notes So, language ? Alphabet Directions (131.11-135.15) : Read the following information carefully and answer the questions which follow : If 'P * Q means 'P is the mother of Q'. If 'P x Q' means 'P is the father of Q'. If 'P + Q' means 'P is the sister of Q'. If 'P - Q' means 'P is the brother of Q'. If 'P > Q' means 'P is the son of Q'. If 'P < Q' means 'P is the daughter of Q'. 131.11 In the expression 'A x B + R > S' how is S related to A? (1) Daughter (2) Son (3) Niece (Ans) (4) Nephew (5) Cannot be determined Explanation : A x B + R > S Father sister son So, S is wife of A 132.12. Which of the following means P is the father of S ? (1) P x Q < R * S (Ans) (2) R x P < Q - S (3) R + S > Q + P (4) S + Q - R * P (5) Cannot be determined Explanation : P x Q < R * S Father son mother So, P is father of S 133. 13.In the expression 'P + Q> A - B' how is P related to B? (1) Daughter (2) Son (3) Niece (Ans) (4) Nephew (5) Cannot be determined Explanation : P + Q > A - B sister son brother ? So, P is niece of B 134. 14. Which of the following means D is the aunt of C? (1) D > B * A * C (2) D + B - C * A (Ans) (3) D - B - A x C (4) D + B x A x C (5) None of these Explanation : D + B - C * A ? sister brother mother So, D is aunt of A 135. 15.In the expression 'W > X < Y * Z 'how is W related to Z? (1) Nephew (Ans) (2) Uncle (3) Son (4) Brother-in-law (5) None of these Explanation : W > X < Y * Z ? W son daughter mother So, W is nephew of Z Directions (136.16 - 140.20) : In each of the questions below are given three statements followed by two conclusions numbered I and II, You have to take the given statements to be true even it they seem to be at variance from commonly known facts and then decide which of the given conclusions logically from the statements disregarding commonly known facts. Give answer (1) if only conclusion I follows. Give answer (2) if only conclusion II follows. Give answer (3) if either conclusion I or conclusion II follows. Give answer (4) if neither conclusion I nor conclusion II follows. Give answer (5) if both conclusion I and II follows. 136. 16. Statements : All medicines are tables. Some tables are tonics. Some tonics are bitter. Conclusions : I. Some tablets are bitter. II. No medicine is a tonic. (Ans : 3) Either conclusions I or II follows 137.17. Statements : All incomes are salaries. Some salaries are perks. Some perks are tangible. Conclusions : I. Some incomes are tangible. II. At least some perks are salaries. (Ans : 2) Only conclusions II follows. 138.18. Statements : All roses are red. Some red are colour. All colour are colour. Conclusions : I. Some red are paints. II. All red are rose. (Ans : 1) only conclusion I follows. 139.19. Statements : Some casual are formal. All formal are expensive. All expensive are elegant. Conclusions : I. All formal are elegant. II. Some casual are expensive. (Ans : 5) Both conclusion I and II follow. 140. 20.Statements : All towns are cities. All cities are urban. Some urban are rural. Conclusions : I. Some towns are rural. II. All rural are towns. (Ans : 4) Neither conclusion I nor II follows. Directions (141.21-145.25) : The following questions are based on five words given below: RAT ONE BUT AND SAW (The new words formed after performing the mentioned operations may or may not necessarily be meaningful English words) 141.21 If in each of the given words, each alphabet is changed to the next letter in the English alphabetical series, in how many words thus formed have the consonants changed to vowels ? (1) One (2) Two (3) Three (4) Four (Ans) (5) Five Explanation : The square will be RAT = SBU BUT = CVU ONE = POF AND = BOE SAW = WBX So, Ans is 4. 142. 22. How many such pairs of letters are there in the word highlighted in bold, each of which has as many letters between them in the word (in both forward and backward directions) as they have between them in the English alphabetical order? (1) None (2) One (Ans) (3) Two (4) Three (5) Four 143. 23.In each of the given words, each consonants is changed to the next letter in the English alphabetical series, in how many words thus formed have the vowels (same or different) two or more times? (1) None (2) One (3) Two (4) Three (Ans) (5) Four Explanation : RAT = SAV BUT = CUV ONE = OOE AND = AOE and SAW = WAX So, The words which have two or more vowels are = 3 144.24. If first letter of each word is changed to the next letter in the English alphabetical series, how many meaningful English words will be formed? (1) One (2) Two (Ans) (3) Three (4) Four (5) Five Explanation : RAT = SAT BUT = CUT ONE = PNE AND = BND and SAW = TAW The meaningful words are SAT, CUT 145. 25.If the given words are arranged from left to right as they come in the dictionary which will be fourth from left? (1) RAT (Ans) (2) ONE (3) BUT (4) AND (5) SAW Explanation : RAT will be fourth from left to right 146 - 150 : The arrangement will be Friends Cities Vehicles P Hyderabad Bus Q Kolkata Airoplane R Manglore Car S Chennai Boat T Delhi Train 146 .26- 150 .30: Read the following information carefully and answer the questions which follow: five friends P, Q, R, S and T start their ;journey separately from Goa to Chennai, Kolkata, Delhi, Hyderabad, Manglore via different means of transportation ie Bus, Train, Plane, Car and Yatch. The person going to Delhi did not use yatch, R reached Manglore by car and is used yatch, Q used plane to reach Kolkata and T completed his journey by train. Goa is not connected with Delhi and Chennai via bus. 146. 26 Which pair of place and transport is correct? (1) Kolkata-Bus (2) Delhi-Plane (3) Manglore-Train (4) Chennai-Yatch (Ans) (5) Hyderabad-Car 147.27 Q : Kolkata as S : ? (1) Hyderabad (2) Chennai (Ans) (3) Manglore (4) Delhi (5) Can not be determined 148. 28.Who of the following trvelled to Delhi? (1) T (Ans) (2) R (3) S (4) P (5) Q 149. 29.Which of the following is not correct for P? (1) the travelled to Kolkata by bus (2) the travelled to Delhi by train (3) the travelled to Chennai by yatch (4) the travelled to Hyderabad by bus (Ans) (5) None of these 150. 30.S travelled to___ by_____ (1) Hyderabaad, train (2) Chennai, yatch (Ans) (3) Chennai, train (4) Delhi, yatch (5) Delhi, train Directions (151.31-160.40) : In each of the questions given below which one of the five answer figures on the right should come after thy problem figures on the left, if the sequence we continued? 151. SBI - (Ans : 3) 152. SBI - (Ans : 2) 153. SBI - (Ans : 5) 154. SBI (Ans : 4) 155. SBI - (Ans : 1) 156. SBI - (Ans : 3) 157. SBI - (Ans : 2) 158. SBI (Ans : 2) 159. SBI - (Ans : 5) 160. SBI - (Ans : 4) SBI Reasoning Questions State Bank of India Clerical Cadre,Probatioanry officers,Associate clerks questions for practice,SBI and SBT Examination solved question papers SBT and SBI largest collection of free solved placement papers Reasoning Part -1 1. In certain code DROWN is written as MXNSC. How is BREAK written in that code? (1) LBFSC (2) JBDSA (Ans) (3) JZDQA (4) LZFQC (5) None of these Explanation : Write the letters in reverse order. The code for 1st, 3rd and 5th letters is the preceding letter and for 2nd and 4th, the next letter. DROWN NWORD BREAK KAERB MXNSC JBDSA 2. Among M, N, T, R and D each having a different height, T is taller than D but shorter than M. R is taller than N but shorter than D. Who among them is the tallest? (1) D (2) T (3) M (Ans) (4) R (5) N Explanation : M T D R N 3. How many such digits are there in the number 5436182 each of which is as far away from the beginning of the number as when the digits are arranged in ascending order within the number? (1) None (2) One (Ans) (3) Two (4) Three (5) More than three Explanation : 3 only 4. What should come next in the letter series given below? D D E D E F D E F G D E F G H D E F G H I D E F G H I J D (1) D (2) E (Ans) (3) F (4) J (5) None of these Explanation : E F G H I J K 5. The letters in the word MORTIFY are changed in such a way that the vowels are replaced by the previous letter in the English alphabet and the consonants are replaced by the next letter in the English alphabet. Which of the following will be the fourth letter from the right end of the new set of letters? (1) S (2) H (3) G (4) N (5) None of these (Ans) Explanation : N N S (SBI -Reasoning Part -1 -Ans No. 5) H G Z 6. Four of the following five are alike in a certain way and so form a group. Which is the one that does not belong to that group? (1) Leaf (2) Flower (3) Petal (4) Fruit (5) Tree (Ans) Explanation : Others are parts of tree. 7. Four of the following five are alike in a certain way and so form a group. Which is the one that does not belong to that group? (1) Garlic (2) Ginger (3) Carrot (4) Radish (5) Brinjal (Ans) Explanation : All the others grow below the surface. 8. How many meaningful English words can be made with the letters ALPE using each letter only once in each word? (1) None (2) One (3) Two (4) Three (Ans) (5) More than three Explanation : PALE, LEAP, PEAL. 9. Four of the following five are alike in a certain way and so form a group. Which is the one that does not belong to that group? (1) 24 (2) 48 (3) 32 (4) 72 (5) 64 (Ans) Explanation : It is a perfect square. (8). 10. How many such pairs of letters are there in the word CHAMBERS each of which has as many letters between them in the word as in the English alphabet? (1) None (2) One (3) Two (Ans) (4) Three (5) More than three Explanation : C and A and R and S. 11-15. In each of the questions below are given three statements followed by two conclusions numbered I and II. You have to take the given statement to be true even if they seem to be at variance from commonly known facts. Read all the conclusions and then decide which of the given conclusions logically follows from the given statements disregarding commonly known facts. Give answer : (1) if only Conclusion I follows. (2) if only Conclusion II follows. (3) if either Conclusion I or II follows. (4) if neither Conclusion I nor II follows. (5) if both Conclusions I and II follow. Statements : 11. Some toys are desks. Some desks are pens. All pens are rods. Conclusions : I. Some rods are toys. II. Some pens are toys. (Ans : 4) Explanation : SBI -Reasoning Part -1 -Ans No. 11 Statements : 12. Some tables are huts. No hut is ring. All rings are bangles. Conclusions : I. Some bangles are tables. II. No bangle is table. (Ans : 2) Explanation : SBI -Reasoning Part -1 -Ans No. 12 Statements : 13. All stars are clouds. All clouds are rains. All rains are stones. Conclusions : I. All rains are stars. II. All clouds are stones. (Ans : 2) Explanation : SBI -Reasoning Part -1 -Ans No. 13 Statements : 14. All windows are doors. Some doors are buildings. All buildings are cages. Conclusions : I. Some cages are doors. II. Some buildings are windows (Ans : 1) Explanation : SBI -Reasoning Part -1 -Ans No. 14 Statements : 15. Some chairs are rooms. All rooms are trees. All trees are poles. Conclusions : I. Some poles are chairs. II. Some trees are chairs. (Ans : 5) Explanation : SBI -Reasoning Part -1 -Ans No. 15 Q. 16-20. Study the following arrangement carefully and answer the questions given below: G M 5 I D # J K E 2 P T 4 W % A F 3 U 8 $ N V 6 Q @ 7 H 1 © B 9 ? Z (SBI -Reasoning Part -1 -Ans No. 16 -25) 16. Four of the following five are alike in a certain way based on their positions in the above arrangement and so form a group. Which is the one that does not belong to that group? (1) D J I (2) F U A (3) H @ 1 (Ans) (4) B ? © (5) I # 5 17. What should come in place of the question mark (?) in the following series based on the above arrangement? D J K 2 T 4 % F 3 ? (1) U S V (2) U $ N (3) 8 N V (Ans) (4) 8 N I (5) None of these 18. How many such numbers are there in the above arrangement, each of which is immediately preceded by a vowel and also immediately followed by a symbol? (1) None (2) One (Ans) (3) Two (4) Three (5) More than three 19. How many such consonants are there in the above arrangement, each of which is immediately preceded by a number but not immediately followed by a consonant? (1) None (2) One (3) Two (4) Three (Ans) (5) More than three 20. Which of the following is the fourth to the right of the twelfth from the right end of the above arrangement? (1) 8 (2) 7 (Ans) (3) K (4) A (5) None of these Q. 21-25. Study the following information carefully and answer the questions given below: A, B, C, D, E , F, G and H are sitting around a circle facing at the centre. F is third to the right of B who is third to the right of H. A is third to the left of H. C is fourth to the left of A. E is third to the right of D who is not a neighbour of A. SBI -Reasoning Part -1 -Ans No. 16 -25 21. In which of the following pairs the second person is to the immediate right of the first person? (1) HC (Ans) (2) BE (3) GB (4) FA (5) None of these 22. Who is second to the right of D? (1) F (2) G (3) A (4) Data inadequate (5) None of these (Ans) 23. Who is third to the left of G? (1) H (2) D (3) C (Ans) (4) F (5) None of these 24. Who is fourth to the left of C? (1) F (2) A (Ans) (3) E (4) Data inadequate (5) None of these 25. What is B's position with respect to D? (A) Fourth to the right (B) Fifth to the left (C) Fifth to the right (1) (A) only (2) (B) only (3) (A) and (B) only (Ans) (4) (C) and (D) only (5) None of these Q. 26-30. In each question below is given a group of letters followed by four combinations of digits/symbols numbered (1), (2), n(3) and (4). You have to find out which of the combinations correctly represents the group of letters based on the following coding system and mark the number of that combination as the answer. If none of the four combinations correctly represents the group of letters, mark (5) i.e. 'None of these' as the answer. Letter : P M A K T I J E R N D F U W B Digit/Symbol : 7 # 8 % 1 9 2 @ 3 © S 4 ? 5 6 Conditions : (i) If both the first and the last letters of the group are consonants, both are to be coded as the code for the last letter. (ii) If the first letter is a consonant and the last letter is a vowel, the codes are to be interchanged. 26. BDATFE (1) 6$8146 (2) 6$814@ (3) @$814@ (4) @$8146 (Ans) (5) None of these 27. AWBRND: (1) $563©8 (2) 8563©$ (Ans) (3) 8365©$ (4) 8536©$ (5) None of these 28. EMNTKU: (1) ?#©1%@ (2) @#©14? (3) @#©1%? (Ans) (4) #@©1%? (5) None of these 29. MDEAJI: (1) 1$@82# (2) #$@821 (3) 1$@821 (4) #$@82# (5) None of these (Ans) 30. RKUMFP: (1) 7%?#43 (2) 3?%#47 (3) 3%?#43 (4) 3%?#47 (5) None of these (Ans) Q.31-35. In the following questions, the symbols S, @, ©, % and ? are used with the following meaning as illustrated below: 'P @ Q' means 'P is not greater than Q'. 'P % Q' means 'P is not smaller than Q'. 'P ? Q' means 'P is neither greater than nor smaller than Q'. 'P © Q' means 'P is neither greater than nor equal to Q'. 'P $ Q' means 'P is neither smaller than nor equal to Q'. Now in each of the following questions assuming the given statements to be true, find which of the two conclusions I and II given below them is/are definitely true? Give answer : (1) if only Conclusion I is true. (2) if only Conclusion II is true. (3) if either Conclusion I or II is true. (4) if neither Conclusion I nor II is true. (5) if both Conclusions I and II are true. Ans: P @ Q P ? Q; P % Q P = Q; P * Q P = Q P © Q P < Q; P $ Q p > Q Statements : 31. R $ M, M © F, F % J. Conclusions : I. R $ J II. F © R (Ans : 4) Explanation : R > M; M < F; F = J Statements : 32. M © D, D @ K, K ? N. Conclusions : I. N $ D II. K $ N (Ans : 2) Explanation : M < D; D ? K; K = N Statements : 33. B @ D, D $ M, M ? N. Conclusions : I. N @ D II. D $ N (Ans : 2) Explanation : B ? D; D > M; M = N Statements : 34. F $ W, W % J, K @ N. Conclusions : I. J @ F II. N % W (Ans : 4) Explanation : F > W; W = J; J ? N Statements : 35. F © T, T % R, R $ W. Conclusions : I. W © T II. R © T (Ans : 4) Explanation : F < T; T = R; R > W Q. 36-40. In each of the questions given below which one of the five answer figures on the right should come after the problem figures on the left, if the sequence were continued? SBI -Reasoning Part -1 -Qno. 36-40 SBI -Reasoning Part -1 -Ans No. 36-40 (Ans : 36 - 2)(Ans : 37 - 1)(Ans : 38 - 1)(Ans : 39 - 1)(Ans : 40 - 3) SBI question papers State Bank of India Clerical Cadre Examination,Probationary officers examiantion,Associate clerks written test examination questions with answers,SBI largest collecction of previous years solved question papers,SBI and SBT free solved sample placement papers,SBI and SBT aptitude,reasoning,english,general awareness,current affaris bank based general awarness questions and answers SBI and SBT Mathematics Section Q. 1 - 5. What should come in place of question Mark (?) in the following questions? 1. 92.5% of 550 = ? (1) 506.45 (2) 521.65 (3) 518.55 (4) 508.75 (Ans) (5) None of these 2. 124 * 1213 = ? (1) 127 (2) 1239 (3) 1217 (Ans) (4) 12-7 (5) None of these 3. 12.22 + 22.21 + 221.12? (1) 250.55 (2) 255.50 (3) 250.05 (4) 255.05 (5) None of these (Ans) Explanation : 255.55 4. 464 ÷ (16 * 2.32) = ? (1) 12.5 (Ans) (2) 14.5 (3) 10.5 (4) 8.5 (5) None of these 5. 78 ÷ 5 ÷ 0.5 = ? (1) 15.6 (2) 31.2 (Ans) (3) 7.8 (4) 20.4 (5) None of these 6. A bus covers a distance of 2,924 kms. in 43 hours. What is the speed of the bus? (1) 72 kms/hr (2) 60 kms/hr (3) 68 kms/hr (Ans) (4) Cannot be determined (5) None of these Explanation : Speed = D/t 7. If (9)3 is subtracted from the square of a number, the answer so obtained is 567. What is the number? (1) 36 (Ans) (2) 28 (3) 42 (4) 48 (5) None of these Explanation : x2 - 93 = 567 ? x = 36 8. What would be the simple interest obtained on an amount of Rs. 5,760 at the rate of 6 p.c.p.a. after 3 years? (1) Rs. 1,036.80 (Ans) (2) Rs. 1,666.80 (3) Rs. 1,336.80 (4) Rs. 1,063.80 (5) None of these Explanation : S. I. = 5760 * 6 * 3/100 = Rs. 1036.80 9. What is 333 times 131? (1) 46,323 (2) 43,623 (Ans) (3) 43,290 (4) 42,957 (5) None of these 10. The product of two successive numbers is 8556. What is the smaller number? (1) 89 (2) 94 (3) 90 (4) 92 (Ans) (5) None of these Explanation : x(x +1) = 8556 ? x = 92 11. The owner of an electronics shop charges his customer 22% more than the the cost price. If a customer paid Rs. 10,980 for a DVD Player, then what was the cost price of the DVD Player? (1) Rs. 8,000 (2) Rs. 8,800 (3) Rs. 9,500 (4) Rs. 9,200 (5) None of these (Ans) Explanation : 122/100 x = 10980 ? x = Rs. 9000 12. What would be the compound interest obtained on an amount of Rs. 3,000 at the rate of 8 p.c.p.a after 2 years? (1) Rs. 501.50 (2) Rs. 499.20 (Ans) (3) Rs. 495 (4) Rs. 510 (5) None of these Explanation : C.I = P |(1+ R/100)n - 1| = 3000 | (1 + 8/100)2- 1| = Rs. 499.20 13. What is the least number of be added to 4321 to make it a perfect square? (1) 32 (2) 34 (3) 36 (4) 38 (5) None of these (Ans) Explanation : Image _ SBI _ Ans No. 13 652 < 4321 < 662 Reqd. No. = 662 - 4321 = 35 14. 45% of a number is 255.6. What is 25% of that number? (1) 162 (2) 132 (3) 152 (4) 142 (Ans) (5) None of these Explanation : 45/100 of x = 255.6 ? x = 255.6 x 100/45 ? 25/100 x 255.6 x 100/45 = 142 15. Find the average of the following Set of Scores : 221, 231, 441, 359, 665, 525 (1) 399 (2) 428 (3) 407 (Ans) (4) 415 (5) None of these 16. If (78)2 is subtracted from the square of the number, the answer so obtained is 6,460. What is the number? (1) 109 (2) 111 (3) 113 (4) 115 (5) None of these (Ans) Explanation : x2 - 782 = 6460 ? x =112 17. In an examination it is required to get 40% of the aggregate marks to pass. A student gets 261 marks and is declared failed by 4% marks. What are the maximum aggregate marks a student can get? (1) 700 (2) 730 (3) 745 (4) 765 (5) None of these (Ans) Explanation : 40/100 x = 261 + 4/100 x ? x = 725 18. Pinku, Rinku and Tinku divide an amount of Rs. 4,200 amongst themselves in the ratio of 7 : 8 : 6 respectively. If an amount of Rs. 200 is added to each of their shares, what will be the new respective ratio of their shares of amount? (1) 8 : 9 : 6 (2) 7 : 9 : 5 (3) 7 : 8 : 6 (4) 8 : 9 : 7 (Ans) (5) None of these Explanation : Share of Pinku, Rinku and Tinku in Rs. 4200 are 7/ 7 + 8 + 6 x 4200, 8/ 21 x 4200, 6/ 21 x 4200 i.e. 1400, Rs. 1600, Rs. 1200 Reqd. ratio = (1400 + 200) : (1600 + 200) : (1200 + 200) = 8 : 9 : 7 19. Ms Suchi deposits an amount of Rs. 24,000 to obtain a simple interest at the rate of 14 p.c.p.a. for 8 years. What total amount will Ms Suchi get at the end of 8 years? (1) Rs. 52,080 (2) Rs. 28,000 (3) Rs. 50,880 (Ans) (4) Rs. 26,880 (5) None of these Explanation : Total Amount = Rs. 24000 + 24000 x 14 x 8/100 = Rs. 50880 20. The average of 5 consecutive even numbers A, B, C, D and E is 52. What is the product of B and E? (1) 2912 (2) 2688 (3) 3024 (4) 2800 (Ans) (5) None of these Explanation : A + B + C + D + E x + (x + 2) + (x + 4) + (x + 6) + (x + 8) = 5 x 52 ? x = 48 = A, B = 50 ? E = 48 + 8 = 56 ? BE = 50 x 56 = 2800 21. The difference between 42% of a number and 28% of the same number is 210. What is 59% of that number? (1) 630 (2) 885 (Ans) (3) 420 (4) 900 (5) None of these Explanation : (42 - 28)% of x = 210 ? x = 210 x 100/14 = 1500 ? 59/100 * 15 = 885 22. What approximate value should come in place of the question mark (?) in the following question? 4275 496 x (21)2 = ? (1) 3795 (2) 3800 (Ans) (3) 3810 (4) 3875 (5) 3995 Explanation : Use BODMAS 23. A canteen requires 112 kgs of wheat for a week. How many kgs of wheat will it require for 69 days? (1) 1,204 kgs (2) 1,401 kgs (3) 1,104 kgs (Ans) (4) 1,014 kgs (5) None of these Explanation : 112/7 x 69 = 1104 kg = Reqd. quantity of wheat 24. If an amount of Rs 41,910 is distributed equally amongst 22 persons. How much amount would each person get? (1) Rs. 1,905 (Ans) (2) Rs. 2,000 (3) Rs. 1,885 (4) Rs. 2,105 (5) None of these 25. The cost of 4 Cell-phones and 7 Digital cameras is Rs 1,25,627. What is the cost of 8 Cell-phones and 14 Digital cameras? (1) Rs. 2,51,254 (Ans) (2) Rs. 2,52,627 (3) Rs. 2,25,524 (4) Cannot be determined (5) None of these Explanation : 4x + 7y = 125627] x 2 ? 8x + 14y = 251254 Q. 26-30. Each of the question below consists of a equation and two statement numbered I and II are given below it. You have to decide whether the data provided in the statements are sufficient to answer the question. Read both the statements and give answer: (1) if the data in Statement I alone are sufficeint to answer the question. while the data in Statement II alone are not sufficient to answer the question. (2) if the data in Statement II alone are sufficient to answer the question, while the data in Statement I alone are not sufficient to answer the question. (3) if the data in Statement I alone or in Statement II alone are sufficient to answer the question. (4) if the data in both the Statement I and Statement II are not sufficient to answer the question. (5) if the data in both the Statements I and II together are necessary to answer the question. 26. What is the area of the circle? I. Perimeter of the circle is 88 cms. II. Diameter of the circle is 28 cms. (Ans : 3) Explanation : 2pr = 88 ? r = 88/2pr r = D/2 = 28/2 = 14 Either (i) or (ii) is reqd. A = pr2 27. What is the rate of interest? I. Simple interest accrued on an amount of Rs. 25,000 in two years is less than the compound interest for the same period by Rs. 250. II. Simple interest accrued in 10 years is equal to the principal. (Ans : 3) Explanation : 250 = 25000 [(1 + R)2 -1] - 25000 x R x 2 or R = x * 100/ x * 10 = 10% From either of statement we can find R 28. What is the number of trees planted in the field in rows and columns? I. Number of columns is more than the number of rows by 4. II. Number of trees in each column is an even number. (Ans : 4) 29. What is the area of the right-angled triangle? I. Height of the triangle is three-fourth of the base. II. Diagonal of the triangle is 5 metres. (Ans : 5) Explanation : x2 + [3/4 x]2 = 52 ? x = 4, h = 3/4 * 4 = 3 Area = 1/2 (x * 3/4 x) = 1/2 (4 *3) = 6 sq. units Both (i) and (ii) statement are reqd. Image _ SBI _ Ans No. 29 30. What is the father's present age? I. Father's present age is five times the son's present age. II. Five years ago the father's age was fifteen times the son's age that time. (Ans : 5) Explanation : Let son's present age be x ? Father's present age = 5x ATS 5x - 5 = 15 (x - 5) ? x = 7 ? Father's present age = 35 yrs Q. 31-35. Study the following graph carefully to answer these questions: 31. What is the ratio between the profit earned by Company A in 2004 and the profit earned by Company B in 2003 respectively? (1) 4 : 3 (2) 3 : 2 (3) 3 :4 (4) 2 :3 (5) None of these (Ans) Explanation : 40/40 = 1/1 32. What is the difference (in Crore Rs.) between the total profit earned by Companies E, F and G together in 2003 and the total profit earned by these companies in 2004? (1) 70 (2) 75 (3) 78 (4) 82 (5) None of these (Ans) Explanation : Diff = (50 + 80 + 60) - (40 + 20 + 50) = 80 33. What is the ratio between the total profit earned by Company C in 2003 and 2004 together and the total profit earned by Company E in these two years respectively? (1) 11 : 9 (Ans) (2) 9 : 10 (3) 10 : 11 (4) 11 : 10 (5) None of these Explanation : Reqd ratio = 50 + 60 / 40 + 50 = 11/9 34. What was the average profit earned by all the companies in 2003? (In Crore Rs Rounded Off to two digits after decimal.) (1) 52.75 (2) 53.86 (3) 52.86 (Ans) (4) 53.75 (5) None of these 35. Profit earned by Company B in 2004 is what per cent of the profit earned by the same company in 2003? (1) 133.33 (2) 75 (Ans) (3) 67.66 (4) 75.25 (5) None of these Explanation : 30 = x% of 40 ? x = 75 Q. 36-40. Study the following table carefully to answer these questions: TABLE GIVING PERCENTAGE OF UNEMPLOYED MALE AND FEMALE YOUTH AND THE TOTAL POPULATION FOR DIFFERENT STATES IN 2005 AND 2006 STATE 2005 2006 M F T M F T A 12 15 32 7 8 35 B 8 7 18 10 9 20 C 9 10 28 10 12 34 D 10 6 24 8 8 30 E 6 8 30 7 6 32 F 7 5 28 8 7 35 M = Percentage of unemployed Male youth over total population F = Percentage of unemployed Female youth over total population T = Total population of the State in lakhs 36. What was the total number of unemployed youth in State A in 2006? (1) 2,20,000 (2) 3,25,000 (3) 5,20,000 (4) 5,25,000 (Ans) (5) None of these 37. How many female youth were unemployed in State D in 2005? (1) 14,400 (2) 1,44,000 (Ans) (3) 1,40,000 (4) 14,000 (5) None of these 38. Number of unemployed male youth in State A in 2005 was what per cent of the number of unemployed female youth in State E in 2006? (1) 66 (2) 50 (3) 200 (Ans) (4) 133 (5) None of these 39. What was the difference between the number of unemployed male youth in State F in 2005 and the number of unemployed male youth in State A in 2006? (1) 70,000 (2) 45,000 (3) 68,000 (4) 65,000 (5) None of these (Ans) Explanation : Ans 49000 40. What was the respective ratio between unemployed male youth in State D in 2005 and the unemployed male youth in State D in 2006? (1) 1 : 1 (Ans) (2) 2 : 3 (3) 3 : 2 (4) 4 : 5 (5) None of these Explanation : 10 * 24 / 100 / 8 * 30/100 = 1 : 1 SBI and SBT Data Intrepreation questions with answers for practice,SBI and SBT aptitude ,reasoning solved question papers,SBT and SBI free solved sample placement papers,SBT and SBI Associate banks PO and associate clerks practice questions with answers Detections (1-5): Study the following Table carefully and answer the questions given below: Numbered of Students Passed and Failed in Five Classes of A School Over the Years 101. What is the average number of failed students from class VII for the given years? (1) 27.5 (2) 28 (3) 26.5 (4) 26 (5) 24.5 (Ans) Explanations : The average number of failed students = 14 + 22 + 19 + 25 + 29 + 38/6 = 147/6 = 24.5 102. What is the ratio between total number of passed students and total number of failed students for the year 2006? (1) 3:1 (2) 56:23 (Ans) (3) 67:13 (4) 68:35 (5) 339:137 Explanations : Total number of passed students in 2006 = 336 Total number of failed students in 2006 = 138 SBI Solved reasoning questions papers SBI reasoning questions with answers,SBI previous years solved questions papers,SBI and SBT associate banke question papers Directions (1-7): Study the following information and answer the questions given below: Eight persons P, Q, R, S, T, U, V, W from two families are taking breakfast around a round table. Three members are from one family and five belong to other family. Four of them are male members. T, a male member is sitting second to right of V, a female member. In all cases R has same position with respect to S, who is second to left of Q, a female member. S is wife of W and is sitting adjacent to her husband. U is sister of W and is not sitting exactly between V and T. Q is immediate left of V. W is sitting immediate right of P. 1. Which of the given statement is wrong? (1) U is immediate right of S (2) T is immediate left of P (3) Q is sitting between U and V (4) R is second to left of W-answer (5) V is second to right of U 2. Who is second to the left of P? (1) Q (2) V (3) R -answer (4) S (5) U 3. How many persons are sitting between P and Q when we count anticlockwise? (1) 2 (2) 3 -answer (3) 4 (4) None (5) Cannot be determined 4. How many members are there in W’s family? (1) 2 (3) 3 (3) 5 (4) Cannot be determined-answer (5) None of these Directions (5-7): One out of five groups does not match on the basis of sitting arrangement. Find that group- 5.(1) USQ (2) RVT (3) TRP (4) WPS (5) None of these-answer 6. (1) TW (2) PS (3) WU (4) SQ (5) TV-answer 7. (1) V (2) Q (3) S (4) U (5) P-answer Directions (8-12): Study the following information to answer the given questions: In a certain code ‘ge ji zo’ means ‘had horrible dream’, ‘lit zo pit’ means ‘realize your dream’ and ‘ge ze pat ze’ means ‘very very horrible experience’. 8. Which of the following is the code for ‘your’? (1) lit (2) zo (3) pit (4) Cannot be determined-answer (5) None of these 9. ‘ji ze pit lit’ may represent- (1) very horrible you realize (2) you had realize your (3) had realize your very -answer (4) your very realize dream (5) your very very had 10. ‘dream had horrible experience’, can be coded as: (1) zo ge ji ze (2) pat ge zo ji-answer (3) zo ji ge pit (4) Cannot be determined (5) None of these 11. Which of the following is the code for ‘very’? (1) ge (2) pat (3) ze -answer (4) Cannot be determined (5) None of these 12. Which of the following is the code of’ had’? (1) ge (2) ji-answer (3) zo (4) Cannot be determined (5) None of these Directions (13-17): In each of the questions /set of questions below are given two statements followed by two conclusions numbered I and II. You have to assume everything in the statements to be true even if they seem to be at variance from commonly known facts and then decide which of the two given conclusions logically follows from the information given in the statement. Give answer- (1) If only Conclusion I follows. (2) If only Conclusion II follows. (3) If either Conclusion I or II follows. (4) If neither Conclusion I nor II follows. (5) Both Conclusions I and II follow. 13. Statements: All doors are windows. Some windows are clips. 5-answer Conclusions: I. Some clips, if they are doors, they are also windows. II. All chips which are not windows are also not doors. 14-15 Statements: No shoe is a chappal. Some chappals are sandals. Conclusions: I. Some sandals are not chappals. II. Sandals which are not chappals are shoes. 1-answer 15. Conclusions: I. No sandal is a shoe. II. Sandals which are chappals are not shoes. 2-answer 16. Statements: Some paints are red. All red which are paints are yellow. Conclusions: I. Some paints are yellow. II. Some yellow are red. 5-answer 17. Statements: All sets are hot. All belts are hot. Conclusions: I. At least some seats are belts. II. All hot are either seats or belts. 2-answer Directions (18-22): An arrangement machine when given an input of words, rearrange them following a particular rule and solve it. Input: this chapter would give you basic concepts. Step I: basic this chapter would give you concepts. Step II: basic chapter this would give you concepts. Step III: basic chapter concepts this would give you. Step IV: basic chapter concepts give this would you. Study the logic and answer the questions that follow: 18. Input: adequate safety arrangements are must for kids. Which of the following will be 2nd last step for the given input? (1) adequate are arrangements for kids safety must.-answer (2) adequate are arrangements for kids must safety. (3) adequate are safety arrangements must for kids. (4) adequate safety are arrangements must for kids. (5) None of these 19. Input: do not imitate others work. Which of the following will be step 3 for the given input? (1) do others not imitate work. (2) do imitate not others work. (3) work do others not imitate. (4) do not others work imitate. (5) There should be no 3rd step.-answer 20. Input: Marcus Brauchli gives inaccurate information. How many steps are required to rearrange the above input properly? (1) 3 (2) 4-answer (3) 5 (4) 2 (5) None of these 21. Input: most Indian leaders cannot even visualize free India. Which of the following will be last step of the above input? (1) most Indian even leaders cannot visualize free India. (2) cannot even free India Indian most leaders visualize. (3) cannot even free India most Indian leaders visualize. (4) cannot even free India Indian leaders most visualize.-answer (5) None of these 22. Step IV: it is figure anti-establishment from media. Which of the following is the input of the above mentioned step? (1) it is figure from anti-establishment media. (2) media it is figure from anti-establishment. (3) it is anti-establishment media from. (4) Can’t be determined.-answer (5) None of these Directions (23-27): Study the following arguments and distinguish between ‘strong’ arguments and ‘weak’ arguments because they relate to the question. ‘Strong’ arguments are these which are both important and directly related to the question. ‘Weak’ arguments are those which are of minor importance and also not directly related to the question. Each question below is followed by two arguments numbered I and II. Decide which of the argument is a ‘strong’ and ‘weak’. Give answer- (1) If only argument I is ‘strong’. (2) If only argument II is ‘strong’ (3) If either I or II is ‘strong’. (4) If neither I nor II is ‘strong’. (5) If both I and II are ‘strong’. 23. Statement: Should the government punish those schools which charge high admission fee? Arguments: I. Yes, where will the poor students go? II. Yes, education has become more of a business than a pious commitment. 2.-answer 24. Statement: Should there be a quota for those who are poor? Arguments: I. Yes, This will decrease the level of unemployment and poor people will not be discriminated by upper caste. II. No, we already have reservations in our jobs. 1.-answer 25. Statement: Should there be no place for interview in selection? Arguments: I. Yes, it is a difficult part of selection. II. No, it is the only way to judge the candidates personality and motives. 2.-answer 26. Statement: Should women be provided more job opportunities? Arguments: I. Yes, they should go into the outside world. II. No, they are not interested in jobs. 1.-answer 27. Statement: Should government provide more benefits to those who are the only child of their parents? Arguments: I. Yes, it will control increase of population. II. Yes, it will given them bright future and love of parents. 4.-answer Directions (28-31): In these questions, a statement is given, which is followed by various assumptions. Read the statements and the assumptions and decide which one of them is implicit. 28. Statement: “Ensure freedom from thieves with this car locking system”. Assumptions: I. This car locking system is the best. II. It is desired to have freedom from thieves. III. There are thieves everywhere. (1) I and II are implicit (2) II and III are implicit (3) I and III are implicit .-answer (4) only II is implicit (5) All I, II and III are implicit 29. Statement: “We deal in used cars. Contact us at phone no. XYZ, at the earliest possible” – an advertisement. Assumptions: I. Some people want to sell old cars. II. The advertisement will be read by the needy people. III. Used cars may not be usually useless. (1) Only I is implicit (2) Only II and III are implicit (3) Only I and III are implicit (4) All I, II, III are implicit.-answer (5) None of I, II, III are implicit. 30. Statement: “Lalu Prasad is expected to announce several schemes for poor people in the budget” – a news reporter. Assumptions: I. The reporter has a fair reporting. II. The news-reporter has genuine report sources. III. Lalu Prasad is capable of announcing schemes. (1) I and II are implicit (2) II and III are implicit.-answer (3) Only III is implicit (4) All are implicit (5) None is implicit 31. Statement: Mr. X tells Mrs. X: “I cannot send my child to that school. Children over there smoke and drink.” Assumptions: I. Smoking and drinking are not desirable of children..-answer II. Their child will agree to their decision. III. The school has a good reputation. (1) I and II are implicit .-answer (2) II and III are implicit (3)III and I are implicit (4) only I is implicit (5) All I, II, III are implicit Directions (32-36): Each of the questions below consists of a question and two statements numbered I and II given below it. You have to decide whether the data provided in the statements are sufficient to answer the question. Read both the statements and give answer- (A) If the data in statement I alone are sufficient to answer the question, while the data in statement II alone are not sufficient to answer the question. (B) If the data in statement II alone are sufficient to answer the question, while the data in statement I alone are not sufficient to answer the question. (C) If the data in statement I alone or in statement II alone are sufficient to answer the question. (D) If the data in both the statement I and II together are necessary to answer the question. (E) If the data in both statements I and II are not sufficient to answer the question. 32. How many sisters does Priya have? I. Rohan, Priya’s brother has only two siblings. II. Priya’s grandparents have two children and two granddaughters. 4.-answer 33. On which day of the week was Shahid born? I. Shashid’s mother correctly mentions that he was born before Friday and after Tuesday. II. Shahid’s after correctly remembers that Shahid was not born on a Wednesday. 4.-answer 34. Who is the haviest amongst P, Q, R, S and T? I. R is heavier than Q but not as heavy as T II. P is heavier only than S. 4.-answer 35. Is the time in the clock 3 o’clock now? I. After fifteen minutes, the minute and the hour hands of the clock will make a straight line. II. The train which is running late by exactly three hours from its scheduled time of arrival i.e. 11 am has reached now. 5.-answer 36. How is got coded in the language? I. ‘I got good marks’ is written as ‘pe lit a so’ and ‘she has got cold’ is written as ‘je ra so ki’ in the code language. II. ‘What have you got’ is written as ‘de wd so me’ and ‘Brazil got four goats’ is written as ‘di bi fe so’ in the code language. 3.-answer 37. SINK is related to FLOAT in the same way as a DESTROY is related to……. (1) ENEMY (2) DEMOLISH (3) ALIVE (4) PEACE (5) CREATE.-answer 38. Four of the following five are alike in a certain way and hence form a group which one does not belong to the group? (1) BLOCK (2) HAMPER (3) IMPEDE (4) AVOID.-answer (5) OBSTRUCT 39. This of the following figures represents the relation between ‘Gas’ water and helium? 3.-answer 40. What will come in place of question mark (?) in the following series? AC DG HL MR ? (1) SY .-answer (2) TZ (3) RX (4) SX (5) SZ 41. If each odd digit in the number 813246 is multiplied by three and each even digit is divided by two, what will be the difference between the digit which are third from the left and third from the right of the new number thus formed? (1) 1 (2) 2 (3) 3 (4) 7 (5) 8.-answer 42. If each alphabet in the word ABSOLUTE is arranged in alphabetical order (from left to right) and then each vowel is changed to next letter in the English alphabetical series and each consonant is changed to previous letter in English alphabetical series. Which of the following will be sixth from the left side of the new arrangement thus formed? (1) F (2) R.-answer (3) V (4) S (5) K 43. How many such pairs of letters are there in the word SAMPLER each of which has as many letters between them in the word (in both forward and backward direction) as they have between them in the English alphabetical series? (1) None (2) One (3) Two .-answer (4) Three (5) More than three 44. In a certain code ‘SHORE’ is coded as ‘QFMPC’ in the same code ___ will be coded as ‘WNDGL’. (1) NIMPY (2) YPMIN .-answer (3) ULIEJ (4) ULIJE (5) JEILU 45. The question given below is based on the given diagram. You have to take the given diagram to be true even if it seem to be at variance from commonly known facts and then decide which of the five alternatives following question logically follows from the given diagram. 3.-answer Directions (46-50): Each of the following questions consist of five figures marked (a), (b), (c), (d) and (e) called the Problem Figures followed by five other figures marked (1), (2), (3), (4) and (5) called the Answer Figures. Select a figure from amongst the Answer Figures which will continue the same series as established by the five Problem Figures. 46-2.-answer 47-4.-answer 48-2.-answer49-2.-answer50-1.-answer SBI reasoning and aptitude questions with answers SBT and SBI Assistant clerks written test Reasoning aptitude Data Interpretation Data sufficiency questions with answers, SBT and SBI free solved questions with answers and explanations, All Banks IBPS PO clerical CWE,IBPS RRBs gramine banks written test examination and questions with answers detailed solutions Bank based computer knowledge, general awareness, current affairs economy questions with answers.... 1. 'Temporary' is related to 'Permanent' in the same way as 'Natural' is related to '__________ '. (A) Nature (B) Artificial -Answer (C) Imaginary (D) Fraud (E) Genuine 2. In a class of 25 students, Sneha's rank is 19th from the bottom and Meena's rank is 23rd fromthe top. If Chhaya's rank is exactly between Sneha and Meena's rank, what is Chhaya's rank from the bottom? (A) 15th (B) 12th (C) 14th (D) 11th -Answer (E) Cannot be determined 3. Four of the following five are alike in a certain way and hence form a group. Which of thefollowing does not belong to that group? (A) 9 -Answer (B) 216 (C) 8 (D) 64 (E) 512 4. How many meaningful English words can be formed with the letters 'EPA' using all the letters but each letter only once in each word? (A) None (B) One (C) Two -Answer (D) Three (E) More than three 5. Four of the following five are alike in a certain way and hence form a group. Which of the following does not belong to that group? (A) Meter (B) Litre -Answer (C) Yard (D) Feet (E) Mile 6. If each vowel in the word 'REMAINS' is changed to next alphabet according to the English alphabetical series and each consonant changed to previous letter, how many alphabets will be there in the English alphabetical series between the letters which are at the extreme ends of the new word thus formed? (A) None -Answer (B) One (C) Two (D) Three (E) Four 7. Pointing to a man's photograph Ajay said "my mother is the only daughter of his father." How is the man in the photograph related to Ajay? (A) Uncle -Answer (B) Brother (C) Son (D) Grandfather (E) Cannot be determined 8. In a certain code language, 'NICE' is coded as 'OIDE', similarly 'WORK' is coded as 'XOSK'. How will 'SAID' be coded in the same code language? (A) TIJD (B) TBJD (C) SBIE (D) TAJD -Answer (E) TBID 9. Which of the following groups of letters should replace the blank spaces (in the same order from left to right) so that the group of letters, given in bold, follow a logical pattern from the preceding and the following group of letters? (A) b, y, e (B) x, y, g (C) b, y, g (D) x, y, e (E) b, w, e -Answer 10. How many such pairs of letters are there in the word 'LENGTH', each of which has as many letters between them in the word (in both forward and backward directions as they have between them in the English alphabetical series? (A) None (B) One (C) Two -Answer (D) Three (E) More than three Directions-(Q.11-15) Study the following information to answer the given questions- Eight friends A, B, C, D, E, F, G and H are sitting around a circle facing the centre. Four out of the eight friends drive a car while four drive a bike. No two friends who drive a car are immediate neighbours of each other. A, who drives a car sits third to left of H. G, who drives a bike sits second to right of E. F and C are immediate neighbours of each other. F does not drive a car. B is an immediate neighbour of F butnot of H. 11. Four of the following five are alike in a certain way based on the above arrangement and so form a group. Which is the one that does not belong to the group? (A) B (B) E -Answer (C) A (D)C (E) D 12. If all the eight friends are made to sit in alphabetical order in the anti-clockwise direction starting from A, positions of how many will remain unchanged (excluding A)? (A) None (B) One -Answer (C) Two (D) Three (E) Four 13. In which of the following pairs do both the members drive a bike? (A) F, D (B) B, D (C) F, C (D) H, B (E) E, H -Answer 14. Which of the following is true about D? (A) D drives a bike (B) H and E are immediate neighbours of D (C) H sits third to left of 0 (D) D sits second to the left of C -Answer (E) None is true 15. What is F's position with respect to E? (A) Immediate right (B) Third to the right (C) Second to the left -Answer (D) Fourth to the left (E) Second to the right Directions-(Q.26-27) Study the following information carefully and answer the given questions- Among M, R, L, N and J each scoring different marks in an examination, R scored more than M. N scored less than L but more than J. J scored more than R. 26. Who amongst them scored the least marks? (A) R (B) M -Answer (C) J (D) N (E) L 27. Whose score was the third highest? (A) R (B) M (C) J -Answer (D) N (E) L 28. If each vowel of the word PERTAIN is changed to the next letter in the English alphabetical series and each consonant is changed to the previous letter in the English alphabetical series and then the alphabets thus formed are arranged in alphabetical order from left to right, which of the following will be third from the left? (A) O (B) Q (C) M (D) B (E) J -Answer 29. How many such pairs of letters are there in the word THINGS, each of which has as many letters between them in the word (in both forward and backward directions) as they have between them in the English alphabetical order? (A) One (B) Two -Answer (C) Three (D) Four (E) Five 30. Four of the following five are alike in a certain way and so form a group. Which is the one that does not belong to that group? (A) Drop (B) Slip (C) Bounce -Answer (D) Topple (E) Fall Directions-(Q.31-35) Study the following information carefully and answer the given questions- A, B, C, D, E, F and G are sitting in a straight line (not necessarily in the same order) facing North. • E sits third to right of A. E sits second to left of F. • Only one person sits between G and B. Neither G nor B is an immediate neighbour of F. • C is not an immediate neighbour of either B or F. 31. Who sits at the extreme right end of the line? (A) G (B) F -Answer (C) B (D) C (E) E 32. How many persons sit between A and C? (A) None (B) One -Answer (C) Two (D) Three (E) Four 33. Four of the following five are alike in a certain way based on their seating positions in the above arrangement and so form a group. Which is the one that does not belong to that group? (A) GC (B) DF (C) BA (D) DE -Answer (E) AG 34. What is the position of D with respect to C? (A) Third to the left (B) Second to the right -Answer (C) Immediate right (D) Second to the left (E) Immediate left 35. If all the persons are made to sit in alphabetical order from left to right, the positions of how many will remain unchanged as compared to the original seating positions? (A) None (B) One -Answer (C) Two (D) Three (E) Four Directions-(Q.36-40) Study the following information carefully and answer the given questions- P, Q, R, S, T, V, W, and Yare sitting around a circular table facing the centre. • P sits second to left of T • W sits third to right of Y. Y is not an immediate neighbour of T. • Q and R are immediate neighbours of each other but neither Q nor R is an immediate neighbour of Y. • V is an immediate neighbour of R. 36. Who sits exactly between P and Y? (A) W (B) T (C) V (D) Q (E) None of these -Answer 37. How many persons sit between T and P when counted in clockwise direction from T? (A) None (B) One -Answer (C) Two (D) Three (E) Four 38. Four of the following five are alike in a certain way based on their seating positions in the above arrangement and so form a group. Which is the one that does not belong to that group? (A) WP (B) VR (C) QT (D) VY -Answer (E) PS 39. What is the position of Y with respect to Q? (A) Fourth to the left (B) Immediate to the right (C) Third to the left (D) Second to the left (E) Third to the right -Answer 40. What will come in the place of the question mark (?) based upon the given seating arrangement? RV SP QR PW? 11 (A) YV (B) VY (C) PW (D) WT (E) YS -Answer Directions-(Q.41-45) Study the following information to answer the given questions- Seven friends -P, Q, R, S, T, V and Ware sitting in a straight line facing North, not necessarily in the same order. T sits at one of the21. extreme ends of the line. Q sits third to left of T. Only one person sits between Sand W. S is an immediate neighbour of Q. P is not an immediate neighbour of T and S. P and R do not sit at the extreme ends of the line. 41. What is R's position with respect to P? (A) Second to the right (B) Fourth to the right -Answer (C) Second to the left (D) Third to the left (E) None of these 42. Four of the following five are alike in a certain way based on their seating positions in the above arrangement and so form a group. Which is the one that does not belong to the group? (A) QS (B) VP (C) TR -Answer (D) PW (E) WQ 43. If all the seven friends are made to sit in alphabetical order from right to left, the positions of how many will remain unchanged? (A) Four (B) Three (C) One (D) Two (E) None -Answer 44. Who sits exactly in the middle of the line? (A) P (B) W (C) S (D) Q -Answer (E) R 45. Who amongst the following sits to the immediate left of W? (A) Q (B) P -Answer (C) R (D) S (E) T Directions-(Q.46-50) Study the following arrangement carefully and answer the questions given below- A $ H * J R S 2 & I T 6 @ Q B K F G U M I © # % E 4 N 5 9 D S 3 L W 7 46. Which of the following is fifth to the left of the sixteenth from the left end of the above arrangement? (A) 4 (B) I (C) E (D) 2 (E) None of these -Answer 47. What should come in place of the question mark (?) in the following series based on the above arrangement? H* RS IT BK? (A) ©# (B) MI (C) #% (D) I© (E) None of these -Answer 48. If all the letters are dropped from the arrangement, which of the following will be the ninth from the right end? (A) # (B) @ -Answer (C) © (D) 6 (E) None of these 49. Four of the following five are alike in a certain way based on their positions in the above arrangement and so form a group. Which is the one that does not elong to the group? (A) AJH (B) G©M (C) TB@ (D) 5DL -Answer (E) %54 50. How many such numbers are there in the above arrangement, each of which is immediately preceded by vowel and also immediately followed by a consonant? (A) None (B) One -Answer (C) Two (D) Three (E) More than three Test-I: Reasoning Ability 1. Four of the following five are alike in a certain way and so form a group. Which is the one that does not belong to that group? 1)215 2)143 3)247 4)91 5 )65 2. In a certain code CROWNED is written as PSDVEFO.How is STREAMS written in that code? 1) SUTDBNT 2) TUSDTNB 3)SUTFTNB 4)QSRDTNB 5) None of these 3. 'DEAN' is related to 'NDAE' and -ROAD' is related to 'DRAO' in the same way as'SOME' is related to l)ESMO 2) EOMS 3)EMOS 4)MSEO 5) None of these 4. In a row of thirtyfive children, M is fifteenth from the right end and there are ten children between M and R. What is R's position from the left end of the row? 1) 15th 2) 5th 3) 30th 4) Data inadequate 5) None of these 5. Four of the following five are alike in a certain way and so form a group. Which is the""one that does not belong to that group? DGold 2) Nickel 3) Platinum 4) Tungsten 5) Diamond 6. Among P, Q, T. A and B, each having a different height,T is taller than P and B but shorter than A and Q. P is not the shortest. Who among them is the tallest? 1)A 2)Q 3)P 4) P or B 5 ) Data inadequate 7. How many meaningful English words can be made with the letters RBAE, using each letter only once in each word? 1) None 2) One 3) Two 4) Three 5) More than three 8. In acertain code TRAIN is written as'39?7%'and MEALis written as ' 4 $?@'. How is ITEM written in that code? 1)7534 2)73$4 3)79$4 4)73H4 5) None of these 9. How many such pairs of letters are there in the word OBJECTIVE each of which has as many letters between them in the word as in the English alphabet? 1) None 2) One 3) Two 4) Three 5) More than three 10. If each consonant in the word TOLERANT is replaced by the previous letter in the English alphabet and each vowel in the word is replaced by the next letter is the English alphabet and the new set of letters is arranged alphabetically, which of the following will be the fourth from the right end after the replacement ? 1)M 2)P 3)Q 4)K. 5) None of these 11. How many such pairs of digits are there in the number 8429516 each of which has as many digits between them in the number as when the digits are arranged in ascending order within the number? 1) None 2) One ) Two 4) Three 5) More than three 12. The positions of the first and the second digits in the number 85341279 are interchanged. Similarly, the positions of the third and the fourth digits are interchanged, and so on, till the positions of the seventh and the eighth digits. Which of the following will be the third to the right of 3 after the rearrangement? 1)9 2)7 3)8 4)2 5) None of these 13. In a certain code language 'over and above' is written as 'da pa ta' and 'old and beautiful' is written as 'sa na pa'. How is 'over' written in that code 'language? l)da 2)ta 3)na 4)daorta 5) None of these 14. What should come next in the following letter series? H G F E D C B A G F E D C B A F E D C B A 1)E 2)G 3)F 4)B 5) None of these 15. Mohan correctly remembers that his father's birthday is before twentieth January but after sixteenth January,whereas his sister correctly remembers that their father's birthday is after eighteenth January but before twentythird January. On which date in January is definitely their father's birthday? 1) Eighteenth 2) Nineteenth 3) Twentieth 4) Data inadequate 5) None of these 16. If'P denotes '?' ; 'R' denotes ' + ' ; 'M' denotes '-'; and 'W denotes'+', then 2 0 R 5 W 1 2 M 3 P 4 = ? 1)4 2)16 3)28 4)52 5) None of these Directions (Q. 17-20): Following questions are based on the five three-digit numbers given below: 832 719 654 967 481 17. If the positions of the second and the third digits within each number are interchanged, which of the following will be the sum of the first and the second digits of the third highest number? 1)16 2)10 3)9 4)15 5) None of these 18. Which of the following is the sum of the first and the third digits of the second lowest number ? 1)16 2)10 3)18 4)5 5)Noneofthese 19. If the positions of the first and the second digits within each number are interchanged, which of the following will be the difference between the highest and the second highest number? 1)203 2)133 3)385 4)182 5)144 20. If the positions of the first and the third digits within each number are interchanged, which of the following will be the sum of the second and third digits of the lowest number? 1)8 2)11 3)15 4)12 5) None of these Directions (Q. 21-27): In each of t h e q u e s t i o n s below a r e given four s t a t e m e n t s followed by t h r e e conclusions numbered I, II & 111. You have to t a k e t h e given statements to be t r u e even if t h ey seem to be at v a r i a n c e with commonly known facts. Read all t h e conclusions and t h e n decide which of t h e given conclusions logically follows from the given statements, disregarding commonly known facts. 21. Statements: All chairs are keys. All keys are balloons. Some balloons are mirrors. Some mirrors are desks. Conclusions: I. Some desks are keys. II. Some balloons are chairs. III. Some mirrors are balloons. 1) Only I follows 2) Only II follows 3) Only I I I follows 4) Only I I and I I I follow 5) All I , I I and I I I follow 22. Statements: Some drums are posters. All posters are windows. Some windows are tablets. All tablets are books. , Conclusions: I. Some windows are drums. II. Some books are posters. III. Some tablets are drums. 1) None follows 2) Only I follows' 3) Only II follows 4) Only I I I follows 5) Only I and II follow 23. Statements: Some boxes are toys. Some toys are nails. Some nails are stores. Some stores are shops. Conclusions: I. Some shops are toys. II. Some nails are boxes. III. No shop is toy. 1) Only I follows 2) Only I I I follows 3) Only either I or I I I follows 4) Only II follows 5) None of these 24. Statements: All doors are windows. No window is house. Some houses are buildings. All buildings are skies. Conclusions: I. Some skies are doors. II. Some skies are houses. III. Some buildings are doors; I ) Only I follows 2) Only I I follows 3) Only I I I follows 4) Only I I and I I I follow 5) None of these 25. Statements: All rivers are walls. All walls are stones. All stones are clothes. All clothes are trees. Conclusions: I. Some trees are stones. II. Some clothes are rivers. III. All walls are clothes. 1) Only 1 and II follow 2) Only I and I I I follow 3)Only I I and I I I follow 4) All I , I I and I I I follow 5) None of these 26. Statements: Some letters are glasses. Some glasses are plates. All plates are buses. All buses are cars. Conclusions: I. Some cars are letters. II. Some cars are glasses. I I I . Some buses are glasses. 1) Only 1 and II follow 2) Only I and III follow 3) Only II follows 4) Only III follows 5) Only II and III follow 27. Statements: All books are pens. Some pens are ropes. All ropes are discs. Some discs are bricks. Conclusions: I. Some bricks are ropes. II. Some discs are books. I I I . Some bricks are pens. 1) None follows 2) Only I follows 3) Only II follows 4) Only 111 follows 5) Only II and III follow Directions (Q. 28-34): Study the following arrangement carefully a n d answer t h e questions given below: B 4 @ D A © 7 9 F % 2 R - 5 H 6 E * N $ 1 U W 3 P T 8 8 V # Y I 28. Which of the following is the twelfth to the left of the twenty-first from the left end of the above arrangement? 1)R 2)1 3)5 4)$ 5) None of these 29. If all the numbers from the above arrangement are dropped, which of the following will be the fifteenth from the right end? 1 )W 2)6 3)* 4)R 5) None o f these 30. How many such consonants are there in the abovearrangement, each of which is immediately preceded by a symbol and immediately followed by a vowel? l ) N o n e 2)One 3)Two 4)Three 5)Four 31. In which of the following combinations the first elementis in between the second and the third elements in the above arrangement? 1) 2 % F 2^9©7 3 ) 5 H 6 4 ) T P 3 5 ) * E N 32. Four of the following five are alike in a certain way based on their positions in the above arrangement and so form a group. Which is the one that does not belong to that g r o u p 9 1 ) 5 6 2 2)WPU 3 ) © 9 D 4 ) 6 # T 5)U3$ 33. How many such symbols are t h e r e in the above arrangement, each of which is immediately preceded by a consonant and immediately followed by a number? 1) None 2) One ) Two 4) Three 5) More than three 34. How many such numbers are there in the above arrangement, each of which is immediately preceded by a letter and immediately followed by a symbol? 1) None 2) One 3) Two 4) Three 5) More than three Directions (Q. 35-41): In each question below is given aletter g r o u p followed by four combinations of digits/symbols numbered 1), 2), 3) a n d 4). You have to find out which of t h e combinations correctly represents the g r o u p of letters based on the following coding system and t h e conditions that follow and m a r k the number of t h a t combination as your answer. If none of t h e combinations c o r r e c t l y represents the g r o u p of l e t t e r s , m a r k 5), i e ' N o n e of t h e s e ' a s y o u r answer. L e t t e r : P M A D E J 1 TO U O F H W B D i g i t / S y m b o l : 6 S 7 1 % 2 5 8 3 © 4 @ 9 5 ? Conditions: (i)If the first letter is a consonant and the last letter is a vowel, their codes are to be interchanged. (ii) If the first letter is a vowel and the last letter is a consonant, both are to be coded as the code for the last letter. (iii) If both the first and the last letters are consonants, both are to be coded as ' Directions (Q. 42-48): Study the following information carefully and answer the given questions: A word and number arrangement machine when given an input line of words and numbers rearranges them following a particular rule in each step. The following is an illustration of input and rearrangement. Input joy far 35 27 16 96 heij >ht star Step I 96 joy far 35 27 16 heij jht star Step J1 % far joy 35 27 . 16 heij ;ht star Step III 96 far 35 joy 27 16 heij iht star Step IV % far 35 height joy 27 16 star StepV 96 far 35 height 27 joy 16 star And Step V is the last step of the rearrangement. As per the rules followed in the above steps, find out in each of the following questions the appropriate step for the given input. 42. Input: organize 19 12 stable room 35 72 house How many steps will be required to complete the arrangement? l)Five 2) Six 3) Seven 4) Four 5) None of these Directions (Q. 49-55): In the following questions, the symbols $, %, @, © and ? are used with the following meaning as illustrated below: ' P % Q' means 'Pis neither greater than nor smal ler than Q'. 'P $ Q' means 'P is neither smaller than nor equal to Q'. 'P ©Q' means 'P is neither greater than nor equal to Q'. 'P ? Q' means 'P is not greaterthan Q'. 'P @ Q' means 'P is not smaller than Q'. Now in each of the following questions, assuming the given statements to be true, find which of the three conclusions I, II and III given below them is/are definitely true and give your answer accordingly. 49. Statements :V©K, K@B, B $ M Conclusions: I. V © B II. M© K III. M©V 1) None is true 2) Only I is true 3) Only II is true 4) Only III is true 5) Only II and III are true 50. Statements: D ? R, R % F, F $ T Conclusions: I. F % D II. F $D III.T©R 1) Only I is true 2) Only II is true 3) Only III is true 4) Only either I or II is true 5) Only either 1 or II and III are true 51. Statements: N @ D, D?K, K $ A Conclusions: I. K@N II. A©D HI.N$A 1) None is true 2) Only I is true 3) Only II is true 4) Only III is true 5) Only II and III are true 52. Statements: K@T, T $ N, N © R Conclusions: I. R $ K II. N ? K III. K $ N 1) None is true 2) Only I is true 3) Only I is true 4) Only III is true 5) Only II and III are true 53. Statements: W%K, K © F, D $ F Conclusions: I. D$K II. D $W III.F@W 1) Only 1 and III are true 2) Only I and II are true 4) All 1, II and III are true 5) None of these 54. Statements: B ? K., K © F, F % R Conclusions: I. R $ K II. R $ B III. F S B 1) Only I and II are true 2) Only I and III are true 3) Only II and III are true 4) All I, II and III are true 5) None of these 55. Statements: H $ M, M % D , D@K. Conclusions: I. H $ D II. K ? M III. K© H l)Only II is true 2) Only II and III are true 3) Only I and III are true 4) Only I and II are true 5) AH I, II and III are true Directions (Q. 56-60): Study the following information carefully and answer the questions given below: A, M, D, P, R, T, B and H are sitting around a circle, facing the centre. M is third to the left of A, who is second to the left of T. D is second to the right of H, who is second to the right of T. R is second to the right of B, who is not an immediate neighbour of T. 56. Which of the following combinations represents the first and the second to the left of B respectively? 1)MD 2)DH 3)AM 4)AR 5)DM 57. Who is third to the right of T? 1)D 2)B 3)H 4)M 5) None of these 58. Who is to the immediate left of H? 1)P 2)M 3)T )R 5) Data inadequate 59. Who is second to the left of B? 1)D 2)H 3)M 4) Data inadequate 5) None of these 60. In which of the following combinations the third person is second to the left of the second person? 1)BAR 2) DBM )TPH 4)PMH 5) None of these Directions (Q. 61-70): Study the following information carefully and answer the questions given below: Following are the conditions for selecting Manager- Finance in an organization: The candidate must i) be at least 30 years and not more than 35 years as on 1.11.2009. (ii) be a graduate in any discipline with at least 55 percent marks. (iii) be a postgraduate degree/diploma holder in Management with Finance specialisation with at least 60 per cent marks. (iv) have post-qualification work experience of at least six years in the Finance department of anorganisation. (v) have secured at least 50 per cent marks in the preliminary interview. (vi) have secured at least 40 per cent marks in the final interview. In the case of a candidate who satisfies all the above conditions EXCEPT (a) at (iv) above, but has post-qualification workexperience of at least three years as Deputy Finance Manager in an organisation, his/her case is to be referred to VP-Finance. (b) at (vi) above, but has obtained at least 60 per cent marks in the preliminary interview, his/her candidature is to be considered under 'wait list'. In each question below, details of one candidate are given. You have to take one of the following courses of action based on the information provided and the conditions and subconditions given above and mark the number of that course ofaction as your answer. You are not to assume anything other than the information provided in each question. All these cases are given to you as on 1.11.2009. Mark answer (1) if the candidate is to be selected. Mark answer (2) if the candidate is not to be selected. Mark answer (3) if the candidate is to be kept on waiting list. Mark answer (4) if the case is to be referred to VP-Finance. Mark answer (5) if the data provided are not adequate to take a decision 61. Neelam Johri has secured 38 per cent marks in the final interview. She has also secured 65 per cent marks in bothB. Com. and postgraduate degree in Finance Management.She has been working in the Finance department of an organisation for the past six years after completing her postgraduate degree. She was born on 16th August 1978. She has secured 63 per cent marks in the preliminary interview. 62. Anirban Chowdhury was born on 8th March 1978. He has secured 65 per cent marks in BSc and 62 per cent marks in postgraduate degree in Finance Management. He has been working in the Finance department of a company for the past seven years after completing his postgraduation. Hehas secured 50 per cent marks in the final interview and 40 per cent marks in the preliminary interview. 63. Vaibhav Joshi has secured 60 per cent marks in both graduation and postgraduate diploma in Finance Management. He has been working as Deputy Finance Manager in an organisation for the past four years after completing his post-graduate diploma. He has secured 53 per cent marks in the preliminary interview and 43 per cent marks in the final interview He was born on 3rd July 1977. 64. Sudha Motwani was born on 24th March 1977. She has secured 58 per cent marks in BA and 68 per cent marks in postgraduate diploma in Finance Management. She has been working in the Finance department of an organisation for the past eight years after completing her post• graduation. She has secured 50 per cent marks in both preliminary and final interviews. 65. Ashok Chandra has been working in the Finance department of an organization for the past seven years after completing his postgraduate diploma in management with Finance specialization with 65 per cent marks. He has secured 55 percent marks in the preliminary interview and 45 per cent marks in the tlnal interview. He was born on 12th April 1976. 66. Suparna Desai has secured 5 8 per cent marks in graduation and 68 per cent marks in postgraduate diploma in Finance Management. She has been working as Deputy Finance Manager in an organization for the past four years after completing her postgraduate diploma. She has secured 50 percent marks in preliminary interview and 45 percent marks in the final interview. She was born on 26th August 1977. 67. Balwant Singh was born on 18th November 1979. He has secured 60 per cent marks in B Com and 55 per cent marks in post graduate degree in Finance Management. He has been working as Deputy Finance Manager in an organisation for the past five years after completing his postgraduate degree. He has secured 54 per cent marks in preliminary interview and 44 per cent marks in final interview. 68. Abhishek Saha was born on 8th October 1978. He has been working in the Finance department of an organisation for the past six years after completing his postgraduate diploma in management with 68 per cent marks. He has secured 63 per cent marks in B Sc. He has also secured 60 percent marks in both preliminary and final interviews. 69. Shikha Rastogi was born on 11th April 1976. She has secured 68 per cent marks in graduation and 62 per cent marks in postgraduate degree in Finance Management.She has secured 65 per cent marks in preliminary interview and 35 per cent marks in final interview. She has been working in the Finance department of an organisation for the past seven years after completing her postgraduate degree. 70. Navin Shukla has secured 60 per cent marks in graduation and 64 per cent marks in postgraduate diploma in Finance Management. He has been working in the Financedepartment of a company for the past six years after completing his postgraduate diploma. He has secured 56 per cent marks in the preliminary interview and 45 per cent marks in the final interview. He was born on 28thMay 1976. Directions (Q. 71-75): Below is given a passage followed by several possible inferences which can be drawn from the facts stated in the passage. You have to examine each inference separately in the context of the passage and decide upon its degree of truth or falsity. Mark answer (1) if the inference is 'definitely true', ie it properly follows from the statement of facts given. Mark answer (2) if the inference is "probably true" thoughnot 'definitely true' in the light of the facts given. Mark answer (3) if the 'data are inadequate', ie from the facts given you cannot say whether the inference is likely to be true or false. Mark answer (4) if the inference is 'probably false', though not 'definitely false' in the light of the facts given. Mark answer (5) if the inference is definitely false', ie it cannot possibly be drawn from the facts given or it contradicts the given facts.In its most ambitious bid ever to house 6 crore slum dwellers and realize the vision of a slum-free India, the government is rolling out a massive plan to build 50 lakh dwelling units in five years across 400 towns and cities. The programme could free up thousands of acres of valuable government land across the country and generate crores worth of business for real-estate developers. Proliferation of slums has had an adverse impact on the GDP growth foryears. Slum dwellers are characterized by low productivity and susceptibilility to poor health conditions. The government believes that better housing facilities will addresssocial issues and also have a multiplier effect and serve as an economic stimulus. 71. Health and sanitary conditions in slums are far below the acceptable norms of human habitat in Indian cities and towns. 72. Cities and towns of developed countries are free from slums. 73. Per capita income of slum dwellers is significantly lower than that of those living in better housing facilities. 74. Majority of the slums in cities and towns in India are on prime private properties. 75. Development of land occupied by slums in cities of India will not have any effect on the common public. Directions (Q. 76-80): In each question below is given a statement followed by two courses of action numbered I and II. A course of action is a step or administrative decision to be taken for improvement, follow-up or further action in regard to the problem, policy, etc. On the basis of the information given in the statement, you have to assume everything in the statement to be true, then decide which of the suggested courses of action logically follow(s) for pursuing. Give answer 1) if only I follows.Give answer 2) if only II follows.Give answer 3) if either I or II follows.Give answer 4) if neither I nor 11 follows.Giveanswer5) if both land II follow. 76. Statement: A very large number of people gathered outside the local police station to submit amemorandum on behalf of the residentshighlighting police inaction in curbing incidents of theft and burglary in the eighbourhood for the past few months Courses of action: I. The police authority should forma team of officers to talk to the representatives of the residents and assure them that propersteps will be taken to stop the menace. II. The police authority should advise the people gathered outside the police station to disburse and promise them quick action. 77. Statement: At least five students were killed due to astampede in one city school as the students triedto leave the school bui Iding, fearing short circuit. Courses of action: I. The principal of the school shouldbe arrested immediately. 1 II. The Govt should immediately order closure of the school permanently. 78. Statement: Two local passenger trains collided while running in opposite directions on the same trackas the signalling system failed for a brief period. Courses of action: I. The services of the motormen ofthe trains should immediately be terminated. II. The Govt should immediately constitute a task force to review the functioning of the signalling system. 79. Statement: Almost ninety per cent of the flights of one of the private airline companies were cancelled for the fourth consecutive day as the pilots refused to join their duties in protest against sacking of two of their colleagues by the airline management. Courses of action: I. The management of the airline company should be ordered by the Govt to immediately reinstate the sacked pilots to end the crisis. II. The Govt should immediately take steps toend the impasse between the management andthe pilots to help the hapless passengers. 80. Statement: A major part of the local market in the city was gutted due to a short circuit causingextensive damage to goods and property. Courses of action: I. The Govt should issue strict guidelines for all establishments regarding installation and maintenance of electrical fittings. 11. The Government should relocate all themarkets to the outskirts of the city. 81. Statement: The Govt has decided to construct an eightlane super highway across the state to facilitate fast movement of vehicles.Which of the following can be an assumption which is implicit in the above statement ? 1) The Govt has adequate resources to construct theproposed super highway. 2) The people of the state may protest against the Govt's decision as their farm land will be taken over by the Govt for constructing the highway. 3) The Govt may find it difficult to enrol a suitable contractor for constructing the highway. 4) There is no other highway which can be used for transportation of goods across the state. 5) None of these 82. Statement: It has been reported in a recent research report that taller people are found to be happier than those who are relatively shorter. Which of the following contradicts the above finding ? 1) People are generally truthful when they respond to the questions of such research studies. 2) Those people who have a comfortable lifestyle are found to be happier than those who lead a strenuous lifestyle irrespective of their height. 3) Shorter people suffer from inferiority complex throughout their life. 4) Taller people are found to be more positive in approach to daily chores of life. 5) None of these 83. Statement: There was a slow decline in the number of patients with flu-like symptoms visiting varioushealth facilities in the city during the last fortnight. Which of the following substantiates the fact mentionedin the above statement? 1) Majority of the people suffering from flu visit the health facilities in the city. 2) There has been a continuous increase in the sale medicines for curing flu in the city limits. 3) People have started visiting the crowded places like malls and cinema halls during the last fortnight after a cautioned gap of one month. 1 4) There is a sudden increase in the number of deaths caused by flu-like symptoms followed by respiratorycomplications. 5) None of these 84. Cause: A severe cyclonic storm swept away most part of the state during the last two days. Which of the following cannot be a possible effect of the above cause ? 1) Heavy rainfall was reported in most part of the state during the last two days. 2) Many people were rendered homeless as their houses were flown away. 3) The communication system of the state was severely affected and continues to be out of gear. 4) Govt has ordered that all the offices and schools should be kept Open. 5) All are possible effects. 85. Effect: The prices of petroleum products have increased by about twenty per cent in the past two months. Which of the following can be a probable cause of the above effect ? 1) The prices of foodgrains and vegetables have shot up by more than thirty per cent. 2) The truck owners' association has decided to increase their rent by about 20 per cent with immediate effect. 3) The prices of crude oil in the international market haveincreased considerably during the past few weeks. 4) People have decided to demonstrate against the Govt's apathy towards rise in prices of essential commodities. 5) None of these Placement Paper SBI Reasoning and aptitude Questions for practice,Sbi model questions with answers,SBI free solved sample palcement papers Directions (Q.1-5) These questions are based on the following letter/number/symbol arrangement. Study it carefully to answer the questions. A 8 B 6 # 7 H U % 3 $ F V R 2 I @ 4 1 W E 9 © L 5 1. Which of the following is seventh to the right of fifteenth from the right end ? (1) 4 - Ans (2) (3) 1 (4) w (5) None of these Expl : Seventh to the right of fifteenth from the right end is 4. 2. If all the symbols are dropped from the above arrangement which of the following will be eighth from the left end ? (1) F (2) $ (3) U (4) 3 - Ans (5) None of these Expl : If all the symbols are dropped from the arrangement A 8 B 6 7 H U 3 F V R 2 I 4 1 W E9 L 5 I Eighth from the left So, eighth from the left is 3. 3. How many such symbols are three in the above arrangement, each of which is immediately followed by a consonant and also immediately preceded by a number? (1) None (2) One (3) Two - Ans (4) Three (5) More than three Expl : 3 Number Symbol Consonant 3 $F and 9 © L 4. How many such numbers are there in the above arrangement each of which is immediately followed by a vowel but not immediately preceded by a number ? (1) None (2) One - Ans (3) Two (4) Three (5) More than three Number Symbol Vowel R2I 5. Four of the following five are alike in a certain way on the basis of their positions in the above arrangement and so form a group. Which is the one that does not belong to the group ? (1) B7# (2) 14 (3) F2 R (4) 95 L (5) W©E - Ans Expl : B ---+3---> 7 ---1----> # I ---+3---> 4 ---1----> F ---+3---> 2 ---1----> R 9 ---+3---> 5 ---1----> L W ---+3---> © ---2---->E So W © E does not belong to the group. 6. Four of the following five are alike in a certain way and so form a group. Which is the one that does not belong to the group ? (1) Iron (2) Copper (3) Zinc (4) Aluminium (5) Brass - Ans Expl : Except Brass all are metal 7. Four of the following five are alike in a certain way and so form a group. Which is the one that does not belong to the group ? (1) 31 (2) 39 - Ans (3) 47 (4) 41 (5) 43 Expl : Except 39 all are prime number 8. In a certain code ROSE is written as #43 $ and FIRST is written as 5 *#37. How is store written in that code ? (1) 473$# (2) 473#$ (3) 374 #$ - Ans (4) 347#$ (5) None of these Expl : R O S E and F I R S T # 4 3 $ 5 * # 3 7 Similarly, S T O R E 3 7 4 # $ 9. How many meaningful English words can be made from the letters ADER, using each letter only once in each word ? (1) None (2) One (3) Two (4) Three (5) More than three - Ans Expl : Meaningful words READ, DEAR, EARD and DARE 10. How many such pairs of letters are there in the word DISTINGUISH, each of which have as many letters between them in the word as they have between them in the English alphabet ? (1) None (2) One (3) Two (4) Three (5) More than three - Ans Expl : D I S T I N G U I S H 4 9 19 20 9 14 7 21 9 19 8 Four such pair of letters are formed. Direction (Q.11-15) In each question below are four statements followed by four conclusions numbered I, II , III and IV. You have to take the four given statements to be true even if they seem to be at variance from commonly known facts and then decide which of the given conclusions logically follows from the four given statements disregarding commonly known facts. Then, decide which of the answers (1), (2), (3) , (4) and (5) is the correct answer and indicate it on the answer sheet. 11. Statements : All stones are pearls. Some pearls are shells. Some shells are boxes . No box is container Conclusions : I. Some stones are shells II. No pearl is container. III. No shell is container IV. Some pearls are containers. (1) Only II follows (2) Only II and III follow (3) Only either II or IV follows - Ans (4) Only III follows (5) None of follows 12. Statements : Some rings are chains. All chains are bangles. All bracelets are bangles. Some bangles are pendants. Conclusion : I. Some rings are bangles. II. Some chains are pendants III. Some bracelets are rings IV. No pendant is ring. (1) None of follows (2) Only I follows - Ans (3) Only II and III follow (4) Only IV follows (5) None of the above 13. Statements : Some schools are colleges. Some colleges are hostels. No hostel is office. All offices are institutes. Conclusions : I. No hostel is institute II. Some hostels are schools III. Some hostels are institutes IV. Some offices are college (1) Only I follows (2) Only II and III follow (3) Only IV follows (4) Only either I or III follows - Ans (5) None of the above 14. Statements : Some pins are needles. Some threads are needles. All needles are nails. All nails are hammers. Conclusions : I. Some pins are hammers II. Some threads are nails III. Some pins are threads. IV. No pin is thread (1) Only I, II and either III or IV follows - Ans (2) Only III and IV follow (3) Only I and II follow (4) All follow (5) None of the above Conclusions I. v II. v III. v IV. v So either III or IV and I, II follows. 15. Statements : Some chairs are rooms. No room is sofa. All sofas are tables. Some tables are desks. Conclusions : I. Some sofas are desks II. No room is table III. Some chairs are tables IV. No desk is room. (1) None follows - Ans (2) Only I follows (3) Only either II or III follows (4) Only III and IV follow (5) All follow Conclusions : I. x II. x III. x IV. x None follows Directions (Q.16-20) Study the following information carefully to answer these questions. A, B, C, D, E, F, G and H are sitting around a circle facing the centre. F is third to the right of C and second to the left of H. D is not an immediate neighbour of C or H. E is to the immediate right of A, who is second to the right of G. 16. Who is second to the left of C ? (1) A - Ans (2) B (3) E (4) D (5) None of these 17. Who is to the immediate right of C ? (1) A (2) B - Ans (3) D (4) B or D (5) None of these 18. Which of the following pairs of persons has first person sitting to the right of the second person ? (1) CB (2) AE (3) FG (4) HA (5) DB - Ans 19. Who sits between G and D ? (1) H (2) D (3) F - Ans (4) E (5) None of these 20. Which of the following is the correct position of B with respect to H ? (1) Only I (2) Only II (3) Only III (4) Both II and III - Ans (5) None of these Expl : Directions (Q.21-25) Each of the questions below consists of a question and two statements numbered I and II are given below it. You have to decide whether the data provided in the statements are sufficient to answer the question. Read both the statements and -------- Give answer (1) if the data in Statement I alone are sufficient to answer the question, while the data in Statement II along are not sufficient to answer the question. Give answer (2) if the data in Statement II alone are sufficient to answer the question, while the data in Statement I alone are not sufficient to answer the question. Give answer (3) if the the data in statement I alone or in Statement II alone are sufficient to answer the question. Give answer (4) if the data in both the Statements I and II are not sufficient to answer the question. Give answer (5) if the data in both the Statements I and II together are necessary to answer the question. 21. who reached the station first among L, M, J, T and R, if no two persons reached together ? I. M reached only after J and T. II. L reached before R. Ans : 4 Expl : From statement I (J and T) > M > (L and R) From statement II, L>R From statements I and II, (J and T) >M>L>R 22. Tower 'P' is in which direction with respect to tower 'Q' I. P is to the West of H, which is to the South of Q. II. F is to the West of Q and to the North of P. Ans : 3 Expl : From statement I, 23. What is the Suneeta's rank from top in the class ? I. In the call of 42 children, Suneeta is 29th from the bottom II. Suneeta is ten ranks below Samir. Ans : 1 Expl : From statement I, The tank of Sunita from top = 42 - 28 = 14th 24. What is the code for 'walks' in the code language ? I. In the code language 'she walks fast' is written as 'he ka to'. II. In the code language 'she learns fast' is written as 'jo ka he'. Ans : 5 Expl : From statement I, She walks fast ---> he ka to From statement II, she learns fast ---> jo ka he :. From statements I and II, walks ----> to 25. How is K related to N? I. N. is brother of M 2who is daughter of K II. F is husband of K Ans : 5 From statement I, Brother Daughter N--------------<-------M----------<--------K :. K may be the mother or father of M. From statement II, Husband F-------------<------K From statement I and II , K is the mother of N. 26.Directions (Q.26-30) In these questions a group of digits is given followed by four combinations of letters and symbols numbered (1), )(2), (3) and (4). Digits are to be coded as per the scheme and conditions given below. You have to find out which of the four combinations correctly represents the group of digits. Serial number of that combination is your answer. If none of the combinations is correct, your answer is (5) ie, 'None of these'. Digit : 5 1 2 8 6 3 9 0 4 7 Letter/Symbol code : H $ T A U % # F R @ Conditions : (i) If first digit is odd and the last digit is even their codes are to be interchanged (ii) If the first as well as the last digit is even both are to be coded as * (iii) if the first digit is even and the last digit is odd both are to be coded as the code for last digit. 26. 471536 (1) *@$H%* - Ans (2) R@$H%U (3) U@$H%U (4) R@$H%R (5) None of these Expl : 4 7 1 5 3 6 * @ $ H % * conditions II follows 27. 697845 (1) U#@ARU (2) U#@ARH (3) H#@ARH - Ans (4) H#@ARU (5) None of these Expl : 6 9 7 8 4 5 H # @ A R H Conditions III follows 28. 590247 (1) @#FTRH (2) H#FTR@ - Ans (3) H#FTRH (4) @#FTR@ (5) None of these Expl : 5 9 0 2 4 7 H # F T R @ 29. 348096 (1) %RAF#U (2) %RAF#% (3) URAF#U (4) *RAF#* (5) None of these - Ans Expl : 3 4 8 0 9 6 U R A F # % 30. 374862 (1) %@RAUT (2) %@RAU% (3) *@RAU* (4) T @ RAU% - Ans (5) None of these Expl : 3 7 4 8 6 2 T @ R A U % Condition (i) follows Directions (Q.31-35) In these questions symbols @, #, $, * and % are used with different meaning as follows . 'A @ B' means 'A is not smaller than B' 'A # B' means 'A is neither smaller than nor equal to B'. 'A$B' means 'A is neither greater than nor smaller than B'. 'A*B' means 'A is not greater than B'. 'A%B' means 'A is neither greater than nor equal to B'. In each of the following questions assuming the given statements to be true, find out which of the two conclusions I and II given below them is/are defenitely true. Give answer (1) if only conclusion I is true Give answer (2) if only conclusion II is true. Give answer (3) if either conclusion I or nor conclusion Give answer (4) if neither conclusion I or nor conclusion II is true Give answer (5) if both conclusions I and II are true. %==> < # ==> > *==> < @==> > $ ==> = 31. Statement N @ W, W # H, H % T Conclusion I. H%N II. T#W Ans : 1 Expl : (1) N @ W ==> N>W W # H ==> W >H H%T ==> H< T :.N> W>H<T 32. Statements : F # R, H%R, L*H Conclusions I. F#L II. R @ L Ans : 1 Expl : F # R ==> F>R H%R ==> H<R L*H ==> L<H :. F>R>H>L 33. Statements : J @ K, K%M, M#T Conclusions : I. K% T II. K@ T Ans : 3 Expl : J @ K ==> J> K K%M ==> K<M M#T ==> M>T :. J>K<M>T Conclusion : I. K% T ==> K<T (False) II. K@ T ==> K>T (False) Either I or II is true. 34. Statements V * W, W$H, H @ I Conclusions I. V*I II. I*W Ans : 2 Expl : V * W ==> V<W W$H ==> W=H H @ I ==> H>I :. V< W=H> I Conclusions I. V* I ==> V<I (False) II. I*W ==> I < W (True) 35. Statements L * P, P%V, V#D Conclusions I. L*V I I. L$D Expl : Directions (Q.36-40) These questions are based on the following information. Study it carefully and answer the questions. Seven members L, H, K, T, F, J and R represent different countries in Olympics viz. USA, China, Korea, France, Russia, Australia and Japan; each one competes for a different sport viz., volleyball, Archery, Rifle Shooting, Tennis, Boxing, Athletics and Football. The order of persons, countries and the games is not necessarily the same. K presents China for Archery. T represents USA but not for Velleyball or Rifle Shooting. The one who represents Japan competes for Boxing. F competes Volleyball but not for Korea. L represents Australia for Athletics. The one who represents Russia competes for Tennis. J does not represent Korea or Japan. R competes for Rifles Shooting. 36. D The one who competes for Rifle shooting, represents which country ? (1) France (2) Korea - Ans (3) Japan (4) USA (5) None of these 37. Which of the following combinations is correct ? (1) J-Tennis-France (2) R-Tennis-Russia (3) R-Tennis-France (4) J-Tennis-Russia - Ans (5) None of these 38. Who represents Japan ? (1) F (2) R (3) J (4) H - Ans (5) None of these 39. For which game does T compete ? (1) Boxing (2) Foot ball - Ans (3) Tennis (4) Cannot be determined (5) None of these 40. F represents which country ? (1) France - Ans (2) Russia (3) Japan (4) Korea (5) None of these Expl: Member Country Game L Australia Athletics H Japan Boxing K China Archery T USA Football F France Volleyball J Russia Tennis R Korea Rifle shooting _______________________________________ Placement Paper State Bank of India SBI Jan 2008 previous years reasoning solved question papers with answers and detailed explanations SBI Aptitude, Reasoning, General Awareness, Computer questions free solved sample placement papers of all IT NON IT Bank PSU, all competitive examinations for learn and practice, SBI and SBT current affairs and general awareness multiple chioce objective type questions with answers, What is the selection procedure for bank recruitment, How to crack bank Interview, SBI Reasoning questions with answers Directions (Qns. 1 to 5) : In the questions given below, certain symbols are used with the following meaning : A @ B means A is greater than B A * B means A is either greater than or equal to B A # B means A is equal to B A $ B means A is either smaller than or equal to B A + B means A is smaller than B Now in each of the following questions assuming the given statements to be true, find which of the two conclusions I and II given below them is/are definitely True ? Give answer (A) if only conclusion I is true. Give answer (B) if only conclusion II is true. Give answer (C) if neither conclusion I or II is true. Give answer (D) if neither conclusion I or II is true. Give answer (E) if both conclusion I and II are true. 1. Statements : D + T; E $ V; F * T; E @ D Conclusions : I. D $ V II. D + F (Ans : B) Explanatory Ans : - D + T; E $ V; F * T; E @ D After conversion D < T; E = V; F = T; E > D or, V = E > D < T = F Conclusions : I. D $ V ? D < V : Not True D is smaller than V. II. D + F ? D < F : True 2. Statements : B + D; E $ T; T * P; P @ B Conclusions : I. P $ D II. P @ D (Ans : C) Explanatory Ans : - B + D; E $ T; T * P; P @ B After conversion B < D; E = T; T = P; P > B or, E = T = P > B < D Conclusions : I. P $ D ? P = D : Not True II. P @ D ? P > D : Not True While considering the relation between two entities three possibilities exists : greater than, equal to or smaller than. Therefore, either I or II is true. 3. Statements : T * U; U $ W; V @ L; W + V Conclusions : I. V @ T II. L # W (Ans : D) Explanatory Ans : - T * U; U $ W; V @ L; W + V After conversion T = U; U = W; V > L; W < V or, T = U = W < V > L Conclusions : I. V @ T ? V > T : Not True II. L # W ? L = W : Not True 4. Statements : P $ Q; N # M; M @ R; R * P Conclusions : I. P + N II. Q $ M (Ans : A) Explanatory Ans : - P $ Q; N # M; M @ R; R * P After conversion P = Q; N = M; M > R; R = P or, N = M > R = P = Q Conclusions : I. P + N ? P < N : True II. Q $ M ? Q < M : Not True 5. Statements : E * F; G $ H; H # E; G @ K Conclusions : I. H @ K II. H * F (Ans : E) Explanatory Ans : - E * F; G $ H; H # E; G @ K After conversion E = F; G = H; H = E; G > K or, K < G = H = E = F Conclusions : I. H @ K ? H > K : True II. H * F ? H = F : True Directions (Qns. 6 to 11) : Study the following letter-number-symbol sequence carefully and answer the question given below : B D 5 F E 3 8 $ M 2 I K * P T @ U 9 A 7 1 £ H J 4 Q 6 Explanatory Ans : - (6-11) (i) There are altogether 27 elements in the above sequence. (ii) There are only 14 letters in the above sequence (iii) There are only 9 digits in the above sequence. (iv) There are only 4 symbols in the above sequence. (v) The middle terms of the sequence is P. 6. What should come in place of the question mark (?) in the following sequence ? 5ES, MIP, ?, IHQ A) TUA B) TU7 (Ans) C) @91 D) T91 E) None of these Therefore, ? = TU7 7. Which of the following is exactly in the midway between the eleventh from the left end and the 7th from the right end ? A) P B) @ (Ans) C) T D) U E) None of these Explanatory Ans : - 11 th from left end ? 1 7th from right end ? 1 Remaining elements between I and I : 8. Which of the following is the sixth to the right of the twentieth from the right end ? A) 5 B) F C) P (Ans) D) K E) None of these Explanatory Ans : - 6th to the right of 20th from the right end means 20 - 6 = 14th from the right end. There are altogether 27 elements in the above sequence and the middle term. i.e. 14th element from either end is P. Therefore, our required answer is option (C). 9. How many such digits are there in the above sequence which are immediately preceded as well as followed by digits ? A) None (Ans) B) One C) Two D) Three E) None of these Explanatory Ans : - Digit Digit Digit There is no such combination. 10. If the first fifteen elements are written in the reverse order then which of the following will be eighth to the left of the thirteenth element from right end ? A) M (Ans) B) 8 C) $ D) * E) None of these Explanatory Ans : - 8th to the left of 13th element from right end means 13 + 8 = 21st from the right end. 21st element from the right end is equivalent to 28 - 21 = 7th element from the left and vice versa. Thus, 1st ? 15th ? 2nd ? 14th 3rd ? 13th ? 4th ? 12th 5th ? 11th ? 6th ? 10th 7th ? 9th Therefore, required element would be 9th from the left in the original sequence. 9th from left ? M. 11. If all the consonants starting from B are given sequentially the value of even numbers such as B = 2, C = 4 and so on and all the vowels are given the value of 5 each, then what will be the value of the letters of the word CUSTOM ? A) 92 B) 86 C) 92 D) 96 (Ans) E) None of these Explanatory Ans : - According to question. A ? 5 B ? 2 C ? 4 D ? 6 E ? 5 F ? 8 G ? 10 H ? 12 I ? 5 J ? 14 K ? 16 L ? 18 M ? 20 N ? 22 O ? 5 P ? 24 Q ? 26 R ? 28 S ? 30 T ? 32 U ? 5 V ? 34 W ? 36 X ? 38 Y ? 40 Z ? 42 Now, 12. How many such 5s are there in the following sequence that the sum of the two immediately following digits is greater than the sum of the two immediately preceding digits ? 3 7 6 5 8 3 2 4 5 5 4 8 7 9 1 5 3 4 8 7 5 9 8 7 6 4 A) One B) Two C) Three (Ans) D) Four E) None of these Such combinations are: 13. If A + B means "A is the sister of B". A * B means "A is the wife of B", A ÷ B means "A is the father of B" and A - B means "A is the brother of B", then which of following expresses the relationship that "T is the daughter of P" ? A) P * Q ÷ R + S - T B) P * Q ÷ R - T + S (Ans) C) P * Q ÷ R + T - S D) P * Q ÷ R + S + T E) None of these Explanatory Ans : - Option (I) P * Q ÷ R + S - T P * Q ? P is the wife of Q. Q ÷ R ? Q is the father of R. R + S ? R is the sister of S. S - T ? S is the brother of T. Note : The sex of T is not known. Deductions (i) Q is the husband of P. (ii) P is the mother of R, S and T. (iii) Q is the father of R, S and T. (iv) R is the sister of S and T. (v) S is the brother of R and T. Option (II) P * Q ÷ R - T + S P * Q ? P is the wife of Q. Q ÷ R ? Q is the father of R. R - T ? R is the brother of S. T + S ? T is the sister of S. Note : The sex of S is not known. Deductions (i) Q is the husband of P. (ii) P is the mother of R, S and T. (iii) Q is the father of R, S and T. (iv) T is the daughter of P and Q. 14. If the position of the first letter of English alphabet is interchanged with the position of the fourteenth letter, second letter wit the fifteenth letter in such a way that M is interchanged with Z, then which of the following letters will be 9th to the right of 17th letter from the right ? A) F (Ans) B) E C) R D) T E) None of these Explanatory Ans : - According to question, the new sequence would be : N O P Q R S T U V W X Y Z A B C D E F G H I J K L M 9th to the right of 17th letter from the right means (17 - 9) = 8th letter from the right. 8th letter from right ? F. Directions (Qns. 15 to 18) : Read the following information and answer the questions given below : (i) Seven friends P, Q, R, S, T, U and W have gathered at the Mumbai airport. Five of them are scheduled to go to five different places - Delhi, Chennai, Lucknow, Bangalore and Calcutta. (ii) Five of them are executives, each specializing in viz. Administration (Admn.), Human Resource Management (HRM), Marketing, Systems and Finance. (iii) T, an executive is going to Chennai and is neither from Finance nor Marketing. (iv) W is a System specialist and is leaving for Delhi. U is an executive but is not going to one of the five places. (v) Q is an executive but not from HRM but has come at the airport to see his friends. (vi) P is an executive but not from Marketing and is flying to one of the destinations but not to Bangalore or Calcutta. Explanatory Ans : - (15-18) : On the basis of given information and conclusions as well as sub-conclusions drawn from them we can construct the following chart : Person Field Destination P Finance Lucknow Q HRM — R — Calcutta/ Bangalore S — Calcutta/ Bangalore T Administration Chennai U Marketing — W System Delhi 15. Who is going to fly to Bangalore ? A) Data inadequate (Ans) B) R C) S D) P E) None of these 16. Who among the following specializes in Marketing ? A) S B) P C) U (Ans) D) Data inadequate E) None of these 17. R has specialization in which of the following fields ? A) Finance B) Marketing C) Either Marketing or Finance D) None (Ans) E) None of these 18. The one who is going to fly to Chennai is A) Not at executive B) From administration (Ans) C) S D) From finance E) None of these 19. How many pairs of letters are there in the word 'NURSING' which have as many letters between them as in the alphabet ? A) One B) Three (Ans) C) Five D) Six E) None of these There are three such pairs. SBI Reasoning Question Paper SBI Reasoning ability solved question papers SBI stands for State Bank of IndiaSBI Computer Knowledge whole solved questions paper Directions (Q. 1-5) In a certain code the symbol for 0 (zero) is • and that for 1 is *. There are no other symbols for numbers and all numbers greater than 1 are written using these two symbols only, the value of the symbol for 1 doubling itself every time it shifts one place to the left. Thus 0 is written • 1 is written * 2 is written * • 3 is written * * 4 is written *•• and so on 1. If *** is multiplied by **, the product will be (1) * • * • * (Ans) (2) * * • • * (3) * • • * * (4) • * * * * (5) * • • • * 2. Which of the following will represent 20% of 45 in that code ? (1) * * * * (2) * * * (3) * • • * (Ans) (4) * * • * (5) * * * • 3. Which of the following stands for 7 in that code ? (1) * * * * * * * (2) * • * • * • * (3) * * • (4) * • • * (5) * * * (Ans) 4. If * * • is added to * • * *, the sum will be (1) * * * * * (2) * • • • * (Ans) (3) * * • • * (4) * • • * * (5) * * * • * 5. Which of the following numbers is written as * •* • in that code? (1) 8080 (2) 202 (3) 42 (4) 10 (Ans) (5) 9 6. Four of the following five are alike in certain way and so form a group. Which is the one that does not belong to that group ? (1) Jasmine (2) Rose (3) Dahlia (4) Marigold (5) Lotus (Ans) 7. ‘Jackal’ is related to ‘Carnivorous’ in the same way as ‘Goat is related to (1) Omnivorous (2) Carnivorous (3) Herbivorous (Ans) (4) Multivorous (5) None of these 8. If blue is called red, red is called green ,green is called black and black is called white, what is he colour of grass ? (1) red (2) black (Ans) (3) white (4) green (5) None of these 9. In a certain code RAID is written as % # * $, RIPE is written as % * @ ©. How is DEAR written in the code (1) @©#% (2) $@#% (3) @$#% (4) $©#% (Ans) (5) None of these 10. ‘Radish’ is related to ‘Root’ in the same way as ‘Brinjal’ is related to (1) Fruit (Ans) (2) Stem (3) Flower (4) Root (5) None of these Directions (Q. 11-15) Each of the questions below consists of a question and two statements numbered I and II are given below it. You have to decide whether the data provided in the statements are sufficient to answer the question. Read both the statements and Give answer (1) if the data in Statement I alone are sufficient to answer the question, while the data in Statement II alone are not sufficient to answer the question. Give answer (2) if the data in Statement II alone are sufficient to answer the question, while the data in Statement I alone are not sufficient to answer the question. Give answer (3) if the data either in Statement I alone or in Statement II alone are sufficient to answer the question. Give answer (4) if the data even in both the Statements I and II are not sufficient to answer the question. Give answer (5) if the data in both the Statements I and Ii together are necessary to answer the question. 11. How many children are there in the group? I. Sangita has scored more marks than 12 children in the group. II. Reena has scored less than Sangita. (Ans. (4) Both the statements are not sufficient to give answer.) 12. What is the value of 36$4*8 ? I. P$Q means divide P by Q. II. A*B means multiply A by B. (Ans. (5) From statements I and II,) 13. What is Samir’s rank form the top in the class of 30 students? I. Sudhir, who is four ranks above Samir, is fifteenth in rank from the bottom. II. Samir is three ranks below Neeta who is eighteenth form the bottom. (Ans. (3) From statements I,) 14. Who among L, N, F, G and Q was the first to reach the college ? I. F reached before L and G but not before Q who was not the first to reach. II. N reached before F and G and L reached after F. (Ans. (1) From statements I,) 15. In the code language what is the code for ‘fat’ ? I. In the code language ‘she is fat’ is written as ‘he ra ca.’ II. In the same code language ‘fat boy’ is written as ‘ra ka’. (Ans. (5) From statements I,) Directions (Q. 16-20) Below in each question are given two statements (A) and (B). These statements may be either independent causes or may be effects of independent causes or a common cause. One of these statements may be the effect of the other statement. Read both the statements and decide which of the following answer choice correctly depicts the relationship between these two statements. Mark answer (1) if statement (A) is the cause and statement (B) is its effect. Mark answer (2) if statement (B) is the cause and statement (A) is its effect. Mark answer (3) if both the statements (A) and (B) are independent causes. Mark answer (4) if both the statements (A) and (B) are effects of independent causes. Mark answer (5) if both the statements (A) and (B) are effects of some common causes. 16. A. Large numbers of primary schools in the villages in the district are closed down this year. B. Severe draught situation gripped the state resulting into acute shortage of drinking water. (Ans. (4)) 17. A. Govt. has imposed a strict ban on use of plastic products are on the verge of closure. B. All the small scale units producing plastic products products are on the verge of closure. (Ans. (1)) 18. A. Police had launched a crackdown on all the criminal activities in the locality last month. B. There has been a significant decline in the cases of criminal activities in the locality. (Ans. (1)) 19. A. Large number of devotees visited the shrine on Saturday. B. Every Saturday special prayers are offered. (Ans. (2)) 20. A. The Village Panchayats in the state are empowered by the Gov.to settle cases of land disputes in the villages. B. There has been significant reduction in the number of criminal cases in the district court. (Ans. (4)) Directions (Q. 21-25) Study the following information carefully and answer the questions given below. Digit : 3 5 7 2 4 6 1 8 9 Code: F K R L D T G H B Following conditions are to be applied: (i) If the first digit is even and the last digit is odd, both are to be coded as X. (ii) If the first digit is odd and the last digit is even, both are to be coded as A (iii) If the first digit as well as the last digit is even, both are to be coded as the code for last digit. (iv) If the first digit as well as the last digit is odd, both are to be coded as the code for the first digit. Applying above conditions you have to find out the correct code for the number in each question and indicate your answer accordingly. If none of the codes is correct, (5) ie. None of these is your answer. 21. 364289 (1) BTDLHB (2) FTDLHB (3) FTDLHF (Ans) (4) BTDLHF (5) None of these 22. 521437 (1) KLGDFK (Ans) (2) RLGDFR (3) KLGDFR (4) KLDGFK 5) None of these 23. 392648 (1) ALBTDA (2) XBLTDA (3) XBLTDX (4) ABLTDA (Ans) (5) None of these 24.279654 (1) LRBTKD (2) LRBTKL (4) DRTBKD (5) None of these 25.725638 (1) ALKTFH (2) ALKTFA (3) XLKTFX (4) XLKTFH (5) None of these Directions (Q. 26-33) In the following questions the symbols @, #, $, % and & are used with different meanings as follows: ‘P @ Q’ means ‘P is neither smaller than nor equal to Q’. ‘P # Q’ means ‘P is not greater than Q’. ‘P $ Q’ means ‘P is not smaller than Q’. ‘P % Q’ means ‘P is neither greater than nor smaller than Q’. ‘P & Q’ means ‘P is neither greater than nor equal to Q’. In each of the following questions assuming the given statements to be true, find out which of the two conclusions I and II given below them is/are definitely true. Give answer (1) if only conclusion I is true. Give answer (2) if only conclusion II is true. Give answer (3) if either conclusion I or conclusion II is true. Give answer (4) if neither conclusion I nor conclusion II is true. Give answer (5) it both conclusions I and II are true. 26. Statements G # H, H $ K, K @ M Conclusions I. M # G II. G & M 27. Statements F $ D, H # M, M % D Conclusions I. F $ H II. F @ H 28. Statements R & M, M # L, L $ Q Conclusions I. M % Q II. M @ Q 29. Statements F # R, Q $ R, Q & M Conclusions I. F # Q II. R & M 30. Statements D & T, R # T, R $ M Conclusions I. M & T II. M % T 31. Statements E % H, H $ M, M # Q Conclusions I. H $ Q II. E $ M 32. Statements S # A, S @ T, L & T Conclusions I. L & A II. S @ L 33. Statements G $ J, J @ K, K % N Conclusions I. G @ N II. G % N Directions (Q. 34-40) These questions are based on the following arrangement of numerals, symbols and letters. W T D I 5 M K % L $ 3 7 F E B # 1 G H A © @ J U V 2 4 34. How many such symbols are there in the above arrangement each of which is immediately precede by a letter ? (1) BTDLHB (2) FTDLHB (3) FTDLHF (Ans) (4) BTDLHF (5) None of these 35.If KL : $7, then B1 : ? (1) GA (2) H © (3) GH (4) © J (5) None of these 36If all the vowels are dropped from the arrangement, which element will be fifth to the right of the thirteenth element form the right? (1) H (2) G (3) A (4) 1 (5) None of these 37. If the first ten elements in the arrangement are reversed, which element will be third to the left of eleventh element from the left ? (1) D (2) % (3) K (4) I ( 5) None of these 38.Four for the following five are alike in a certain way on the besis of their position in the above arrangement and so form a group. Which is the one that does not belong to the group? (1) D5W (2) L3K (3) E#7 (4) H © (5) U2@ 39.How many such vowels are there in the above arrangement which are immediately preceded as well as immediately followed by a consonant? (1) None (2) One (3) Two (Ans) (4) Three (5) None of these 40.Which element is exactly midway between the eighth element form the left and the tenth element from the right? (1) E (2) 7 (3) B (4) # (5) None of these (Ans) Directions (Q.41-45) Given an input, a coding machine generates pass – codes for six batches every day as follows: Input : the shopkeeper offered discount to customers, Pass-code for Batch I : customer the shopkeeper offered discount to Batch II : customers discount the shopkeeper offered to Batch III : customers discount offered the shopkeeper to and so on until the arrangement is completed. After the arrangement is completed the next batch gets the same code as that for Batch I. Duration of each batch is 1 hour. There is a break of one hour after the fourth batch. Sixth batch is the last batch. Now answer the following questions: 41.If the pass-code for the second batch is “do lean window out of not”, what will be the pass-code for the fourth batch? (1) do lean of not out window (2) do lean of out not window (3) do lean of not out window of (4) do lean of not of window out (Ans) (5) None of the above 42If the input is “please do not delay the matter”, what will be the pass-code for third batch? (1) do please not delay the matter (2) delay do matter please not the (3) delay do matter not please the (4) delay do matter not the please (5) None of the above 43.If the pass-code for third batch is “brisk every for morning go walk, what will definitely be the input? (1) morning brisk go walk for every (2) morning go brisk walk for every (3) morning go walk brisk for every (4) Cannot be determined (Ans) (5) None of the above 44.If the pass-code for the second batch is “children for not is good watching television”, what will be the pass-code for the fifth batch? (1) children for good not is watching television (2) children for good is not watching television (3) children good for is not television watching (4) Cannot be determined (5) None of the above 45.If the input is “necessary arrangements have already been made”, how many batches are required to complete the arrangement? (1) Three (2) Four (3) Five (4) Six (5) None of these Directions (Q. 46-50) Study the following information carefully and answer the questions given below. An organization wants to recruit System Analysts. The following conditions apply. The candidate must (i) be an engineering graduate in Computer/IT with at least 60% marks. (ii) have working experience in the field of Computer at least for 2 years after acquiring the requisite qualification. (iii) have completed minimum 25 years and maximum 30 years of age as on 1.12.2005. (iv) be willing to sign a bond for Rs. 50000. (v) Have secured minimum 55% marks in selection test. However, if a candidate fulfills all other r conditions except: (a) at (i) above, but is an Electronics Engineer with 65% or more marks the case is to be referred to the General Manager (GM)-IT. (b) at (iv) above, but has an experience of at least 5 years as a Software Manager, the case is to be referred to the VP. In each question below, detailed information of candidate is given. You have to carefully study the information provided in each case and take one of the following courses of actions based on the information and the conditions given above. You are not to assume anything other than the information provided in each question. All these cases are given to you as on 01.12.2005. You have to indicate your decision by marking answer to each question as follows: Mark answer (1) if the case is to be referred to VP. Mark answer (2) if the case is to be referred to GM. Mark answer (3) if the data provided is not sufficient to take a decision. Mark answer (4) if the candidate is to be selected. Mark answer (5) if the candidate is not to be selected 46.Ms. Suneeta is an IT Engineer with 60% marks at graduation as well as in selection test. She is working as a Software Engineer for last 3 years after completing engineering degree and has completed 27 years of age. She is willing to sign the bond of Rs.50000. (Ans. 4) 47. Rakesh Rao is Computer Engineer Graduate and thereafter is working as a Software Manager for past 6 years. He has secured 72% mark at graduation and 67% marks in selection test. His date of birth is 5th December 1976. He is not willing to sign the bond for Rs.50000 (Ans.1) 48. Ramkumar is an engineering graduate in Computer with 78% marks passed out in 1999 at the age of 23 years. Since then he is working as a Software Manager in an engineering firm. He doesn’t want to sign the bond for Rs. 50000. He has cleared the selection test with 72% marks. (Ans. 1) 49. Nishant is an Electronics Engineer passed out in June 2002 at the age of 22 years. Since then he is working as a Programmer in a software company. He has passed the selection test with 66% marks and is willing to sign the bond. (Ans. 3) 50. Kalyani is an Engineer with 72% marks in Telecommunication. She has just completed 27 years of age, She has cleared the selection test with 59% marks, She is willing to sign the bond. (Ans.5) 51. Sangita is an IT Engineer and is working as an EDP Officer in a bank for past 5 years. She has completed 28 years of age and is willing to sign the bond of Rs. 50000. She has obtained 65% marks in the selection test. (Ans. 3) 52. Abhijit is a Computer Engineer with 65% marks passed out in 2001 at the age of 22 year. Since then he is working as a Software Engineer in a private firm. He is willing to sign the bond for the required amount. He has secured 63% marks in selection test. (Ans) 53. Giridhar is working as a Software Engineer in a reputed company for past 4 years after completing Computer Engineering with 68% marks. He has cleared the selection test with 75% marks and is willing to sign the bond. His date of birth is 17th December, 1978. 54. Nikita has just completed 26 years of age. She has passed IT engineering Examination in 2002 with 66% marks and is working as a Sr. Programmer since then. She has no problem in signing the bond of Rs. 50000. She has secured 53% marks in selection test. 55. Mr. Rajan is working as a Programmer for the last 6 years in an engineering firm after passing Engineering Examination. He has passed Electronics Engineering with 76% marks. His date of birth is 16.05.78. He has cleared the selection test with 62% marks and is willing to sign the bond for Rs. 50000. Directions (Q. 56-57) Study the following information to answer these questions. Seven Professors A, B, C, D, E, F and G are engaged in evaluation of answer papers in three different subjects English, Mathematics and History. At least two persons evaluate the papers in each subject. Each of the evaluators stay in different buildings P, Q, R, S, T, V and W not necessarily in the same order. A evaluates English papers only with E and stays in building R. D stays in building W and does not evaluate Maths papers. The one who stays in building V evaluates History papers. B and C do not evaluate the papers in the same subject. Those who evaluate English papers do not stay in building Q.F Stays in building P but does not evaluate History papers. G evaluates same papers as F.C stays in building T. 56. Who stays in building V? (1) E (2) F (3) G (4) B (Ans) (5) None of these 57. Which of the following combinations of subject, person and building is definitely correct? (1) Maths - F - Q (2) Maths - G - Q (Ans) (3) History - D - T (4) History - E - S (5) None of these 58. Which of the following groups of persons evaluate the Mathematics paper? (1) CF (2) EFG (3) CFG (Ans) (4) FG (5) None of these 59. Papers in which subject are evaluated by D? (1) History (Ans) (2) Maths (3) English (4) English or Mathematics (5) History or Mathematics 60. E stay in which building ? (1) P (2) Q (3) T (4) Cannot be determined (5) None of these (Ans) Directions (Q. 61-65) In each question given below, use the following notations: A J B’ means ‘Add A to B’, ‘A | B’ means ‘Subtract B from A’, ‘A $ B’ means ‘Multiply A by B’, and ‘A @ B’ means ‘Divide A by B’. 61. Profit percentage is computed by using the following method. Cost Price (C) is subtracted from the Sale Price (S) and the difference is multiplied by 100 and the product is then divided by Cost Price (C). Which of the following expressions indicates the profit percentage? (1) (C | S) $ 100 @ C (2) (S | C) $ 100 @ C (3) (S | C) @ 100 @ C (4) C | S $ 100 ] C (5) None of these 62. IQ is worked out by dividing mental age (MA) of a person by his chronological age (CA) and the quotient so obtained is multiplied by 100. Which of the following expressions indicates the IQ of a person? (1) MA $ 100 @ CA (2) MA $ CA @ 100 (3) CA $ MA ] 100 (4) MA $ CA | 100 (5) None of these 64. The average salary of the Managers is computed by the following method. (i) The average salary (A) is multiplied by the Grand Number (GN) (ii) The average salary of other employee (AE) is multiplied by ;number of employees (N). (iii) (ii) is subtracted from (i), and (iv) the difference so obtained is divided by the number of Managers (NM)Which of the following expressions indicates the average salary of the Managers? (1) A $ GN | ( AE $ N @ NM) (2) A $ GN ] ( AE $ N @ NM) (3) GN | AE @ N @ NM (4) (GN $ A | AE $ N) @ NM (5) None of the above 65. For earthquake relief fund, each employee of a company had contributed an amount equal to the total number of employees (N) in the company. To this amount was added the contribution made by 10 Directors, each of whom had paid Rs. 100 more than what each employee had paid. Which of the following expressions indicate the total contribution made by the company? Which of the following expressions indicates the average salary of the Managers? (1) N ] N $ 10 @ (N ] 100) (2) N $ N ] 10 $ (100 ] N) (3) N $ N @ 10 S (N | 100) (4) N $ N ] 10 $ (100 $ N) (5) None of the above Directions (Q. 66-) Study the following information carefully to answer these questions. Seven executive A, B, C, D, E, F and G from a company have to visit seven different place Ahmedabad, Kolkata, Delhi, Chennai, Hyderabad, Bengaluru and Jaipur to market their newly launched product. The order of persons and cities may not be necessarily the same. Each one flies by a different airline Spice jet, Kingfisher, Sahara, Jet, Air Deccan, Indian Airlines, Air India, not necessary in the same order. C goes to Kolkata but not by Sahara or Jet Airlines. D flies by Air India to Bengaluru. The one who goes to Jaipur does not travel by Air Deccan or Sahara. E travels by Air Deccan. A does not go to Ahmedabad. F travels to Hyderabad by Spice jet. B goes to Chennai by Kingfisher. E does not go to Ahmedabad. G does not go to Jaipur. 66 Who travels by Sahara Airlines? (1) A (2) C (3) G (4) Cannot be determined (5) None of these 67. Who goes to Jaipur? (1) A (2) B (3) G (4) D (5) None of these 68. The one who travels by Air Deccan, visits which place? (1) Ahmedabad (2) Delhi (3) Chennai (4) Bengaluru (5) None of these Directions (Q. 6-9-75) Study the following information carefully to answer these questions.A word and number arrangement machine when given an input line of words and numbers rearranges them following a particular rule in each step. The following is an illustration of input and rearrangement. Input : day case 20 13 now for 49 56 Step I : 13 day case 20 now for 49 56 Step II : 13 now day case 20 for 49 56 Step III : 13 now 20 day case for 49 56 Step IV : 13 now 20 for day case 49 56 Step V : 13 now 20 for 49 day case 56 Step VI : 13 now 20 for 49 day 56 case and Step VI is the last step As per the rules followed in the above steps, find out in each of the following questions the appropriate step for the given input 69. Input : yes 21 far 32 17 12 wide goal Which of the following will be step V of the above input ? (1) 12 yes 21 wide 32 far 17 goal (2) 12 yes 21 wide far 32 17 goal (3) 12 yes 21 far 32 17 wide goal (4) There will be no such step (5) None of the above 70. Step IV of an input is: 17 hotel 29 father 83 76 door eye. Which of the following will be Step VII? (1) 17 hotel 29 father 76 door 83 age (2) 17 hotel 29 father 76 age 83 door (3) 17 hotel 29 father 76 83 door age (4) There will be no such step (5) None of the above 71. Step III of an input is: 25 with 32 85 73 tax break home. How many more steps will be required to complete the rearrangement? (1) Five (2) Four (3) Six (4) Seven (5) None of these 72. Step III of an input is: 37 Yellow 42 61 53 violet green red. How many more steps will be required to complete the rearrangement? (1) 3 (2) 4 (3) 5 (4) 6 (5) None of these 73. Input : can you go there 22 36 13 46 How many steps will be required to complete the rearrangement? (1) 7 (2) 8 (3) 6 (4) 5 (5) None of these 74. Input : 42 36 go and come back 20 15 Which of the following steps will be the last but one? (1) IV (2) V (3) VII (4) VII (5) None of these 75. Step II of an input is: 39 sure 72 63 height over 42 lamp. Which of the followings is definitely the input? (1) height 39 sure 72 63 over 42 lamp (2) 72 63 39 sure height over 42 lamp (3) 63 39 sure 72 height over 42 lamp (4) Cannot be determined (5) None of the above SBI Aptitude-Reasoning SBI PO Preliminary Exam Paper Reasoning Ability Directions (Qs. 1- 4) Study the following information carefully and answer the given questions.When a word and number arrangement machine is given an input line of words and numbers, it arranges them following a particular rule. The following is an illustration of input and rearrangement. (All the numbers are two digit numbers) Input: tall 48 13 rise alt 99 76 32 wise jar high 28 56 barn Step I: 13 tall 48 rise 99 76 32 wise jar high 28 56 barn alt Step II: 28 13 tall 48 rise 99 76 32 wise jar high 56 alt barn Step III: 32 28 13 tall 48 rise 99 76 wise jar 56 alt barn high Step IV: 48 32 28 13 tall rise 99 76 wise 56 alt barn high jar Step V: 56 48 32 28 13 tall 99 76 wise alt barn high jar rise Step VI: 76 56 48 32 28 13 99 wise alt barn high jar rise tall Step VII: 99 76 56 48 32 28 13 alt barn high jar rise tall wise And Step VII is the last step of the above input, as the desired arrangement is obtained. As per the rules followed in the given steps, find out the appropriate steps for the given input. Input: 84 why shit 14 32 not best ink feet 51 27 vain 68 92(All the numbers are two digit numbers) 1. Which step number is the following input? 32 27 14 84 why sit not 51 vain 92 68 feet best ink (1) Step V (2) Step VI (3) Step IV (4) Step III (5) There is no such step 2. Which word/number would be at 5th position from the right in Step V? (1) 14 (2) 92 (3) feet (4) best (5) why 3. How many elements (word or numbers) are there between ?feet and 32? as they appear in the last step of the input? (1) One (2) Three (3) Four (4) Five (5) Seven 4. Which of the following represents the position of ?why? in the fourth step? (1) Eighth from the left (2) Fifth from the left (3) Sixth from the left (4) Fifth from the left (5) Seventh from the left Directions (Qs. 6 -10) Study the following information carefully and answer the question given below: Eight persons - Harish, Ishaan, Jamal, Kunal, Lokesh, Manish, Nitin and Omkar - are standing in a straight line at equidistant positions. Some of them are facing north while others are facing south.Manish is standing third to the right of Harish. Manish is standing at one of the extreme ends. Lokesh is standing third to the left of Harish. The immediate neighbours of Jamal face north. Nitin is not an immediate neighbour of Harish. The persons standing at the extreme ends face the same direction (both are facing either North or South). The immediate neighbours of Harish face the opposite direction as that of Manish. The immediate neighbours of Omkar face opposite direction with respect to each other. One of the immediate neighbour of Lokesh is Kunal who is facing north. Ishaan is standing between Jamal and Manish. Not more than four persons are facing north. 5. Who among the following is third to the left of Nitin? (1) Kunal (2) Jamal (3) Harish (4) Ishaan (5) Omkar 6. The immediate neighbours of Lokesh are: (1) Manish and Nitin (2) Nitin and Omkar (3) Kunal and Nitin (4) Nitin and Harish (5) Jamal and Harish 7. Exactly how many persons are standing between Ishaan and Omkar? (1) Three (2) Four (3) One (4) Two (5) None 8. Who among the following is to the immediate left of Harish? (1) Omkar (2) Jamal (3) Ishaan (4) Lokesh (5) Kunal 9. Four of the following five are alike in a certain way based on the above arrangement and hence form a group. Which of the following does not belong to that group? (1) Nitin (2) Lokesh (3) Omkar (4) Jamal (5) Kunal 10. Who among the following is exactly between Lokesh and Jamal? (1) Nitin (2) Omkar (3) Harish (4) Ishaan (5) None Directions (11 - 13): Each of the questions below consists of a question and two statements numbered I and II given below it. You have to decide whether the data provided in the statements are sufficient to answer the question. Read both the statements and give the answer. (1) If the data in statement I alone are sufficient to answer the question, while the data in statement II alone are not sufficient to answer the question (2) If the data in statement II alone are sufficient to answer the question, while the data in statement I alone are not sufficient to answer the question (3) If the data either in statement I alone or in statement II alone are sufficient to answer the question (4) If the data given in both statement I and II together are not sufficient to answer the question and (5) If the data in both statements I and II together are necessary to answer the question 11. How many brothers does Shilpa have? I. Deepa who is Shilpa?s sister has two siblings. II. Kaushal is the only son of Deepa?s parents. 12. Amongst four friends A, B, C and D seated in straight line facing north, who sits to the immediate right of B? I. A sits third to the right of B and C is not immediate neighbour of B. II. C sits to the immediate left of A. A sits at one of the extreme ends. D does not sit at the extreme end of the line. 13. Which direction is Sudha facing? I. If Sunny, who is currently facing West, turns 900 towards his left, he would face the same direction as Sudha. II. If Harry, who is currently facing East, turns 900 towards his left, he would face the direction just opposite to the direction Sudha is facing. Directions (Qs. 14 - 15): Study the following information carefully and answer the questions given below: Point A is 5 m towards the West of point B. Point C is 2 m towards the North of point B. Point D is 3 m towards the East of point C. Point E is 2 m towards the South of point D. 14. If a person walks 2 m towards the North from point A, takes right turn and continues to walk, which of the following points would he reach first? (1) D (2) B (3) E (4) C (5) Cannot be determined 15. Which of the following points are in a straight line? (1) ABE (2) DCA (3) CED (4) BDA (5) ACE Directions (16-20): In each of the questions below are given two/three statements followed by two conclusions numbered I & II. You have to take the given statements to be true even if they seem to be at variance from commonly known facts. Read all the conclusion and then decide which of the given conclusions logically follows from the given statements disregarding commonly known facts. 16. Statements: All metals are black.Some pots are black. No black is thread. Conclusions: I. At least some thread are pots. II. All thread being pot is a possibility III. Some metals are black. (1) Only I follows (2) Only II & III follow (3) All follow (4) None follows (5) Only I & III follow 17. Statements: Some boys are rude. Some rude are human. All human are good. Conclusions: I. All human being boys is a possibility. II. At least some rude are good. III. Some good are not boys. (1) Only I follows (2) Only II & III follow (3) All follow (4) Only I & III follow (5) None of these 18. Statements: Some trains are metro.No bus is a train. All taxi is a metro. Conclusions: I. Some metro is not a bus. II. At least some taxi is a bus. III. All bus being metro is a possibility. (1) Only I follows (2) Only II & III follow (3) All follow (4) Only I & III follow (5) None follows 19. Statements: All managers are clerk.Some boss is a clerk. All peons are boss. Conclusions: I. All boss is a peon. II. No manger is boss. III. All manager being peon is a possibility. (1) Only I follows (2) Only III follows (3) Only II follows (4) None follows (5) Only I & III follow 20. Statements: No chain is gold.Some metal is chain. Some pots are gold. Conclusions: I. Some pots are not chain. II. All metal being gold is a possibility. III. Some pots are metal. (1) Only I follow (2) Only II & III follows (3) All follows (4) None follows (5) Only I & III follow Directions (21-25): In the following questions,relationship between different elements is shown in the statements. The statements are followed by three conclusions. Read the conclusions and then decide which of the given conclusions logically follows from the given statements. 21. Statement: A >B > C < D, P < C Conclusion: I. P > D II. B <P III. A = C (1) Only I follows (2) Only II follows (3) Only III follows (4) None follows (5) Only I & III follow 22. Statement: K ? L < M > N, L > P Conclusion: I. P < N II. M > N III. M = N (1) Only I & III follow (2) Only I & II follow (3) All follow (4) Either II or III follows (5) None follows 23. Statement: R = O > L, B < A, A < L Conclusion: I. L = R II. O > B III. A < R (1) Only I & III follow (2) Only I & II follow (3) All follow (4) Only II & III follow (5) None follows 24. Statement: S > N > M > P < K, S < Q Conclusion: I. Q > N II. Q >P III. M > S (1) Only I & III follow (2) Only I & II follow (3) All follow (4) Only II & III follow (5) None follows 25. Statement: E > K > T < M, K < N> T Conclusion: I. N > E II. T < E III. N > M (1) Only I & III follow (2) Only I & II follow (3) All follow (4) Only II & III follow (5) None of these Directions (26-30): Study the following information carefully and answer the questions given below: Eight colleagues A, B, C, D, E, F, G and H are sitting around a circular table facing the centre but not necessarily in the same order. Each one of them holds a different post viz. Manager, Company Secretary, Chairman, President, Vice - President, Group Leader, Financial Advisor and Managing Director.A sits third to the right of the Managing Director. Only two people sit between the Managing Director and H. Vice - President and the Company Secretary areimmediate neighbours of each other. Neither A nor H is a Vice - President or a Company Secretary. Vice -President is not an immediate neighbour of the Managing Director. Manager sits second to the left of E. E is not an immediate neighbour of H. The Manager is an immediate neighbour of both Group Leader and Financial Advisor. Financial Advisor sits third to the right of B. B is not the Vice - President. C sits to the immediate right of the Chairman. A is not the Chairman. F is not an immediate neighbour of A. G is not an immediate neighbour of the Manager. 26. Who amongst the following sits third to the left of E? (1) The Manager (2) G (3) A (4) The Financial Advisor (5) B 27. Four of the following five are alike in a certain way based on the given arrangement and thus form a group. Which is the one that does not belong to that group? (1) F - Chairman (2) G - President (3) D - Manager (4) A - Financial Advisor (5) B - Managing Director 28. Who amongst the following is the President of the company? (1) A (2) C (3) H (4) G (5) D 29. Which of the following is true with respect to the given seating arrangement? (1) The Group Leader of the company is an immediate neighbour of the Vice - President (2) G sits second to right of D (3) The Group Leader and the Company Secretary are immediate neighbours of each other (4) The Chairman of the company sits to the immediate left of the Managing Director (5) The Group Leader sits second to the left of D 30. Which of the following posts does B holds in the company? (1) Chairman (2) Manager (3) Company Secretary (4) Vice - President (5) Financial Advisor 31. Read the following information carefully and answer the questions which follows: If ?A X B? means ?A is father of B? If ?A + B? means ?A is daughter of B? If ?A ÷ B? means ?A is son of B? If ?A - B? means ?A is sister of B? What will come in the place of the question mark, to establish that P is the son - in-law of S in the expression: P X Q + R - T ? S (1) + (2) X (3)- (4) ÷ (5) Either + or ÷ 32. How many such pairs of letters are there in the word SEARCHES each of which has as many letters between them in the word as in the English alphabet? (1) None (2) One (3) Two (4) Three (5) More than three Directions (Qs. 33- 35): Study the following information and answer the following questions. In a certain code, ?he is waiting there? is written as ?la pa ro ta?, ?there is the train? is written as ?zo ro ji la?, ?waiting at the station? is written as ?ma ta fu ji? and ?is this a station? is written as ?fu bi ro vi?. 33. What is the code for ?he?? (1) la (2) pa (3) ro (4) ta (5) Either la or zo 34. Which of the following represents ?the train station?? (1) zo la ma (2) fu ji ta (3) fu ji zo (4) ro zo fu (5) Cannot be determined 35.Which of the following may represent ?guard is waiting?? (1) ro ta zo (2) ta ki ro (3) fu zo ki (4) ta ro ji (5) la ma ro SBI Preliminary Paper Reasoning Ability 1. How many such pairs of letters are there in the word TRIBUNAL each of which has as many letters between them in the word as in the English alphabet? a) None b) One c) Two d) Three e) More than three 2. In a certain code DOWN as ?5@9#? and NAME is written as ?#6%3?. How is MODE written in that code? a) %653 b) %@63 c) %5@3 d) %@53 e) None of these 3. How many meaningful English words can be formed with the letters LGEU using each letter only once in each word? a) None b) One c) Two d) Three e) More than three 4. In a certaincode THRIVES is written as SIUHRDU. How is SOULFUL written in that code? a) VPTKKTE b) VPTKETK c) TPVKKTE d) TNRKMVG e) None of these 5. The positions of how many digits in the number 59164823 will remain unchanged after the digits are rearranged in descendign order within the number? a) None b) One c) Two d) Three e) More than three 6. Mohan walked 30 metres towards South, took a left turn and walked 15 metres. He then took a right turn and walked 20 metres. He again took a right turn and walked 15 metres. How far is he from the starting point? a) 95 metres b) 50 metres c) 70 metres d) Can?t be determined e) None of these 7. What should come next in the following letter series? PQRSTABCDEPQRSABCDEPQRSABCDPQ a) R b) T c) A d) B e) None of these 8. In a certain code language, ?how can you go? is written as ?ja da ka pa?. ?can you come here? is written as ?na ka sa ja? and ?come and go? is written as ?ra pa sa?. How is ?here? written in that code language? a) ja b) na c) pa d) Data inadequate e) None of these 9. What should come next in the following letter series based on English alphabet? CEA IKG OQM ? a) STW b) WUS c) SWU d) UWS e) None of these Directions (Q. 10-14) In each of the questions below are given four statements followed by four conclusions numbered I, II, III and IV. You have to take the given statements to be true even if they seem to be at variance from commonly known facts. Read all the conclusions and then decide which of the given conclusions logically follows from the given statements disregarding commonly known facts. 10. Statements: Some trains are cars. All cars are branches. All branches are nets. Some nets are dresses. Conclusions: I. Only I and II follow II. Only II and III follow III. Only I and IV follow IV. Some roads are clips. a) Only I and II follow b) Only II and III follow c) Only I and IV follow d) Only II, III and IV follow e) None of these 11. Statements: All papers are clips. Some clips are boards. Some boards are lanes. All lanes are roads. Conclusions: I. Some roads are boards. II. Some lanes are clips. III. Some boards are papes. IV. Some roads are clips. a) Only I and II follow b) Only I and III follow c) Only I, II and III follow d) Only II, III and IV follow e) None of these 12. Statements: Some pencils are kites. Some kites are desks. All desks are jungles. All jungles are mountains. Conclusions: I. Some mountains are pencils. II. Some jungles are pencils. III. Some mountains are desks. IV. Some jungles are kites. a) Only I and III follow b) Only I, II and III follow c) Only III and IV follow d) Only II, III and IV follow e) None of these 13. Statements: All stones are hammers. No hammer is ring. Some rings are doors. All doors are windows. Conclusions: I. Some windows are stones. II. Some windows are rings. III. No window is stone. IV. Some rings are stones. a) Only I follows b) Only II follows c) Only III follows d) Only eithre I or III follows e) Only either I or III and II follow 14. Statements: All pens are clocks. Some clocks are tyres. Some tyres are wheels. Some wheels are buses. Conclusions: I. Some buses are tyres. II. Some wheels are clocks. III. Some wheels are pens. IV. Some buses are clocks. a) None follows b) Only I follows c) Only II follows d) Only III follows e) Only IV follows Directions (Q. 15-19) Study the following information carefully and answer the questions given below: A, B, C, D, E, F, G, H and K are sitting around a circle facing the centre. F is 4th to the right of A who is 3rd to the right of B. K is 4th to the left of B and 3rd to the right of D. C is 3rd to the right of H. E is 2nd to the left of G. 15. Who is to the immediate right of F? a) B b) G c) E d) Data inadequate e) None of these 16. Who is 3rd to the right of K? a) F b) E c) G d) Data inadequate e) None of these 17. What is E?s position with respect to B? a) 2nd to the left b) 3rd to the right c) 4th to the right d) 3rd to the left e) 5th to the right 18. Who is 4th to the left of G? a) C b) A c) D d) K e) Data inadequate 19. In which of the following combinations is the 3rd person sitting betweenthe 1st and the 2nd persons? a) GFB b) BGH c) ADC d) KEC e) EGF Directions (Q. 20-24) In the following questions, the symbols !, @, ©, % and * are used with the following meaning as illustrated below: ?P © Q? means ?P is not smalle than Q?. ?P % Q? means ?P is neither smaller than nor equal to Q?. ?P * Q? means ?P is neither greater than nor equal to Q?. ?P ! Q? means ?P is not greater than Q?. ?P @ Q? means ?P is neither greater than nor smaller than Q?. Now in each of the following questions assuming the given statements to be true, find which of the three conclusions I, II, III and IV given below them is/are definitely true and give your answer accordingly. 20. Statements: D ! T, T @ R, R © M, M % K Conclusions: I. R @ D II. R % D III. K * T IV. M ! T a) Only either I or II is true b) Only III and IV are true c) Only either I or II and III are true d) Only either I or II and IV are true e) Only either I or II and III and IV are true 21. Statements: J @ F, F ! N, N % H, H © G Conclusions: I. G * N II. N © J III. F * J IV. J ! G a) Only I and II are true b) Only I, II and III are true c) Only II, III and IV are true d) All I, II, III and IV are true e) None of these 22. Statements: R * K, K % D, D @ V, V ! M Conclusions: I. R * D II. V * R III. D @ M IV. M % D a) None is true b) Only III is true c) Only IV is true d) Only either III or IV is true e) None of these 23. Statements: B © T, T * R, R % F, F @ K Conclusions: I. B % R II. F * T III. R % K IV. K & T a) None is true b) Only I is true c) Only II is true d) Only III is true e) Only IV is true 24. Statements: F % N, N © W, W ! Y, Y - T Conclusions: I. F % W II. T % N III. N % Y IV. T % W a) Only I and III are true b) Only I and IV are true c) Only II and III are true d) Only I, II and IV are true e) None of these Directions (Q. 25-29) In making decision about important question, it is desirable to be able to distinguish between ?strong? arguments are those which are both important and directly related to the question. ?Weak? arguments are those which are of minor importance and also may not be directly related to the question or may be related to a trivial aspect of the question. Each question below is followed by three arguments numbered (A), (B) and (C). You have to decide which is the arguments is a ?strong? arguments and which is a ?weak? argument. 25. Statement Should there be complete ban on setting up of thermal power plants in India? Arguments: (A) Yes, this is the only way to arrest further addition to environmental pollution. (B) No, there is a huge shortage of electricity in most parts of the country and hence generation of electricity needs to be augmented. (C) No, many developed countries continue to set up thermal power plants in their countries. a) None is strong b) Only (A) is strong c) Only (B) is strong d) Only (C) is strong e) Only either (A) or (B) is strong 26. Statements: Should road repair work in big cities be carried out only late at night? Arguments: (A) No, this way the work will never get completed. (B) No, there will be unncessary use of electricity. (C) Yes, the commuters will face lot of problems due to repair work during the day. a) None is strong b) Only (A) is strong c) Only (C) is strong d) Only (B) and (C) are strong e) Only (A) and (B) are strong 27. Statements: Should all the deemed universities be derecognized and attached to any of the central of state universities in India? Arguments: (A) Yes many of these deemed universities do not conform to the required standards of a full-fledged university and hence the level of education is compromised. (B) No, these deemed universities have been able to introduce innovative courses suitable to the requirement of various industries as they are free from strict government controls. (C) Yes, many such universities are basically money spinning activities and education takes a backseat in these institutions. a) Only (A) and (B) are strong b) Only (B) and (C) are strong c) Only (A) and (C) are strong d) All (A), (B) and (C) are strong e) None of these 28. Statement: Should there be a cap on drawing groundwater for irrigation purposes in India? Arguments: (A) No, irrigation is of prime importance for food production in India and it is heavily dependent on groundwater in many parts of the country. (B) Yes, water tables have gone down to alarmingly low levels in some parts of the country where irrigation is primarily dependent on groundwater, which may lead to serious environmental consequences. (C) Yes, India just cannot afford to draw groundwater any furthre as the international agencies have cautioned India against it. a) Only (A) and (B) are strong b) Only (B) and (C) are strong c) Only (A) and (C) are strong d) All (A), (B) and (C) are strong e) None of these 29. Statements: Should there be a restriction on the construction of high rise building in big cities in India? Arguments: (A) No, big cities in India do not have adequate open land plots to accommodate the growing population. (B) Yes, only the builders and developers benefit from the construction of high rise buildings. (C) Yes, the Government should first provide adequate infrastructural facilities to existing building before allowing the construction of new high rise buildings. a) Only (B) is strong b) Only (C) is strong c) Only (A) and (C) are strong d) Only (A) is strong e) None of these Directions (Q. 30-34) In each question below is given a statement followed by three assumptions (A), (B) and (C). An assumption is something supposed or taken for granted. You have to consider the statements and the following assumptions and decide which of the assumptions is implicit in the statement. 30. Statement: The Government has decided to auction construction of highways to private entities in several blocks across the country on build-operate-transfer basis. Which of the following assumption(s) is/are implicit in the above statement? (A) An adequate number of private entities may not respond to the Government?s auction notification. (B) Many private entities in the country are capable of constructing highways within reasonable time. (C) The Government?s proposal of build-operate-transfer may financially benefit the private entities. a) Only (A) and (B) are implicit b) Only (B) and (C) are implicit c) Only (B) is implicit d) Only (A) and (C) are implicit e) None of these 31. Statement: Government has urged all the citizens to use electronic media for carrying out their daily activities, whenever possible, instead of using paper as the manufacture of paper requires the cutting down of a large number of trees causing severe damage to the ecosystem. Which of the following assumption(s) is/are implicit in the above statement? (A) Most people may be capable of using electronic media to carry out various routines. (B) Most people may have access to electronic media for carrying out their daily routine activities. (C) People at large may reject the Government?s appeal and continue using paper as before. a) Only (A) is implicit b) Only (B) is implicit c) Only (A) and (B) are implicit d) Only (C) is implicit e) None of these 32. Statement: The apex body controlling universities in the country has decided to revise the syllabus of all the technical courses to make them focused towards the present needs of the industry thereby making the technical graduates more employable than they are at present. Which of the following assumption(s) is/are implicit in the above statement? (A) Technical colleges affiliated to different universities may not welcome the apex body?s decision and may continue with the same syllabus as at present. (B) The industry may welcome the decision of the apex body and scale up their hiring from these colleges. (C) The Government may not allow the apex body to implement its decision in all the colleges as it may lead to chaos. a) None is implicit b) Only (A) is implicit c) Only (B) is implicit d) Only (C) is implicit e) Only (A) and (B) are implicit 33. Statement: Police authority cordoned off the entire locality for the entire day and stopped all vehicular movement for the visit of a top functionary of the government in view of the threat perception and advised all the residents in the area to limit their movement outside their dwellings. Which of the following assumption(s) is/are implicit in the above statement? (A) Police personnel may not be able to control the vehicular movement in the locality and may seek help from the armed forces. (B) People living in the locality may move out of their houses for the day to avoid inconvenience. (C) The Government functionary may request the police authority to lift the ban on movement residents of the locality outside their dwellings. a) None is implicit b) Only (A) is implicit c) Only (B) is implicit d) Only (C) is implicit e) None of these 34. Statement: The airlines have requested all their bonafide passengers to check the status of flight operations before leaving their homes as heavy fog is causing immense problems to ormal flight operations. Which of the following assumption(s) is/are implicit in the above statement? (A) The majority of the air passengers may check the flight status before starting their journey to the airport. (B) The Government may take serious objection to the notice issued by the airline company. (C) Majority of the passengers may cancel their tickets and postpone their journey till the situation becomes normal. a) None is implicit b) Only (A) is implicit c) Only (B) is implicit d) Only (C) is implicit e) None of these SBI Data Intrepretation SBI Data Interpretation solved question papersReasoning ability solved question papers SBI stands for State Bank of India, SBI Central Recruitment and promotion of Department recruit clerical cadre in Associate Banks of State Bank of India SBI latest bank profile, SBI, free solved sample placement papers of clerk, SBI Associate Banks Clerks Recruitment 2012 notification and job details, SBI Dates of Written Examination : 07.10.2012 and 14.10.2012,SBI previous years solved question papers, SBI 2010,2011,2012 Clerks and po officer job written test examination syllabus and selection procedure, SBI PO and Associate banks clerks examination questions papers, SBIAssociate Banks Clerks Recruitment 2012 notification and detailed explanation with answers and solutions, How to crack Sbi written test examination tips and trick,SBI, General Awareness, General English, Quantitative Aptitude, Reasoning Ability, Marketing Aptitude / Computer Knowledge whole solved questions paper, Latest general awareness SBI Associate Bank clerical cadre question papers and selection procedure and test pattern Study the following table and answer the questions based on it. Number of Candidates Appeared, Qualified and Scheduled in a Competitive Examination from Five States Delhi, H.P, U.P, Punjab and Haryana Over the Years 1994 to 1998 1. For which state the average number of candidates selected over the years is the maximum? A. Delhi B H.P C. U.P D. Punjab Answer: Option A Explanation: The average number of candidates selected over the given period for various states are: For Delhi = 94 + 48 + 82 + 90 + 70 /5 = 384 /5 = 76.8. For H.P. = 82 + 65 + 70 + 86 + 75 /5 = 378 /5 = 75.6. For U.P. = 78 + 85 + 48 + 70 + 80 /5 = 361 /5 = 72.2. For Punjab = 85 + 70 + 65 + 84 + 60 /5 = 364/5 = 72.8. For Haryana = 75 + 75 + 55 + 60 + 75 /5 = 340 /5 = 68. Clearly, this average is maximum for Delhi. 2. The percentage of candidates qualified from Punjab over those appeared from Punjab is highestin the year? A. 1997 B. 1998 C. 1999 D. 2000 Option D Explanation: The percentages of candidates qualified from Punjab over those appeared from Punjab during different years are: For 1997 = ( 680 x 100 ) /8200 % = 8.29%. For 1998 = ( 600 x 100 ) 6800 % = 8.82%. For 1999 = ( 525 x 100 ) /6500 % = 8.08%. For 2000 = ( 720 x 100 ) /7800 % = 9.23%. For 2001 =( 485 x 100 ) /5700 % = 8.51%. Clearly, this percentage is highest for the year 2000. 3. In the year 1997, which state had the lowest percentage of candidates selected over the candidates appeared? A. Delhi B. H.P C. U.P D. Punjab Answer: Option D Explanation: The percentages of candidates selected over the candidates appeared in 1997, for various states are: (i) For Delhi = ( 94 x 100 )/ 8000 % = 1.175%. (ii) For H.P. = ( 82x 100 )/ 7800 % = 1.051%. (iii) For U.P. =( 78 x 100 ) /7500 % = 1.040%. (iv) For Punjab(85 x 100 ) /8200 % = 1.037%. (v) For Haryana ( 75 x 100 )/6400 % = 1.172%. Clearly, this percentage is lowest for Punjab. 4. The number of candidates selected from Haryana during the period under review is approximately what percent of the number selected from Delhi during this period? A. 79.5% B. 81% C.84.5% D. 88.5% Answer: Option D Explanation: Required percentage = [ (75 + 75 + 55 + 60 + 75)/(94 + 48 + 82 + 90 + 70) x 100 ] % = [ 340 x 100 ] /384%== 88.54% = 88.5% 5. The percentage of candidates selected from U.P over those qualified from U.P is highest in the year? A. 1997 B. 1998 C. 1999 D. 2001 Answer: Option B Explanation: The percentages of candidates selected from U.P. over those qualified from U.P. during different years are: For 1997 = ( 78 x 100 ) /720% = 10.83%. For 1998 = ( 85 x 100 ) /620% = 13.71%. For 1999 = ( 48 x 100 ) /400 % = 12%. For 2000 = ( 70 x 100 ) /650% = 10.77%. For 2001 = (80 x 100 ) /950% = 8.42%. Clearly, this percentage is highest for the year 1998. 1. Directions to Solve The following bar chart represents the GDP of different countries during the half decades 2001 - 2005 and 2006 - 2010. All figures are in Rs. billion. Which of the countries listed below accounts for the maximum GDP during the half decade 2006 to 2010 ? A. UAE B. US C. India D. China Answer: Option B Explanation: It's clear that US is the highest amongst the given options. 2 The GDP of UAE is what fraction of GDP of the UK for the decade (approximately) ? A. (1/4)th B. (1/5)th C. (1/6)th D. Data inadequate Answer: Option B Explanation: (10/50) = 20% = (1/5)th 3. Which of the countries listed below accounts for the highest GDP during the half decade 2001 to 2005 ? A. Russia B. China C. India D. UAE Answer: Option A Explanation: It's clear that Russia is the answer 4. Out of every Rs. 10,000 spent during the decade 2001 - 2010 approximately how much was the GDP of Russia during the half decade 2001 - 2005 ? A. Rs. 700 B. Rs. 1,400 C. Rs. 2,800 D. None of these Answer: Option D Explanation:Russia will account for 10 out of 125, i.e, 8% of the total, i.e, 800 out of 10,000 (approximately values based on visual interpretation). Study the following pie-chart and the table and answer the questions based on them. Proportion of Population of Seven Villages in 1997 1. If the population of village R in 1997 is 32000, then what will be the population of village Y below poverty line in that year? A. 14100 B.15600 C. 16500 D. 17000 Answer: Option B Explanation: Population of village R = 32000 (given). Let the population of village Y be y. Then, 16 : 15 = 32000 : y=> y = ( 15 x 32000) /16 = 30000. Therefore Population of village Y below poverty line = 52% of 30000 = 15600. 2 The ratio of population of village T below poverty line to that of village Z below poverty line in 1997 is: A. 11 : 23 B. 13 : 11 C. 23 : 11 D. 11 : 13 Answer: Option C Explanation: Let N be the total population of all the seven villages. Then, population of village T below poverty line = 46% of (21% of N) and Population of villages Z below the poverty line = 42% of (11% of N) Therefore Required ratio = 46% of (21% of N) = 46 x 21 = 22000. 42% of (11 % of N) 42 x 11 3. Find the population of village S if the population of village X below poverty line in 1997 is 12160. A. 18500 B. 20500 C. 22000 D. 26000 Answer: Option C Explanation: Let the population of village X be x. Then, 38% of x = 12160 => x = 12160 x 100 /38= 32000. Now, if s be the population of village S, then 16 : 11 = 32000 : s => s = ( 11 x 3200 ) /16= 22000. 4. If in 1998, the population of villages Y and V increase by 10% each and the percentage of population below poverty line remains unchanged for all the villages, then find the population of village V below poverty line in 1998, given that the population of village Y in 1997 was 30000. A. 11250 B. 12760 C. 13140 D.13780 Answer: Option B Explanation: Population of village Y in 1997 = 30000 (given). Let the population of village V in 1997 be v. Then, 15 : 10 = 30000 : v => v = 30000 x 10 /15= 20000. Now, population of village V in 1998 = 20000 + (10% of 20000) = 22000. Therefore Population of village V below poverty line in 1998 = 58% of 22000 = 12760. Placement Paper SBI OBJECTIVE GENERAL ENGLISH practice questions, Sbi Aptitude, Reasoning and verbal ability Questions with answers and detailed solutions SBI SBI PRACTICE SET 1 Directions : In each of the following questions, out of the given alternatives, choose the one which best expresses the meaning of the given word. 1. EAGER (a) Clever (b) Enthusiastic (c) Curious (Ans) (d) Devoted 2. PLACID (a) Clear (b) Calm (Ans) (c) Enjoyable (d) Dull 3. COMMEMORATE (a) Boast (b) Remember (Ans) (c) Manipulate (d) Harmonise 4. DEIFY (a) Face (b) Worship (Ans) (c) Flatter (d) Challenge 5. AMENITIES (a) Pageantries (b) Privileges (Ans) (c) Facilities (d) Courtesies 6. DISPARITY (a) Injustice (b) Unlikeness (Ans) (c) Partiality (d) Distortion 7. BARBARIAN (a) Arrogant (b) Impolite (c) Uncivilized (Ans) (d) Unkind 8. LETHAL (a) Unlawful (b) Deadly (Ans) (c) Sluggish (d) Smooth 9. SERENE (a) Solemn (b) Meak (c) Delicate (d) Calm (Ans) 10. GRUESOME (a) Hateful (b) Painful (c) Tragic (d) Frightful (Ans) 11. SECURE (a) Secret (b) Comfortable (c) Safe (Ans) (d) Independent 12. TYRANNY (a) Misrule (b) Power (c) Madness (d) Cruelty (Ans) 13. OBLIGATORY (a) Useful (b) Required (Ans) (c) Stubborn (d) Agreeable 14. CONNOISSEUR (a) Ignorant (b) Interpreter (Ans) (c) Delinquent (d) Lover of art 15. IMPETUOUS (a) Violent (b) Resourceful (c) Pleasing (d) Rash (Ans) 16. NEBULOUS (a) Tiny (b) Vague (Ans) (c) Insignificant Placement Paper SBI Bank Probationary Officers Examination PO Written Test of English Language Previous years solved question papers for practice, Test of Reasoning, High level, Test of English Language, Test of Data analysis, Data Interpretation, General awareness solved question papers with detailed explanations, SBI SBT IBPS PO clerical specialists officers, All bank PSU IT Non IT UPSC,SSC,SSB,AFCAT CAT GATE all examinations calendar, solved question papers, free solved sample placement papers, Tips tricks, Recruitment details, written Test examination Procedure, Test pattern etc.. SBI English Language Question with answers Direction (1 - 5) : Read the following passage carefully and answer the questions given below it. Certain words/phrases are printed in bold to help you to locate them while answering some of the questions. The outside world has pat answers concerning extremely impoverished countries, especially those in Africa. Everything comes back, again and again, to corruption and misrule. Western officials argue that Africa simply needs to behave itself better, to allow market forces to operate without interference by corrupt rulers. Yet the critics of African governance have it wrong. Politics simply can't explain Africa's prolonged economic crisis. The claim that Africa's corruption is the basic source of the problem does not withstand serious scrutiny. During the past decade I witnessed how relatively well-governed countries in Africa, such as Ghana, Malawi, Mail and Senegal, failed to prosper, whereas societies in Asia perceived to have extensive corruption, such as Bangladesh, Indonesia and Pakistan, enjoyed rapid economic growth What is the explanation ? Every situation of extreme poverty around the world contains some of its own unique cases, which need to be diagnosed as a doctor would a patient. For example. Africa is burdened with malaria like no other part of the world, simply because it is unlucky in providing the perfect conditions for that disease : high temperatures, plenty of breeding sites and particular species of malaria transmitting mosquitoes-that prefer to bite humans rather than cattle. Another myth is that the developed world already gives plenty of aid to the world's poor. Former U.S. Secretary of the Treasury, Paul O'Neil expressed a common frustration when he remarked about aid for Africa : "We've spend trillions of dollars on these problems and we have damn near nothing to show for it". O'Nell was no foe of foreign aid. Indeed, he wanted to fix the system so that more U.S. aid could be justified. But he was wrong to believe that vast flows of aid to Africa had been squandered. President Bush said in a press conference in April 2004 that as 'the greatest power on the face of the earth, we have an obligation to help the spread of freedom. We have an obligation to feed the hungry". Yet how does the U.S. fulfill its obligation ? U.S. aid to fanners in poor countries to help them grow more food runs at around $ 200 million per year, far less than S1 per person per year for the hundreds of millions of people living in subsistence farm households. From the world as a whole, the amount of aid per African per year is really very small, just $30 per sub-Saharan African in 2002. Of that modest amount, almost $5 was actually for consultants from the donor countries, more than $3 was for emergency aid, about $4 went for servicing Africa's debts and $5 was for debt-relief operations. The rest, about $12, went to Africa. Since the "money down the drain" argument is heard most frequently in the U.S. it's worth looking at the same calculations for U.S. aid alone In 2002, the U.S. gave $3 per sub Saharan African. Taking out the parts for U.S. consultants and technical co-operation, food and other emergency aid, administrative costs and debt relief, the aid per African came to grand total of 6 cents. The U.S. has promised repeatedly over the decade, as a signatory to global agreements like the Monterrey Consensus of 2002, to give a much larger proportion of its annual output specifically up to 0.7% of GNP, to officially development assistance. The U.S.'s failure to follow through has no political fallout domestically, of course. because not one in a million U.S. citizens even knows f statements like the Monterrey Consensus, But no one should underestimate the salience that it has around the world. Spin as American might about their nation's generosity, the poor countries are fully aware of what the U.S. is not doing. 1. Which of the following statement is TRUE about U.S. aid to the Sub-Saharan African countries ? (a) the U.S. aid meant for of capital African does not reach the incumbent (b) the U.S. aid to African coun tries is more than that for any other developing or under developed nation (c) the U.S. aid for farmers in African Countries is $200 m. per year (d) the donor country charges $5 per individual as the constancy charges (e) U.S. has been contributing more than 0.7% of its GNP for development assistance (ANS) 2. President Bush's statement in Press Conference in April 2004 indicates that ..... (a) the aid given by the U.S. to the poor countries is substantial and sufficient (b) the spread of freedom cannot be achieved through financial aid (c) feeding the hungry million outside the U.S. is not possible (d) the U.S. on its own, assumes the obligation of helping the poor countries (ANS) (e) U.S. has spent trillions of dollars on aid 3. The author has mentioned Ghana as a country with ....... (a) reasonably good-governance (ANS) (b) corrupt leadership (c) plenty of natural resources (d) rapid economic growth (e) none of these 4. The cases of malaria in Africa are mainly due to............ (A) high temperature (B) climatic conditions conducive for breeding (C) malaria carriers' liking for human blood in preference to that of cattle. (a) None of these (b) Only B & C (c) Only A & C (d) Only A & B (e) All the three (ANS) 5. The purpose of the author in writing this passage seems to ............ (a) criticize USA for not providing adequate financial help (b) make Africans realize their own problems (c) analyze the actual quantum of aid against the perceived one (d) highlight how American leaders are power-hungry (e) none of these (ANS) 6. The author has given the example of Bangladesh, Indonesia and Pakistan in support of his argument that ............ (a) corruption is the major culprit in the way of prosperity (b) mis-governance hampers the prosperity of nations (c) despite rampant corruption, nations may prosper (ANS) (d) developed nations arrogantly neglect underdeveloped countries (e) none of these 7. The remark of former U.S. Secretary of the Treasury, Paul O'Neil, is according to the author ........ (a) a statement of fact (b) not factually correct (ANS) (c) an underestimation of U.S and (d) a ruthless remark by an arrogant bureaucrat (e) none of these 8. The passage seem to emphasize that the outside world has (a) correct understanding about the reasonable aid provided by the USA to the poor countries (b) definite information about what is happening in under developed countries (c) stopped extending any financial aid to underdeveloped countries (d) misconceptions about the aid given to the poor nations by developed countries (e) none of these (ANS) 9. According to the Westerners the solution to eradicate poverty of African nations lies in ............. (a) corruption (b) improving their own national behavior (c) misrule (d) prolonged economic crisis (ANS) (e) none of these Directions (10-12) : Which of the following word/group of words is most OPPOSITE in meaning of the word given in bold as used in the passage. 10. extensive (a) intensive (b) abominable (c) inherent (d) rampant (e) negligible (ANS) 11. prolonged (a) immediate (b) shortened (c) brevity (d) short-lived (ANS) (e) narrow 12. myth (a) reality (ANS) (b) mystery (c) misery (d) misconception (e) exaggeration Directions (13-15) : Which of the following word/group of words is MOST NEARLY THE SAME in meaning as the word printed in bold as used in the passage ? 13. squander (a) use economically (b) spend wastefully (ANS) (c) siphon judiciously (d) donate generously (e) donate with ulterior motive 14. modest (a) humble (ANS) (b) sufficient (c) meagre (d) sober (e) unpretentious 15. obligation (a) lip sympathy (b) true empathy (c) self pity (d) conditional responsibility (e) moral binding (ANS) Directions (16-25) : Read each sentence to find out whether there is any grammatical error in it. The error if any, will be in one part of the sentence. The number of that part is the answer. If there is no error; the answer is (e) i.e. 'No Error'. (Ignore the errors of punctuation, if any). 16. He was grateful (a) / to me for the help (b) / that I had extended for him (c) / in the hour of his need. (d) / No Error (e) Ans : e 17. While undergoing a (a) / strenuous workout, (b) / he suffered a massive heart attack (c) / but luckily survival. (d) / No Error (e) Ans : d 18. Taking care of yourself (a) / cannot be (b) / and should not be considered (c) / as a selfish thing. (d) / No Error (e) Ans : b 19. The smooth and easier (a) / we allow people to navigate (b) / through any device at any place (c) / in the world, the better. (d) / No Error (e) Ans : a 20. What really agonize them (a) / is the presence of (b) / an unwanted and unscrupulous (c) / member on the panel. (d) / No Error (e) Ans : d 21. People express their expectations (a) / that their leaders should not (b) / resorting to (c) / corrupt practices and nepotism, (d) / No Error (e) Ans : c 22. Asian culture will, (a) / sooner or later (b) / become international norm (c) / for entertainment industry. (d) / No Error (e) Ans : d 23. The business lobby wanted (a) / he to take over (b) / as the new Chairman (c) / of their coveted Board. (d) / No Error (e) Ans : b 24. He thinks that (a) / a popular magazine would like to (b) / know what are other topics most (c) / no people's minds. (d) / No Error (e) Ans : b 25. We as the editors of (a) / a popular magazine would like to (b) / know what are other topics most (c) / on people's minds. (d) / No Error (e) Ans : c Directions (26-30) : Rearrange the following six sentences (A), (B), (C), (D), (E) and (F) in the proper sequence to form a meaningful paragraph, then answer the questions given below them. (A) It is a general term used to describe over 200 individual diseases. (B) The abnormal cells grow with-out any control, invade through normal tissue barriers and reproduce indefinitely. (C) The word "cancer" comes from Latin, meaning a crab. (D) These characteristics include development within any tissue of a malignant growth. (E) A tumour was called cancer because of swollen veins around the area resembling a crab's limbs. (F) These diseases progress differently over a period of time and share certain characteristics. 26. Which of the following should be the SECOND sentence after rearrangement ? (a) A (b) B (c) C (d) D (e) E (ANS) 27. Which of the following should be the THIRD sentence after rearrangement ? (a) A (b) B (ANS) (c) C (d) D (e) E 28. Which of the following should be the FOURTH sentence after rearrangement ? (a) A (b) B (c) C (d) D (ANS) (e) E 29. Which of the following should be the FIFTH sentence after rearrangement ? (a) A (ANS) (b) B (c) C (d) D (e) None of these 30. Which of the following should be the SIXTH (LAST) sentence after rearrangement ? (a) A (b) B (c) C (d) D (e) None of these (ANS) Directions (31-35) : In each of the following sentences there are two blank spaces. Below each sentence there are five pairs of words denoted by numbers (a), (b), (c), (d) and (e). Find out which pair of words can be filled up in the blanks in the sentence in the same sequence to make the sentence grammatically correct and meaningfully complete. 31. Liberalization has removed all the legal .......... and .............. flood-gates to multinational companies. (a) hurdles ............. awarded (b) barriere ............ opened (ANS) (c) obstacles ........... guarded (d) manipulation .......... closed (e) battles ............. threw 32. The .......... on some of the towns has created ........ among the residents of the other part of the country. (a) attack .............. ambition (b) raid .......... awareness (c) bombardment ......... panic (ANS) (d) spell ........... satisfaction (e) shower ............. dampness 33. ............. to the popular belief that every astrologer nurtures blind faith in fate, our astrologer believes in (a) Contrary ........ action (ANS) (b) According .......... thoughts (c) Bowing ............... present (d) Proving ............. forecasting (e) Pointing .............. devotion 34. His ........... has yielded him the ............. fruit. (a) fate ......... undesirable (b) efforts ........ unwanted (ANS) (c) action .......... viable (d) perseverance ............... desired (e) emphasis ....... expected 35. His ............ contribution to the Tsunami relief fund was ............. by his staff members. (a) meagre ............ admired (b) spontaneous ............ nullified (c) negligible ............ sanctioned (d) noteworthy ............ improved (e) generous ............ appreciated (ANS) Directions (36-40) : For each statement there are three different sentences given below it. Pick out the sentence (s) that most appropriately conwey (s) the meaning of the statement The number of that sentence or combination of sentences is the answer. 36. I could not observe his unwillingness while donating to charity. (A) I observed his willingness to donate to charity. (B) I failed to detect his reluctance, if there was any, while he was donating to charity. (C) I noticed his enthusiasm while donating to charity. (a) Only A (b) Only B (ANS) (c) Only C (d) Only A and B (e) Only B and C 37. If you hesitate to participate freely and vigorously, your performance will be adversely affected. (A) Participate freely and vigorously if you desire to demonstrate your full performance. (B) Do not participate with reservations if you want your participation to be affected adversely. (C) your free and vigorous participation is most likely to affect your performance adversely. (a) Only A (ANS) (b) Only B (c) Only C (d) Only A and B (e) Only B and C 38. Who do you think will be able to handle the work effectively ? (A) In your opinion, there is none who can handle the work effectively. (B) Who, in your opinion, can handle the work effectively ? (C) Can you name the person who, as per your assessment, can handle the work effectively ? (a) Only A (b) Only B (c) Only C (d) Only A and B (e) Only B and C (ANS) 39. How much you speak is less important than how well you speak ? (A) The quality of your talk is less important than the quantum of your talk. (B) How good you speak is as important as how much you speak. (C) The quality of your talk as also the quantum are equally important. (a) Only A and B (b) Only B and C (ANS) (c) All the three (d) None (e) Only A 40. All our efforts were wasted due to the negligence of the clients. (A) The negligence of the clients was responsible for making our efforts futile. (B) Our efforts would have been fruitful had there been no negligence on the part of the clients. (C) The negligence of the clients saved all our efforts from being wasted. (a) Only A (b) Only B (c) Only A and B (ANS) (d) Only B and C (e) All the three Directions (41-50) : In the following passage, there are blanks. each, of which has been numbered. These numbers are printed below the passage and against each, five words are suggested, one of which Fits the blank appropriately. Find out the appropriate word in each case, New technology has led directly to (41) standards of living yet science tends to follow market forces as well as to (42) them, It is not surprising that the rich get richer in a continuing cycle of (43) while the poorest are often left behind. A special (44) should be made by the powerhouses of world science to address the unmet challenges of the poor. Ending (45) poverty can relieve many of the pressures on the environment. When impoverished households are (46), (47) on their farms, for example, they face less pressure to cut down neighboring forests in (48) of new farmland. Still, even as extreme poverty ends, we must not fuel prosperity with a lack of (49) for industrial pollution and the (50) burning of fossil fuels. 41. (a) visible (b) declining (c) Improved (ANS) (d) amicable (e) rigorous 42. (a) fail (b) claim (c) market (d) avoid (e) lead (ANS) 43. (a) wealth (b) growth (c) poverty (d) improvement (ANS) (e) economy 44. (a) effort (b) care (ANS) (c) practice (d) occasion (e) sanction 45. (a) marginal (b) apparent (c) superficial (d) extreme (ANS) (e) dismal 46. (a) abnormally (b) less (c) more (ANS) (d) excessively (e) unreasonably 47. (a) efficient (b) meticulous (c) careful (d) dependent (ANS) (e) productive 48. (a) view (b) search (ANS) (c) expectation (d) lust (e) place 49. (a) attitude (b) mobility (c) initiative (d) concern (ANS) (e) ease 50. (a) unchecked (b) repeated (ANS) (c) periodical (d) occasional (e) limited General - other SBI Reasoning ability solved question papers SBI stands for State Bank of India, SBI Central Recruitment and promotion of Department recruit clerical cadre in Associate Banks of State Bank of India SBI latest bank profile, SBI, free solved sample placement papers of clerk, SBI Associate Banks Clerks Recruitment 2012 notification and job details, SBI Dates of Written Examination : 07.10.2012 and 14.10.2012,SBI previous years solved question papers, SBI 2010,2011,2012 Clerks and po officer job written test examination syllabus and selection procedure, SBI PO and Associate banks clerks examination questions papers, SBIAssociate Banks Clerks Recruitment 2012 notification and detailed explanation with answers and solutions, How to crack Sbi written test examination tips and trick,SBI, General Awareness, General English, Quantitative Aptitude, Reasoning Ability, Marketing Aptitude / Computer Knowledge whole solved questions paper, Latest general awareness SBI Associate Bank clerical cadre question papers and selection procedure and test pattern SBI PO GENERAL AWARENESS SOLVED PAPER 1. On 7th May 2008, agni-I was test-fired. It was (A) Ballistic Missile (B) Surface to air missile (C) Land to sea missile (D) Air to air missile 2. Which of the following High Courts has recently quashed the merit list issued in 2006 and 2007 by the centre and the UPSC (A) Bhopal High Court (B) Rajasthan High Court (C) Madras High Court (D) Guwahati High Court 3. Recently the Acronym START came into News. START stands for” (A) Strategic Association for Restructuring Trade (B) Strategic Arms Limitation Treaty (C) System Tracking and Reprocessing Team (D) None of the above 4. People™s Democratic Party emerged second largest party with 33·01% in elections held in” (A) Bhutan (B) Nepal (C) Zimbabwe (D) Italy 5. According to the World Conservation Union Report, the number of plant species in India listed as threatened is (A) 247 plant species (B) 250 plant species (C) 275 plant species (D) 255 plant species 6. Which of the following states passed a Bill in March 2008, amending the Registration Act 1908 to prohibit foreigners from buying land in the state (A) Kerala (B) Andhra Pradesh (C) Goa (D) Maharashtra 7. The renowned scientist John Wheeler who died in April 2008, was originally hailed from (A) Australia (B) U.S.A. (C) Finland (D) Britain 8. Who received Indira Gandhi Prize for Peace, Disarmament and Development for 2007 (A) Bill Gates (B) Nelson Mandela (C) Bill and Melinda Gates Foundation (D) Arthur C. Clarke 9. The 2008 UNESCO/Guillermo Cano World Press Freedom Prize awarded to (A) Lydia Cacho Ribeiro (B) Anna Politkovskaya (C) May Chidiac (D) U Win Tin 10. Which of the following Indian players has not completed 10,000 runs in Test cricket (A) Rahul Dravid (B) Sachin Tendulkar (C) Sunil Gavaskar (D) Virendra Sehwag 11. On March 26, 2008, NASA™s space shuttle which returned safely to Earth after completing 16 day mission to International Space Station was (A) Discovery (B) Endeavourer (C) Columbia (D) None of the above 12. Jules Verne is Europe (A) Space shuttle (B) Missile shield system (C) Biggest spacecraft (D) Mission to Antarctica 13. Indias National Vector Borne Disease Control Programme has prepared the countries first (A) Dirrohoea map (B) Malarial map (C) Bird flu map (D) Cancer map 14. The World Is What It Is™ is the biography of (A) V. S. Naipal (B) Khushwant Singh (C) Amrita Preetam (D) A.P.J. Abdul Kalam 15. According to a World Bank Report India got remittances in 2007 worth A) $ 30 billion (B) $ 35 billion (C) $ 25 billion (D) $ 27 billion 16. The Summit of NATO (NORTH ATLANTIC TREATY) was held in April in A) Romania (B) Morocco (C) Pakistan (D) Indonesia 17. ACcording to latest WHO report the number of estimated TB cases found every year in India is (A) 2·2 million (B) 1·9 million (C) 1·5 million (D) 1·7 million 18. Tata purchased Jaguar and Rover from (A) Hyundai (B) Maruti Suzuki (C) Ford Motor (D) None of the above 19. According to a survey, the country which has the highest number of internet users is (A) U.S.A. (B) China (C) Japan (D) Germany 20. Which country has won the Women's Asia Cup Cricket 2008 which was held in Colombo in the month of May 2008 ? (A) Brisbane (B) Melbourne (C) Sydney (D) Perth 21. According to Prime Minister™s Economic Advisory Council (EAC) headed by C. Rangrajan the economic growth rate for 2008-09 will remain at A) 8% (B) 8·5% (C) 9% (D) 8·9% 22. The Pritzkar Architecture Prize has been won by (A) Bill Gates Foundation (B) William Harvey (C) Jea Nouvel (D) None of the above 23. Which of the following cricket players has been banned for violating the player™s code of conduct by PCB ? (A) Danish Kaneria (B) Shoaib Malik (C) Mohammad Yousuf (D) Shoaib Akhtar 24. The National Rural Employment Guarantee Scheme has now been extended to all 604 districts of the country with a total budget outlay of” (A) 18000 crore (B) 16000 crore (C) 15000 crore (D) 14000 crore 25. Inflation which soared above 7% in the month of April is measured by A) Consumer Price Index (B) Retail Price Index (C) Whole Sale Price Index (D) Industrial Price Index 26. The winner of Bahrain Grand Prix is (A) Lewis Hamilton (B) Kartikeyan (C) Felipe Massa (D) Robert Kubica 27. The winner of Miss India Earth 2008 is (A) Tanvi Vyas (B) Suman Kaur (C) Rashmi Sehgal (D) Rani Sharma 28. In face of rising prices, Central Government has decided to set strategic Reserve of foodgrains over and above buffer stock. The limit of the reserve is (A) 10 million tonneS (B) 7 million tonnes (C) 6 million tonnes (D) 5 million tonnes 29. Which of following newspapers/magazines won six Pulitzer prizes (A) The New York Times (B) The Times Magazine (C) The Washington Post (D) The Guardian 30. ˜Leftism in India , 1917-1947’ is a book written by (A) S. R. Chaudhuri (B) K. Kanwar (C) R. Nagaswamy (D) Sudipto Chatterjee 31. Recently President Pratibha Patil visited three countries. Which of the following was not the part of her trip (A) Argentina (B) Chile (C) Mexico (D) Brazil 32. GJM stands for (A) General Justice Movement (B) Gorkha Janmukti Morcha (C) Gorkhaland Janshakti Morcha (D) Gorkha Janjagriti Morcha 33. The elections of which of the following countries were marred by controversy (A) ItalY (B) Spain (C) Zimbabwe (D) Nepal 34. Supreme Court has upheld the law enacted in 2006 providing 27% for candidates belonging to (A) SCs (B) STs (C) OBC (D) Minorities OBC 35. At which of the following places have the remains of early human history been found in excavation (A) Chittorgarh (B) Bhavnagar (C) Kolkata (D) Karaikal 36. Judum™ is (A) Terrorist organization (B) A branch of naxalite (C) Name of a police force (D) An anti naxalite movement 37. When was Nepal declared a Secular Nation (A) in 2005 (B) in 2007 (C) in 2008 (D) in 2006 38. The Director General of UN Food and Agriculture Organization of present is (A) Jacques Diouf (B) Kundekeh K. Yumkella (C) Jammel Al Hiyilan (D) Michael Griffin 39. According to a research, stem cells from skin can treat (A) Alzhumer (B) Schizophrenia (C) Parkinsons disease (D) None of the above 40. When did the Olympic torch relay begin (A) in 1950 (B) in 1952 (C) in 1953 (D) in 1963 Answers with Hints 1. (A) 2. (C) 3. (B) The Strategic Arms Limitation Treaty (START-II) is a treaty between Russia and U.S.A. for reducing their nuclear arsenals. The treaty is due to expire in 2009. 4. (A) 5. (A) 6. (C) 7. (B) 8. (C) 9. (A) 10. (D) 11. (B) 12. (C) 13. (B) India™s Vector Borne Disease Control Programme has prepared the country™s first malarial map identifying India™s 60 most malarial endemic districts that report over 50% of the country™s malaria™s cases. 14. (A) 15. (D) 16. (A) 17. (B) 18. (C) 19. (B) 20. (A) India won fourth consecutive times in Women's Asia Cup Cricket 21. (B) 22. (C) The Pritzkar Architecture Prize, considered to be equivalent of Nobel Prize for architecture has been won french architect Jea Nouvel for his creative experimentation in architecture. 23. (D) 24. (B) 25. (C) 26. (C) 27. (A) 28. (D) 29. (C) The Washington Post has won six Pulitzer Prizes, including Public service award for its reporting on conditions of U.S. War Veterans. 30. (A) 31. (A) 32. (B) 33. (C) 34. (C) The Supreme Court on April 10, 2008 upheld the law enacted in 2006 providing a quota of 27 per cent to the other Backward Classes in Central Educational Institutions. 35. (C) 36. (D) Salva Judam is an antinaxalite movement formed by group of people against attack from maxalites. The movement started from Dantewada district (Chhattisgarh). 37. (D) 38. (A) 39. (C) 40. (B) SBI Associate Clerk Exam - Previous Year Solved Model Question Paper General Awareness: 1. Poverty line is expressed in terms of overall 1) per capita income 2) per capita consumption expenditure 3) per capita entertainment expenditure 4) per capita development expenditure 5) per capita GDP 2. Who among the following is the author of the English novel 'The Namesake'? 1) Shobhaa De 2) Jhumpa Lahiri 3) Shashi Tharoor 4) Upamanyu Chatterjee 5) V.S. Naipaul 3. The abbreviation UIDAI stands for 1) Unique Identification Division of All India 2) Uuique Identification Department of Ancient India 3) Unique Identity Division of Ancient India 4) Unique Identification Authority of India 5) None of these 4. Kanha Tiger Reserve is situated in Mandla district of- 1) Rajasthan 2) Uttar Pradesh 3) Chattisgarh 4) Madhya Pradesh 5) Maharashtra 5. At the official march past in the opening ceremony of the 2012 London Olympics, the Indian contingent was led by- 1) Shuttler Parupali Kashyap 2) Sushil Kumar 3) Paddler Soumyajit 4) Saina Nehwal 5) Chef-de-Mission Brig P.K.M. Raja 6. Who amongst the following Indian contestants in the 2012 London Olympics 10-meter Air Rifle event won a bronze medal? 1) Gagan Narang 2) Vijay Kumar 3) Sushil Kumar 4) Parupali Kashyap 5) Amit Kumar 7. Which of the following is a global international organisation dealing with the rules of trade between nations? 1) IMF 2) IDA 3) UNO 4) WTO 5) UNESCO 8. Excise duty is the tax levied on- 1) production of goods 2) import of goods 3) sale of goods 4) profits on the sale of goods 5) income from other sources 9. Union Budget is presented in the Parliament by the- 1) Prime Minister 2) Commerce Minister 3) Minister for Parliamentary Affairs 4) Union Finance Minister 5) Lok Sabha Speaker 10. In the Union Budget for 2011-12, Very Senior Citizens were defined as those who are over- 1) 90 years of age 2) 85 years of age 3) 80 years of age 4) 75 years of age 5) 70 years of age 11. A tax based on the value of the property/ product is called- 1) Fringe Benefit Tax 2) Value Added Tax 3) Minimum Alternative Tax 4) Turnover Tax 5) Ad Valorem Tax 12. Who is the present Governor of the Reserve Bank of India? 1) Dr. K.C. Chakraborty 2) Dr. D. Subbarao 3) Dr. Y.V. Reddy 4) Dr. S. Gokarn 5) None of these 13. The term 'Plastic Money' refers to- 1) ATM/ Credit Cards 2) Plastic-coated Currency Notes 3) Foreign Currency Notes 4) Traveller's Cheque 5) None of these 14. It is mandatory to quote which of the following identification number on all large-value financial transactions? 1) PAN 2) Voter ID 3) PIN 4) Ration Card number 5) Phone number 15. Which of the following typhoons hit the east coast of China on 8th August 2012? 1) Saola 2) Haikui 3) Damrey 4) Ferdie 5) Gener 16. The term 'FTZ' stands for- 1) Free Trade Zone 2) Food Trade Zone 3) Foreign Trade Zone 4) Fiscal Trade Zone 5) Financial Trade Zone 17. Which of the following is an American multinational retailer corporation that runs chains of large discount department stores and warehouse stores? 1) WalMart 2) Alstom 3) IKEA 4) More 5) Spencer's 18. Who amongst the following is a Nobel laureate in Economics? 1) Kaushik Basu 2) Subir Gokarn 3) Dr. Amartya Sen 4) Montek Singh Ahluwalia 5) C. Rangarajan 19. 'Outlook.com' is a new e-mail service recently launched by- 1) Google 2) Apple 3) Microsoft 4) Rediff 5) Sify 20. Jamaican sprinter Usain Bolt bagged gold in the 2012 London Olympics in men's 100-metre race as he set an Olympic record of- 1) 10.23 seconds 2) 9.93 seconds 3) 9.63 seconds 4) 9.33 seconds 5) 9.23 seconds English Language: Directions (Q.1-5): Read each sentence to find out whether there is any grammatical or idiomatic error in it. The error, if any, will be in one part of the sentence. The number of that part is the answer. If there is no error, the answer is 5), i.e. 'No Error'. (Ignore errors of punctuations, if any.) 1. 1) Mangal Pandey was well known/ 2) because he was involved/ 3) in the initial stages/ 4) of the Indian rebellion./ 5) No Error 2. 1) Most of the Indian populations still lives/ 2) in its villages and thus the contribution of/ 3) agriculture to Indian economy/ 4) becomes very important. 5)No Error 3. 1) Catherine's grandfather always/ 2) lost his balance while walking/ 3) and would be found fallen/ 4) on the road./ 5) No Error 4. 1) Her doctor was/ 2) annoyed because she/ 3) ignore her health/ 4) even after being hospitalised twice./ 5) No Error 5. 1) Raghav was worry/ 2) about telling his parents/ 3) that he wanted to move out/ 4) and live independently. / 5) No Error Directions (Q. 6-10): Each question below has a blank, each blank indicating that something has been omitted. Choose the word that best fits the meaning of the sentence as a whole. 6. Rohit --- a huge library and has a large collection of books in it. 1) wants 2) has 3) wanted 4) had 5) needs 7. Rachna liked her room to be exactly the way she left it and she would ---- allow anyone to touch her things. 1) always 2) sometimes 3) willingly 4) never 5) certainly 8. Elephants are the largest living land animals --- earth today. 1) in 2) and 3) on 4) like 5) at 9. Josephine was elated because the doctor confirmed the news --- her pregnancy. 1) like 2) of 3) with 4) after 5) in 10. It --- been estimated that there may be many millions of species of plants, insects and micro organisams still undiscovered in tropical rainforests. 1) have 2) will 3) should 4) has 5) shall Directions (Q. 11-15): In each question below, four words printed in bold type are given. These are numbered (1), (2), (3) and (4). One of these words printed in bold might either be wrongly spelt or inappropriate in the context of the sentence.Find out the word that is inappropriate or wrongly spelt, if any. The number of the word is your answer. If the words printed in bold are correctly spelt and appropriate in the context of the sentence then mark (5), ie 'All Correct', as your answer. 11. The kidnappers(1) asked for ransom(2) and threatened(3) to kill Mr.Gopalan's son if their demands(4) were not met. All correct( 5) Ans : 5 12. Ramesh spiled(1) juice all over Raj's new( 2) clothes and did not even care(3) to apologise(4). All Correct(5) Ans : 1 13. Ravi met(1) with an accident( 2) and broke( 3) his leg the day he bought(4) his new car. All correct(5) Ans : 2 14. Daisy loved(1) children and so she would distribute(2) sweets in an orphanage(3) on her birthday(4). All Correct(5). Ans : 2 15. Roshni wanted(1) to play(2) with her pet dog and so she started(3) throwing tantrums(4). All Correct(5). Ans : 5 Directions: (Q. 16-20): Rearrange the following eight sentences/ group of sentences (A), (B), (C), (D), (E), (F), (G) and (H) in the proper sequence to form a meaningful paragraph; then answer the questions given below them. (A) During the examination, the invigilator noticed the chits and despite Rajesh's plea for innocence asked him to leave the examination hall. (B) At this point Ravish realised his mistake and felt guilty, so he immediately confessed his misdeed to the invigilator and left the examination hall. (C) Rajesh forgave Ravish because Ravish had not only accepted his mistake on time but also had not let Rajesh be punished for the wrong reason. (D) Ravish and Rajesh were in college and had been friends since childhood.However, Ravish did not trust Rajesh. (E) Another student, Satish, who had seen Ravish hiding something in Rajesh's desk, stood up and informed the invigilator of what he had seen. (F) One day Ravish decided to test Rajesh's friendship and so during one of the college examinations Ravish went early to class and hid some chits in Rajesh's desk. (G) Rajesh did not believe Satish, instead was furious that Satish had falsely blamed his friend and agreed that he would leave his examination only if Ravish was kept out of the matter. (H) After the examination was over, Ravish apologised to Rajesh, and promised that he would be a good friend from then onwards. 16. Which of the following should be the FIFTH sentence after rearrangement? 1) G 2) H 3) E 4) D 5) C 17. Which of the following should be the FOURTH sentence after rearrangement? 1) C 2) D 3) E 4) F 5) G 18. Which of the following should be the EIGHTH (LAST) sentence after rearrangement? 1) A 2) B 3) C 4) D 5) E 19. Which of the following should be the FIRST sentence after rearrangement? 1) A 2) B 3) C 4) D 5) F 20. Which of the following should be the SECOND sentence after rearrangement? 1) A 2) B 3) G 4) F 5) E Quantitative Aptitude: Directions (Q. 1-15): What should come in place of question mark (?) in the following questions? 1. 415.25 ? 627.10 + 958.55 = ? 1) 747.5 2) 674.7 3) 750.7 4) 747.9 5) None of these. 2. 34928 ? 2591 ? 14986 = ? 1) 17546 2) 17355 3) 17351 4) 17390 5) None of these 3. 147 ÷ 5 ÷ 0.3 = ? 1) 76 2) 82 3) 54 4) 80 5) None of these 4. 311 × 17 ? 2482 = ? 1) 2650 2) 2392 3) 2805 4) 2788 5) None of these 5. 12.5 × 6.7 × 4.2 = ? 1) 315.55 2) 376.75 3) 351.75 4) 358.55 5) None of these 6. 27% of 450 ? ?% of 375 = 76.5 1) 14 2) 19 3) 12 4) 15 5) None of these 7. 4567.8 + (14 × 9.8) = 5038 ? ? 1) 348 2) 522 3) 541 4) 333 5) None of these 8. 51 × ? = 1632 1) 34 2) 29 3) 48 4) 24 5) None of these 9. 17 2/3 × 1 17/106 = ? 1) 20 1/2 2) 21 1/2 3) 19 4/5 4) 20 4/5 5) None of these 10. 4 × ? = 6924 ÷ 15 1) 115.6 2) 125.05 3) 151.2 4) 117.4 5) None of these 11. 8888 ? 4444 + 222 = ? 1) 5668 2) 4666 3) 4888 4) 3999 5) None of these 12. 1734.5 × 1768.9 ÷ 1727.4 = 17? 1) 74.5 2) 82 3) 56 4) 76 5) None of these 13. 23% of 468 = ? 1) 110.45 2) 106.47 3) 114.65 4) 107.64 5) None of these 14. 63 × 24 ? 92 = ? 1) 3375 2) 3248 3) 3095 4) 3585 5) None of these 15. ? ÷ 26 × 17 = 7225 1) 11950 2) 11050 3) 10950 4) 11235 5) None of these 16. On a particular day, sweets were to be equally distributed among 960 students of a school. However, on that particular day, 360 students remained absent. Hence each student present on that day got three sweets extra. Had all 960 students remained present that day, how many sweets would each student have got? 1) 3 2) 5 3) 7 4) 4 5) None of these 17. What should come in place of question mark (?) in the following number series?9 10 24 81 340 ? 1) 1376 2) 1780 3) 1570 4) 1725 5) None of these 18. What would be the compound interest (in Rs.) accrued on an amount of Rs.9000 at the rate of 11 pcpa in two years? 1) Rs.2089.90 2 ) Rs.2140.90 3) Rs.2068.50 4) Rs.2085.50 5) None of these 19. What is the least number to be added to 2530 to make it a perfect square? 1) 50 2) 65 3) 75 4) 80 5) None of these 20. The difference between 20% of a number and 7/15 of the same number is 124. What is 40% of that number? 1) 186 2) 200 3) 196 4) 465 5) None of these Marketing Aptitude: 1. EMI is a marketing tool if- 1) it is very high 2) it is very low 3) it is increasing 4) it is fluctuating 5) It has no role as a marketing tool 2.Which of the following may be classified as FMCG? 1) Printing Machines 2) Tobaco Product 3) Life Saving Drugs 4) Coin Vending Machines 5) Industrial Goods 3. Buyer resistance in a sales deal can be overcome by means of- 1) lengthy sales talk 2) confusing sales talk 3) IT jargons 4) perseverance 5) arguing skills 4. The best, important and central activity of a business is known as its- 1) Portfolio 2) Product line 3) Nuclear Activity 4) Mainstay 5) Core Competency 5. The target group for Home Loans is- 1) All body-builders 2) Tiles manufacturers 3) Housing societies 4) Farmers' societies 5) Individuals not owning any house 6. Delivery Channels means- 1) Sales Outlets 2) Purchased Goods 3) Product Shelf-life 4) Courier Person 5) Channel Finance 7. Using a customer's buying history to select them for related offers is known as- 1) Up-Selling 2) Prospecting 3) Marketing 4) Channel Selling 5) Cross-Selling 8. The concept of selling is different from marketing and aims at profit maximisation through 1) increasing sales volume of quality products 2) customer satisfaction 3) solution of customer problems 4) satisfaction of customer needs 5) innovation and market research 9. The segmentation of markets based on the gender of the customer is a type of- 1) Geographic Segmentation 2) Demographic Segmentation 3) Psychographic Segmentation 4) Socio-cultural Segmentation 5) Lifestyle-based Segmentation 10. Mass communication with customers or potential customers, usually through paid public media, is known as- 1) Publicity 2) Sales Promotion 3) Advertising 4) Public Relations 5) Brand Building 11. A 'Call' means- 1) an internet terminology 2) contacting a prospective customer 3) visiting a call centre 4) attending to a complaint 5) websites 12. 'Prospect' means 1) existing customers 2) a likely buyer 3) mission document 4) a religious leader 5) new targets to be achieved 13. KYC means 1) Keep Your Customers cool 2) Keep Your Credit card 3) Know Your Customers 4) Keep Your Cool 5) Know Your Credits 14. The collective perceptions and impressions people have formed about an organisation,its products and/or its services, is known as its- 1) Brand Value 2) Brand Alert 3) Brand Architecture 4) Brand Image 5) Brand attribute 15. Good customer service is an extended arm of- 1) Service marketing 2) Web marketing 3) Process marketing 4) Indirect Marketing 5) Internet marketing 16. A company's ability to perform in one or more ways that competitors cannot or will not match is known as its- 1) Attribute competition 2) Attribute positioning 3) Brand image 4) Competitive advantage 5) Brand positioning 17. An audio or video advertising announcement, usually presented on television, radio or in a movie theatre, is called- 1) Publicity 2) Creative 3) Banner 4) Sponsorship 5) Commercial 18. Electrical goods such as TVs, videos, stereo systems etc, used for home entertainment, are known as- 1) White Goods 2) Green Goods 3) Red Goods 4) Blue Goods 5) Brown Goods 19. A very large market segment or wide collection of smaller segments is called- 1) Niche Market 2) Meta Market 3) Mass Market 4) Multi Level Market 5) None of these 20. Customer Retention can be ensured by- 1) Offering freebies 2) Offering loans at low rates 3) Catchy slogans 4) Giving incentives 5) Personalised services SBI Question-Bank previously asked questions State Bank of India SBI Questions with answers, SBI previous years solved Question papersSBI sample model questions with answers SBI model aptitude questions with answers with detailed explanations 1. The average age of 20 students in a class is 20. When the age of their class teacher commuted with the total age of students, their average age, including the class teacher's raises to 21. If so, what is the age of the class teacher ? (a) 40 (b) 36 (c) 41 (Ans) (d) 44 Answer : Total age of 20 students = 20 * 20 = 400. After inclusion of teacher, total age = 21 [No of members] * 21 [New average age] = 441. Age of new entrant [teacher] = 441 - 400 = 41 2. Rajan travels from Kannur to Kozhikode by a car at speed of 60 Km/hour and returns back by hiring a gipsy at a speed of 50 Km/ hour. Then what will be the average speed of his vehicles through out their to and fro journey. ? (a) 54-6/11 km/hour (Ans) (b) 57 km/hour (c) 55-6/11 Km/hour (d) 54-5/11 Km/hour Answer : If 'a' & 'b' are the speed in to and fro journey of vehicles By short method formula to arrive average speed = 2ab/a+b ie, 2*60*50 /60+50 = 6000/110 = 54-6/11 3. Mr. X complete a piece of work with in 8 hours and whereas Mr. Y complete the same piece of work in 6 hours. If both Mr. X & Y operate the same work jointly, in how may hours they complete the same work ? (a) 7/14 Hrs (b) 24/7 Hrs (Ans) (c) 5 hours (d) Not given Answer : If 'x' and 'y' be the number of days required individually by two persons, then by Short method formula, the number of days required to complete the same work by joining them is = xy = 6*8 = 48 x+y 6+8 14 = 24/7 [If three persons worked the short method formula becomes as xyz .] xy + xz + zx 4. Raman & Krishnan will complete a job in 6 days and 12 days respectively according to their capacity. Govindan will complete the same of piece of work in a single day that which Raman & Krishnan both do in a single day. If Krishnan and Govindan wishes to complete the same work, in how many days they take to finish the work (a) 5 days (b) 3 days (Ans) (c) 4 days (d) 4.5 days Answer : Here Raman and Krishnan do the work in 6*12 days ie 72 ie in 4 days 6+12 18 It is stated that the job of Raman & Krishnan in one day is equals to Govindan's one day work. Otherwise it means that Govindan complete the assigned work in 4 days. If Krishnan & Govindan work jointly, the work will be completed in 12*4 = 48 = 3 days 12+4 16 5. A train running in 90Km/hour, having 100 metres long, crosses a train running in another track in opposite direction with speed of 54 Km per hours having 140 metres. How many seconds will take the trains to cross each other fully ? (a) 9 seconds (b) 6 seconds (Ans) (c) 8 seconds (d) No given Answer : Here we want to calculate the relative speed or velocity. Since the trains are moving in opposite directions the relative speed will arrive by summing up i.e. 90 Km hr + 54 Km/hr = ie. 144 Km/hour = 40 m/secs [to convert Km/hour to metres/seconds we want to multiply the term by 5/18.] The total distance of crossing, [i.e. length of two trains] = 100m + 140m = 240m The time required to cross 240m by 40m/secs speed = 240/40 = 6 seconds [Conversion of metres/seconds to KM/seconds will be arrived by multiplying by 18/5.] SBI English previous years solved question papers 1. Find out which part of the sentence has an error (a) They were traveling (b) in the metro when they heard (Ans) (c) a weird sound very close to them (d) No Error 2. The ........... growth of the jungle hid the ancient ruins completely. (a) thickly (b) luxurious (c) densely (d) luxuriant (Ans) 3. It is the house ....... door is painted red. (a) that (b) whose (Ans) (c) which (d) none of these 4. He did time for more than six months. This sentence means ;- (a) He fought against time until his views accepted for more than six months. (b) He gave his service to the prisoners for more than six months. (c) He was in prison for more than six months. (Ans) (d) He spent his time for recognition for more than six months. 5. Hardly ................................... (a) I had started than it began to rain. (b) I had started when it began to rain. (c) had I started than it began to rain. (d) had I started when it began to rain. (Ans) 6. That marriage won't last. I will give them two months ........ the outside. (a) on (b) at (Ans) (c) by (d) over 7. I can see from your expression that you are thinking deep ...... (a) thought (b) think (c) thinking cap (d) thoughts (Ans) Choose the alternative which best expresses the meaning of the words 8. Decrepit (a) lurid (b) dangerous (c) healthy (d) feeble (Ans) 9. jettison (a) discard (Ans) (b) defeat (c) deduce (d) yield The antonyms of the following words 10. doleful (a) melancholy (b) cheerful (Ans) (c) mournful (d) grim 11. dovish (a) draconian (b) hawkish (Ans) (c) peevish (d) lavish 12. The longer you keep this fruit, it will depreciate. (a) easier (b) much (c) the more (Ans) (d) the most SBI latest current affairs national international 1. Consider the following statements about parliamentry standing committees : (a) Commonly known as miniparliament, these committees include numbers from all political parties and even independent MPs. (b) There are 31 members in each committee-21 from Lok Sabha and 10 from Rajya Sabha. (c) The Committees monitor the functioning of all major central ministries and. departments. Which of the above statements are true? (1) only (a) (2) Only (b) (3) only (c) (4) All of the above (Ans) 2. Consider the following statements about Lok Pal Bill: (a) The submit their reports which are placed in both Houses of parliament. (b) The governmen t is not bound to accept these reports, but generally it does accept some of the suggestions. (c) The Lokpal Bill has been reffered to the parliamentary standing committee for personel public grievances, law and justice. (d) Headed by Rajya Sabha Congress MP, Abhishek Singhvi, it has three months to give it report. Which of the above statements are true? (1) only (a) and (b) (2) Only (b) and (c) (3) only (c) and (d) (4) All of the above (Ans) 3. Recently in news Teesta water deal between? (1) India and Bangladesh (Ans) (2) India and Nepal * (3) India and Pakistan (4) India and China 4. The central government on announced a 7% hike in the dearness allowance (DA) of its employees to give them relief from the near double-digit inflation ahead of the festival season. (1) 14 September, 2011 (2) 15 September, 2011 (Ans) (3) 16 September, 2011 (4) 17 September, 2011 5. Who of the following is regarded as "Father of Employment Guarantee Scheme" and a postage stamp was released on his name by the President of India recently ? (1) K M M a d h e w (2) Kasu Brahmanand Reddy (3) Daulat Singh Kothari (4) Vitthal Sakharam Page (Ans) SBI Model Questions and Answers Section A : Quantitative Aptitude Directions(1-10): What should come in the place of question mark(?) in the following questions . 1. 7960 + 2956 - 8050 + 4028 = ? a) 6984 b) 6884 c) 6894 d) 6954 Answer : c 2. 25 ×3.25 + 50.4 ÷24 = ? a) 84.50 b) 83.35 c) 83.53 d) 82.45 Answer : b 3. 350% of ? ÷ 50 + 248 = 591 a) 4900 b) 4890 c) 4850 d) 4950 Answer : a 4. ½ of 3842 + 15 % of ? = 2449 a) 3520 b) 3250 c) 3350 d) 3540 Answer : a 5. (833.25 -384.45)÷ 24 = ? a) 1.87 b) 20.1 c) 18.7 d) 2.01 Answer : a 6. 325 314 288 247 191 ? a) 126 b) 116 c) 130 d) 120 Answer : d 7. 45 46 70 141 ? 1061.5 a) 353 b) 353.5 c) 352.5 d) 352 Answer : b 8. 620 632 608 644 596 ? a) 536 b) 556 c) 656 d) 646 Answer : c 9. 15 25 40 65 ? 170 a) 115 b) 90 c) 105 d) 120 Answer : c 10. 120 320 ? 2070 5195 13007.5 a) 800 b) 920 c) 850 d) 900 e) None of these Answer : e 11. A train moves past a telegraph post and a bridge 264 m long in 8 seconds and 20 seconds respectively. What is the speed of the train? a) 69.5 km/hr b) 70 km/hr c) 79 km/hr d) 79.2 km/hr Answer : d 12. A 300 metre long train crosses a platform in 39 seconds while it crosses a signal pole in 18 seconds. What is the length of the platform? a) 320 m b) 350 m c) 650 m d) Data inadequate Answer : b 13. P can complete a work in 12 days working 8 hours a day. Q can complete the same work in 8 days working 10 hours a day. If both P and Q work together, working 8 hours a day, in how many days can they complete the work? a) 5/11 b) 6/11 c) 7/12 d) 8/12 Answer : a 14. Ravi and Kumar are working on an assignment. Ravi takes 6 hours to type 32 pages on a computer, while Kumar takes 5 hours to type 40 pages. How much time will they take, working together on two different computers to type an assignment of 110 pages? a) 7 hours 30 minutes b) 8 hours c) 8 hours 15 minutes d) 8 hours 25 minutes Answer : C 15. A trader mixes 26 kg of rice at Rs. 20 per kg with 30 kg of rice of other variety at Rs. 36 per kg and sells the mixture at Rs. 30 per kg. His profit percent is: a) No profit, no loss b) 5% c) 8% d) 10% Answer : b 16. The cost price of 20 articles is the same as the selling price of x articles. If the profit is 25%, then the value of x is: a) 15 b) 16 c) 18 c) 25 Answer : B 17. Present ages of Kiran and Syam are in the ratio of 5 : 4 respectively. Three years hence, the ratio of their ages will become 11: 9 respectively. What is Syam's present age in years? a) 28 b) 27 c) 26 d) 24 Answer : d 18. A is two years older than B who is twice as old as C. If the total of the ages of A, B and C be 27, then how old is B? a) 7 b) 8 c) 9 d) 10 Answer : d 19. The average weight of 8 person's increases by 2.5 kg when a new person comes in place of one of them weighing 65 kg. What might be the weight of the new person? a) 76 kg b) 76.5 kg c) 85 kg d) Data inadequate Answer : c 20. 9 In how many different ways can the letters of the word 'LEADING' be arranged in such a way that the vowels always come together? a) 360 b) 480 c) 720 d) 5040 Answer : c 21. What will be the least number which when doubled will be exactly divisible by 12, 18, 21 and 30 ? a) 196 b) 630 c) 1260 d) 2520 Answer : B 22. A group of students decided to collect as many paise from each member of group as is the number of members. If the total collection amounts to Rs. 59.29, the number of the member is the group is: a) 57 b) 67 c) 77 d) 87 Answer : C 23. An industrial loom weaves 0.128 metres of cloth every second. Approximately, how many seconds will it take for the loom to weave 25 metres of cloth? a) 178 b) 195 c) 204 d) 488 Answer : b 24. A container contains 40 litres of milk. From this container 4 litres of milk was taken out and replaced by water. This process was repeated further two times. How much milk is now contained by the container? a) 26.34 litres b) 27.36 litres c) 28 litres d) 29.16 litres Answer : d 25. Sakthi invests a part of Rs. 12,000 in 12% stock at Rs. 120 and the remainder in 15% stock at Rs. 125. If his total dividend per annum is Rs. 1360, how much does he invest in 12% stock at Rs. 120? a) Rs. 4000 b) Rs. 4500 c) Rs. 5500 d) Rs. 6000 Answer : a 26. The true discount on a bill of Rs. 540 is Rs. 90. The banker's discount is: a) Rs. 60 b) Rs. 108 c) Rs. 110 d) Rs. 112 Answer : b 27. Two trains running in opposite directions cross a man standing on the platform in 27 seconds and 17 seconds respectively. If they cross each other in 23 seconds, what is the ratio of their speeds? Insufficient data a) 3 : 1 b) 1 : 3 c) 3 : 2 d) 1: 4 Answer : d 28. An automobile financier claims to be lending money at simple interest, but he includes the interest every six months for calculating the principal. If he is charging an interest of 10%, the effective rate of interest becomes: a) 10% b) 10.25% c) 10.5% d) None of these Answer : b 29. The last day of a century cannot be a) Monday b) Wednesday c) Tuesday d) Friday Answer : c 30. A and B started a partnership business investing some amount in the ratio of 3 : 5. C joined then after six months with an amount equal to that of B. In what proportion should the profit at the end of one year be distributed among A, B and C? a) 3 : 5 : 2 b) 3 : 5 : 5 c) 6 : 10 : 5 d) Data inadequate Answer : c 31. If 3(x - y) = 27 and 3(x + y) = 243, then x is equal to: a) 0 b) 2 c) 4 c) 6 Answer : C 32. Three pipes A, B and C can fill a tank in 6 hours. After working at it together for 2 hours, C is closed and A and B can fill the remaining part in 7 hours. The number of hours taken by C alone to fill the tank is: a) 10 b) 12 c) 14 d) 16 Answer : c 33. A rectangular field is to be fenced on three sides leaving a side of 20 feet uncovered. If the area of the field is 680 sq. feet, how many feet of fencing will be required? a) 34 b) 40 c) 68 d) 88 Answer : d 34. In a class, there are 15 boys and 10 girls. Three students are selected at random. The probability that 1 girl and 2 boys are selected, is: a) 21/46 b) 25/117 c) 1/50 d) 3/25 Answer : A 35. Find the odd one out 2, 5, 10, 17, 26, 37, 50, 64 a) 50 b) 26 c) 37 d) 64 Answer : d 36. A man standing at a point P is watching the top of a tower, which makes an angle of elevation of 30º with the man's eye. The man walks some distance towards the tower to watch its top and the angle of the elevation becomes 60º. What is the distance between the base of the tower and the point P? a) 43 units b) 8 units c) 12 units d) Data inadequate Answer : d 37. A hall is 15 m long and 12 m broad. If the sum of the areas of the floor and the ceiling is equal to the sum of the areas of four walls, the volume of the hall is: a) 720 b) 900 c) 1200 d) 1800 Answer : c 38. How many times in a day, are the hands of a clock in straight line but opposite in direction? a) 20 b) 22 c) 24 d) 48 Answer : b 39. A boat running upstream takes 8 hours 48 minutes to cover a certain distance, while it takes 4 hours to cover the same distance running downstream. What is the ratio between the speed of the boat and speed of the water current respectively? a) 2 : 1 b) 3 : 2 c) 8 : 3 d) Cannot be determined Answer : c 40. At a game of billiards, A can give B 15 points in 60 and A can give C to 20 points in 60. How many points can B give C in a game of 90? a) 30 points b) 20 points c) 10 points d) 12 points Answer : c Section B : Logical Reasoning Questions 1. In the following question, various terms of an alphabet series are given with one or more terms missing as shown by (?). Choose the missing terms out of the given alternatives. A, B, N, C, D, O, E, F, P, ?, ?, ? a) G, H, I b) G, H, J c) G, H, Q d) J, K, L Answer : c Directions (2-6): Study the following information carefully and answer the given questions: In a sports event, different games are scheduled to be held on seven days, starting on Monday and ending on Sunday. Two games are scheduled to be held on Saturday as well as on Sunday and one game on each of the remaining five days. The games to be held in these seven days are: Basketball, Football, Boxing, Sprinting, and Swimming, shooting, weightlifting, wrestling, and cycling, but not necessarily in the same order. Shooting is scheduled to be held on Thursday. Boxing and Cycling are scheduled to be held on the same day. Wrestling is scheduled to be held three days before basketball, i.e. two sports are scheduled between wrestling and basketball. Wrestling is not scheduled to be held on Wednesday. Weightlifting is scheduled immediately after the day boxing is scheduled. Football is scheduled immediately after the day wrestling is scheduled. Swimming is not scheduled on Monday. 2. Which of the following games is scheduled to be held on Friday? a) Basketball b) Wrestling c) Swimming d) Weightlifting Answer : a 3. Which of the following games is scheduled to be held on Sunday? a) Football b) Wrestling c) Basketball d) Cycling e) Swimming Answer : e 4. On which of the following days is sprinting scheduled? a) Saturday b) Friday c) Wednesday d) Monday Answer : d 5. On which of the following days is weightlifting scheduled? a) Monday b) Tuesday c) Wednesday d) Friday e) Sunday Answer : e 6. Shooting is related to Basketball in the same way as Wrestling is related to a) Sprinting b) Football c) Shooting d) Weightlifting Answer : b 7. In a certain code, COMPUTER is written as RFUVQNPC. How is MEDICINE written in the same code? a) EOJDJEFM b) EOJDEJFM c) MFEJDJOE d) MFEDJJOE Answer : a 8. In a code if THRIVES is denoted by SIUHRDU then SOULFUL is denoted in the same code by a) VPTKKTE b) VPTKETK c) TPVKKTE d) TNRKMVG Answer : a Direction (9 to 13): These questions are based on the figure given above in which (1) Rectangle represents Males, (2) Circle represents the urban (3) Square represents the educated and (4) Triangle represents the civil servants. 9. The number indicating the uneducated urban males is a) 4 b) 5 c) 7 d) 11 Answer : b 10. The number indicating the educated civil servants who are males but not urbans is a) 7 b) 8 c) 9 d) 10 Answer : d 11. The number indicating the educated urban males who are not civil servants is a) 8 b) 9 c) 10 d) 11 Answer : b 12. The number indicating the educated males who are urban civil servants is a) 4 b) 7 c) 8 d) 9 Answer : c. 13. The number indicating the uneducated females who are urban civil servants is a) 6 b) 9 c) 10 d) 11 Answer : d 14. Introducing Suresh, Subha said, ?The father of his brother is the only son of my Grandmother?. How is Subha related to Suresh? a) Mother b) Cousin c) Aunt d) Daughter Answer : b 15. A B @ # $ 1 2 f G h 9 2 u V n M N Q O? % = 8 If the first half of the above sequence is written in reverse order, which symbol or letter will be 11th of the right of 21th symbol or letter from the left end? a) G b) H c) # d) @ e) None of these Answer : e 16. The question given below consists of a statement, followed by two arguments numbered I and II. You have to decide which of the arguments a ?strong? argument is and which a ?weak? argument is. Statement: Should the railways in India be privatized in a phased manner like other public sector enterprises? Arguments: I.Yes. This is the only way to bring in competitiveness and provide better services to the public. II.No. This will pose a threat to the national security of our country as multinationals will enter into the fray. a) Only argument I is strong b) Only argument II is strong c) Either I or II is strong d) Neither I nor II is strong e) Both I and II are strong Answer : d 17. Read the conclusion and then decides which of the given conclusions logically follows from the two given statements, disregarding commonly known facts. Statements: 1. Some swords are sharp. 2. All swords are rusty Conclusions: I. Some rusty things are sharp. II. Some rusty things are not sharp. a) Only conclusion I follows b) Only conclusion II follows c) Either I or II follows d) Neither I nor II follows e) Both I and II follow Answer : a 18. In a code BOY is written as $@% and HOUR is written as ×@£O then RUBY is written in the indicating language is ?.. a) O£$% b) O$£% c) %$£O d) %£$O Answer : a 19. Vijay started to move towards the South. After moving 15 m, he turned to his left and moved 15 m. Again he turned to his left and moved 15 m. Now how far is he from his starting point and in what direction? a) 15 m, North b) 15 m, South c) 30 m, East d) 15 m, West e) 15 m, East Answer : e 20. In a code of PROVIDE is denoted by OSNWHED then MORNING is denoted in the same code by a) LPQOHOF b) LPQOOHF c) LPOQHOF d) LQPOHOF e) None of these Answer : a 21. In a row of girls, Rita is eight from the left and Reena is seventeenth from the right. If they interchange their positions, Rita becomes fourteenth from the left. How many girls are there in the row? a) 25 b) 26 c) 29 d) 32 e) 30 Answer : e 22. What will come in the place of question mark? G D B A C? a) E b) F c) G d) H e) A Answer : b 23. A good way to figure out the relationship in a given question is to make up a sentence that describes the relationship between the first two words. Then, try to use the same sentence to find out which of the answer choices completes the same relationship with the third word. Play is to actor as concert is to a) symphony b) musician c) piano d) percussion Answer : b 24. In the following question contains a small paragraph followed by a question on it. Read each paragraph carefully and answer the question given below it. The school has always been the most important means of transferring the wealth of tradition form one generation to the next. This applies today in an even higher degree than in former times for, through the modern development of economy, the family as bearer of tradition and education has become weakened. This passage best supports the statement that for transferring the wealth of tradition from one generation to the next - a) there are means other than the school. b) several different sources must be tried. c) economic development plays a crucial role d) modern technology must be put to use. Answer : c 25. Read the paragraph carefully and determine the main point the author is trying to make. What conclusion can be drawn from the argument? Each paragraph is followed by five statements.One statement supports the author's argument better than the others do. The criminal justice system needs to change. The system could be more just if it allowed victims the opportunity to confront the person who has harmed them. Also, mediation between victims and their offenders would give the offenders a chance to apologize for the harm they have done. This paragraph best supports the statement that victims of a crime should a) learn to forgive their offenders. b) have the right to confront their offenders. c) learn the art of mediation. d) insist that their offenders be punished. Answer : b 26. In a beach, Rina is walking with her children?s and there she met Samuel and Susan. Samuel introduces Susan to those children as ?this person is the daughter of your mother?s father?s mother and is my sister. How is Samuel and Susan related to Rina? a) Brother, sister b) Uncle, aunt c) Cousin, aunt d) Uncle, Cousin Answer : b 27. Which of the following is exactly in the middle of the eleventh from the left end and the fifteenth from the right end? B A 5 D % R I * F H 6 # V 9 $ 3 E 7 G 1 ÷2 M K X 8 U F W Z N a) V b) $ c) 7 d) E e) None Answer : e 28. Direction: The question given below consists of a statement, followed by two arguments numbered I and II. You have to decide which of the arguments a ?strong? argument is and which a ?weak? argument is.Give answer as (1) If only argument I is strong (2) If only argument II is strong (3) If either I or II is strong (4) If neither I nor II is strong and (5) If both I and II are strong. Statement: Should there be a complete ban on manufacture of firecrackers in India? Arguments: I : No. This will render thousands of workers jobless. II : Yes. The firecracker manufacturers use child labour to a large extent. a) 1 b) 2 c) 3 d) 4 d) 5 Answer : a 29. Direction: Read the conclusion and then decide which of the given conclusions logically follows from the two given statements, disregarding commonly known facts. Statements: All windows are doors No door is wall Conclusions: I. No window is wall II. No wall is door a) If only conclusion I follows b) If only conclusion II follows c) If either I or II follows d) If neither I or II follows e) If both I and II follows Answer : a 30. In a certain code ?a friend of mine? 'is written as ?4 9 1 6 ?, ?mine lots of metal ?is written as ?3 1 0 9 ?and ?a piece of metal ?is written as ? 7 1 6 3? . What is the code for 'piece '? a) 3 b) 2 c) 1 d) 7 Answer : d 31. A school bus driver starts from the school, drives 2 km towards the North takes a left turn and drives for 5 km. He then takes a left turn and drives for 8km before taking a left turn again and driving for 5km. The driver finally takes a left turn and drives 1 km before stopping. How far and towards which direction should the driver to reach the school again? a) 3 km towards North b) 7 km towards East c) 6 km towards South d) 6 km towards West e) 5 km towards North Answer : e 32. In a code of COURAGE is denoted by UOCREGA then JOURNAL is denoted in the same code by a) UOJRLAN b) OUJRANL c) OJURANL d) UOJLANR e) UOJRLAR Answer : a 33. In a queue, Vijay is fourteenth from the front and Jack is seventeenth from the end, while Mary?s position is in the middle of Vijay and Jack. If Vijay be ahead of Jack and there be 48 persons in the queue, how many persons are there between Vijay and Mary? a) 8 b) 7 c) 6 d) 5 Answer : a 34. Athul finds that he is twelfth from the right in a line of boys and fourth from the left, how many boys should be added to the line such that there are 28 boys in the line? a) 12 b) 13 c) 14 d) 15 Answer : b 35. Look carefully the pattern and then fill the blank QAR, RAS, SAT, TAU??. a) UAV b) UAT c) TAS d) TAT Answer : a Section C : Verbal Ability Questions 1. Direction: In the following question, find out which part of the sentence has an error. The error may be idiomatic or grammatical. If there is no mistake, the answer is ?No error?. Removing seeds from (1)/cotton plants was (2)/a slowest job until (3)/Eli Whitney invented the cotton gin (4)/No error (5). a) 1 b) 2 c) 3 d) 4 e) 5 Answer : c 2. Choose the correct meaning of proverb/idiom, if there is no correct meaning 'None of these' will be the answer To set one's face against. a) To oppose with determination b) To judge by appearance c) To get out of difficulty d) To look at one steadily Answer : a 3. Direction: Choose the correct SYNONYM of the word in capital letters from the five options given below. FASTIDIOUS a) Dormant b) Delicious c) Fussy d) Faint Answer : c 4. Direction: Fill the blank with suitable one. Although, he is reputed for making very candid statements ............. a) his today?s speech was not fairly audible b) his promises had always been realistic c) his speech was very interesting d) people follow whatever he instructs to them e) his today?s statements were very ambiguous Answer : e 5. Grammatically correct the sentence. Along with a sharp rise in ??., a recession would eventually result in more men, women and children living in ???. . a) Crime, apathy b) Fatalities, poor c) Deaths, slums d) Unemployment, poverty e) Migrations, streets Answer : d 6. Direction: In the following question, the first and the last parts of the sentence are numbered 1 and 6. The rest of the sentence is split into four parts and named, P, Q, R and S. These four parts are not given in their proper order. Read the parts and find out which of the four combinations is correct. Find the correct answer. 1. A study P. success increase Q. concludes that R. and chances for S. commitment to future tasks 6. future success a) RQPS b) SRQP c) QPSR d) PSRQ Answer : C 7. Direction: Choose the correct ANTONYM of the word in capital letters from the five options given below. CONGENIAL a) Disagreeable b) Inconvenient c) Delightful d) Unsuitable Answer : a 8. Direction: A statement given below is capable of being expressed by a one word. The sentence is followed by four such words of which only one is correct. Pick out the correct choice A lady?s umbrella is??.. a) Parasol b) Granary c) Epitaph d) Aviary Answer : a 9. Direction: In the following question, find out which part of the sentence has an error. The error may be idiomatic or grammatical. If there is no mistake, the answer is ?No error?. India is a land of (1)/great political leaders (2)/who ruled the country effectively (3)/and also by protecting its national interest. (4)/No error (5). a) 1 b) 2 c) 3 d) 4 e) 5 Answer : C 10. In the following question five alternatives are given for the idiom/phrase capitalized in the sentence. Choose the alternative which best expresses the meaning of idiom/phrase. Why do you wish to TREAD ON THE TOES? a) To give offence to them b) To follow them grudgingly c) To treat them indifferently d) To be kicked by them e) To be treat them Answer : a 11. Direction: Choose the correct ANTONYM of the word in capital letters from the five options given below. SPARTAN a) Decadent b) Teutonic c) Slavish d) Autonomous Answer : a 12. Direction: Choose the correct SYNONYM of the word in capital letters from the five options given below. ALACRITY a) Briskness b) Fear c) Frankness d) Duplicity Answer : a 13. Direction: Fill the blank with suitable one. The ...... of the Minister's statement cannot be verified by people who have no access to official records. a) Veracity b) Verbosity c) Ambiguity d) Validity Answer : a 14. Direction: In this question, the first and the last parts of the sentence are numbered 1 and 6. The rest of the sentence is split into four parts and named, P, Q, R and S. These four parts are not given in their proper order. Read the parts and find out which of the four combinations is correct. Find the correct answer. 1. As we move P. We face increasing number of stressful events Q. for instance R. through middle age S. that are not 6. easily controlled a) SRQP b) PSRQ c) QPSR d) RQPS Answer : d 15. Direction: A statement given below is capable of being expressed by a one word. The sentence is followed by four such words of which only one is correct. Pick out the correct choice. A person who renounces the world and practices self-discipline in order to attain salvation. a) Sceptic b) Ascetic c) Devotee d) Antiquarian Answer : b 16. Grammatically correct the sentence. If I??..a more reliable car, I????to Spain rather than fly. a) Would have?..Would drive b) Had?..Had driven c) Had??Would drive d) Would have had??Would drive Answer : c Direction (17- 20): Read the passage carefully and then answer the questions given below. Harold a professional man who had worked in an office for many years had a fearful dream. In it, he found himself in a land where small slug-like animals with slimy tentacles lived on people's bodies. The people tolerated the loathsome creatures because after many years they grew into elephants which then became the nation's system of transport, carrying everyone wherever he wanted to go. Harold suddenly realized that he himself was covered with these things, and he woke up screaming. In a vivid sequence of pictures this dream dramatized for Harold what he had never been able to put in to words; he saw himself as letting society feed on his body in his early years so that it would carry him when he retired. He later threw off the "security bug" and took up freelance work. 17. In his dream Harold found the loathsome creatures a) in his village b) in his own house c) in a different land d) in his office Answer : c 18. Which one of the following phrases best helps to bring out the precise meaning of 'loathsome creatures'? a) Security bug and slimy tentacles b) Fearful dream and slug-like animals c) Slimy tentacles and slug-like animals d) Slug-like animals and security bug Answer : c 19. The statement that 'he later threw off the security bug' means that a) Harold succeeded in overcoming the need for security b) Harold stopped giving much importance to dreams c) Harold started tolerating social victimization d) Harold killed all the bugs troubled him Answer : a 20. Harold's dream was fearful because a) It brought him face to face with reality b) It was full of vivid pictures of snakes c) He saw huge elephant in it d) In it he saw slimy creatures feeding on people's bodies Answer : a SBI marketing and computer SBI Associate Banks Clerks Exam Exam Held On : 16-01-2011 Marketing Aptitude/ Computer Knowledge, SBI Associate Banks Clerks Exam Exam Held On : 16-01-2011 ,State Bank of India Clerical Cadre, Probationary officers, Associate clerks questions for practice, SBI and SBT Examination solved question papers SBT and SBI largest collection of free solved placement papers SBI Reasoning Ability solved question paper for practice, SBI Model question papers for practice SBI Marketing and Computer Awareness questions and answers 1. Marketing in Banks is required for _____ (1) Getting new customers (2) Retaining existing customers (3) Lending (4) Accepting of deposits (5) All of these (Ans) 2. Service Marketing is the same as _____ (1) Relationship Marketing (2) Transaction Marketing (3) Internal Marketing (4) All of these (5) None of these (Ans) 3. Market size Means ___ (1) Size of marketing staff (2) Size of the Organization (3) Scope for profits (Ans) (4) Scope for marketing (5) None of these 4. Market share means ___ (1) Market price of shares (2) Sensex (3) Share market (4) Share of business among peers (Ans) (5) Market volatility 5. Internal Marketing means ____ (1) Selling to oneself (2) Selling to the employees (3) Selling of samples (4) Selling to foreign markets (5) None of these (Ans) 6. A 'Call' means ____ (1) A Profession (2) Teletalk (3) Calling on a Prospect (Ans) (4) A Call Centre (5) All of these 7. A 'Call Centre' is ___ (1) A Place where Salesmen meet every day (2) A training Centre (3) A Back Office set up where customer queries are answered (Ans) (4) Customers meeting place (5) All of these 8. A presentation means ___ (1) Display of products (2) Explaining the utility of products (Ans) (3) A gift (4) Display of Communication skills (5) All of these 9. A DSA means ____ (1) Directly Selling Authority (2) Directly Selling Agent, (3) Direct Selling Authority (4) Direct Selling Agent (Ans) (5) Distributors & Selling Agencies 10. 'Benchmark' means _____ (1) Products lined up on a bench (2) Salesmen sitting on a bench (3) Set standards (Ans) (4) Marks on a bench (5) None of these 11. Value - added services means ______ (1) Giving full value for money (2) Better value for higher price (3) Costlier service (4) Additional service (Ans) (5) All the above 12. CRM means ____ (1) Customer Relationship Management (Ans) (2) Customer Retention Manager (3) Customer Relatives Meet (4) Channel Route Market (5) Channel Representative Manager 13. A 'Target Group' in Marketing ______ (1) To whom the sales should be directed (Ans) (2) A Group of Sellers (3) A Group of Buyers (4) A Group of Products (5) All the above 14. The Target Group for Credit Cards is ____ (1) All Card holders (2) All existing borrowers (3) Individuals with taxable income (4) All of these (5) None of these (Ans) 15. Bancassurance means ___ (1) Assurance given by banks to Loanees (2) Assurance to bank with one bank (3) Assurance to repay loans (4) Assurance to give good service (5) Selling insurance products through banks (Ans) 16. Cross - selling means _____ (1) Selling with a cross face (2) Selling to Red Cross members (3) Selling to Blue Cross members (4) Selling across countries (5) Selling other products to existing customers (Ans) 17. ATM means _______ (1) Any Time Marketing (2) Any Time Money (3) Any Time Machine (4) Automatic Teller Machine (Ans) (5) Automatic Teller Money 18. Digital Marketing is ______ (1) a way of Direct Marketing (2) a way of Indirect Marketing (Ans) (3) a strategy of the DSAs (4) All of these (5) None of these 19. Market segmentation helps in ________ (1) Identifying the target group (2) Focussed marketing (3) Improved lead generation (4) Only (1) & (2) (5) All (1), (2) & (3) (Ans) 20. Only one of the following is true. Find the same. (1) Marketing is not required in a monopolistic situation (2) Marketing is not required in Public enterprises (3) Marketing depends on competitors performance (4) Marketing has no role to play in production levels (Ans) (5) Motivated employees do not require sales incentives. 21. Price is an important factor for effective marketing. What are the other Ps? (1) Product, Promotion & Place (Ans) (2) Place, Persuasion & Promotion (3) Pride, Person & Place (4) Product, Paper & Person (5) None of these 22. Advertisements are required for ______ (1) Boosting the production levels (2) Motivating the employees (3) Boosting the sales (Ans) (4) All of these (5) None of these 23. Sales forecasting involves study of _____ (1) Proper selling price (2) Sales planning (3) Distribution outlets (4) Consumer needs and demands (5) All of these (Ans) 24. Market Research is needed for _____ (1) Choosing the right sales persons (2) Choosing the right products (3) Marking proper marketing decisions (Ans) (4) Deciding the sales incentives (5) None of these 25. Marketing is termed as successful, when _________ (1) There is a rise in the number of salesmen (2) There is no need for post-sales service (3) There is no need to give discounts (4) There is rise in sales through more clients (Ans) (5) Marketing is an ongoing process and hence, an never be successful. 26. What happens ;when you press the Ctrl + V key? (1) A Capital V letter is typed into your document at the cursor point (2) The selected item is pasted from the Clipboard (Ans) (3) The selected item is pasted to the Clipboard (4) The selected drawing objects are distributed vertically on he page (5) None of these 27. Which do you press to enter the current date in a cell ? (1) CTRL + : (semicolon) (Ans) (2) CTRL + SHIFT + : (colon) (3) CTRL + F10 (4) CTRL + F11 (5) None of these 28. Which of the following describes a relational database? (1) It provides a relationship between integers (2) It consists of separate tables of related data (Ans) (3) It retrieves data related to its queries (4) It provides a relationship between floats (5) None of these 29. How do you print your slides in a handout that includes lines for notes? (1) In the Print dialog box, select Handouts and set the number of slides per page to 3 (Ans) (2) In the Print dialog box, select Handouts and a number of slides per page, then select the Include comment pages option (3) In the Print dialog box, select Notes Pages instead of Handouts (4) It cannot be done (5) None of these 30. Suppose you have columns of data that span more than one printed page. How can you automatically print the column headings on each page? (1) Click Page Setup on the File menu, click the Sheet tab, and enter the row that contains these column headings under Print titles (2) Click Page Setup on the File menu, click the Page tab, click the Options button, then enter your choices (3) Click Page Setup on the File menu, click the Sheet tab, and make a selection under the Print heading (4) Click Page Preview on the File menu, click the Sheet tab, and enter the row that contains these column headings under Print titles (Ans) (5) None of these 31. The Assistant is ______ (1) an application that allows you to take notes and save them in a file (2) an animated character that provides help and suggestions (Ans) (3) a button on the standard Toolbar that executes the Print command (4) a collection of frequently misspelled words in a dictionary file (5) None of these 32. Which of the following place the common data elements in order from smallest to largest ? (1) Character, file, record, field, database (2) Character, record, field, file, database (3) Character, field, record, file, database (Ans) (4) Bit, byte, character, record, field, file, database (5) None of these 33. Select the odd one out ______ (1) Interpreter (2) Operating System (Ans) (3) Compiler (4) Assembler (5) None of these 34. Which of these is a quick way to copy formatting from a selected cell to two other cells on the same worksheet ? (1) Use CTRL to selected all three cells, then click the Paste Button image button on the Standard toolbar (2) Copy the selected cell, then select the other two cells, click Style on the Format menu, then click Modify (3) Click Format Painter Button image on the Formatting toolbar twice, then click in each cell you want to copy the formatting to (Ans) (4) Use Alt to select all three cells, then click the Paste Button image button on the Standard toolbar (5) None of these 35. How do you prevent emailed word document from always opening in the Reading Layout ? (1) From the Tools Menu > Options > General Tab > uncheck the 'Allow starting in Reading Layout' (Ans) (2) From the View Menu > Reading Layout > General Tab > uncheck the 'Allow starting in Reading Layout' (3) Form the Format Menu > Auto format > Edit Tab > uncheck the 'Use with emailed attachments' (4) All the above (5) None of these 36. By default your document prints in __ mode. (1) Landscape (2) portrait (Ans) (3) page steup (4) Print view (5) None of these 37. Which of the following is small single site network? (1) LAN (Ans) (2) DSL (3) PAM (4) USB (5) None of these 38. How can page break be done in word? (1) Press f1 key putting cursor on an appropriate place (2) Press ctrl+ Enter putting cursor on an appropriate place (Ans) (3) Use Insert/section break (4) Change font size of document (5) None of these 39. What do the contents of Active cell show? (1) Name box (2) Reheading (3) formula bar (Ans) (4) Task pen (5) None of these 40. To search and load the saved file ____ (1) select the close command (2) select the new command (Ans) (3) select the same command (4) select open command (5) None of the SBI question papers SBI Associate Banks Clerks Exam,SBT and SBI previous years solved question papers,SBT and SBI Aptitude/Reasoning, General awareness questions with answers,SBI and SBI free solved placement papers,SBT and SBI Associate clerks, Probabtionary officers,sepecialists officers recruitment pattern ,Test pattern,Written test solved question papers,All IT Non IT(PSU Companies) and Banks IBPS,PO,CWE,model question papers for practice Exam Held On : 16-01-2011 (Based on Memory) SBI GENERAL AWARENESS 1. Hillary Clinton who was in India recently is the U.S. Secretary of _____ (1) Commerce (2) State (Ans) (3) Treasury (4) Defence (5) None of these 2. If Indian Banking industry had demonstrated its resilience to external shocks in 2008-09, the credit goes to _______ (A) Policies of the RBI and the Govt. of India (B) Strength of Indian Business Houses (C) Foreign aid/investments to Indian companies (1) Only (A) (Ans) (2) Only (B) (3) Only (C) (4) All (A), (B) and (C) (5) None of these 3. Tembhli village which was in news is in _____ (1) Bihar (2) Rajasthan (3) Orissa (4) West Bengal (5) Maharashtra (Ans) 4. Rohan Bopanna who represented Indian in an international event is a _____ (1) Billiards Player (2) Golf Player (3) Chess Player (4) Table Tennis Player (5) Lawn Tennis Player (Ans) 5. AIBA Women's World Boxing championship was organized in September 2010 in ___ (1) Rome (2) New Delhi (3) London (4) Vienna (5) Barbados (Ans) 6. Mr. A. K. Antony a Union Minister in the Indian Cabinet recently signed an agreement with South Korea. This mean the agreement is for the Cooperation in the field of ______ (1) Agriculture (2) Rural Development (3) Defence (Ans) (4) Steel & Iron (5) None of these 7. What is financial inclusion? (1) To provide a permanent employment to the unemployed (2) To Provide a 100 days job to all those who are in need of a job (3) To provide banking services to all those loving in remote areas (Ans) (4) To ensure that all financial transactions amounting Rs. 5,000/- and above are done througth banks. (5) To provide finances for all 8. Recently the term 3G was very much in news and even a conference on it was organized in India. What is the letter 'G' denotes in 3G (1) Grade (2) Group (3) Global (4) Guild (5) Generation (Ans) 9. Some major banks and financial institutions in various Western countries were to wind up their business and/or declare themselves in financial problems during last few years. This trend is technically known as (1) Devaluation (2) Deformation (3) Global Slowdown (Ans) (4) Political backdrop (5) Globalization 10. Which of the following International sports events was organized in Delhi in October 2012 in which several countries participated? (1) Asian Games (2) World Cup Cricket Tournament (3) Asian Hockey Tournament (4) Commonwealth Games (Ans) (5) None of these 11. Exporters in India get insurance cover and risk cover from which of the following organizations? (1) SIDBI (2) NABARD (3) ECGC (Ans) (4) RBI (5) None of these 12. Which of the following place in India does not have a Stock Exchange? (1) Kolkata (2) Ahmedabad (3) Mumbai (4) Delhi (5) Udaipur (Ans) 13. Sir Mota Singh who was Knighted by the Queen Elizabeth II earlier this year is _______ (1) first Sikh Judge in UK (Ans) (2) first Sikh Minister in UK (3) first Indian to become Vice Chancellor in Cambridge (4) first Indian to become Governor of a State in USA (5) None of these 14. 'Ayodhya' which was in news recently is a place in _____ (1) Madhya Pradesh (2) Bihar (3) Rajasthan (4) Jharkhand (5) Uttar Pradesh (Ans) 15. Which of the following is the name of a private sector Bank in India? (1) IDBI Bank (Ans) (2) Axis Bank (3) Corporation Bank (4) UCO Bank (5) All are Private Banks 16. The Govt. of India does not provide any direct financial assistance to which of the following schemes? (1) Mahatma Gandhi National Rural Employment guarantee scheme (2) Rural Health Mission (3) Indira Aawas Yojana (4) Jeevan Stathi Yojana (Ans) (5) Rajiv Gandhi Grameen Vidyutikaran Yojana 17. What does the letter 'S' denotes in 'AFSPA' an abbreviation we very often see in newspapers? (1) Slow (2) Special (Ans) (3) State (4) Solid (5) Sovereign 18. Which of the following terms is used in banking and finance? (1) Abiotic (2) Demand Deposit (Ans) (3) Fat scales (4) A diabetic (5) Cathode 19. Which of the following is a food crop? (1) Maize (Ans) (2) Cotton (3) Palm (4) Jute (5) Jatropha 20. Vedanta Alurnina is a company operating in the area of _______ (1) Shipping (2) Mining (Ans) (3) Cement (4) Textiles (5) Banking 21. Name of which of the following rates/ratios cannot be seen in financial newspapers? (1) Bank Rate (2) Repo Rate (3) Cash Reserve Ratio (4) Pulse Rate (Ans) (5) Statutory Ratio 22. An individual visiting UAE will have to make hi/her payments in which of the following currencies? (1) Dirham (Ans) (2) Taka (3) Baht (4) Peso (5) Dinar 23. Who amongst the following is NOT a famous writer? (1) Sania Mirza (Ans) (2) V. S. Naipaul (3) Chetan Bhagat (4) Khushwant Singh (5) Namita Gokhale 24. 'Peepli Live' is a film directed by______ (1) Anusha Rizvi (Ans) (2) Aamir Khan (3) Kiran Rao (4) David Dhawan (5) None of these 25. Which of the following is the Chemical name of the salt we use in our kitchen? (1) Calcium Carbonate (2) Calcium Chloride (3) Sodium Carbonate (4) Sodium Chloride (Ans) 26. The judges of the Supreme Curt of India now retire at the age of ____ (1) 58 years (2) 60 years (3) 62 years (4) 65 years (Ans) (5) 68 years 27. Operation New Dawn is the biginning of stability in____ (1) Afghanistan (2) Iran (3) Sudan (4) Sri Lanka (5) Iraq (Ans) 28. Wchih of the following books is written by Krian Desai? (1) A house for Mr. Biswas (2) 3 Mistakes of My Life (3) A Tale of Two Sisters (4) The Inheritance of Loss (Ans) (5) Tale of Two Cities 29. Jimena Navarrete who was crowned Miss Universe 2010 is from ___ (1) Mexico (Ans) (2) Venezuela (3) Brazil (4) New Zealand (5) Italy 30. India has an agreement with USA for producing energy in the form of___ (1) Hydraulic (2) Thermal (3) Nuclear (Ans) (4) Solar (5) All of these forms 31. Which of the following is an economic term? (1) Plaintiff (2) Bunker Blaster (3) Deflation (Ans) (4) Lampoon (5) Zero Hour 32. Which of the following awards is given to sports persons in India? (1) Shantiswarup Bhatnagar Awards (2) Arjuna Award (Ans) (3) Kalidas Samman (4) Saraswati Samman (5) None of these 33. Which of the following is the trophy/cup associated with the game of cricket? (1) Derby (2) Merdeka Cup (3) Santhosh Trophy (4) Duleep Trophy (Ans) (5) Durand Cup 34. Who amongst the following can take benefit of the Social Security Fund established by the Govt. of India for unorganized workers? (1) Primary school teacher (2) Employee of Sugar Factory (3) Textile Mazdoor (4) Rickshaw Puller (Ans) (5) Clerk in a factory 35. Shushil Kumar who won a Gold Medal at an international event is a famous____ (1) Wrestling champion (Ans) (2) Golf player (3) Table Tennis Player (4) Weight lifter (5) None of these 36. In case a major war breaks out between two nations, which of the following organizations/ agencies will be see in the battle field, in addition to the armies of the fighting parties? (1) WTO (2) Red Cross (Ans) (3) CARE (4) OPEC (5) All of these 37. Which of the following is the abbreviated name associated with food security? (1) ASHA (2) PDS (Ans) (3) WTO (4) OPEC (5) NPA 38. Which of the following is a good and rich source of vitamin 'C'? (1) Milk (2) Radish (3) Mango (4) Lemon Juice (Ans) (5) Banana 39. Who amongst the following Indian Prime Ministers was honoured Posthumously by the Govt. of Bangladesh for his/her contributions to the independence of the the nation? (1) Rajeev Gandhi (2) Jawaharlal Nehru (3) Indira Gandhi (Ans) (4) Lal Bahadur Shastri (5) None of these 40. Who amongst the following was never a Government of the RBI ? (1) Bimal Jalan (2) Y.V. Reddy (3) Arup Roy Choudhury (Ans) (4) C. Rangarajan (5) D. Subbarao SBI English questions with answers Directions (41-50) : Read the following passage carefully and answer the question given below it. Certain words are printed in hold to help you to locate them while answering some of the questions. Once upon a time there lived a queen in the city of Benaras. Her name was Khema and she was the wife of King Bahuputtaka. One night, the Queen had a dream of beautiful golden goose that spoke with great wisdom, almost as if he was a sage. She told her husband that she desperately wanted to see a bird just like the one in her dream. So the King asked his ministers to find out all that they could about a bird such as this. He was told that such a bird did exist but was extremely rare and difficult to find. They advised him to build a beautiful lake on the outskirts of Benaras so that be may attract such rare and lovely creatures to reside there. In this way the queen might have her wish. Towards the north, on Mount Cittakuta, there lived about ninety thousand wild geese headed by a beautiful golden goose called King Dhatarattha. He got to her of this exquisite lake surrounded by water lilies and lotuses floating on the surface. The King had invited all the birds to come and live on it, promising that none of them would ever be harmed. Corn was scattered on a daily basis in order to attract the birds So a couple of geese went up to their King, the golden goose and lold him that they were quite tired of living up on the mountains and would like to see this wonderful lake where they had been promised food and protection. The king agreed to their request and took the flock down south towards Benaras Meanwhile, at the lake king Bahuputtaka had placed hunters all around in order to capture any golden goose that happened to pass by. So the next morning when the headhunter saw this flock of geese approaching he was very excited to see their golden leader. He immediately went about setting up a snare amongst the water lilies and lotuses, as he knew that the leader would definitely be the first to alight. The whole flock came flying down in one mighty swoop and as expected it was the King's foot that touched the water first. He was ensnared and could not escape. Seeing this, the other geese flew back to Mount Cittacuta for safety. All except one. He was the chief captain, Sumukha. King Dhatarattha entreated him to fly to safety too, as he would surely be captured if he stayed by his side. But Sumukha replied that he would never desert his master in the face of danger and would either try to save him or die by his side. Al this point the hunter approached and as Sumukha saw him he decided to appeal to his compassion. The hunter asked the golden goose how come he had not noticed the trap that was set. The golden goose replied that when one's time was up it was no used to struggle against what was fated and one must just accept it. The huntsman was very impressed with his grace and wisdom. He then turned to Sumukha and asked why he had not fled with the other birds even though he was free to do so. Sumukha answered that the golden goose was his King, best friend and master and that he could never desert him even at the cost of his own life. Hearing this, the hunter realised that these were a couple of rare birds of great nobility. He did not much care for his own King's reward and decided to do the right thing and set them free. He told Sumukha that as he was ready to die for his King he would set them both free to fly wherever they wish. 41. Why were the geese keen on visiting the lake in Benaras? (1) They were invited personally by king Bahuputlaka. (2) They were tired of their old lake. (3) They were amused by the water lilies and lotuses. (4) The lake was not very far from the mountains. (5) To lake was to be a source of good food and protection. (Ans) 42. Why did the king approach the ministers? (1) In order to find out the best location for the lake. (2) In order to find out if a golden goose existed. (Ans) (3) To enquire about his wife's dreams. (4) To devise a plan to capture all rare bird species. (5) To enquire about the best hunter in the locality. 43. Why did the head hunter release the king, the golden goose? (1) He was impressed by his wisdom and nobility. (Ans) (2) The geese were a rare species of birds. (3) The golden goose, king Dhatarattha promised him a handsome rewards. (4) The hunter sympthised with the golden goose. (5) The hunter did not want the chief captain to lose a friend. 44. What was the king's intention behind building a lake ? (1) To attract all rare species of living animals and birds. (2) To beautify the city of Benaras. (3) He did not want to go against the wishes of his wife. (4) He wanted to capture the golden goose. (Ans) (5) He enjoyed bird watching and wanted to create a habitat for them. 45. Why did the flock of geese panic and retreat to Mount Cittacuta? (1) Their king took the wrong route to the lake and they lost their way. (2) Their king, the golden geese was captured. (Ans) (3) Their chief captain, Sumukha betrayed the king. (4) They spotted many hunters by the lake. (5) None of these 46. What advice did the ministers give to the king? (1) That no such golden goose existed and he was only wasting his time searching for one. (2) To create an artificial golden goose for the queen. (3) To build a lake in order to attract the golden goose. (Ans) (4) To open a sanctuary of rare birds for the queen. (5) To have a beautiful garden surrounded by flowers and trees. 47. Which of the following statements would best describe the qualities of Sumukha? (A) Betrayer of the flock. (B) The Selfless Goose. (C) Loyal towards the king. (D) The naive and ignorant goose. (1) Only (A) (2) Only (B) (3) Only (C) and (D) (4) Only (A) and (D) (5) Only (B) and (D) (Ans) Directions (48-50) : Choose the word which is most nearly the same in meaning as the word printed in bold as used in the passage. 48. Exquisite : (1) Expensive (2) Delicate (3) Elaborate (4) Wonderful (Ans) (5) Efficient 49. Snare : (1) Trap (Ans) (2) Alarm (3) Plan (4) Arrangement (5) Protection 50. Desert : (1) Displease (2) Encourage (3) Instigate (4) Escort (5) Abandon (Ans) Direction (51-55) : In each of the following sentences, an idiomatic expression or a proverb is highlighted. Select the alternative which best describes its use in the sentence. 51. The course of events made it necessary for Joseph to start working. (1) events that were planned. (2) long list of future events. (3) A succession of unexpected events. (Ans) (4) nature of events that followed after Joseph joined work. (5) None of these 52. The team put their plant into execution the very next day, (1) proposed a plan. (2) discussed their plan (3) started thinking about a plan. (4) started carrying out their plan. (Ans) (5) None of these 53. Mrs Nayak opened the discussion on the "alarming rate of poverty in India" (1) stated the discussion, (Ans) (2) gave her opinion in the discussion. (3) did not agree on the discussion. (4) welcomed the people to the discussion. (5) None of these 54. The new law on "Right to Food Safety" will come into force next month. (1) be forced upon the people. (2) be associated from next month onwards. (3) be implemented next month. (Ans) (4) be withdrawn next month. (5) be widely rejected next month. 55. When the girl wanted to stay out past midnight, her father put his foot down. (1) gave in to her request. (2) walked away disapprovingly. (3) obstructed her from leaving the house. (Ans) (4) requested her to be home on time. (5) None of these Directions (556-60) : Read each sentence to find out whether there is any grammatical error in it. The error, if any, will be in one part of the sentence. The number of that part is the answer. If there is no error, the answer is (S) i.e. 'No Error' (Ignore the errors of punctuation, if any.) 56. A public safety advertising (1)/ campaign in Russia (2)/ hope to draw attention (3)/ of pedestrians crossing the road (4). No Error (5) (Ans : 3) 57. Nuclear waste will still being (1)/ radioactive even after twenty thousand years, (2)/ it must be disposed (3)/ of very carefully (4). No Error (5) (Ans : 1) 58. My friend lived at the top (1)/ of an old house (2)/ which attic had been (3)/ converted into a flat. (4) No Error (5) (Ans : 3) 59. The Renaissance was (1)/ a time to 're-awakening' (2)/ in both the arts (3)/ and the sciences (4) No Error (5) (Ans : 2) 60. A cash prize was (1)/ award to the most (2)/ successful salesman of the year (3)/ by the President of the company. (4) No Error (5) (Ans : 2) Directions (61-65) : Each sentence below has a blank/s, each blank indicates that something has been omitte. Choose the word/s that best fit/s the meaning of the sentence as a whole. 61. The state-of-the-art school is ___ with a medical clinic and fitness center. (1) establish (2) illustrative (3) having (4) equipped (Ans) (5) compromising 62. Many leading members of the opposition party ___ to justify the party's decision. (1) having tried (2) has tried (3) have been trying (Ans) (4) tries (5) is trying 63. The charity ____ most of its money through private donations. (1) receives (Ans) (2) borrows (3) uses (4) proposes (5) invests 64. The Bhagavad Gita is a part of the Mahabharata, but it stands ____ and is _____ in itself. (1) dependent, incomplete (2) together, justified (3) separate, dignified (4) apart, complete (Ans) (5) united, connected 65. The artist's work of art is worthy ___ praise. (1) for (2) of (Ans) (3) to (4) about (5) to be Directions (66-70) : In each question below four words printed in bold type are given. These are numbered (1), (2), (3) and (4). One of these words printed in bold may either be wrongly spelt or inappropriate in the context of the sentence, Find out the word that is inappropriate or wrongly spelt, if any. The number of that word is your answer. If all the words printed in bold are correctly spelt and appropriate in the context of the sentence then mark (5). i.e. 'All Correct' as your answer. 66. Employees must familiarise (1)/ themselves with the health (2)/ and safety (3)/ mannual (4)/ of the rganisation, (5) All Correct (Ans : 4) 67. The political (1)/ environment (2)/ is not condusive (3)/ to economic reforms., (4) All Correct (5) (Ans : 3) 68. The ship (1)/ was at sea for many days and finally (2)/ anchored (3)/ near the costline. (4)/ All Correct. (Ans : 4) 69. Some of our regular listenors (1)/ have complained (2)/ about the new programme (3)/ schedule (4). All Correct (5) (Ans : 1) 70. The report (1)/ found that the saefty (2)/ equipment had been very poorly (3)/ maintained (4), All Correct (5) (Ans : 2) Directions (71-75). Rearrange the following six sentences/group of sentences (A), (B), (C), (D), (E) and (F) in the proper sequence to form a meaningful paragraph; then answer the questions given below them. (A) The merchant greedily counted his gold and said, "The purse I dropped had 200 pieces of gold in it. You've already stolen more than the reward ! Go away or I will tell the police." (B) The judge, looking towards the merchant said, "you stated that the purse you lost contained 200 pieces of gold. Well, that's a considerable cost. But, the purse this beggar found had only 100 pieces of gold". (C) "This purse therefore cannot be the one you lost." And, with that, the judge gave the purse and all the gold to the beggar. (E) A beggar found a leather purse that someone had dropped in the marketplace. One opening it, he discovered that it contained 100 pieces of gold. Then he heard a merchant, shout, "A reward! A reward to the one who finds my leather purse". (F) "I am an honest man," said the beggar defiantly. "Let us take this matter to the court. The judge patiently listened to both sides of the story. 71. Which of the following should be the FIFTH sentence after the rearrangement? (1) E (2) D (3) B (Ans) (4) C (5) F 72. Which of the following should be the SECOND sentence after the rearrangement? (1) A (2) D (3) F (4) B (5) C (Ans) 73. Which of the following should be the SIXTH (LAST) sentence after the rearrangement? (1) A (2) D (Ans) (3) F (4) E (5) C 74. Which of the following should be the FIRST sentence after the rearrangement? (1) E (Ans) (2) B (3) D (4) C (5) F 75. Which of the following should be FOURTH sentence after the rearrangement? (1) D (2) F (Ans) (3) B (4) E (5) C Directions (76-80) : In the following passage, there are blanks, each of which has been numbered. These numbers are printed below the passage and against each five words are suggested, one of which fits the blank appropriately. Find out the appropriate word in each case. The Bhagavad Gita is a poem of 700 verses which is a part of the Mahabarata. It is the only philosophical song existing in all languages. Its popularity and influence have never waned. It (760 light and guidance to the troubled mind in times of crisis. It is in the (77) of a dialogue between Arjuna and Krishna on the battlefield. Arjuna's mind is troubled at the thought of the killings of his friends and relatives. He cannot conceive of any gain Arjuna is the (78) of the tortured spirit of man torn by conflicting obligations and moralities. The dialogue proceeds and takes us to the higher level of individual duty and social behaviourl, application of ethics to practical life and social outlook that should govern all. An attempt is (79) to reconcile the three path of action and the path of faith. But more (80) is laid on faith. There is a call of action to meet the obligations of life, keeping in view the spiritual background and the large purpose of the universe. 76. (1) shines (2) enforces (3) secures (4) seeks (5) provides (Ans) 77. (1) constitution (2) part (3) formation (4) form (Ans) (5) programme 78. (1) dream (2) source (3) figures (4) symbol (Ans) (5) conceived 79. (1) made (Ans) (2) established (3) coined (4) given (5) generated 80. (1) significant (2) declaration (3) emphasis (Ans) (4) blessings (5) important Placement Paper SBI General knowledge and Computer Objective Type Questions SBI previous years solved question papers, SBI aptitude and reasoning questions with answers , SBI aptitude, reasoning,generalawareness,marketing and computer questions with answers 1. What does 'DOS' stands for a) Disk Original System b) Dynamic Operating System c) Disk Operating System (Ans) d) Default Operating System e) None of these 2. Which of the following are functions of DOS 1) Controlling the input and output operations 2) Controlling the input and output operations 3) Controlling the interaction between the user and the computer a) Only (1) b) Only (1) and (2) c) Only (2) and (3) d) All (1), (2) and (3) (Ans) e) None of these 3. What do you understand by the term DOS a) Downloading the System Program from the Internet b) Down loading Operating Program from the Internet c) The master control program of the computer which helps computer to manage the files d) The program which helps computer to check the various peripheral devices e) Both (c) and (d) (Ans) 4. What was the last version of MS-DOS that was released separately a) 6.22 (Ans) b) 6.0 c) 6.11 d) 5.0 e) none of these 5. One of the following statements is incorrect on MS-DOS a) S-DOS is a command driven operating system b) MS-DOS is a windows driven operating system (Ans) c) MS-DOS exhibits a system prompt which is normally represented as C:\> d) MS-DOS is made up of external and internal commands e) None of these 6. Which of the following is not a usual file extension in DOS a) .EXE b) .BAT c) .o (Ans) d) .class e) None of these 7. What do you understand by the term 'booting' a) The process of starting the computer from the power-off position (Ans) b) Connecting the computer to the electric switch c) Increasing the memory of the computer d) The process of shut down the computer e) None of these 8. A bootable disk is ______________ a) a disk in the computer, which can be easily updated b) a disk that contains all the system files of DOS (Ans) c) a disk loaded with the information from the internet d) a disk loaded with all the software e) None of these 9. What is the difference between cold booting and warm booting 1) Cold booting uses a floppy for booting while warm booting uses hard disk for booting 2) Cold booting uses floppy is starting the computer from the power-off position and warm booting is the restarting the computer with CAD (Control+Alt+Delete) command or reset button when it hangs. a) only (1) b) Only (2) (Ans) c) Both (1) and (2) d) Either (1) or (2) e) None of these 10. Which file in MS-DOS contains internal commands that are loaded during booting process a) CONFIG.SYS b) MSDOS.SYS c) IO.SYS d) COMMAND.COM (Ans) e) None of these 11. What is the name of the batch file that is automatically run when MSDOS is booted a) CONFIG.SYS b) CONFIG.BAT c) AUTORUN.BAT (Ans) d) RUN.BAT e) None of these 12. One of the following statements on Internal commands is not true a) At the time of booting, Internal commands are automatically loaded into memory b) Internal commands are executed instantly without refering to disk c) Internal commands are those that have been built into MS-DOS d) All of these e) None of these (Ans) 13. One of the following on External command is correct a) External commands are stored onto disk in the form of a file b) For executing these commands path must be set to DOS director c) For executing external commands they must be first loaded memory d) All of the above e) None of the above (Ans) 14. FAT stands for a) File Activity Table b) File Allocation Table (Ans) c) File Access Tape d) File Accomodation Table 15. The REN command in DOS is used to a) change the name of the file (Ans) b) move the file from one location to another c) delete a file and its contents d) All of the above e) None of the above 16. The purpose of the MD command is to a) create a directory (Ans) b) move a folder c) make a database d) All of the above e) None of these 17. The SCANDISK program a) is a disk analysis and repair tool (Ans) b) make a database c) scans disk and eliminate viruses d) All of the above e) None of these 18. The function of DELTREE command is to a) delete an entire directory that contains files, subdirectories or both files and subdirectories (Ans) b) pick similar named files from different directories and delete them automatically c) delete and entire disk with complex directory and file structures so as to format the disk faster d) All of the above e) None of these 19. One of the following statements is not true about DIR command a) if the DIR command is used without parameters of switches, it displays the disk's volume label and serial number, one directory or filename per line, including the filename extension, the file size in bytes and data and time of the last modification of the file b) used to display a list of the files and/or subdirectories in the specified path. c) is to read the contents of a file (Ans) d) /P switch of DISKCOPY command is to e) None of these 20. The FILES command a) can be issued from the system prompt-C:\> b) assigns the amount of memory that is to be set aside for the file handlers (Ans) c) is to be issued only in AUTOEXEC.BAT files d) All of the above e) None of these 21. The purpose of the FORMAT command is to a) create a new root directory and file allocation table for the disk b) locate and mark all bad sectors on a disk c) prepare a floppy disk for use with the MS-DOS d) All of the above (Ans) e) None of these 22. The purpose of the MOVE command is to a) move one or more files to the location you gave (Ans) b) rename directories c) both of the a and b d) to move the text within file e) None of the above listed 23. The PATH command a) specifies directories that MS-DOS is to search when trying to locate executable files (Ans) b) displays detailed technical specification of your disk drivers c) displays the port address of all the installed ports and dynamically display the status of each port including the I/O ports d) displays the information of adapters e) None of these 24. The PRINT command a) can print in the background if you have an output device connected to one of your systems' serial or parallel ports. b) is used to print a text file c) both a and b mentioned above (Ans) d) to print all the files in the directory e) None of the above 25. The REPLACE command a) selectively copies files from a source file to the target disk b) adds new files to the target disk if they are not already there c) replaces the files target with more recent versions of files d) All of the above (Ans) e) None of these 26. The TREE command a) is an external command b) displays the structure of the directories or the disk in the specified drive (Ans) c) displays the true name of logical drives and directories d) All of these e) None of these 27. Which command in DOS is used to display the version of MSDOS a) VER (Ans) b) VERSION c) VERSN d) RESTORE e) None of these 28. Which among the following best tools for fixing errors on display a) CHKDSK b) FIXDSK c) FDISK d) SCANDISK (Ans) e) None of these 29. Which command displays the list of all previous command entered the user a) DIR/ALL b) COMMANDS/ALL c) KEYDOS d) DOSKEY (Ans) e) None of these 30. VIRUS stands for a) Very Important Record User Searched b) Verify Interchanged Result Until Sources c) Virtual Information Resources Under Search (Ans) d) Very Important Resource Under Size e) None of these 31. Which of the following file names are invalid in MSDOS a) MYFILE.DOS b) CHECKS (1) c) VERIFIED###3 (Ans) d) QWERTY.1? e) None of these 32. Which one of the following DOS commands sends contents of the screen to an output device a) BREAK b) DISKCOPY c) MORE (Ans) d) ASSIGN e) None of these 33. The DOS command, which cannot be executed with version 1 and a) GRAPHICS b) FIND c) LABEL (Ans) d) MODE e) None of these 34. Which one of the following file names is invalid in DOS a) RT.BAT b) LISTEN.BIN c) RLUA.BAT (Ans) d) TALK.BAS e) None of these 35. Which one of the following DIR- commands lists a group of files a) DIR INVOICE.BAS b) DIR RESCUE.BAS c) DIR PAY ROLL.BAS d) DIR TOOL ? BAS (Ans) e) None of these 36. Which one of the following is a valid DOS command a) RENAMEA:SAMPLE:TEXT b) LIST ???.??? c) RECOVER A: (Ans) d) LIST e) None of these 37. Which one of the following is an invalid command a) DIR>>LIST>TCT b) DIR>>LIST>TXT c) DIR>>LPTI: (Ans) d) All of these e) None of these 38. Which one of the following is not an Internal DOS command a) MORE b) REN c) DATE (Ans) d) CLS e) None of these 39. Which one of the following is not an internal DOS command a) TIME (Ans) b) RD c) FORMAT d) TYPE e) None of these 40. Which one of the following is not an internal DOS command a) TIME b) RD c) FORMAT (Ans) d) TYPE e) None of these 41. Which one of the following is not an Internal DOS command a) CLS b) PROMPT (Ans) c) DEL d) CHKDSKS e) None of these Questions on MS WORD 42. What is MS Word a) A game b) An operating system. c) Word Processing Software (Ans) d) Typing tuitor software e) None of these 43. Which of the following companies developed MS Office 2000 ? a) Novell b) Corel c) Lotus d) Microsoft (Ans) e) None of these 44. MS-Word allows creation of ___ type of documents by default a) .DOC (Ans) b) .WPF c) .TXT d) .DOT e) None of these 45. Which of the following is the word Processor in MS Office a) Word pad b) Word (Ans) c) Word perfect d) Word Star e) None of these 46. Which of the following is not a part of a standard officer suit a) File manager (Ans) b) Image editor c) Database d) Word Processor e) None of these 47. What word is used to describe the act of correcting something a) Erase b) Correcting c) Edit (Ans) d) White out e) None of these 48. Outlook Express is a ________ a) E-mail Client b) Schedular c) Address Book d) All of the above (Ans) e) None of these 49. Which of the following is not a special program in MS-Office a) Office Art b) Clip Art c) Word Art d) Paint Art (Ans) e) None of these 50. Which of the following are valid Minimum & Maximum zoom size MS-Office a) 10,100 b) 10,1000 c) 20,250 d) 10,500 (Ans) e) None of these SBI current affairs SBI bank based general awareness questions and answers for practice, SBI and SBT previous years solved question papers,SBI bank based current affairs general awareness questions and answers Bank based general awareness,terms 1. Which of the following statements is true? (1) Banks cannot accept demand and time deposits from public (2) Banks can accept only demand deposits from public (3) Banks can accept both demand and time deposits from public (4) Banks can accept both demand and time deposits from public -Answer (5) Banks can accept demand and time deposits only from government 2. Which of the following is the correct statement? (1) State Bank of India is the sole authority to issue and manage currency in India (2) A nationalized bank is the sole authority to issue and manage currency in India (3) A cooperative bank is the sole authority to issue and manage currency in India (4) RBl is the sole authority to issue and manage currency in India-Answer (5) None of the above 3. Interest payable on savings bank accounts is (1) not regulated by RBI (2) regulated by Sate Governments (3) regulated by Central Government (4) regulated by RBI -Answer(5) regulated by Finance Minister 4. The usual deposit accounts of banks are (1) current accounts, electricity accounts and insurance premium accounts (2) current accounts post office savings bank accounts and term deposit accounts (3) loan accounts, savings bank accounts and term deposit accounts (4) current accounts, savings bank accounts and term deposit accounts-Answer (5) current bill accounts and term deposit accounts 5. Fixed deposits and recurring deposits are (1) repayable after an agreed period-Answer (2) repayable on demand (3) not repayable (4) repayable after death of depositors (5) repayable on demand or after an agreed period as per bank’s choice 6. Accounts are allowed to be operated by cheques in respect of (1) both savings bank accounts and fixed deposit accounts (2) savings bank accounts and current accounts-Answer (3) both savings bank accounts and loan accounts (4) both savings bank accounts and cash accounts only (5) both Current accounts and fixed deposit accounts 7. Which of the following is correct statement? (1) Normally no interest is paid on current deposit accounts-Answer (2) Interest is paid on current accounts at the same rate as term deposit accounts (3) The rate of interest on current account and savings account are the same (4) No interest is paid on any deposit by the bank (5) Savings deposits are the same as current deposits 8. Mortgage is a (1) security on movable property for a loan given by a bank (2) security on immovable property for a loan given by a bank-Answer (3) concession on immovable property for a loan given by a bank (4) facility on immovable property for a loan given by a bank (5) security on immovable property for a deposit received by a bank 9. Which of the following is known as cross selling by banks? (A) Sale of a debit card to a credit card holder. (B) Sale of Insurance policy to a depositor. (C) Insurance of cash against cheque presented by a third party. (1) Only (A) (2) Only (B) (3) Only (C) (4) Both (A) and (C) (5) All (A), (B) and (C)-Answer 10. Financial inclusion means provision of (1) financial services namely, payments, remittances, savings, loans and insurance at affordable cost to persons not yet given the same-Answer (2) ration at affordable cost to persons not yet given the same (3) house at affordable cost to persons not yet given the same (4) food at affordable cost to persons not yet given the same (5) education at affordable cost to persons not yet given the same 11. When a bank returns a cheque unpaid, it is called (1) payment of the cheque (2) drawing of the cheque (3) canceling of the cheque (4) dishonour of the cheque-Answer (5) taking of the cheque 12. NEFT means (1) National Electronic Funds Transfer system-Answer (2) Negotiated Efficient Fund Transfer system (3) National Efficient Fund Transfer solution (4) Non Effective Fund Transfer system (5) Negotiated Electronic Foreign Transfer system 13. Upper limit prescribed for RTGS transaction is (1) Rs. 1 lac (2) Rs. 2 lacs (3) Rs. 5 lacs (4) Rs. 50 lacs (5) No upper limit is prescribed-Answer 14. Distribution of insurance products and insurance policies by banks as corporate agents is known as (1) General Insurance (2) Non-life Insurance (3) Bancassurance-Answer (4) Insurance Banking (5) Deposit Insurance 15. In order to attract more foreign exchange the Government of India decided to allow foreign investment in LLP firms. What is full form of ‘LLP’ as used in this reference? (1) Local Labour Promotion (2) Low Labour Projects (3) Limited Loan Partnership (4) Longer Liability Partnership (5) Limited Liability Partnership-Answer 16. Interest on Saving bank account is now calculated by banks on (1) minimum balance during the month (2) minimum balance from 7th to last day of the month (3) minimum balance from 10th to last day of the month (4) maximum balance during the month (5) daily product basis-Answer 17. Largest shareholder (in percentage shareholding) of a nationalized bank is (1) RBI (2) NABARD (3) LICI (4) Government of India -Answer(5) IBA 18. When the rate of inflation increases (1) purchasing power of money increases (2) purchasing power of money decreases-Answer (3) value of money increases (4) purchasing power of money remains unaffected (5) amount of money in circulation decreases 19. A centralized databases with online connectivity to branches, internet as well as ATM-network which has been adopted by almost all major banks of our country is own as (1) Investment Banking (2) core Banking-Answer (3) Mobile Banking (4) National Banking (5) Specialized Banking 20. The Unit Trust of India came into existence in (1) 1960 (2) 1962 (3) 1964-Answer (4) 1966 (5) 1968 21. Which of the following is example of financial assets? (1) National Saving Certificates (2) Infrastructure Bonds (3) Indira Vikas Patra (4) Krishi Vikas Patra (5) All of the above-Answer 22. Capital market is a market which deals in (1) short-term funds (2) long-term funds-Answer (3) gilt-edge securities (4) All of the above (5) None of the above 23. Regional Rural Banks fall within supervisory purview of (1) SBI (2) RBI -Answer(3) SEBI 4) IRDA (5) None of these 24. IRDA with its headquarters at ……… is the regulatory authority for all insurance companies in India including the Life Insurance Corporation of India. (1) Hyderabad-Answer (2) Bengaluru (3) Mumbai (4) Delhi (5) Chandigarh 25. Mutual Funds fall within 7 supervisory purview of (1) SBI (2) RBI (3) SEBI -Answer(4) IRDA (5) None of these 26. Which of the following does not come under the category of Development Banks? (1) Industrial Development Bank of India (2) Small Industries Development Bank of India (3) Industrial Investment Bank of India (4) State Finance Corporation (5) Export-import Bank-Answer 27. Main financial instruments of corporate sector are (1) Shares (ii) Debentures (iii) Public Deposits (iv) Loan from Institutions Select the correct answer by using of the following codes (1) i and ii (2) ii and iii (3) iii and iv (4)1, ii and iv (5) All I, ii, iii and iv-Answer 28. Financial institutions (1) promote savings (2) mobilise savings (3) allocate savings among different users (4) All of the above -Answer(5) None of the above 29. Which of the following is not an / example of primary securities? (1) Bills (2) Bonds (3) Shares (4) Book debts (5) New currency-Answer 30. Indian Financial System / comprises of (1) Scheduled Commercial Banks (2) Non-banking Financial Institutions (3) Urban Cooperative Banks (4) All of the above (5) None of the above-Answer 31. The Bombay Stock Exchange was 7 made functional as early as (1) 1870 -Answer(2) 1901 (3) 1935 (4) 1951 (5) 1949 32. The Unit Trust of India come into existence in (1) 1964-Answer (2) 1970 (3) 1975 (4) 1980 (5) 1982 33. 19 July 1969, how commercial Banks were nationalised? (1) 13 (2) 14 -Answer(3) 15 (4) 16 (5) 20 34. New Private Banks are being given licenses since (1) 1991 (2) 1992 (3) 1993-Answer (4) 1995 (5) 2001 35. The gilt-edged market refers to the market for (i) Government securities (ii) Semi-government securities (iii) Corporate securities Select the correct answer (1) only i (2) i and ii -Answer(3) ii and iii (4) i, ii and iii (5) only iii 36. First share market in India established in (1) Delhi (2) Mumbai-Answer (3) Kolkata (4) Chennai (5) None of these 37. Consider the following statements: (i) Securities that have an original maturity that is greater than one year are traded in capital markets. (ii) The best known capital market securities are stocks and bonds. Select the correct answer (1) (i) is true and (ii) is false (2) (i) is false and (ii) is true (3) Both are true-Answer (4) Both are false (5) None of the above 38. Consider the following statements: (i) Securities that have an original maturity that is greater than one year are traded in money markets. (ii) The best known money market securities are stocks and bonds. (1) (i) is true and (ii) is false (2) (i) is false and (ii) is true (3) Both are true (4) Both are false-Answer (5) None of the above 39. The primary issuers of capital market securities include (1) the Central Government (2) the local Government (3) corporations 4) the Central and Local Governments and corporations-Answer (5) Local Government and corporations 40. Which of the following is a / characteristic of a capital market instrument? (a) Liquidity (b) Marketability (3) Long maturity (4) Liquidity premium 5) All of the above-Answer 41. Which one of the following is a money market instruments? (1) A Treasury bill (2) A negotiable certificate of deposit was (3) Commercial paper (4) All of the above -Answer (5) None of the above 42. T-bills are financial instruments initially sold by ________ to raise funds. (1) Commercial Banks (2) the government-Answer (3) corporations (4) agencies of the State Government (5) None of the above 43. Commercial paper is a short-term security issued by ________ to raise funds. (1) the Reserve Bank of India (2) Commercial Banks (3) large and well-known companies-Answer (4) National Stock Exchange (5) State and Local Governments 44. Which of the following statements is true regarding a corporate bond? (1) A corporate callable bond gives the holder the right to exchange it for a specified number of the company’s common shares (2) A corporate debenture is a secured /‘ bond (3) A corporate indenture is a secured bond (4) A corporate convertible bond gives the holder the right to exchange thebond for a specified number of the company’s common shares -Answer (5) Holders of corporate bonds have voting rights in the company 45. Which one of the following is not a money market instrument? (1) A Treasury bill (2) A negotiable certificate of deposit (3) Commercial paper (4) Treasury bond-Answer (5) Repo 46. Money lend for 15 days or more in Inter-bank market is called (1) call money (2) notice mone (3) term money-Answer (4) All of these (5) None of these 47. Money lent for one day is called (1) call money -Answer(2) notice money (3) term money (4) All of these (5) None of these 48. Specified interest rate on a fixed maturity security fixed at the time of issue is called (1) market rate of interest (2) call rate (3) repo rate (4) coupon rate -Answer5) discount rate 49. Lending of scheduled Commercial Banks, on a fortnightly average basis, should not exceed — of their capital fund. (1) 25 per cent -Answer(2) 35 per cent (3) 15 per cent (4) 50 per cent (5) None of these 50. A short-term credit investment created by a non-financial firm and guaranteed by a bank to make payment is called (1) bankers acceptance market-Answer (2) collateral loan market (3) treasury bill market (4) call money market (5) repo market 51.From the given options A to E , please find out the one which is not an objective of a central Bank of a country? (A)The central Bank of a country aims at profit-Answer (B)The central bank is given powers to control and regulate the working of the commercial banks (C)Central Bank generally is a organ of the government and run by government officials (D)Central Bank generally controls the credit (E)Central bank is generally banker of the government 52.From the given options, bring out the one which is not a function / power of Reserve Bank of India: (A)To assume the responsibility of meeting directly or indirectly all reasonable demands for accommodation (B)To hold cash reserves of the commercial banks and make available financial accommodation to them (C)To enjoy monopoly of the note issue (D)To assume responsibility of all banking operations of the government (E)To assume the responsibility of statistical analysis of data related to macro economy of India-Answer 53.Many a times we read in the newspapers about margin requirements. From the given options, find out the one which correctly indicates margin requirements: (A)Margin requirements aim at the regulation of the volume of credit as well as flow of the credit (B)Margin requirements imply that every bank has to keep certain minimum cash reserves with the reserve bank of India (C)Margin requirements imply that every bank has to keep certain proportion of its total deposits in the form of cash with it self (D)Margin requirements imply that RBI has certain rate at which it discounts the bills of exchange of the commercial bank (E)Margin requirements imply to a cushion against the decline in the value of the security-Answer 54.What will be the impact on the cash reserves of commercial banks if RBI conducts a sale of securities ? (A)Increase (B)Decrease-Answer (C)Remain constant (D)Increase or decrease (E)Increase if the securities are sold in open market operations 55.Which among the following is a major qualitative control measure in India ? (A)Bank Rate Policy (B)Open market Operations (C)ways and means advances (D)Reserve Ratio Requirements (E)Margin Requirements-Answer 56.Many a times we read in the financial newspapers that Reserve bank of India is "Lender of Last Resort (LOLR) " In India. Which among the following statement gives the most correct definition of "Lender of Last Resort"? (A)If a person or firm which is eligible to get a loan, does not get it from any commercial bank, may approach to Reserve Bank of India for loan. (B)If the state governments are in crisis and need money for short term , they can approach RBI for this purpose (C)If a commercial bank is in crisis, it may place its reasonable demand for accommodation to Reserve Bank of India-Answer (D)A scheduled commercial bank meets all of its demands in all weathers from Reserve bank of India (E)Whenever the government declares a debt relief, the RBI will have to bear the brunt of it. 57.(The figures in this questions are imaginary). We suppose that Cash Reserve Ratio in country's economy is 10%. The banking system wish a cash deposits of Rs. 1000 Crore, creates total deposits of Rs. 10,000 Crores. The reserve Bank wishes that bank should create more deposits. Which among the following step will be taken by the Reserve Bank? (A)It will lower the Cash Reserve Ratio-Answer (B)It will raise the Cash Reserve Ratio (C)It will increase the margin Requirements (D)It will start selling Government securities (E)It will start rationing of credit 58.Many a times we read in the newspapers that RBI takes certain steps to curb the menace of Inflation. In this context, which among the following will not help RBI in controlling the inflation in the country? (A)An increase in the Bank Rate (B)An increase in the Reserve Ratio Requirements (C)A purchase of securities in the open market-Answer (D)Rationing of the credit (E)Increasing the Repo Rate 59.We suppose that Reserve Bank of India would like to increase the cash Reserves of the commercial Banks. Which among the following would be most appropriate action of the RBI to achieve this aim? (A)RBI would release gold from its reserves (B)RBI would raise the reserve ratio (C)RBI would buy the bonds in the open market-Answer (D)RBI will stop the transactions which involve the bills of exchange (E)RBI will put more reserves in the International Monetary Fund 60.Which among the following is a incorrect statement: (A)The Reserve Bank of India has the special powers to control and regulate the commercial banking system (B)A rise in the bank rate is a strong anti-deflationary monetary tool-Answer (C)Minimum Reserve Requirements are fixed to ensure the liquidity and solvency of individual commercial banks (D)Reserve Ratio Requirement is a quicker method than bank rate and OMO (Open Market Operations) in general credit regulations (E)Rationing of credit is a powerful qualitative monetary tool 61.Since which year, Reserve bank of India is using the Selective Credit Control measures to control the amount of bank advances against the commodities having limited supply ? (A)1949 (B)1956-Answer (C)1969 (D)1973 (E)1982 62.The "Service area Approach" was an strategy launched to improve which of the following? (A)Micro, Small and medium Enterprising (B)Unorganized Sector (C)Rural lending-Answer (D)Urban Industrial Lending (E)A & B 63.Which among the following bank / banks in India have set up the Financial Literacy & Credit Counseling centers ? (A)RBI (B)Scheduled Commercial banks-Answer (C)Foreign Banks working in India (D)Regional Rural Banks (E)All of above 64.Which of the following imaginary circumstances, the Reserve bank of India will opt to sell Government securities in the open market? (A)When the Foreign funds inflow is meek (B)When there is enormous Foreign Funds Inflow in the Indian Economy-Answer (C)When banks have low liquidity and need liquidity (D)When Government of India asks the RBI to sell securities (E)None of the above 65.Which among the following is correct full form of CAS in context with banking markets in India ? (A)Cash Authorization Scheme (B)Credit Authorization Scheme-Answer (C)Credit Access System (D)Credit Arrangement System (E)Cash Accreditation Scheme SBI Marketing Aptitude Questions SBI TEST Marketing Aptitude questions with answers for practice's and SBT previous years solved question papers,SBT and SBI free solved sample placement papers,SBT and SBI Aptitude reasoning marketing computer,general knowledge questions with answers,All Bank IT and Non IT free solved question papers IBPS PO CWE Grmain Bank solved practice papers SBI and SBI marketing Aptitude Questions 1. Marketing persons need (1) Innovative skills (Ans) (2) Laborious attitude (3) Fighting nature (4) Quality of imitating others (5) Sympathetic approach Ans : A marketing person should posses innovative skills in order to generate new ideas and plans to execute them to get the desired result. 2. Communication skills in the case of a DSA means (1) Oratory excellence (2) Delivering long speeches (3) Loquacious (4) Ability to convince the customer with the right choice of words (Ans) (5) Ability to talk very fast Ans : A direct selling agent must have the communication skills as he has to convince the customers with the proper choice of words. 3. A 'Prospect' means (1) A rude person (2) A polite person (3) A likely buyer (Ans) (4) A religious head (5) A team leader Ans : A prospect is a person who can become a buyer of the product after being properly convinced. 4. Social Marketing refers to (1) Share market prices (2) Marketing by the entire society (3) Internet Marketing (4) Marketing for a social cause (Ans) (5) Society bye-laws Ans : Social Marketing is done for a social concern. 5. A 'Call' means (1) to call out to someone (2) a profession (3) a speech (4) a new product (5) a sales person visiting likely a buyer (Ans) Ans : In terms of marketing, when a sales person visits the likely buyer then it is termed that he is making a call. 6. Effective selling skills depends on (1) knowledge level of competitors (2) information about marketing staff (3) information regarding Share Market (4) knowledge of related markets (Ans) (5) information regarding political leaders Ans : Knowledge of the related markets is the sale criteria for the effective selling skills. 7. Modern styles of marketing include ____ find the wrong answer. (1) digital marketing (2) tele-marketing (3) door-to-door marketing (4) e-mail solicitation (5) All of these (Ans) Ans : Modern style of marketing includes the following: (i) Digital Marketing (ii) Tele Marketing (iii) Door-to-Door Marketing (iv) e-mail solicitation 8. A DSA means (1) Detective Service Agency (2) Direct Selling Agent (Ans) (3) Direct Supplying Agent (4) Distribution & Sales Agency (5) Disciplined Sales Agent Ans : A DSA is the person who sells the goods directly which is known as Direct Selling Agent. 9. Incentives are paid to sales persons (1) for missing the targets (2) for surpassing the targets (Ans) (3) for identifying leads (4) for designing products (5) for travelling Ans : When the sales persons attain defined targets they are paid the incentives in addition to the salaries. 10. Online Marketing is useful for (1) selling old products (2) sending e-mails (3) increased products (Ans) (4) increased job opportunities (5) increased expenses Ans : Online marketing helps in case of increased production of the goods as they can be marketed in the short span of time. 11. Service marketing is the same as (1) Internet Marketing (2) Telemarketing (3) Internal Marketing (4) Relationship Marketing (Ans) (5) Marketing done by service class employees Ans : In relationship marketing, a good relationship is maintained between the seller and the customer by offering a good quality services. 12. 'Push' marketing style requires (1) proper planning (2) good pushing strength (3) team work (4) ability to identify the products (5) aggressive marketing (Ans) Ans : An aggressive marketing approach is desired to be successful in the push marketing strategy. 13. The Securities Market is governed by the rules which are framed by (1) IRDA (2) SEBI (Ans) (3) AMFI (4) NSE (5) BSE Ans : SEBI governs the entire activities of security market by framing the rules and regulations. 14. Planning for Retirement Saving is a type of (1) Banking Option (2) Stock Market Option (3) Branding (4) Financial Planning (Ans) (5) Mutual Fund Benefit Ans : Planning for retirement saving is a type of Financial Planning. 15. Absence of a proper Financial Planning can lead to (1) balance investment in Mutual Funds (2) overspending and Debt problems (Ans) (3) inadequate exposure to share market (4) capital gains (5) planned Future Ans : Overspending and Debt problems are the results of the absence of a proper financial planning. 16. One of the following is not included in the 7 P's of Marketing. Find the same. (1) Product (2) Price (3) Production (Ans) (4) Promotion (5) People Ans : 7 P's of Marketing are as follows: (i) Product (ii) Price (iii) Place (iv) Promotion (v) People (vi) Process (vii) Physical Evidence 17. 'SIP' in Mutual Fund business means (1) Salaried Individuals and Plans (2) Systemized Insurance Plans (3) Systemized MF Plans (4) Systematic Investment Plans (Ans) (5) None of the above Ans : SIP stands for Systematic Investment Plan. 18. Mutual Fund business from existing bank customers can be mobilized by (1) Outdoor Marketing (2) Telemarketing (3) Cross-selling (Ans) (4) Internal Marketing (5) All of the above Ans : Cross-selling is the process through which more than one product of the same firm can be sold out. 19. Conversion means (1) Grouping of prospective clients (2) Interacting with a prospective client (3) Converting a seller to a buyer (4) Converting a prospect into a buyer (Ans) (5) Converting an employer into an employee Ans : When a prospect is converted into a buyer then this process is termed as conversion in the marketing point of view. 20. Market Segmentation means (1) Territory allocation (2) Market space (3) Market place (4) Dividing the target group as per their needs (Ans) (5) Sales persons' groups Ans : Dividing the entire market into small target groups as per their needs is termed as market segmentation. 21. Marketing Opportunities means (1) Availability of sales persons (2) Availability of data (3) Melas (4) Scope for marketing (Ans) (5) Evaluation of performance Ans : Marketing opportunities are the scopes for marketing. 22. Marketing Research is useful for (1) Deciding proper marketing strategies (Ans) (2) Deciding the sales persons (3) Choosing old products (4) Recruitment of staff (5) Payment of bonus Ans : If a proper marketing strategies has to be decided then market research tool comes into the scene. 23. The main target group for SIP schemes are (1) All HNIs (2) All NRIs (3) All salaried persons (Ans) (4) All agriculturists (5) All students Ans : All salaried persons can be the main target group for the Systematic Investment Plan. 24. Innovation in marketing is same as (1) Motivation (2) Inspiration (3) Aspiration (4) Creativity (Ans) (5) Team work Ans : Innovation in marketing mean to create or generate an idea or plan in order to have a new product or service. 25. 'Buyer Resistance' means (1) Buyer's interest in the product being sold (2) Buyer fighting with the seller (3) Buyer's hesitation in buying the product (Ans) (4) Buyer becoming a seller (5) Buyer buying the product Ans : When a buyer hesitates to buy the product then this is termed as buyer resistance. 26. Bulk Savings Accounts can be mobilized by means of (1) Door-to-door canvassing (2) Making cold calls (3) e-mail solicitation (4) Contacting all employees of an institution (Ans) (5) Writing letters Ans : Contracting all employees of an institution can be a means of the bulk saving accounts mobilization as at one point of time & place a number of customers can be convinced by the seller. 27. Diversification in marketing means (1) Marketing to different countries (2) Marketing in many companies (3) Marketing of the same product by many, diverse persons (Ans) (4) Marketing of new, diverse product (5) All of the above Ans : When the same product is marketed by many diverse persons then it is termed as diversification in marketing. 28. One of the following is not a pre-sales activity. Find the same. (1) Lead generation (2) Product design (3) Sales presentation (4) After-sales service (Ans) (5) Scanning the Yellow Pages Ans : 'After Sales Service' is the service delivered after the goods being sold out. 29. Corporate Loans are given to (1) Individuals (2) Blind persons (3) Schools & Colleges (4) Proprietary concerns (5) Limited Companies (Ans) Ans : Corporate loans are handsome bulk amount given to the limited companies having the authentic credibility. 30. Financial Inclusion needs canvassing the Accounts of (1) Financial Institutions (Ans) (2) NRIs (3) HNIs (4) Housewives (5) Persons from the weaker sections Ans : Financial inclusion needs canvassing the account of financial institutions. 31. Value added sevices means (1) Costlier products (2) More number of products (3) Additional services (Ans) (4) At par services (5) New products Ans : When additional services are offered along with a particular product at the same price then it is termed as Value Added Services. 32. The USP of a Current Accounts is (1) High minimum balance (2) No restrictions on transactions (Ans) (3) No interest payable (4) Restricted deposits (5) Any number of accounts Ans : The best benefit to have a current account is there is no any restriction on transactions. 33. Situation Analysis is useful for (1) SWOT Analysis (Ans) (2) Analysis of Sales persons performances (3) Analysis of capital markets (4) Staff placement (5) Area allocation Ans : Situation analysis is useful for SWOT analysis. 34. RTGS facility can be best canvassed among (1) Inter-state traders (Ans) (2) Tailors (3) Carpenters (4) Politicians (5) Film stars Ans : Real Time Gross Settlement (RTGS) facility can be best canvassed among the inter state traders. 35. 'USP' of a product connotes (1) High selling features of a product (Ans) (2) Drawbacks of a product (3) New additions to a product (4) Efficient staff (5) Large organization Ans : The High Selling features of a product is known as the USP of the product. 36. CRM (Customer Relationship Management) is (1) a pre-sales activity (Ans) (2) a tool for lead generation (3) an on going daily activity (4) the task of a DSA (5) customer complaints Ans : CRM (Customer Relationship Management) is an ongoing daily activity under that relationships with customers are maintained by handling their grievances. 37. Recurring accounts can best be canvassed among (1) farmers (2) traders (Ans) (3) salaried persons (4) minors (5) students Ans : Recurring accounts can best be canvassed among the traders. 38. ELSS means (1) Entry Load Shares Scheme (2) Entry Load Starting Shares (3) Entry Linked Savings Scheme (Ans) (4) Equity Linked Shares Sold (5) Employee Level Salary Scheme Ans : ELSS Stands for Equity Linked Saving Scheme. 39. Societies and Trusts are the target groups for opening (1) Current Accounts (2) Saving Accounts (3) Car Loan Accounts (4) Corporate Loan Accounts (Ans) (5) Education Loan Account Ans : Societies and trusts are the target groups for opening the corporate loan accounts. 40. De-mat accounts are useful for (1) Online trading of shares (Ans) (2) Locker operations (3) ATMs (4) Quick sanction of loans (5) Export transactions Ans : In order to do online trading of shares one must have the Demat account. 41. Home Loans are basically (1) short terms finance (2) loans given to minors (3) long term loans (Ans) (4) loans given for boosting profits (5) loans given more to industries Ans : Home loans are long term loans. 42. What does the term HUF means? (1) Hindu Undivided Family (Ans) (2) Hindu Single Family (3) Heavy Facilities (4) Hindered Remittances (5) Term is used for failed transactions Ans : 'HUF' stands for 'Hindu Undivided Family'. 43. EMI can be a marketing tool if (1) EMI is increasing (2) It is very high (3) It is very low (Ans) (4) EMI has no impact on marketing (5) EMI is a flat rate Ans : Equated Monthly Installment (EMI) being low can be marketing tool. 44. Channel Finance can be considered only among (1) Supply-chain distributors (Ans) (2) Chain snatchers (3) NRIs (4) Pensioners (5) Housewives Ans : Channel Finance can be considered among the Supply-Chain distributors. 45. Direct Marketing is useful for (1) Designing Products (2) Sending e-mails (3) Increased production (4) Increased job opportunities (5) None of the above (Ans) Ans : The benefits of direct marketing are as follows: (i) Flexible targeting (ii) Multiple Uses (iii) Cost Effectiveness (iv) Ease of Management (v) Rapid Delivery 46. The pricing strategy for credit cards depends on (1) Competition (2) Customer's income (Ans) (3) Customer relations (4) Customer awareness (5) Customer usage Ans : On the basis of customers' income the pricing strategy is adopted. 47. Offsite Marketing means (1) Salespersons (2) Marketing in the Production Department (3) ATMs (4) Marketing inside the Branch (5) Marketing function outside the Branch (Ans) Ans : Marketing Activities carried outside the branch is known as offsite marketing. 48. One of the following is not a target group for Saving Accounts (1) Salaried persons (2) Doctors (3) Public servant (4) Insurance agents (5) Loss making companies (Ans) Ans : Loss marketing companies are not generating or earning any revenue then it can not be the target group for saving accounts. 49. Rural bank marketing has good scope for (1) Industrial loans (2) Corporate loans (3) NRI loans (4) Tractor loans (Ans) (5) All of these Ans : For the marketing of tractor loans rural bank marketing can be a good tool. 50. ASBA accounts are a type of (1) Loans accounts (2) Credit card facilities (3) Savings accounts (Ans) (4) Export-Import remittances (5) Internet banking Ans : ASBA (Application Supported by Blocked Amount) is a saving account. SBI Marketing Aptitude and Computer Knowledge Question Paper 2011-2012 ,SBI previous years solved question papers, SBI quantitative aptitude, reasoning, DI, computer knowledge, marketing aptitude, economy bank based general awareness questions with answers, SBI PO Clerical cadres written test examination question papers with detailed solutions, SBI preparation tips and tricks Q. 1. An email account includes a storage area, often called a(n) __ (1) attachment (2) hyperlink (3) mailbox-Answer (4) IP address (5) None of these Q. 2. Data becomes __ when it is presented in a format that people can understand and use. (1) processed (2) graphs (3) informationAnswer (4) presentation (5) None of these Q.3. A set of computer programs that helps a computer monitor itself and function more efficiently isajan __ (1) Windows (2) System SoftwareAnswer (3) DBMS (4) Application Software (5) None of these Q. 4. An area of a computer that temporarily holds data waiting to be processed is __ (1) CPU (2) MemoryAnswer (3) Storage (4) File (5) None of these Q.5. A __ is a microprocessor-based computing device. (1) personal computerAnswer (2) mainframe (3) workstation (4) server (5) None of these Q. 6. You use a(n) __ , such as a keyboard or mouse, to input information. (1) output device (2) input deviceAnswer (3) storage device (4) processing device (5) None of these Q.7. The term __ designates equipment that might be added to a computer system to enhance its functionality. (1) digital device (2) system add-on (3) disk pack (4) peripheral deviceAnswer (5) None of these Q. 8. The instructions that tell a computer how to carry out the processing tasks are referred to as computer __ (1) programsAnswer (2) processors (3) input devices (4) memory modules (5) None of these Q. 9. RAM can be thought of as the __ for the computer's processor. (1) factory (2) operating room (3) waiting roomAnswer (4) planning room (5) None of these Q. 10. C, BASIC, COBOL, and Java are examples of __ languages. (1) low-level (2) computer (3) system programming (4) high-levelAnswer (5) None of these Q.11. __ is the ability of a device to "jump" directly to the requested data. (1) Sequential access (2) Random accessAnswer (3) Quick access (4) All of the above (5) None of these Q.12. The __ is the amount of data that a storage device can move from the storage medium to the Computer per second. (1) data migration rate (2) data digitizing rate (3) data transfer rate Answer (4) data access rate (5) None of these Q.13. A __ converts all the statements in a program in a single batch and the resulting collection of instructions is placed in a new file. (1) converter (2) compilerAnswer (3) interpreter (4) instruction (5) None of these Q.14. One thousand bytes is a __ (1) kilobyteAnswer (2) megabyte (3) gigabyte (4) terabyte (5) None of these Q.15. 'Benchmark' means __ (1) Benches for customers to sit (2) Benches for salesmen to sit (3) Products displayed on a bench (4) Set standardsAnswer (5) All of the above Q. 16 A Call Centre is __ (1) a meeting place for DSAs (2) a Training Centre for DSAs (3) a meeting place for customers (4) Data Centre (5) a back office set up where customer queries are answered Answer Q .17. The sequence of a sales process is __ (1) Lead generation, Call, Presentation & Sale Answer (2) Sale, Presentation, Lead generation & Call (3) Presentation, Lead generation, Sale & Call (4) Lead Generation, Call, Sale & Presentation (5) There is no sequence required Q.18. Home loans are granted to - (1) IndividualsAnswer (2) Institutions (3) Builders (4) All of these (5) None of these Q.19. To 'Close a Call' means __ (1) 'fo end the conversation (2) To put the phone down (3) To close the doors (4) To clinch the saleAnswer (5) To close the business Q.20. A __ is an additional set of commands that the computer displays after you make a selection from the main menu. (1) dialog boxAnswer (2) submenu (3) menu selection (4) All of the above (5) None of these Q.21. Information kept about a file includes ---- (1) print settings (2) deletion date (3) (1) & (2) only Answer (4) size (5) None of these Q.22. __ provides process and memory management services that allow two or more tasks, jobs, or programs to run simultaneously. (1) MultitaskingAnswer (2) Multithreading (3) Multiprocessing (4) Multicomputing (5) None of these Q.23. The ALU performs __ operations. (1) arithmeticAnswer (2) ASCII (3) algorithm-based (4) logarithm-based (5) None of these Q. 24. A (n) __ is software that helps a computer control itself to operate efficiently and keep track of data. (1) application system (2) hardware system (3) software system (4) oprating system Answer (5) None of these Q.25. 'Customisation' means __ (1) Tailor-made products for each customerAnswer (2) Customers selling goods (3) Tailor-made products for each staff 4) A selling process (5) None of these Q.26. A 'Call' in Marketing language means __ (1) Calling On a salesperson (2) Calling on a customerAnswer (3) Making a phone-call (4) Telemarketing (5) None of these Q.27. Computers manipulate data in many ways, and this manipulation is called __ (1) upgrading (2) processingAnswer (3) batching (4) utilizing (5) None of these Q.28. A Marketing Survey is required for __ (1) deciding marketing strategies (2) deciding Product strategies (3) deciding pricing strategies (4) All of theseAnswer (5) None of these Q.29. The Target Group for Education Loans is ---- (1) all colleges (2) all parents (3) research scholars (4) meritorious students seeking higher education Answer (5) all of these Q.30. Cross-selling means __ (1) Selling with a cross face (2) Cross country marketing (3) Selling other products to existing customersAnswer (4) Selling to friends (5) Selling to employees Q.31. Market Segmentation is useful for--- (1) Preferential marketing (2) Targeting existing clients (3) Identifying prospects (4) Knowing customers' tastes (5) All of the above Answer Q.32. The Target Group for Savings Deposit Accounts is __ (1) Newborn babies (2) students (3) parents (4) businessman (5) all of the above Answer Q. 33. Market Segmentation can be resorted to by means of __ (1) segmenting by age (2) segmenting by income (3) segmenting geographically (4) all of theseAnswer (5) none of these Q.34. The Target Group for a Car Loan is --- (1) all auto drivers (2) all auto dealers (3) all car owners (4) any individual needing a carAnswer (5) all of these Q.35. Market information means --- (1) knowledge of industries (2) knowledge of households (3) knowledge of peers (4) knowledge of customers' tastes (5) All of theseAnswer Q.36. Credit cards are used for - (1) Cash withdrawals (2) Purchase of air tickets (3) Purchase of consumable items from retail outlets (4) All of theseAnswer (5) None of these Q.37. TheTarget Group for Agricultural Loans is __ (1) any farmer (2) farm labourers (3) any individual dealing in agricultural or related activity Answer (4) farmers'societies (5) all of these Q.38. The Target Group for Credit Cards is ---- (1) existing cardholders (2) all graduates (3) all minors (4) individuals with taxable incomeAnswer (5) all of these Q.39. Market Segmentation means __ (1) segmenting the salesmen (2) segmenting the employees (3) segmenting the customers as per their Heeds Answer (4) segmenting the products (5) All of these Q. 40. A 'Target Group' means--- (1) a group of sellers (2) a group of buyers (3) a group of products (4) a group of persons to whom sales should be focused (5) all of these Answer SBI stands for State Bank of India, SBI Central Recruitment and promotion of Department recruit clerical cadre in Associate Banks of State Bank of India SBI latest bank profile, SBI,free solved sample placement papers of clerk, SBI Associate Banks Clerks Recruitment 2012 notification and job details, SBI Dates of Written Examination : 07.10.2012 and 14.10.2012,SBI previous years solved question papers, SBI 2010,2011,2012 Clerks and po officer job written test examination syllabus and selection procedure, SBI PO and Associate banks clerks examination questions papers, SBIAssociate Banks Clerks Recruitment 2012 notification and detailed explanation with answers and solutions, How to crack Sbi written test examination tips and trick,SBI, General Awareness, General English, Quantitative Aptitude, Reasoning Ability, Marketing Aptitude / Computer Knowledge whole solved questions paper, Latest general awareness SBI Associate Bank clerical cadre question papers and selection procedure and test pattern SBI Written Test Questions and answers |Reasoning ability,Aptitude,General Awareness SBI Latest Placement paper Questions and Answers SBI I-General Awareness Questions 1. What is the name of the cyclone, which hit W.Bengal in May in which at least 94 people died? 1) Nila 2) Aila 3) Kila 4) Fila 5) None of these (Ans:2) 2. Which Govt. refuses Suu Kyi, even after the intervention of UN Security Council? 1) Srilanka 2) Nepal 3) Myanmar 4) Bhutan 5) None of these (Ans:3) 3. How many applicants are there in the final list who had booked Nano car? 1)103500 2)206703 3)305802 4) 553000 5) None of these (Ans:2) 4. What is the amount of loan sanctioned by the World Bank for renovation and modernization of four coal fired units? 1)105mn 2)$225.5 mn 3) $158.26 mn 4) $342.5 mn 5) None of these (Ans:2) 5. Who has been named as CEO of Mahindra Satyam? 1) C.P Gurnani 2)S.Durgashankar 3) A.S Munfay 4) N. narayanan 5) None of these (Ans:1) 6. Katchatheevu was ceded to Srilanka in the year? 1) 1990 2) 1974 3) 1975 4) 2001 5) None of these (Ans:2) 7. The location of recent NAM? (Non Alignment meet)? 1) Israel 2) India 3) China 4) Egypt 5) None of these (Ans:4) 8. Ram Pradhan Committee is connected with? 1) Gujarat riots 2) Bhopal Tragedy 3) Banking reforms 4) Mumbai attack 5) None of these (Ans:4) 9. The international Festival of Films and Tribal Art & Culture was held in? 1) Bhopal 2) Luknow 3) Pune 4) Mumbai 5) one of these (Ans:1) 10. Ashok Chawla is the Secretary of? 1) Defence 2) Commerce 3) Finance 4) Home 5) None (Ans:3) SBI General English Questions with answers II-General English Q (1-10): In the following passage there are blanks each of which has been numbered. These numbers are printed below the passage and against each five words are suggested one of which fits the blank appropriately. Find out the appropriate word in each case. I had (1) to become a multipurpose manager. I took three steps to accomplish this. First I took very (2) part in professional associations. I (3) the National President of the Indian Society for Training and Development-one of the (4) organizations for HR professionals. This helped me to grow professionally. My communication skills (5). It also taught me (6) to conduct meetings in an effective manner. My job (7) toe to different parts of the country and (8) believe that (9) provided me with an education. I also wrote a lot. Newspaper editors often approached me with (10) to write articles for their publications. 1. a) achieved b) aim c) try d) dreamed e) wanted (Ans:e) 2. a) selected b) active c) interesting d) often e) joint (Ans:b) 3. a) elected b) voted c) became d) applied e) decided (Ans:c) 4. a) respected b) status c) impressed d) aged e) common (Ans:a) 5. a) lacked b) grown c) learnt d) improved e) earned (Ans:d) 6. a) that b) why c) how d) never e) anyhow (Ans:c) 7. a) saw b) showed c) posted d) discovered e) took (Ans:e) 8. a) quite b) firmly c) strong d) first e) not (Ans:b) 9. a) traveling b) journey c) visit d) migrating e) shift (Ans:a) 10. a) wish b) offer c) appointments d) requests e) commands (Ans:d) SBI Quantitative Aptitude Questions III-Quantitative Aptitude 1. v17956=? 1) 134 2) 144 3) 124 4) 104 5) None of these (Ans:1) 2. 30 % of 200+ v?= 48 % of 550- 10 % of 150 1) 600 2) 21 3) 189 4) 35721 5) None of these (Ans:4) 3. 11.6 x 8.9 x 5.1 = ? 1) 398.264 2) 664.358 3) 468.428 4) 526.524 5) None of these (Ans:4) 4. 45 % of 1200 = 54 % of ? 1) 1080 2) 1320 3) 1240 4) 720 5) None of these (Ans:5) 5. 1354 + 1184 = ? % of 5640 1) 36 2) 42 3) 45 4) 52 5) None of these (Ans:3) 6. ? ÷ 40 x 9 =378 1) 1616 2) 1648 3) 1696 4) 1680 5) None of these (Ans:4) 7. 2637 ÷ 36 = ? 1) 73.25 2) 68 3) 66.5 4) 71 5) None of these (Ans:1) 8. 11160 ÷ 45 ÷ 8 = ? 1) 29 2) 31 3) 43 4) 47 5) None of these (Ans:2) 9. 18 x 8 + (?) 2 = (15)2 1) 9 2) 81 3) 18 4) 27 5) None of these (Ans:1) 10. 54679 + 34521 = ? + 43668 1) 45352 2) 45232 3) 42455 4) 45552 5) None of these (Ans:5) SBI Reasoning Ability Questions IV-Reasoning Ability 1. Four of the following five are alike in a certain way and to form a group. Which is one that does not belong to that group? 1) Orange 2) Apple 3) Guava 4) Papaya 5) Mango (Ans:1) 2. How many such pairs of letters are there in the word BEHAVIOUR each of which has as many letters between them in the word as in the English alphabet? 1) None 2) One 3) Two 4) Three 5) More than one (Ans:3) 3. How many such digits are there in the number 764528 each of which is as far away from the beginning of the number as when the digits are arranged in descending order within the number? 1) None 2) One 3) Two 4) Three 5) More than three (Ans:2) Study the following arrangement carefully and answer the questions given below: H T 7 L E $ 2 Y # W 4 I F @ V 3 A 8 % K 1 M © 5 G U 6 C 4. Which of the following is the fifth to the left of the fourteenth from the left end of the above arrangement? 1) # 2) W 3) Y 4) K 5) None of these (Ans:1) 5. How many such vowels are there in the above arrangement, each of which is immediately preceded by a number but not immediately followed by a symbol? 1) None 2) One 3) Two 4) three 5) More than three (Ans:3) 6. Four of the following five are alike in a certain way based on their positions in the above arrangement and so form a group. Which is the one that does not belong to that group? 1) H E 7 2) 2 4 # 3) K 5 M 4) 3 K 8 5) F V A (Ans:5) 7. How many such consonants are there in the above arrangement each of which is immediately preceded by a number and also immediately followed by a symbol. 1) None 2) One 3) Two 4) Three 5) More than three (Ans:3) 8. What should come next in the letter series given below? A B A B C A B C D A B C D E A B C D E F A B C D E F G A B C 1) D 2) E 3) F 4) H 5) None of these (Ans:1) 9. How many meaningful English words can be made with the letters EPRY using each letter only once in each word? 1) None 2) One 3) Two 4) Three 5) More than three (Ans:3) 10. J, D, L, H and F each traveling to station, each one reaches at a different time. L reaches only after J and D reaches only before F. Who amongst them is third to reach? 1) F 2) L 3) D 4) Cannot be determined 5) None of these (Ans:5) V- Computer Knowledge 1. Compiler is the (a) name given to the computer operator (b) part of the digital machine to store the information (c) translator of source program to object (d) part of arithmetic logic unit (e) operator of Boolean Algebra Ans (c) 2. Main memory is (a) Random Access Memory (b) Read only Memory (c) Serial Access Memory (d) None of these Ans (a) 3. Which of the following is the smallest and fastest computer imitating brain working? (a) Supercomputer (b) Quantum Computer (c) Param-10,000 (d) IBM chips Ans (b) 4. A compact disc (CD) is a data storage of the type (a) Magnetic (b) Optical (c) Electrical (d) Electromechanical Ans (a) 5. Which of the following is not as language for computer programming? (a) WINDOWS (b) PASCAL BASIC (d) COBOL (e) All of these Ans (a) 6. What are the two basic types of memory that your computer uses? (a) RAM (b) RW/RAM (c) ROM ERAM (e) POST Ans (a) 7. The term gigabyte refers to (a) 1024 bytes (b) 1024 kilobytes (c) 1024 megabytes (d) 1024 gigabytes (e) None of the above Ans (c) 8. A computer with CPU speed around 100 million instructions per second & with the word length of around 64 bits is known as (a) Super computer (b) Mini computer (c) Micro computer (d) Micro computer (e) None of the above Ans (a) 9. What digits are representative of all binary numbers? (a) 0 (b) 1 (c) Both (a ) and (b) (d) 3 (e) None of the above Ans (c) 10. To locate a data items for storage is (a) Field (b) Feed (c) Database (d) Fetch (e) None of the above Ans (d) SBI Marketing Aptitude SBI associate clerical cadre marketing and general awareness questions with answers,SBI Reasoning ability solved question papers SBI stands for State Bank of India, SBI Central Recruitment and promotion of Department recruit clerical cadre in Associate Banks of State Bank of India SBI latest bank profile, SBI, free solved sample placement papers of clerk, SBI Associate Banks Clerks Recruitment 2012 notification and job details, SBI Dates of Written Examination : 07.10.2012 and 14.10.2012,SBI previous years solved question papers, SBI 2010,2011,2012 Clerks and po officer job written test examination syllabus and selection procedure, SBI PO and Associate banks clerks examination questions papers, SBIAssociate Banks Clerks Recruitment 2012 notification and detailed explanation with answers and solutions, How to crack Sbi written test examination tips and trick,SBI, General Awareness, General English, Quantitative Aptitude, Reasoning Ability, Marketing Aptitude / Computer Knowledge whole solved questions paper, Latest general awareness SBI Associate Bank clerical cadre question papers and selection procedure and test pattern MARKETING APTITUDE QUESTIONS FOR BANK & PSC EXAMINATIONS 1. The Indian Economy will grow as per the estimates given by the Asian Development Bank (ADB) at which of the following rates in 2008 ? (A) 6.7% (B) 7.5% (C) 8.5% (D) 8% Ans (D) 2. Government has notified the hike in Foreign Direct Investment (FDI) limit for Public Sector Undertakings refineries. The new FDI limit is? (A) 26% (B) 49% (C) 51% (D) 75% Ans: (B) 4. 13th Finance Commission has been constituted under the Chairmanship of? (A) Y. S. P. Thorat (B) Vijai L. Kelkar (C) T. S. Vijayan (D) Laxmi Narayan Ans: (C) 5. SEBI is a? (A) Statutory body (B) Advisory body (C) Constitutional body (D) Non-statutory body Ans: (A) 6. 15th SAARC Summit is scheduled to be held in July-August 2008. The changed venue of the summit is? (A) New Delhi (B) Sri Lanka (C) Maldives (D) Bangladesh Ans: (B) 7. When was the SAARC established ? (A) On December 8, 1984 (B) On January 1, 1984 (C) On December 8, 1985 (D) On January 1, 1985 Ans: (C) 8. What is the present number of member countries of European Economic Community ? (A) 15 (B) 12 (C) 25 (D) 20 Ans: (B) 9. Which Committee recommended abolition of tax rebates under section 88 ? (A) Chelliah Committee (B) Kelkar Committee (C) Shome Committee (D) None of the above Ans: (B) 10. Which of the following countries is not the members of European Union ? (A) Switzerland (B) Malta (C) The Czeck Republic (D) Poland Ans: (A) 11. Pradhanmantri Bharat Jodo Pariyojna is related to? (A) Communication (B) Social Integration (C) Linking of Rivers (D) Development of Highways Ans: (D) 12. ?Sagarmala? is a name associated with? (A) A drilling vessel (B) A project of port development (C) Oil well in Indian Ocean (D) None of the above Ans: (B) 13. Tarapore Committee submitted its report on ?Full Convertibility on Rupee? in? (A) Current account (B) Capital account (C) Both in current as well as in capital account (D) Special Drawing Rights (SDR) Ans: (B) 14. The new President of FICCI (Federation of Indian Chambers of Commerce and Industry) is? (A) K. C. Raina (B) V. P. Shetty (C) Rajiv Chandrashekhar (D) Som Mittal Ans: (C) 15. As per Revised Estimates for 2006-07 released by CSO, the growth rate for Indian economy has been estimated to be? (A) 9.0% (B) 9.2% (C) 9.4% (D) 9.6% Ans: (D) 16. CENVAT is related to? (A) Sales Tax (B) Excise Duty (C) Custom Duty (D) Service Tax Ans: (B) 17. Aam Admi Bima Yojana provides social security to? (A) All labours in rural areas (B) All landless labours living below poverty line in rural areas (C) All labours in urban areas (D) All labours in both rural as well as urban areas Ans: (B) 18. According to 2001 census urbanrural population ratio is about? (A) 35 : 65 (B) 32 : 68 (C) 28 : 72 (D) 25 : 75 Ans: (C) 19. As per the final estimates of Ministry of Agriculture, the rice production for 2006-07 is estimated at? (A) 80.11 MT (B) 90.0 MT (C) 93.4 MT (D) 101.60 MT Ans: (C) 20. According to 2001 census the state having highest urban population is? (A) U.P. (B) Maharashtra (C) Tamil Nadu (D) Kerala Ans: (B) 21. The growth rate of construction sector for the year 2006-07 has been estimated at the level? (A) 9.2% (B) 8.5% (C) 11.0% (D) 12.0% Ans: (D) 22. International Finance Corporation (IFC) has decided to fund an Ultra Mega Power Project in Gujarat. This project belongs to? (A) Reliance Group (B) Tata Group (C) Birla Group (D) NTPC Ans: (B) 23. Which states ranks first and last respectively in Education Development Index (EDI) prepared to track the progress of states towards providing universal elementary education ? (A) Kerala and Orissa respectively (B) Kerala and Bihar repectively (C) Andhra Pradesh and Orissa respectively (D) Karnataka and Bihar respectively Ans: (B) 24. Targeted power capacity addition for 11th plan period is? (A) 41110 MW (B) 66463 MW (C) 68963 MW (D) 86283 MW Ans: (B) 25. During 2006-07 which sector has shown the highest growth rate ? (A) Service Sector (B) Manufacturing Sector (C) Transport and communication (D) Agriculture Sector Ans: (C) 26. The Ex-officio Secretary of NDC is? (A) Secretary of Finance Ministry (B) General Secretary of Lok Sabha (C) Secretary of Planning Commission (D) Vice Chairman of Planning Commission Ans: (C) 27. 15th SAARC Summit was held in July?August 2008. The venue of the summit was? (A) New Delhi (B) Sri Lanka (C) Maldives (D) Bangladesh Ans: (B) 28. In National Mineral Policy (1993) which mineral was allowed for having investment from private sector? (A) Coal (B) Iron (C) Gold (D) Platinum Ans: (A) 29. The share of road transport in total transport of the country is? (A) 20% (B) 40% (C) 60% (D) 80% Ans: (D) 30. 11th National Conference on e-governance was held on 7?8 February, 2008 at? (A) Gurgaon (Haryana) (B) Chandigarh (C) Mohali (Punjab) (D) Panchkula (Haryana) Ans: (D) 31. Which percentage of Central Taxes have been recommended by the 12th Finance Commission to be transferred to States ? (A) 28.5% (B) 29.5% (C) 30.5% (D) 31.5% Ans: (C) 32. Which state possesses the maximum percentage of SC population ? (A) U.P. (B) M.P. (C) Kerala (D) Punjab Ans: (D) 33. Government has decided to cover all districts of the country in National Rural Employment Guarantee Programme (NREGP)? (A) Upto January 1, 2008 (B) Upto March 31, 2008 (C) w.e.f April 1, 2008 (D) w.e.f. April 1, 2009 Ans: (B) 34. What is ?NIKKEI? ? (A) Share Price Index of Tokyo Share Market (B) Name of Japanese Central Bank (C) Japanese name of Country?s Planning Commission (D) Foreign Exchange Market of Japan Ans: (A) 35. Which statement is correct for Indian Planning Commission ? (A) It is not defined in Indian Constitution (B) Members and vice-chairman of it do not have fixed working duration (C) Members do not require any minimum education (D) All of these Ans: (D) 36. Which State of the following has adopted VAT (Value Added Tax) w.e.f. January 1, 2007 ? (A) Tamil Nadu (B) Uttar Pradesh (C) Gujarat (D) Goa Ans: (A) 37. Service Tax revenue collection for 2007?08 (Budget estimates) was proposed at Rs. 50,200 crore but the revised estimates remained at? (A) Rs. 50,603 crore (B) Rs. 52,603 crore (C) Rs. 64,460 crore (D) Rs. 64,640 crore Ans: (A) 38. NABARD was established on the recommendation of? (A) Public Account Committee (B) Shivaraman Committee (C) Narsimham Committee (D) None of these Ans: (B) 39. Sampurna Gramin Rojgar Yojana has been launched from? (A) 1st April, 2001 (B) 25th Sept., 2001 (C) 30th Sept., 2001 (D) No scheme of such title has yet launched Ans: (B) 40. Which company is providing mobile service with name ?Cell one? to the consumers ? (A) MTNL (B) BSNL (C) Reliance Infocom (D) Bharti Tele Ans: (B) 41. VAT is imposed? (A) Directly on consumer (B) On final stage of production (C) On first stage of production (D) On all stages between production and final sale Ans: (D) 42. The newly elevated person as Joint Chairman?cum?Managing Director of National Aviation Company of India Ltd. (NACIL) is? (A) S. Sridhar (B) Viswapati Trivedi (C) Rajiv Chandrashekhar (D) Venugopal Dhoot Ans: (B) 43. Kutir Jyoti scheme is associated with? (A) Promoting cottage industry in villages (B) Promoting employment among rural unemployed youth (C) Providing electricity to rural families living below the poverty line (D) All of these Ans: (C) 44. Novelis has been acquired and merged with? (A) Tata Steels (B) SAIL (C) HINDALCO (D) Jindal Steels Ans: (C) 45. OTCEI is? (A) Atomic Submarine of China (B) Economic policy of USA (C) An Indian Share Market (D) A defence research organisation Ans: (C) 46. The New Chairman of SEBI (Securities and Exchange Board of India) is? (A) M. Damodaran (B) C. B. Bhave (C) Venugopal Dhoot (D) Sunil K. Munjal Ans: (B) 47. Gross Budgetary Support (GBS) for 2008?09 as per document of 11th plan stands at Rs. 2,28,725 crore but in budget proposals for 2008?09 it has been raised to? (A) Rs. 2,23,386 crore (B) Rs. 2,43,386 crore (C) Rs. 2,63,386 crore (D) Rs. 28,456 crore Ans: (B) 48. The base of Consumer Price Index for Industrial Workers is being shifted from 1982 to? (A) 1995 (B) 1998 (C) 2000 (D) 2001 Ans: (D) 49. In Budget proposals for 2008?09, which of the following gives 24% contribution in revenue collection of union government ? (A) Income Tax (B) Excise (C) Corporation Tax (D) Non-tax Revenue Ans: (C) 50. The base year of present Consumer Price Index (CPI) for Urban Non-Manual Employees (CPI?UNME) is? (A) 1980-81 (B) 1981-81 (C) 1982-83 (D) 1984-85 Ans: (D) 51. CAPART is related with? (A) Assisting and evaluating rural welfare programmes (B) Computer hardware (C) Consultant service of export promotion (D) Controlling pollution in big industries Ans: (A) 52. Note issuing deptt. of RBI should always possess the minimum gold stock of worth? (A) Rs. 85 crore (B) Rs. 115 crore (C) Rs. 200 crore (D) None of these Ans: (B) 53. Which of the following does not grant any tax rebate ? (A) National Saving Certificate (B) Indira Vikas Patra (C) National Saving Scheme (D) Public Providend Fund Ans: (B) 54. Ad hoc Treasury Bill System of meeting budget deficit in India was abolished on? (A) 31 March, 1997 (B) 31 March, 1996 (C) 1 April, 1992 (D) 1 April, 1995 Ans: (A) 55. SEBI was established in? (A) 1993 (B) 1992 (C) 1988 (D) 1990 Ans: (C) 56. The working of SEBI includes? (A) To regulate the dealings of share market (B) To check the foul dealings in share market (C) To control the inside trading of shares (D) All of these Ans: (D) 57. The ?Ad hoc Treasury Bill System? of meeting budget deficit in India was replaced by ?Ways and Means Advances System? which has come into force on? (A) March 31, 1997 (B) April 1, 1996 (C) April 1, 1997 (D) None of these Ans: (C) 58. Fiscal deficit as a percentage of GDP was 4?0% in 2004?05 which sliped down in 2008?09 (Budget Estimates) to? (A) 3.2% (B) 2.8% (C) 2.5% (D) 2.1% Ans: (C) 59. According to the Economic Survey 2007?08 during the period April 3, 2007 and February 6, 2008 Indian rupee appreciated against US dollar by? (A) 5.6% (B) 6.8% (C) 8.9% (D) 9.3% Ans: (C) 60. Which statement of the following is true for IMF ? (A) It is not an agency of UNO (B) It can grant loan to any country of the world (C) It can grant loan to state Govt. of a country (D) It grants loan only to member nations Ans: (D) 61. For 2006-07, agriculture growth rate has been estimated at? (A) 3.8% (B) 5.9% (C) 0.0% (D) 6.3% Ans: (A) 62. Which of the following is public sector organisation ? (1) FCI Food Corporation of India (2) FCI Fertilizer Corporation of India (3) Cotton Corporation of India (4) Jute Corporation of India (A) Only 1 and 2 (B) Only 2, 3 (C) Only 3, 4 (D) All of these Ans: (D) 63. Which day has been declared as ?Balika Diwas? (Girl Day) by the Ministry of Woman and Children Development ? (A) April 5, every year (B) July 9, every year (C) October 9, every year (D) December 9, every year Ans: (D) 64. For RIDF-XIII, allocation in Union Budget 2006-07 was? (A) Rs. 4500 crore (B) Rs. 5500 crore (C) Rs. 10000 crore (D) Rs. 12000 crore Ans: (D) 65. The birth rate and death rate for the year 2006 have been estimated as? (A) 26.2 and 8.1 per thousand respectively (B) 24.8 and 8.0 per thousand respectively (C) 23.8 and 7.6 per thousand respectively (D) 23.5 and 7.5 per thousand respectively Ans: (D) 66. Central Issue price of foodgrains under TPDS includes price for BPL and APL (below poverty line and above poverty line). What is the difference between the two ? (A) Price for APL is double of BPL price (B) Price for BPL is one-third of APL price (C) Difference of Rs. 500 per Qt. (D) Difference of Rs. 250 per Qt. Ans: (A) 67. What growth target government has estimated for the domestic crude production for the 11th Plan (2007?12) ? (A) 42% (B) 33% (C) 29% (D) 26% Ans: (D) 68. Rural women can avail the benefit of Mahila Samriddhi Yojana if they open their account in? (A) Rural Post Offices (B) Commercial Banks (C) Rural Development Bank (D) Any of the above Ans: (A) 69. Agriculture sector registered 6% growth in 2005-06 and it is estimated for year 2007-08 at? (A) Again 6% (B) 4.3% (C) 3.3% (D) 2.6% Ans: (D) 70. As per the second advanced estimates of Agricultural Production for the year 2007-08 released by the Ministry of Agriculture, Kharif production has been estimated at? (A) 110.52 Million Tonnes (B) 112.24 Million Tonnes (C) 114.42 Million Tonnes (D) 115.9 Million Tonnes Ans: (D) 71. Mistry Committee in its final report recommends full capital account convertibility by? (A) 2008 end (B) 2009 end (C) 2010 end (D) 2011 end Ans: (A) 72. Which committee recommended tax on Agriculture Holdings ? (A) Bhootlingam Committee (B) Wanchoo Committee (C) Raj Committee (D) None of these Ans: (d) 73. The Present Service Tax rate as proposed in Union Budget 2008-09 is? (A) 8% (B) 9% (C) 10% (D) 12% Ans: (D) 74. The cause of deflation is? (A) Lack of goods and services as compared to money supply (B) Lack of imports as compared to exports (C) Lack of money supply as compared to supply of goods and services (D) None of these Ans: (C) 75. Which of the following is a better measure of economic development ? (A) Employment (B) Size of exports (C) Rural consumption (D) National Income Ans: (D) 76. Which bank in India performs duties of Central Bank ? (A) Central Bank of India (B) State Bank of India (C) Reserve Bank of India (D) Above (A) and (B) Ans: (C) 77. Out of one Rupee expenditure, how much paise have been allotted for subsidy in 2008-09 budget proposals? (A) 12 Paise (B) 10 Paise (C) 8 Paise (D) 7 Paise Ans: (C) 78. ?India Brand Equity Fund? was established in? (A) 1996 (B) 1997 (C) 1995 (D) 1992 Ans: (A) 79. Mixed Economy means? (A) Co-existence of small and large industries (B) Promoting both Agriculture and Industries in the economy (C) Co-existence of public and private sectors (D) Co-existence of rich and poor Ans: (C) 80. Ministry of HRD has announced to hike FDI limit in Education to? (A) 65% (B) 70% (C) 75% (D) 100% Ans: (D) 81. ?Pure Banking, Nothing Else? is a slogan raised by? (A) ICICI Bank (B) HDFC Bank (C) SBI (D) UTI Bank Ans: (C) 82. During 2006-07, External debt to GDP ratio in India stood at? (A) 16.4% (B) 17.4% (C) 17.8% (D) 15.8% Ans: (A) 83. Indian State having the lowest Infant Mortality Rate is? (A) Maharashtra (B) Goa (C) Gujarat (D) Kerala Ans: (D) 84. ?Smart Money? is a term used for? (A) Internet Banking (B) Credit Card (C) Cash with Bank (D) Cash with Public Ans: (B) 85. Which of the following has the maximum share in GSM Mobile Phone Service Market ? (A) Vodafone (earlier Hutch) (B) Airtel (C) BSNL (D) Reliance Ans: (B) 86. The main objective of TRYSEM was? (A) To train rural youth for self employment (B) To train urban youth for self employment (C) Both of the above (D) None of these Ans: (A) 87. The establishment of IORARC (Indian Ocean Rim Association for Regional Co-operation) was officially declared on? (A) March 5, 1996 (B) March 5, 1997 (C) April 1, 1997 (D) August 15, 1947 Ans: (B) 88. Inside Trading is related to? (A) Share Market (B) Horse racing (C) Taxation (D) Public Expenditure Ans: (A) 89. The largest source of National Income in India is? (A) Service Sector (B) Agriculture (C) Industrial Sector (D) Trade Sector Ans: (A) 90. ?Public Sector? means? (A) Government ownership on commerce and trade (B) Capitalist ownership on commerce and trade (C) Private ownership on trade (D) None of these Ans: (A) 91. NABARD is? (A) A bank (B) A board (C) A block (D) A department Ans: (A) 92. Indian Green Revolution is the most successful in? (A) Wheat and Potato (B) Jwar and Oil Seeds (C) Wheat and Rice (D) Tea and Coffee Ans: (C) 93. The period of 10th plan in India was? (A) 2000-2005 (B) 2001-2006 (C) 2002-2007 (D) 2003-2008 Ans: (C) 94. Economic Planning is in? (A) Union list (B) State list (C) Concurrent list (D) Not any specified list Ans: (C) 95. Presently (from April 29, 2003) bank rate in India is? (A) 7.0% (B) 6.75% (C) 6.25% (D) 6.0% Ans: (D) 96. Gross domestic savings as a proportion of GDP has been improving. What is the average percentage of gross domestic savings during the 10th plan ? (A) 26.4% (B) 31.4% (C) 34.8% (D) 35.2% Ans: (B) 97. 12th Finance Commission has recommended to merge and determine one single interest rate on various outstanding central loans to states having different interest rates. What is this recommended interest rate ? (A) 7.0% (B) 7.5% (C) 8.0% (D) 8.5% Ans: (B) 98. MRTP is related to? (A) Monopoly and trade restrictions (B) Inflation control (C) Transport control (D) Foreign Exchange Regulations Ans: (A) 99. Interest Rate Policy is a part of? (A) Fiscal Policy (B) Industrial Policy (C) Monetary Policy (D) None of these Ans: (C) 100. The basis of determining dearness allowance to employees in India is? (A) National Income (B) Consumer Price Index (C) Standard of Living (D) Per Capita Income Ans: (B) Computer General Awareness Sample Questions for SBI PO Recruitment Exam(General Awareness) 1. A keyboard is this kind of device? (A) black (B) input (C) output (D) word Processing (E) None of these Ans : (B) 2. IT stands for? (A) information Technology (B) integrated Technology (C) intelligent Technology (D) interesting Technology (E) None of these Ans : (A) 3. Which of the following refers to the fastest, biggest and most expensive computers ? (A) Personal Computers (B) Supercomputers (C) Laptops (D) Notebooks (E) None of these Ans : (B) 4. A collection of related information sorted and dealt with as a unit is a? (A) disk (B) data (C) file (D) floppy (E) None of these Ans : (C) 5. The process of a computer receiving information from a server on the Internet is known as? (A) pulling (B) pushing (C) downloading (D) transferring (E) None of these Ans : (C) 6. Which part of the computer helps to store information ? (A) Disk drive (B) Keyboard (C) Monitor (D) Printer (E) None of these Ans : (A) 7. ??????is the process of carrying out commands. (A) Fetching (B) Storing (C) Executing (D) Decoding (E) None of these Ans : (C) 8. The role of a???generally is to determine a buyer?s needs and match it to the correct hardware and software. (A) computer Scientist (B) computer sales representative (C) computer consultant (D) corporate trainer (E) None of these Ans : (B) 9. Which of the following groups consist of only output devices ? (A) Scanner, Printer, Monitor (B) Keyboard, Printer, Monitor (C) Mouse, Printer, Monitor (D) Plotter, Printer, Monitor (E) None of these Ans : (D) 10. The rectangular area of the screen that displays a program, data, and/or information is a? (A) title bar (B) button (C) dialog box (D) window (E) None of these Ans : (D) 11. A(n)???contains commands that can be selected. (A) pointer (B) menu (C) icon (D) button (E) None of these Ans : (B) 12. An error is also known as? (A) bug (B) debug (C) cursor (D) icon (E) None of these Ans : (A) 13. Arithmetic Operations? (A) involve matching one data item to another to determine if the first item is greater than, equal to, or less than the other item (B) sort data items according to standard, predefined criteria in ascending order or descending order (C) use conditions with operators such as AND, OR and NOT (D) include addition, subtraction, multiplication and division (E) None of these Ans : (D) 14. Sending an e-mail is similar to? (A) picturing an event (B) narrating a story (C) writing a letter (D) creating a drawing (E) None of these Ans : (C) 15. Promotion in Marketing means? (A) passing an examination (B) elevation from one grade to another (C) selling the products through various means (D) selling the product in specific areas (E) None of these Ans : (C) 16. Microsoft Word is an example of? (A) an operating system (B) a processing device (C) application software (D) an input device (E) None of these Ans : (B) 17. Sources of Sales Leads are? (A) Data Mining (B) Market Research (C) Media Outlets (D) Promotional Programs (E) All of these Ans : (E) 18. Any data or instruction entered into the memory of a computer is considered as? (A) storage (B) output (C) input (D) information (E) None of these Ans : (A) 19. Which part of the computer displays the work done ? (A) RAM (B) printer (C) monitor (D) ROM (E) None of these Ans : (C) 20. One of the methods for Market Monitoring is? (A) to watch TV serials (B) to discuss with other sales persons (C) to monitor media outlets (D) All of these (E) None of these Ans : (C) 21. Market Expansion means? (A) hiring more staff (B) firing more staff (C) buying more products (D) buying more companies (E) None of these Ans : (E) 22. A series of instructions that tells a computer what to do and how to do it is called a? (A) program (B) command (C) user response (D) processor (E) None of these Ans : (A) 23. Effective marketing helps in? (A) developing new products (B) creating a competitive environment (C) building demand for products (D) All of these (E) None of these Ans : (D) 24. Which is the part of a computer that one can touch and feel ? (A) Hardware (B) Software (C) Programs (D) Output (E) None of these Ans : (A) 25. A Call in Marketing means? (A) to phone the customers (B) to visit the customers (C) to visit the marketing site (D) to call on prospective customers (E) None of these Ans : (D) 26. Delivery Channel means? (A) maternity wards (B) handing over the products to the buyers (C) places where products are made available to the buyers (D) All of these (E) None of these Ans : (C) 27. Processing involves? (A) inputting data into a computer system (B) transforming input into output (C) displaying output in a useful manner (D) providing relevant answers (E) None of these Ans : (B) 28. One of the following is a target group for the marketing of Internet Banking. (A) All the customers (B) All the educated customers (C) All the computer educated customers (D) Only creditors (E) All of these Ans : (C) 29. Innovation mean? (A) Product Designing (B) New ideas (C) Impulse (D) Both (A) and (B) (E) None of these Ans : (D) 30. One of the following is a target group for the marketing of educational loan. (A) All the customers (B) Students (C) Only poor students (D) Students having promising educational track record (E) All of these Ans : (B) 31. Service after sale is not the function of? (A) Marketing staff (B) Seller (C) Director of the company (D) Employees of the company (E) All of the above are wrong Ans : (A) 32. If done through???the rural marketing would be more effective. (A) fairs (B) village fairs (C) door to door campaign (D) All of these (E) None of these Ans : (B) 33. Market Survey means? (A) Market Research (B) Market Planning (C) Marketing Strategies (D) Market Monitering (E) All of these Ans : (A) 34. ???can be done through digital Banking ? (A) Mobile phone (B) Internet (C) Telephone (D) All of these (E) None of these Ans : (A) 35. A good seller should have the following quality/qualities ? (A) Devotion to the work (B) Submissive (C) Sympathy (D) All of these (E) None of these Ans : (D) 36. The rural marketing is not required because? (A) rural people do not understand marketing (B) its not practical from the cost point of view (C) it is sheer wastage of time (D) All are wrong (E) All are correct Ans : (D) 37. Planned-cost service means? (A) Costly products (B) Extra profit on the same cost (C) Extra work by seller (D) All of these (E) None of these Ans : (B) 38. Internet marketing means? (A) Self-marketing (B) Core Groups monitering (C) Employees marketing (D) All of these (E) None of these Ans : (E) 39. The aim of successful marketing is? (A) to increase the sale (B) to increase the profit (C) to increase the output of sellers (D) All of these (E) None of these Ans : (D) 40. The networking becomes??through networking. (A) very difficult (B) dull (C) easy (D) has no role in marketing (E) None of these Ans : (C) 41. Which of the following States will soon have its first Civil Airports? (A) Jammu & Kashmir (B) Maharashtra (C) Assam (D) Meghalaya (E) Sikkim Ans : (E) 42. Who amongst the following made the 41st Test Century of his career recently? (A) Saurav Ganguly (B) V. V. S. Laxman (C) Rahul Dravid (D) Sachin Tendulkar (E) None of these Ans : (D) 43. Which of the following was/were the objective(s) of the Mission Chandrayaan 1 ? (a) Preparing dimensional atlas of the lunar surface. (b) Chemical Mapping of the entire lunar surface. (c) Locating minerals in the soil of the moon. (A) Only (a) (B) Only (b) (C) Only (c) (D) All (a), (b) & (c) (E) None of these Ans : (D) 44. Which of the following States introduced a high tech foodgrain rationing system to ensure timely supply of the foodgrains to people living below poverty line ? (A) Maharashtra (B) Tamil Nadu (C) Delhi (D) West Bengal (E) Karnataka Ans : (E) 45. Which of the following is NOT a part of India?s Money Market ? (A) Bill Markets (B) Call Money Market (C) Banks (D) Mutual Funds (E) Indian Gold Council Ans : (E) 46. Which of the following is the objective of the project ?Bhoomi Keralam? launched by the State Govt. ? (A) To conduct a survey of the land (B) To bring those people back who have left the State and settled in other States (C) To provide financial security to farmers who are in distress (D) To identify those tribals who need jobs or financial assistance (E) None of these Ans : (A) 47. Which of the following countries adopted its new constitution recently ? (A) Russia (B) Equadore (C) South Korea (D) North Korea (E) None of these Ans : (B) 48. Which of the following awards was given to Pt. Bhimsen Joshi recently (2008) ? (A) Padma Bhushan (B) Sangit Natak Academy Award (C) Maharashtra Bhushan (D) Karnataka Ratna (E) Bharat Ratna Ans : (E) 49. Which of the following States got its first Rail link recently ? (A) Haryana (B) Arunachal Pradesh (C) Sikkim (D) Manipur (E) Jammu & Kashmir Ans : (E) 50. Barack Hussain Obama belongs to which of the following political parties ? (A) Republican (B) Democratic (C) Labour (D) American National Congress (E) None of these Ans : (B) 51. Which of the following is/are NOT the features of India?s Foreign Trade policy (2004 to 2009)? (a) To double India?s percentage share of global trade from present 0?7 per cent to 1?5 per cent by 2009. (b) Simplifying the procedures and bringing down the cost. (c) Make SAARC countries India?s most preferred foreign trade partners by 2009. (A) Only (a) (B) Only (b) (C) Only (c) (D) All (a), (b) & (c) (E) Only (a) & (c) Ans : (C) 52. Stephen Harper whose name was in news recently is the ???. (A) President of Canada (B) Prime Minister of Canada (C) President of Italy (D) Prime Minister of Switzerland (E) Foreign Minister of U.S.A. Ans : (B) 53. Which of the following is NOT a fertilizer product ? (A) Urea (B) Murate of Potash (C) Di Amonium Phosphate (D) Calcium Carbonate (E) All are fertilizers Ans : (E) 54. A Rs. 35,000 crore JSW Steel Plant project was inaugurated in West Bengal recently. The project is setup / developed in ???. (A) Singur (B) Nandigram (C) Salboni (D) Malda (E) Joynagar Ans : (C) 55. Which of the following State Govts. Has announced that it will provide a special package of incentives to Employment Intensive Industries? (A) Punjab (B) Karnataka (C) Tamil Nadu (D) Uttar Pradesh (E) Bihar Ans : (C) 56. Which of the following nations has decided to increase its co-operation in the field of energy with India? (A) Russia (B) China (C) Bangladesh (D) Nepal (E) Japan Ans : (A) 57. Which of the following services is NOT provided by the post offices in India ? (A) Savings Bank Scheme (B) Retailing of Mutual Funds (C) Sale of stamp Papers (Judicial) (D) Issuance of Demand Drafts (E) Life Insurance cover Ans : (D) 58. Mohammed Anni Nasheed is the newly elected President of ???. (A) Fiji (B) Maldives (C) Iraq (D) Iran (E) None of these Ans : (B) 59. Who amongst the following is the winner of the Nobel Prize for Literature 2008 ? (A) Jean Marie Gustave Le Clezio (B) Martti Ahtisaari (C) Yoichiro Nambu (D) Francoise Barre Sinoussi (E) None of these Ans : (A) 60. A national level commission of India has asked about twenty States to set up which of the following commissions in their States as early as possible ? (A) Farmers Commission (B) Women Commission (C) Child Welfare Commission (D) Law Commission (E) None of these Ans : (D) 61. Which of the following is/are the measure(s) taken by the Reserve Bank of India (RBI) to ease the liquidity crunch in the country ? (a) Cut in Cash Reserve Ratio and Statutory Liquidity Ratio. (b) Increase the flow of foreign direct investment. (c) Supply of additional currency notes in the market. (A) Only (a) (B) Only (b) (C) Only (c) (D) All (a), (b) & (c) (E) None of these Ans : (A) 62. The third India Brazil and South Africa (IBSA) Summit took place in which of the following dies recently ? (A) Brazilia (B) New Delhi (C) Suncity (D) Salvador (E) Durban Ans : (B) 63. Which of the following States has dcided to set up Arsenic Removal Plants in all the districts of the State to enable itself to provide arsenic free drinking water to all the people by 2010-11 ? (A) Karnataka (B) Kerala (C) West Bengal (D) Orissa (E) Maharashtra Ans : (C) 64. Who amongst the following is the author of the book ?The Exile? ? (A) B. G. Verghese (B) Philip Roth (C) Aravind Adiga (D) Navtej Saran (E) None of these Ans : (D) 65. Which of the following programmes is being implemented in all the districts of the country ? (A) Sarva Shiksha Abhiyan (B) Navodaya Vidhyalay (C) Ultra Mega Power Project (D) Rajiv Gandhi Grameen Vidyuthikara Yojana (E) None of these Ans : (A) 66. The Govt. of India decided to provide a bonus price of Rs. 50 per quintal on which of the following products over and above its minimum support price of Rs. 850 per quintal ? (A) Wheat (B) Paddy (C) Sugarcane (D) Cotton (E) None of these Ans : (B) 67. Who amongst the following is the author of the book ?The Namesake? ? (A) Vikram Seth (B) V. S. Naipaul (C) Arun Bhagat (D) Anita Desai (E) Jhumpa Lahiri Ans : (E) 68. Besides USA India has signed Nuclear Agreement with which of the following countries and is named as ?Co-operation Agreement for Peaceful uses of Nuclear Energy? ? (A) Italy (B) Germany (C) France (D) Canada (E) Australia Ans : (C) 69. Tzipi Livini whose name was in news recently is from which of the following countries ? (A) Singapore (B) South Korea (C) Austria (D) Israel (E) None of these Ans : (D) 70. World Ozone day is observed on ???. (A) 16th September (B) 16th October (C) 16th November (D) 26th September (E) 26th October Ans : (A) 71. Which of the following is NOT a Govt. Sponsored organization ? (A) Small Industries Development Bank of India (B) NABARD (C) National Housing Bank (D) ICICI Bank (E) All are Govt. sponsored Ans : (D) 72. ?Merdeka Cup? is associated with the game of ???. (A) Badminton (B) Football (C) Hockey (D) Golf (E) Tennis Ans : (B) 73. India?s Space Rocket Launching Centre is in ???. (A) Portblair (B) Hassan (C) Tirupati (D) Kochi (E) Sri Harikota Ans : (E) 74. The National Games scheduled in 2011 will be organized in ???. (A) Bengaluru (B) Goa (C) Kolkata (D) Kochi (E) Jaipur Ans : (B) 75. Which of the followiing diseases is not covered under Integrated Disease Surveillance project ? (A) Cholera (B) Tuberculosis (C) AIDS (D) Polio (E) Cancer Ans : (E) 76. Which of the following is TRUE about ?Antyodaya Anna Yojana? ? (a) Scheme is for Below Poverty Line families. (b) People are provided cooked meals under the scheme. (c) All beneficiaries of National Rural Employment Guarantee Act are covered in this scheme. (A) Only (a) (B) Only (b) (C) Only (c) (D) All (a), (b) & (c) (E) None of these Ans : (E) 77. Jelena Jankovic of Serbia won which of the following titles of tennis after defeating Svetlana Kuznetsova ? (A) China Open (B) Japan Open (C) New Zealand Open (D) Australian Open (E) None of these Ans : (A) 78. Who amongst the following is the Minister of Home Affairs in Union Cabinet of India at present ? (A) Shivraj Patil (B) Lalu Prasad Yadav (C) P. Chidambaram (D) Sharad Pawar (E) None of these Ans : (C) 79. Banks are required to pay how much percentage of their net bank credit to Priority Sector is advance ? (A) 5% (B) 7% (C) 10% (D) 15% (E) None of these Ans : (C) 80. Dronacharya Award is given for excellence in ???. (A) Literacy Work (B) Social Service (C) Coaching in Sports (D) Journalism (E) None of these Ans : (C) 81. This ......... tier processes HTTP protocol, scripting tasks, performs calculations, and provides access to data. (a)Client (b)Applications/Web server (c)Enterprise server (d)DBA (e)None of these Ans. Client 82. A polymorphic virus: (a)modifies its program code each time it attaches itself to another program or file (b)is a malicious-logic program that copies itself repeatedly in memory or on a disk drive until no memory or disk space remains (c)a malicious logic program that hides within or looks like a legitimate program (d)infects a program file, but still reports the size and creation date of the original, uninfected program (e)None of these Ans. modifies its program code each time it attaches itself to another program or file 83. The Secure Electronic Transaction (SET) specification : (a)is a notice, issued and verified by a certificate authority, that guarantees a user or Website is legitimate (b)provides private key encryption of all data that passes between a client and a server (c)allows users to choose an encryption scheme for the data that passes between a client and a server (d)uses a public key encryption to secure credit-card transaction systems (e)None of these Ans. Uses a public-key encryption to secure credit-card transaction systems 84. ......... allows wireless mobile devices to access the Internet and its services such as the Web and e-mail: (a)TCP/IP (b)Ethernet (cWAP (d)Token ring (e)None of these Ans. WAP 85. 'DOS' floppy disk does not have: (a)A Boot Record (b)A File Allocation Table (c)A Root Directory (d)Virtual Memory (e)BIOS Ans. Virtual Memory 86. A passive threat to computer security is: (a)Malicious Intent (b)Sabotage (c)Accident Errors (d)Espionage Agents (e)None of these Ans. Malicious Intent 87. All of the following are basic principles of net works, except: (a)each computer must have a network card (b)there must be communications media connecting the network hardware devices (c)there must be at least one connecting device (d)each computer must have software that supports the movement of information (e)None of these Ans. there must be communications media connecting the network hardware devices 88. 'MICR' technology used for clearance of cheques by banks refers to: (a)Magnetic Ink Character Recognition (b)Magnetic Intelligence Character Recognition (c)Magnetic Information Cable Recognition (d)Magnetic Insurance Cases Recognition (e)None of these Ans. Magnetic Ink Character Recognition 89. All the information collected during database development is stored in a: (a)repository (b)data warehouse (c)RAD (d)CASE (e)None of these Ans. data warehouse 90. The ......... component is used by the data analyst to create the queries that access the database. (a)data extraction (b)end-user query tool (c)end-user presentation tool (d)data store (e)None of these Ans. end-user query tool 91. A ......... allows network users to share a single copy of software, which resides on the network server. (a)single user license agreement (b)network site license (c)end user license agreement (d)business software license (e)None of these Ans. network site license 92. Microwave transmission is used in environments where installing physical transmission media is: (a)difficult or impossible and where line of sight transmission is unavailable (b)easy or promising and where line of sight transmission is unavailable (c)difficult or impossible and where line of sight transmission is available (d)easy or promising and where line of sight transmission is available (e)None of these Ans. easy or promising and where line of sight transmission is available 93. Which is NOT a good Web security strategy? (a)Restrict access to the Web server; keep a minimum number of ports open (b)Limit the users who can load software, edit or add files. (c)Add demo programs, so users can test system without accessing production data. (d)Remove unnecessary compilers and interpreters. (e)None of these Ans. Remove unnecessary compilers and interpreters. 94. The word FIP stands for: (a)File Translate Protocol (b)File Transit Protocol (c)File Typing Protocol (d)File Transfer Protocol (e)None of these Ans. File Translate Protocol 95. Voice mail: (a)is the exchange of text messages and computer files transmitted via a communications network such as a local area network or the Internet (b)permits users to converse in real time with each other via the computer while connected to the Internet (c)functions much like an answering machine, allowing callers to leave a voice message for the called party (d)involves using video and computer technology to conduct a meeting between participants at geographically separate locations (e)None of these Ans. functions much like an answering machine, allowing callers to leave a voice message for the called party 96. A communication processor that connects dissimilar networks by providing the translation from one set of protocol to another is: (a)Bridge (b)Gateway (c)Router (d)Modem (e) All of these Ans. Gateway 97. How is power supplied to a low power USB device? (a)Through a power cable (b)From an external power supply (c)Directly from the computer's power supply (d)Through the USB cable (e)None of these Ans. Through the USB cable 98. RSA is: (a)Symmetric Cryptosystem (b)Asymmetric Cryptosystem (c)Block Cypher (d)Digital Signature (e)None of these Ans. Digital Signature 99. When you are selecting a mouse for a particular computer system, what is the most important consideration? (a)The type of drivers that come with the mouse (b)The length of the mouse cord (c)The type of connector the mouse is equipped with (d)The number of buttons the mouse has (e)None of these Ans. The type of connector the mouse is equipped with 100. The ......... data mining technique derives rules from real world case examples. (a)Rule discover (b)Signal processing (c)Neural nets (d)Case based reasoning (e)None of these Ans. Neural nets 101. ......... are used to identify a user who returns to a Website. (a)Cookies (b)Plugins (c)Scripts (d)ASPs (e)None of these Ans. Cookies 102. A DVD-RAM is similar to a ......... except it has storage capacities up to 5.2 GB. (a)CD-R (b)floppy disk (c)CD-RW (d)hard disk (e)None of these Ans. CD-RW 103. Which of the following is a general-purpose programming language, designed by Sun Microsystems, and well suited for use on the Web? (a)VB Script (b)Java Script (c)CSS (d)Java (e)None of these Ans. Java Script 104. Which one of the following objects is passed to a Java Bean when one of its properties is set via a JSP action? (a)Servlet Request (b)Http Servlet Request (c)Servlet Response (d)http Servlet Response (e)None of these Ans. Servlet Response 105. The altering of data so that it is not usable unless the changes are undone is: (a)Biometrics (b)Compression (c)Encryption (d)Ergonomics (e)None of these Ans. Encryption 106. Which of the following is a part of the Central Processing Unit? a. Printer b. Key board c. Mouse d. Arithmetic & Logic unit e. None 107. CAD stands for a. Computer aided design b. Computer algorithm for design c. Computer application in design d. All of the above e. None 108. Which of the following printer cannot print graphics? a. Ink-jet b. Daisy Wheel c. Laser d. Dot-matrix e. None 109. A program written in machine language is called? a. Assembler b. Object c. Computer d. Machine e. None 110 The father of Modern Computer is a. Charles Babbage b. Von-nuumann c. Danies Ritchel d. Blaise Pascal e. None 111. The Word FTP stands for a. File Translate Protocol b. File Transit Protocol c. File Transfer protocol d. file typing protocol e. None 112. The lowest form of Computer language is called a. BASIC b. FORTRAN c. Machine Language d. COBOL e. None 113. Best Quality graphics is produced by a. Dot Matix b. Laser Printer c. Inkjet Printer d. Plotter e. None 114. Memory which forgets every thing when you switch off the power is known as a. Corrupted b. Volatile c. Non-Volatile d. Non-Corrupted e. None 115. The linking of computers with a communication system is called a. Networking b. Pairing c. Interlocking d. Assembling e. Sharing 116. The 16 bit Microprocessor means that it has a. 16 address lines b. 16 Buses c. 16 Data lines d. 16 routes e. None 117. Data going into the computer is called a. Output b. algorithm c. Input d. Calculations e. flow chart 118. Which of the following refers to a small, single-site network? a. LAN b. DSL c. RAM d. USB e. CPU 119 Microsoft Office is a. Shareware b.Public domain software c. Open-sourse software d. A vertical market application e. An application suite 120. How many options does a BINARY choice offer a. None b. One c. Two d. it depends on the amount of memory on the computer e. It depends on the speed of the computer's processor 121. A collection of program that controls how your computer system runs and processes information is called a. Operating System b. Computer c. Office d. Compiler e. Interpreter 122. Computer connected to a LAN (Local Area Network) can a. run faster b. go on line c. share information and /or share peripheral equipment d. E-mail e. None 123 Information travels between components on the mother board through a. Flash memory b. CMOS c. Bays d. Buses e. Peripherals 124How are data organized in a spreadsheet? a. Lines & spaces b. Layers & Planes c. Height & Width d. Rows & Columns e. None 125. The blinking symbol on the computer screen is called the a. mouse b. logo c. hand d. palm e. cursor 126 A fault in a computer program which prevents it from working correctly is known as a. Boot b. Bug c. Biff d. Strap e. None 127 A self replicating program, similar to a virus which was taken from a 1970s science fiction novel by John Bruner entitled the Shockwave Rider is _________ a. Bug b. Vice c. Lice d. Worm e. None 128. A _______ is a bi-stable electronic circuit that has two stable states. a. Multivibrator b. Flip-flop c. Logic gates d. laten e. None 129. Unwanted repetitious messages, such as unsolicited bulk e-mail is known as a. Spam b. Trash c. Calibri d. Courier e. None 130 DOS stands for a. Disk Operating System b. Disk operating session c. Digital Operating System d. Digital Open system e. None 131. Who is the chief of Miocrosoft a. Babbage b. Bill Gates c. Bill Clinton d. Bush e. None 132 Which of the following are input devices. a. Keyboard b. Mouse c. Card reader d. Scanner e. All of these 133. Examples of output devices are a. Screen b. Printer c. Speaker d. All of these e. None 134 Which of the following is also known as brain of computer a. Control unit b. Central Processing unit c. Arithmatic and language unit d. Monitor e. None 135IBM stands for a. Internal Business Management b. International Business Management c. International Business Machines d. Internal Business Machines e. None 136___________ translates and executes program at run time line by line a. Compiler b. Interpreter c. Linker d. Loader e. None 137___________ is an OOP principle a. Structured programming b. Procedural programming c. Inheritance d. Linking e. None 138. COBOL is widely used in _________ applications a. Commercial b. Scientific c. Space d. Mathematical e. None 139. RAM stands for a. Random origin money b. Random only memory c. Read only memory d. Random access memory e. None 140. 1 Byte = ? a. 8 bits b. 4 bits c. 2 bits d. 9 bits e. None 141. SMPS stands for a. Switched mode power supply b. Start mode power supply c. Store mode power supply d. Single mode power supply e. None 142. The device used to carry digital data on analog lines is called as a. Modem b. Multiplexer c. Modulator d. Demodulator e. None 143. VDU is also called a. Screen b. Monitor c. Both 1 & 2 d. printer e. None 144. BIOS stands for a. Basic Input Output system b. Binary Input output system c. Basic Input Off system d. all the above e. None 145. Father of 'C' programming language a. Dennis Ritchie b. Prof Jhon Kemeny c. Thomas Kurtz d. Bill Gates e. None SBI stands for State Bank of India, SBI Central Recruitment and promotion of Department recruit PO in Associate Banks of State Bank of India, written test solved question papers SBI PO computer awareness, general knowledge, marketing aptitude, quantitative aptitude, reasoning questions with answers,SBI computer knowledge question banks ,SBI previous years solved question papers, SBI latest sample free solved placement papers with answers 1. Compiler is the (a) name given to the computer operator (b) part of the digital machine to store the information (c) translator of source program to object (d) part of arithmetic logic unit (e) operator of Boolean Algebra Ans (c) 2. Main memory is (a) Random Access Memory (b) Read only Memory (c) Serial Access Memory (d) None of these Ans (a) 3. Which of the following is the smallest and fastest computer imitating brain working? (a) Supercomputer (b) Quantum Computer (c) Param-10,000 (d) IBM chips Ans (b) 4. A compact disc (CD) is a data storage of the type (a) Magnetic (b) Optical (c) Electrical (d) Electromechanical Ans (a) 5. Which of the following is not as language for computer programming? (a) WINDOWS (b) PASCAL (c) BASIC (d) COBOL Ans (a) 6. What are the two basic types of memory that your computer uses? a) RAM (b) RW/RAM (c) ROM (d) ERAM Ans (a) 7. The term gigabyte refers to (a) 1024 bytes (b) 1024 kilobytes (c) 1024 megabytes (d) 1024 gigabytes Ans (c) 8. A computer with CPU speed around 100 million instructions per second & with the word length of around 64 bits is known as (a) Super computer (b) Mini computer (c) Micro computer (d) Micro computer Ans (a) 9. What digits are representative of all binary numbers? (a) 0 (b) 1 (c) Both (a ) and (b) (d) 3 Ans (c) 10. To locate a data items for storage is (a) Field (b) Feed (c) Database (d) Fetch Ans (d) 11. Off-line operation is the operation of devices without the control of (a) Memory (b) CPU (c) ALU (d) Control unit Ans (b) 12. A type of line printer that uses an embossed steel band to form the letters printed on the paper is (a) Golf ball printer (b) Dot-matrix printer (c) Laser printer (d) Band printer Ans (d) 13. A software used to convert source program instructions to object instruction is known as (a) Compiler (b) Assembler (c) Interpreter (d) Language processor 14. The 'IC' chip, used in computers, is made of (a) Chromium (b) Iron oxide (c) Silica (d) Silicon Ans (d) 15. Name the first general purpose electronic computer Ans : UNIVAC 16. The size of commonly used Floppy disk is (a) 4.5? (b) 3.5? (c) 3.25? (d) 5.5? Ans (b) 17. Which of the following statement is wrong (a) Windows XP is an operating system (b) Linux is owned and sold by Microsoft (c) Photoshop is a graphical design tool by Adobe (d) Linux is free and open source software Ans (b) 18. Operating system of a computer (a) Enables the programmer to draw a flow chart (b) Links a program with subroutine with references (c) Provides a layer, user friendly interface (d) None of the above Ans (c) 19. The term ?Operating System? means (a) A set of programs which control computer working (b) The way a user operates the computer system (c) Conversion of high level language to machine language (d) The way computer operator works Ans (a) 20. Wild card operators specifies (a) can be used when writing into multiple files (b) allows several files to be read simultaneously (c) Provide an easy way of groups of related files (d) Are only used when printing the contents of a file Ans (c) 21.Which one of the following is not a broadband communication medium (a) Microwave (b) Fibre optic cable (c) Twisted pair (d) Coaxial cable Ans (c) 22.Which of the following performs modulation and demodulation (a) Coaxial cable (b) Satellite (c) Modem (d) Optical fibre Ans (c) 23. Which one of the following is not an application software package (a) Redhat Linux (b) Microsoft Office (c) Adobe PageMaker (d) Open Office Ans (a) 24. Who is the father of computer Ans : Charles Babbage 25.The function of key F4 is Ans : To repeat the last action 26.The function of key F4 is Ans : To repeat the last action 27. The 0 and 1 in the binary numbering system are called binary digits or ....... (a) bytes (b) kilobytes (c) decimal bytes (d) bits Ans (D) 28. A monitor's ...... is the distance between the holes in the mask behind the screen. This helps determine how sharp the dots appear. (a) refresh rate (b) dot pitch (c) resolution (d) colour depth Ans (B) 29. A directly accessible appointment calendar is a feature of a ....... resident package. (a) CPU (b) memory (c) Buffer (d) ALU Ans (B) 30. Perforated paper used as input or output media is known as (a) Paper tape (b) Magnetic tape (c) Punched paper tape (d) Card punch Ans (A) 31. The invention of the slide rule is attributed to (a) Babbage (b) Oughtred (c) Pascal (d) Napier Ans (B) 32. Which of the following is not an advantage of magnetic disk storage. (a) The access time of magnetic disk is much less than that of magnetic tape (b) Disk storage is longer lasting than magnetic tape (c) Disk storage is less expensive than tape storage (d) All of the above Ans (e) 33. The memory sizes in mainframe computers and advanced technology micro computer are expressed as (a) Bytes (b) Kilo-bytes (c) Bits (d) Megabytes Ans (d) 34. A prefix for billion which is equal to .... is called as billi. (a) 100 (b) 10000 (c) 1000 (d) 10 Ans d 35. Each model of a computer has a unique (a) Assembly language (b) Machine language (c) High level language (d) All of the above Ans (b) 36. One kilobyte = ____ byte. (a) 1250 (b) 2088 (c) 1024 (d) 1000 Ans (c) 37. Which of the following circuit is used as a 'Memory device' in computers? (a) Rectifier (b) Flip-Flop (c) Comparator (d) Attenuator Ans (b) 38. To move the cursor to the end of the document press Ans : Ctrl + End 39. In Word Processing the red underline indicates Ans : Spelling mistakes 40. The shortcut key to print documents is Ans : Ctrl + P 41. First layer in the OSI reference model is (a) Data link (b) Network (c) Physical (d) Application Ans (c) 42. Viruses, Trojan horses and Worms are (a) able to harm computer system (b) unable to detect if affected on computer (c) user-friendly applications (d) harmless applications resident on computer Ans (a) 43. Program threats are (a) Trap doors (b) Trojan horse (c) Both (a) & (b) (d) All of the above Ans (b) 44. Failure of passwords security due to exposure can result from (a) Electronic monitoring (b) Visual (c) Both (a) & (b) (d) All of the above Ans (c) 45. Data security threats include (a) Hardware failure (b) Privacy invasion (c) Fraudulent manipulation of data (d) All of the above Ans (b) 46. The bar which displays information about the current page number is Ans : Status bar 47. Graphical pictures that represent an object like file, folder etc. are (a) Task bar (b) Windows (c) Desktop (d) Icons Ans (d) 48. Who invented the super-computer? (a) P.T Farnsworth (b) J.R Whinfield (c) J.H. Van Tassell (d) Charles Ginsberg Ans (c) 49. Accessing records from a file directly without searching from the beginning of the file is (a) Time sharing (b) Random (c) Direct access (d) Access time (e) None of the above Ans (c) 50. Time during which a job is processed by the computer is (a) Delay time (b) Real time (c) Execution time (d) Down time Ans (c 51. This ......... tier processes HTTP protocol, scripting tasks, performs calculations, and provides access to data. (a)Client (b)Applications/Web server (c)Enterprise server (d)DBA (e)None of these Ans. Client 52. A polymorphic virus: (a)modifies its program code each time it attaches itself to another program or file (b)is a malicious-logic program that copies itself repeatedly in memory or on a disk drive until no memory or disk space remains (c)a malicious logic program that hides within or looks like a legitimate program (d)infects a program file, but still reports the size and creation date of the original, uninfected program (e)None of these Ans. modifies its program code each time it attaches itself to another program or file 53. The Secure Electronic Transaction (SET) specification : (a)is a notice, issued and verified by a certificate authority, that guarantees a user or Website is legitimate (b)provides private key encryption of all data that passes between a client and a server (c)allows users to choose an encryption scheme for the data that passes between a client and a server (d)uses a public key encryption to secure credit-card transaction systems (e)None of these Ans. Uses a public-key encryption to secure credit-card transaction systems 54. ......... allows wireless mobile devices to access the Internet and its services such as the Web and e-mail: (a)TCP/IP (b)Ethernet (cWAP (d)Token ring (e)None of these Ans. WAP 55. 'DOS' floppy disk does not have: (a)A Boot Record (b)A File Allocation Table (c)A Root Directory (d)Virtual Memory (e)BIOS Ans. Virtual Memory 56. A passive threat to computer security is: (a)Malicious Intent (b)Sabotage (c)Accident Errors (d)Espionage Agents (e)None of these Ans. Malicious Intent 57. All of the following are basic principles of net works, except: (a)each computer must have a network card (b)there must be communications media connecting the network hardware devices (c)there must be at least one connecting device (d)each computer must have software that supports the movement of information (e)None of these Ans. there must be communications media connecting the network hardware devices 58. 'MICR' technology used for clearance of cheques by banks refers to: (a)Magnetic Ink Character Recognition (b)Magnetic Intelligence Character Recognition (c)Magnetic Information Cable Recognition (d)Magnetic Insurance Cases Recognition (e)None of these Ans. Magnetic Ink Character Recognition 59. All the information collected during database development is stored in a: (a)repository (b)data warehouse (c)RAD (d)CASE (e)None of these Ans. data warehouse 60. The ......... component is used by the data analyst to create the queries that access the database. (a)data extraction (b)end-user query tool (c)end-user presentation tool (d)data store (e)None of these Ans. end-user query tool 61. A ......... allows network users to share a single copy of software, which resides on the network server. (a)single user license agreement (b)network site license (c)end user license agreement (d)business software license (e)None of these Ans. network site license 62. Microwave transmission is used in environments where installing physical transmission media is: (a)difficult or impossible and where line of sight transmission is unavailable (b)easy or promising and where line of sight transmission is unavailable (c)difficult or impossible and where line of sight transmission is available (d)easy or promising and where line of sight transmission is available (e)None of these Ans. easy or promising and where line of sight transmission is available 63. Which is NOT a good Web security strategy? (a)Restrict access to the Web server; keep a minimum number of ports open (b)Limit the users who can load software, edit or add files. (c)Add demo programs, so users can test system without accessing production data. (d)Remove unnecessary compilers and interpreters. (e)None of these Ans. Remove unnecessary compilers and interpreters. 64. The word FIP stands for: (a)File Translate Protocol (b)File Transit Protocol (c)File Typing Protocol (d)File Transfer Protocol (e)None of these Ans. File Translate Protocol 65. Voice mail: (a)is the exchange of text messages and computer files transmitted via a communications network such as a local area network or the Internet (b)permits users to converse in real time with each other via the computer while connected to the Internet (c)functions much like an answering machine, allowing callers to leave a voice message for the called party (d)involves using video and computer technology to conduct a meeting between participants at geographically separate locations (e)None of these Ans. functions much like an answering machine, allowing callers to leave a voice message for the called party 66. A communication processor that connects dissimilar networks by providing the translation from one set of protocol to another is: (a)Bridge (b)Gateway (c)Router (d)Modem (e) All of these Ans. Gateway 67. How is power supplied to a low power USB device? (a)Through a power cable (b)From an external power supply (c)Directly from the computer's power supply (d)Through the USB cable (e)None of these Ans. Through the USB cable 68. RSA is: (a)Symmetric Cryptosystem (b)Asymmetric Cryptosystem (c)Block Cypher (d)Digital Signature (e)None of these Ans. Digital Signature 69. When you are selecting a mouse for a particular computer system, what is the most important consideration? (a)The type of drivers that come with the mouse (b)The length of the mouse cord (c)The type of connector the mouse is equipped with (d)The number of buttons the mouse has (e)None of these Ans. The type of connector the mouse is equipped with 70. The ......... data mining technique derives rules from real world case examples. (a)Rule discover (b)Signal processing (c)Neural nets (d)Case based reasoning (e)None of these Ans. Neural nets 71. ......... are used to identify a user who returns to a Website. (a)Cookies (b)Plugins (c)Scripts (d)ASPs (e)None of these Ans. Cookies 72. A DVD-RAM is similar to a ......... except it has storage capacities up to 5.2 GB. (a)CD-R (b)floppy disk (c)CD-RW (d)hard disk (e)None of these Ans. CD-RW 73. Which of the following is a general-purpose programming language, designed by Sun Microsystems, and well suited for use on the Web? (a)VB Script (b)Java Script (c)CSS (d)Java (e)None of these Ans. Java Script 74. Which one of the following objects is passed to a Java Bean when one of its properties is set via a JSP action? (a)Servlet Request (b)Http Servlet Request (c)Servlet Response (d)http Servlet Response (e)None of these Ans. Servlet Response 75. The altering of data so that it is not usable unless the changes are undone is: (a)Biometrics (b)Compression (c)Encryption (d)Ergonomics (e)None of these Ans. Encryption SBI Clerk Exam pattern Exam 2014: General Awareness: 40 points. Quantitative Aptitude: 40 points. Reasoning ability: 40 points. General English: 40 points. Computer Knowledge / Marketing: 40 points. A) Quantitative Aptitude Questions with Answers 1) If a:b = 2:3 and b:c = 4:3, then find a:b:c a) 8:12:9 b) 2:3:8 c) 2:3:9 d) 2:3:12 Answer: a Explanation: a:b=2:3b:c=4:3=(4?34:3?34)=3:94a:b:c=2:3:94=8:12:9 2) The average age of 20 students in a class is 20. When the age of their class teacher commuted with the total age of students, their average age, including the class teacher's raises to 21. If so, what is the age of the class teacher? a) 40 b) 36 c) 41 d) 44 Answer: c 3) Rajan travels from Kannur to Kozhikode by a car at speed of 60 Km/hour and returns back by hiring a gipsy at a speed of 50 Km/ hour. Then what will be the average speed of his vehicles throughout their to and fro journey. a) 54-6/11 km/hour b) 57 km/hour c) 55-6/11 Km/hour d) 54-5/11 Km/hour Answer: a 4) Mr. X complete a piece of work within 8 hours and whereas Mr. Y complete the same piece of work in 6 hours. If both Mr. X & Y operate the same work jointly, in how many hours they complete the same work? a) 7/14 Hrs b) 24/7 Hrs c) 5 hours d)Not given Answer: b 5) if x:y = 1:3, then find the value of (7x+3y):(2x+y) a) 14:5 b) 15:5 c) 16:5 d) 17:5 Answer: Option C Explanation: let x = 1k and y = 3k, so =7(k)+3(3k)2(k)+1(3k)=16k5k=16:5 6) Raman & Krishnan will complete a job in 6 days and 12 days respectively according to their capacity. Govindan will complete the same of piece of work in a single day that which Raman & Krishnan both do in a single day. If Krishnan and Govindan wishes to complete the same work, in how many days they take to finish the work? a) 5 days b) 3 days c) 4 days d) 4.5 days Answer: b 7) A and B together have Rs. 1210. If 4/15 of A's amount is equal to 2/5 of B's amount. How much amount B have. a) Rs 484 b) Rs. 480 c) Rs 478 d) Rs 470 Answer: a 8) A train running in 90Km/hour, having 100 meters long, crosses a train running in another track in opposite direction with speed of 54 Km per hours having 140 meters. How many seconds will take the trains to cross each other fully? a) 6 seconds b) 9 seconds c) 8 seconds d) Not given Answer: a B) General English Questions with Answers Read the passage carefully then answer the questions: Now the question arises, what is the secret of the longevity and imperishability of Indian culture? Why is it that such great empires and nations is Babylion, Assyria, Greece, Rome and Persia, could not last more than the footprints of a camel m the shifting sands of the desert, while India which faced the same ups and downs, the same mighty and cruel hand of time, is still alive and with the same halo of glory and splendour ? The answer is given by Prof. J. B. Pratt of America. According to him Hindu religion is the only religion in the world which is 'self-perpetuating and self-renewing.' Unlike other religions 'not death, but development' has been the fate of Hinduism. Not only Hindu religion but the whole culture of the Hindus has been growing changing and developing in accordance with the needs of time and circumstance without losing its essential and imperishable spirit. The culture of the vedic ages, of the ages of the Upanishads, the philosophical systems, the Mahabharata, the Smirities, the Puranas, the commentators, the medieval ^aints and of the age of the modern reformers is the same in Spirit and yet very different in form. Its basic principles are so broad based that they can be adapted to almost any environment of development. 1) The author has compared India with all the following except- a) Greece b) Rome c) Babylon d) Assyria e) Egypt Answer: e 2) In what respect is India implied to be superior to all other nations and empires ? a) Material development b) Cultural advancement c) Military strength d) Territorial expansion e) Empire building Answer: b 3) What, according to J. B. Pratt, is the secret of the longevity and imperishability of Indian culture ? a) It has its origin in the remote past b) It issues from the minds and hearts of its sages c) It is self-perpetuating and self-renewing d) It is founded on religion e) It is founded on morality Answer:c 4) What changes has the spirit of Indian culture undergone during its long history right from the vedic age down to the present times ? a) The precedence of moral values was eclipsed at certain periods of time b) Materialism was the hall-mark of Indian culture during certain periods of time c) During certain periods military development was the be-all and end-all of Indian culture d) There is no such thing as any spirit of Indian culture e) The spirit of Indian culture has practically been the same from the ancient times down to the present Answer: e 5) What according to the author has always characterised the Hindu religion ? a) Development b) Stagnation c) Sometimes development and sometimes stagnation d) Lack of moral values e) 1 Precedence of material over moral values Answer: a 6) Which other religion has been mentioned in the passage as self-developing and self-renewing as Hindu religion ? a) Christianity b) Islam c) Buddhism d) Jainism e) None of these Answer: e 7) "Could not last more than the footprints of a camel on the shifting sands of the desert." What does it mean ? a) It lost itself in deserts b) It was transient c) It lacked solidity d) It was limited only to desert areas e) It lacked cohesion Answer: b Pick out the most appropriate equivalent (synonym) of the following words taken from the above passage: 8) Longevity : a) Living long b) Diseased life c) Depressed spirit d) The period of living e) Misery Answer: a 9) Shifting: a) Piercing b) Penetrating c) Changing d) Mincing e) Obstructing Answer:c Pick out the most appropriate word exactly opposite in meaning of the following words taken from the above passage: 10) Last: a) End b) Grow c) Diminish d) Increase e) Finish Answer: a 11) Mighty: a) Forcible b) Forceful c) Weak d) Tough e) Haughty Answer: c 12) Glory: a) Splendour b) Notoriety c) Fame d) Reputation e) Debasement Answer: e C) General Awareness Questions with Answers 1) Maximum number of factories in India are located in __? a) Gujarat b) West Bengal c) Tamil Nadu c) Maharashtra Tamil Nadu Answer: c 2) Which of the following entities has been recently granted banking license by the Reserve Bank of India? a) Bajaj Finance and IFCI b) Bandhan Financial Services Pvt Ltd and IDFC c) India Infoline and INMAC d) JM Finance and LIC Housing Finance Bandhan Financial Services and IDFC answer: b 3) According to RBI guidelines, what fraction of the branches are mandatory to be opened in rural areas by newly licensed banks? a) 15 b) 25 c) 35 d) 45 Answer: b 4) RBI?s guidelines on setting up new banks require new banks to hold minimum capital of Rs.500 crore. RBI has made it mandatory for new banks to open at least 25% of branches in rural centres. 5. Who among the following has been appointed as next Prime Minister of France? a) Manuel Valls b) Marine Le Pen c) Anne Hidalgo d) Jean Claude Answer: a 5) Lee Kun-hee, who ris facing an arrest warrant in a 1.4 million dollar cheating case is chairman of ___? a) Nokia b) Samsung c) LG d) IBM Answer: b 6) The World Autism Awareness Day is observed every year on ___? a) March 28 b) March 30 c) April 1 d) April 2 Answer: d 7) Who among the following has become the first Indian piston shooter to be rank no.1 in the world ranking, recently? a) Heena Sidhu b) Anjali Bhagwat c) Rani Sarnobat d) Annu Raj Singh Answer: a D) Marketing Questions with Answers 1) Market research is useful for ____? a) Deciding proper marketing strategies b) Deciding the selling price c) choosing the right products d) choosing the sales person e) All of these Answer: e 2) A call means: a) shout out to somebody b) profession or business c) visiting friends d) visiting prospective customers e) after-sales service Answer: d 3) Conversion means: a) meeting a prospective client b) interacting with a prospective client c) converting an employer into an employee d) converting a seller into a buyer e) converting a prospective client into a buyer Answer: e 4. Customization means: a) acquiring more customers b) regulating customers c) special products to suit each customer d) more products per customer d) All of these Answer: c 5) Modern style of marketing include a) digital marketing b) tele-marketing c) e-commerce d) e-mails solicitation e) All of these Answer: e 6. E-Marketing is the same as a) virtual marketing b) digital marketing c) real time marketing d) all of these e) None of these Answer: d 7. Value-added services means a) costlier products b) large number of products c) additional services d) at par services e) None of these Answer: c 8) Efficient Marketing styles require a) proper planning b) good communication skills c) team work d) knowledge of products e) All of these Answer: e 9) The performance of a salesperson depends on a) salary paid b) sales incentive paid c) sizes of the sales team d) ability and willingness of the salesperson e) team leader aggressiveness Answer: d 10) Lead generation means a) tips for selling tactics b) tips for better production c) generating leaders d) likely sources for prospective clients e) All of these Answer: d E) Reasoning ability Questions with Answers 1) If FRIEND is coded as HUMJTK, how can CANDLE be written in that code? a) DEQJQM b) DCQHQK c) EDRIRL d) ESJFME Answer : c In the code, the first letter is the second alphabet, the second letter is the third alphabet, the third letter is the fourth alphabet and so on after the corresponding letter in the word. 2) In a certain code ADVENTURES is written as TRDESAUVEN. How is PRODUCED written in that code ? a) IUIPGSSRNP b) IUIPGSRSNR c) IUINGSSRRP d) IRIPGSSNRR Answer : a The first and sixth, third and eighth, fifth and tenth letters of the word interchange places in the code. 3) If FRAGRANCE is written as SBHSBODFG, how can IMPOSING be written? a) NQPTJHOJ b) NQPTJOHJ c) NQTPJOHJ d) NQPTJOHI Answer : d Each letter in the word is moved one step forward and the first letter of the group. so obtained is put at the end, to obtain the code. 4) If ROBUST is coded as QNATRS in a certain language, which word would be coded as ZXCMP? a) AWDLQ b) AYDNQ c) BZEOR d) YYBNO Answer : b Each letter of the word is one step ahead of the corresponding letter of the code. 5) If PALE is coded as 2134, EARTH is coded as 41590, how can is PEARL be coded in that language? a) 25430 b) 29530 c) 25413 d) 24153 Answer : d The letters are coded accordingly P as 2, E as 4, A as 1, R as 5 and L as 3. So PEARL is coded as 24153. 6) If in a certain code, GLAMOUR is written as IJCNMWP and MISRULE is written as OGUSSNC, then how will TOPICAL be written in that code? a) VMRJECN b) VMRHAGJ c) VMRJACJ d) VNRJABJ Answer : c The first, third and sixth letters of the word are each moved two steps forward, the second, fifth and seventh letters are each are moved two steps backward, while the fourth letter is moved one step forward. 7) If 'eraser' is called 'box', 'box' is called 'pencil', 'pencil' is called 'sharpener', and 'sharpener' is called 'bag', what will a child write with? a) Eraser b) Bag c) Pencil d) Sharpener Answer : d A child will write with a 'pencil', and pencil is called 'sharpener'. So, a child will write with a 'sharpener'. 8) A told B that C is his father's nephew. D is A's cousin but not the brother of C. What relationship is there between D and C ? a) Father c) Sisters c) Aunt d) Mother Answer : d. A has two cousins C and D. Since C is male, D must be female. And, both are the nephew and niece of A's father. 9) A, B, C, D, E, F and G are members of a family consisting of four adults and three children, two of whom, F and G are girls. A and D are brothers and A is a doctor. E is an engineer married to one of the brothers and has two children. B is married to D and G is their child. Who is C ? a) E's daughter b) F's father c) G's brother d) A's son Answer : d E is married to A or D. But B is married to D. Thus, E is married to A. Thus, A, B, D, E are the four adults and C, F, G are the three children in the family. B and D have a child G. A and E have two children. They are C and F. Now, only F and G are girls. So, C is a boy. Thus, C is A's or E's son. 10) lion: animal : : flower : ___________ a) plant b) roots c) grass d) rose Answer : a F) Computer KnowledgeQuestions with Answers 1) WWW stands for ? a) World Whole Web b) Wide World Web c) Web World Wide d) World Wide Web Answer: d 2) Which of the following are components of Central Processing Unit (CPU) ? a) Arithmetic logic unit, b) Mouse Arithmetic logic unit, c) Control unit Arithmetic logic unit, d) Integrated Circuits Control Unit, e) Monitor Answer: b 3) Where is RAM located ? a) Expansion Board b) External Drive c) Mother Board d) All of above Answer:C 4) If a computer provides database services to other, then it will be known as ? a) Web server b) Application server c) Database server d) FTP server Answer: C 5) Full form of URL is ? a) Uniform Resource Locator b) Uniform Resource c) Link Uniform d) Registered Link e) Unified Resource Link Answer: a 6) ____________ controls the way in which the computer system functions and provides a means by which users can interact with the computer. a) The operating system b) The motherboard c) The platform d) Application software Answer : a 7) The difference between people with access to computers and the Internet and those without this access is known as the: a) digital divide. b) Internet divide. c) Web divide. d) cyberway divide. Answer : a. 8) Servers are computers that provide resources to other computers connected to a: a) mainframe b) supercomputer c) network d) client Answer :c SBI Whole-Testpaper SBI Clerk Exam - Previous Year Solved Question Paper General Awareness: 1. Who amongst the following was the legendary classical singer, an exponent of the 'Khayal' form of singing, who passed away in 2011? 1) Jagjit Singh 2) Arjun Singh 3) M.F. Hussain 4) Bhupen Hazarika 5) Bhimsen Joshi 2. Commonwealth Games 2014 will be held at- 1) Glasgow 2) Montreal 3) Queensland 4) Gold Coast 5) Beijing 3. Who amongst the following is an American business magnate, investor and philanthropist, Chairman & CEO of Berkshire Hathaway, who is widely considered the most successful investor of the 20th century? 1) Duncan Fletcher 2) Warren Buffett 3) Warren Anderson 4) Niera Radia 5) None of these 4. On 8th August 2012, a 14-member 'Naresh Chandra Task Force' has submitted to the Government of India its report relating to- 1) electoral reforms 2) corruption 3) black money 4) infrastructure sector 5) national security 5. A network of power lines that evacuates electricity from a generating station and operates in a narrow frequency band is known as- 1) Web 2) Mesh 3) Grid 4) Interlock 5) Knit 6. Which of the following is a large ocean wave that is caused by sudden motion on the ocean floor which could be in the form of an earthquake, volcanic eruption or an underwater landslide? 1) El Nino 2) Lunar Eclipse 3) High Tide 4) Low Tide 5) Tsunami 7. The form 'ITR-1' is a form for using 1) Income Tax Challan 2) Excise Duty 3) Customs duty 4) Income Tax Return 5) VAT 8. Which of the following is NOT a viral disease? 1) Chicken-pox 2) Influenza 3) Epilepsy 4) Measles 5) Rabies 9. International day of Non-Violence is observed every year on- 1) 15th August 2) 2nd October 3) 26th January 4) 31st January 5) None of these 10. A new scheme has been announced in the Budget 2012-13, giving tax exemptions to first-time equity market investors. The scheme is titled as- 1) Rajiv Gandhi Equity Savings Scheme 2) Rashtriya Equity Savings Scheme 3) National Equity Savings Scheme 4) Indira Gandhi Equity Savings Scheme 5) Prime Minister's Equity Savings Scheme 11. In addition to India and China, the following nations are also members of BRICS'- 1) Bangladesh, Russia and Sri Lanka 2) Belgium, Romania and South Africa 3) Belgium, Russia and Saudi Arabia 4) Brazil, Russia and South Africa 5) Bhutan, Russia and Sri Lanka 12. Which of the following is a voluntary force raised to assist the police in controlling civil disturbances and communal riots? 1) Rapid Action Force 2) Central Industrial Security Force 3) National Security Guard 4) Central Reserve Police Force 5) Home Guards 13. Who amongst the following is the recipient of Padma Vibhushan in 2012? 1) Jatin Das 2) Mario De Miranda 3) Mira Nair 4) N. Vitthal 5) Limba Ram 14. Who amongst the following is the modern-day spiritual icon and the founder of Art of Living? 1) Baba Ram Dev 2) Acharya Rajneesh 3) Shri Narayan Guru 4) Narendra Dev Acharya 5) Shri Ravi Shankar 15. Which of the following is the national flower of India? 1) Rose 2) Jasmine 3) Lily 4) Lotus 5) Tulip 16. Which of the following is NOT the name of a space shuttle? 1) Enterprise 2) Columbia 3) Iris 4) Discovery 5) Challenger 17. Which of the following is effectively a tax on the carbon dioxide emissions from burning fossil fuels? 1) Fuel Surcharge 2) Carbon Surcharge 3) Carbon tax 4) Hazardous emissions tax 5) Fuel Tax 18. The term 'ATM' stands for 1) Alternate to Money 2) Automated Transaction Machine 3) Automated Teller Mechanism 4) Automatic Teller Mode 5) Automated Teller Machine 19. Which of the following is a measure of the impact of our activities on the environment and in particular climate change? 1) Carbon Trajectory 2) Carbon Trail 3) Carbon Track 4) Carbon Footprint 5) Carbon Imprint 20. Which of the following is a statutory quasi-judicial authority mandated by the Parliament to preserve the freedom of press and maintain and improve the standard of newspapers? 1) Press Trust of India 2) Press Information Bureau 3) United News of India 4) NAM News Network 5) Press Council of India English Language: Directions (Q. 1-10): In the following passage, there are blanks, each of which has been numbered. These numbers are printed below the passage and against each, five words/ phrases are suggested, one of which fits the blank appropriately.Find out the appropriate word/ phrase in each case. Once upon a time, Amarasakti ruled the city-state of Mahilaropyam in the south of India. He had three witless sons who became a matter of endless (1) for him. (2) that his sons had no interest in learning, the king summoned his ministers and said,"You know I am not happy with my sons. According to men of learning, an unborn son is better than a son who is a (3). A son who is stupid will bring dishonour to his father. How can I make my sons fit to be my (4)? I turn to you for advice". One of the ministers (5) the name of Vishnu Sharman, a great scholar enjoying the (6) of hundreds of his disciples. "He is the most (7) person to tutor your children. Entrust them to his care and very soon you will see the change". The king summoned Vishnu Sharman and pleaded with him "Oh, venerable scholar, take pity on me and please train my sons. Turn them into great scholars and I will be (8) to you all my life". Vishnu Sharman said, "Oh, king, count six months from today. If I do not make your children great scholars, you can ask me to change my (9)". The king immediately called his sons and handed them over to the care of the learned man. Sharman took them to his monastery where he (10) teaching them. Keeping his word, he finished the task the king entrusted to him in six months. Thus, the king, now, had scholars for sons. 1. 1) ache 2) worry 3) joy 4) pity 5) hazard 2. 1) fact 2) belief 3) since 4) realising 5) being 3. 1) stupid 2) brilliant 3) fool 4) uneducated 5) scholar 4. 1) self 2) places 3) successors 4) level 5) siblings 5. 1) suggested 2) requested 3) called 4) pointed 5) said 6. 1) teachings 2) attendance 3) glamour 4) rights 5) respect 7. 1) competent 2) right 3) easy 4) actual 5) skill 8. 1) happy 2) oblige 3) beside 4) indebted 5) disciple 9. 1) importance 2) name 3) clothes 4) location 5) life 10. 1) embark 2) mustered 3) begun 4) decided 5) started   Directions (Q. 11-20): Read the following passage carefully and answer the questions given below it. Certain words/ phrases have been printed in bold to help you locate them while answering some of the questions. A long time ago, on a big tree in the lap of a mountain, lived a bird named Sindhuka. It was a rather special bird because its droppings turned into gold as soon as they hit the ground. One day, a hunter came to the tree in search of prey and he saw Sindhuka's droppings hit the ground and turn into gold. The hunter was struck with wonder. He thought, "I have been hunting birds and small animals since I was a boy, but in all my 80 years, I have never seen such a miraculous creature". He decided that he had to catch the bird somehow. He climbed the tree and skillfully set a trap for the bird. The bird, quite unaware of the danger it was in, stayed on the tree and sang merrily. But it was soon caught in the hunter's trap. The hunter immediately seized it and shoved it into a cage. The hunter took the bird home joyfully. But as he had time to think over his good fortune later, he suddenly realised, "If the king comes to know of this wonder, he will certainly take away the bird from me and he might even punish me for keeping such a rare treasure all to myself. So it would be safer and more honourable if I were to go to the king and present the unique bird to him". The next day, the hunter took the bird to the king and presented it to him in court with great reverence. The king was delighted to receive such an unusual and rare gift. He told his courtiers to keep the bird safe and feed it with the best bird food available. The king's prime minister, though, was reluctant to accept the bird. He said "O Rajah, how can you believe the word of a foolish hunter and accept this bird? Has anyone in our kingdom ever seen a bird dropping gold? The hunter must be either crazy or telling lies. I think it is best that you release the bird from the cage". After a little thought, the king felt that his prime minister's words were correct. So he ordered the bird to be released. But as soon as the door of the cage was thrown open, the bird flew out, perched itself on a nearby doorway and defecated. To everyone's surprise, the dropping immediately turned into gold. The king mourned his loss. 11. Which of the following can possibly be the most appropriate title for the story? 1) The Skilled Hunter 2) The King's Prime Minister 3) The King's Defeat 4) The Bird With The Gold Dropping 5) The Trials And Tribulations Of The Foolish Bird Sindhuka 12. Which of the following emotions made the hunter gift the bird to the king? 1) Respect 2) Joy 3) Pride 4) Fear 5) Awe 13. Which of the following is TRUE according to the story? 1) Birds like Sindhuka were very common in the area near the mountain. 2) Sindhuka remained caged for the rest of its life. 3) Sindhuka was unaware of the trap laid by the hunter. 4) The king, when told to not accept the bird, did not listen to his prime minister. 5) All are true. 14. Why was the king's prime minister reluctant to accept the bird? 1) He believed that the bird would die if caged. 2) He knew about the hunter's habit of lying. 3) He believed that the bird would bring bad luck to the king. 4) His sources had informed him that the hunter was crazy. 5) None of these. 15. How did the hunter find Sindhuka? 1) He had read stories about the bird and had set traps at various locations in the city. 2) He followed the bird's droppings. 3) He was on the lookout for a prey when he chanced upon it. 4) People from the city had informed him about the bird's whereabouts. 5) He was attracted by the bird's calls. Directions (Q.16-18): Choose the word/ group of words which is most similar in meaning to the word/ group of words printed in bold as used in the passage. 16. RATHER 1) regular 2) quite 3) instead 4) but 5) known 17. RELEASE 1) free 2) vacate 3) vent 4) let expire 5) make public 18. REVERENCE 1) respect 2) detail 3) astonishment 4) hope 5) remembrance Directions (Q. 19-20): Choose the word which is most OPPOSITE in meaning to the word printed in bold as used in the passage. 19. RELUCTANT 1) true 2) clever 3) averse 4) hesitant 5) keen 20. SKILFULLY 1) angrily 2) haphazardly 3) highly 4) cheaply 5) deftly Quantitative Aptitude: Directions (Q. 1-5): What approximate value should come in place of question mark (?) in the following questions?(Note: You are not expected to calculate the exact value) 1. 11.304 × (6.839 ? 4.331) = ? 1) 30 2) 45 3) 16 4) 52 5) 12 2. 61 × 24.879 ÷ (14.059?6) = ? 1) 110 2) 190 3) 220 4) 348 5) 98 3. (3.805)2 × 14.018 ? 5991 = ? 1) 165 2) 305 3) 278 4) 128 5) 200 4. ?230 ÷ 2.017 ÷ 58.794 = ? 1) 80 2) 102 3) 68 4) 96 5) 77 5. 3451 ÷ 9.895 × 3.0126 = ? 1) 1050 2) 1235 3) 990 4) 1360 5) 1248 6. A train running at the speed of 108 kmph, crosses a 365-metre long platform in 21 secs. What is the length of the train? 1) 260 metres 2) 275 metres 3) 265 metres 4) 285 metres 5) None of these 7. In order to pass in an examination, a student is required to get 280 marks out of the aggregate marks. Hema got 264 marks and was declared failed by 2 per cent. What is the minimum passing percentage of the examination? 1) 33 2) 35 3) 40 4) 44 5) None of these 8. The cost of 8 fans and 14 ovens is Rs.36520. What is the cost of 12 fans and 21 ovens (in Rs.)? 1) Rs.56,800 2) Rs.54780 3) Rs.57950 4) Cannot be determined 5) None of these 9. The radius of a circular garden is 7 metres more than the length of a rectangle whose perimeter is 364 metres and breadth is 84 metres. What will be cost of fencing the garden (only at the circumference), if the cost of fencing is Rs.8 per metre? 1) Rs.5456 2) Rs.6144 3) Rs.5296 4) Rs.5280 5) None of these 10. If 43x + 43y = 4816, what is the average of x and y? 1) 56 2) 112 3) 62 4) 124 5) None of these 11. The average age of a man and his son is 28 years. The ratio of their ages is 3 : 1. What is the man's age? 1) 30 years 2) 38 years 3) 44 years 4) 42 years 5) None of these 12. A car manufacturing plant manufactures 96 dozen cars in eight days. How many dozen cars will the plant manufacture in 17 days? 1) 210 2) 224 3) 204 4) 209 5) None of these 13. A and B together can complete a piece of work in 16 days. B alone can complete the same work in 24 days. In how many days can A alone complete the same work? 1) 34 days 2) 50 days 3) 48 days 4) 42 days 5) None of these 14. Find the average of the following set of scores:142, 93, 102, 206, 115, 98. 1) 122 2) 106 3) 138 4) 117 5) None of these 15. The average of four consecutive odd numbers P, Q, R and S respectively (in increasing order) is 104. What is the sum of P and S? 1) 204 2) 208 3) 206 4) 212 5) None of these Directions (Q.16-18): Study the given information carefully and answer the given questions. A is mother of B. B is sister of C. D is son of C. E is brother of D. F is mother of E. G is granddaughter of A. H has only two children- B and C. 16. How is F related to H? 1) Son-in-law 2) Daughter-in-law 3) Father-in-law 4) Granddaughter 5) Cannot be determined 17. How is C related to E? 1) Father 2) Son 3) Mother 4) Cousin brother 5) Cannot be determined 18. Who is mother of G? 1) G 2) B 3) F 4) Either B or F 5) Either C or F Directions (Q. 19-21): The following questions are based on the five three-letter words given below: FAN HOP GET CUB MID (NOTE: The words formed after performing the given operations may or may not be meaningful English words.) 19. If the positions of the first and the second letters of all the words are interchanged, how many words will form meaningful English words? 1) None 2) Three 3) One 4) Two 5) More than three 20. If all the letters in each of the words are arranged alphabetically (within the word), how many words will remain unchanged? 1) Two 2) Three 3) One 4) None 5) More than three 21. If the first letter of all the words is changed to the next letter of the English alphabetical series, how many words will have more than one vowel? (Same or different vowel) 1) None 2) Three 3) Two 4) More than three 5) One 22. Which of the following will come next in the following series? a z a b y a b c x a b c d w a b c d 1) f 2) u 3) a 4) v 5) e 23. In a certain code language, 'TONIC' is coded as 'CINOT' and 'SCALE' is coded as 'ACELS'. In the same code language, 'PLANK' will be coded as- 1) KNALP 2) AKNLP 3) AKLNP 4) AKPNL 5) Cannot be determined 24. Which of the following will come in place of question mark (?) in the following series based on the English alphabetical order? YS RM LH ? CA 1) DG 2) FD 3) GD 4) DF 5) GC 25. How many such pairs of letters are there in the word NEUTRAL, each of which has as many letters between them in the word (in both forward and backward directions) as they have between them in the English alphabetical series? 1) Two 2) Three 3) None 4) One 5) More than three Directions (Q. 26-30): Study the given information carefully and answer the given questions. Eight people- A, B, C, D, E, F, G and H - are sitting around a circular table facing the centre, not necessarily in the same order. Three people are sitting between A and D. B is sitting second to the right of A. C is on the immediate right of F. D is not an immediate neighbour of either F or E. H is not an immediate neighbour of B. 26. What is E's position with respect to G? 1) Third to the left 2) Second to the right 3) Third to the right 4) Second to the left 5) Fifth to the right 27. Four of the following five are alike in a certain way based on their seating positions in the above arrangement and so form a group. Which of the following does not belong to the group? 1) GE 2) DC 3) AF 4) AB 5) CE 28. Who is sitting third to the right of the one who is sitting on the immediate right of H? 1) A 2) B 3) E 4) C 5) G 29. Which of the following is true regarding the given arrangement? 1) E is second to the left of C. 2) B is an immediate neighbour of G. 3) H is an immediate neighbour of A. 4) D is not an immediate neighbour of H. 5) None is true. 30. How many people are sitting between H and A when counted from the right side of H? 1) Three 2) None 3) More than three 4) One 5) Two Computer Knowledge: 1. ROM stands for- 1) Random Only Memory 2) Readable Only Memory 3) Real Online Memory 4) Random Other Memory 5) Read Only Memory 2. Another name for a logic chip is- 1) PROM 2) memory 3) microprocessor 4) ROM 5) None of these 3. The number system based on "0" and "1" only is known as- 1) Binary System 2) Barter System 3) Number System 4) Hexadecimal System 5) Special System 4. The capacity of 3.5 -inch floppy disk is- 1) 1.40 MB 2) 1.44 GB 3) 1.40 GB 4) 1.45 MB 5) 1.44 MB 5. Which of the following is not an integral part of a computer- 1) CPU 2) Mouse 3) Monitor 4) UPS 5) None of these 6. Which of the following is not a part of the CPU? 1) Primary storage 2) Registers 3) Control unit 4) ALU 5) None of these 7. The device that reconciles differences between computers and phones is the- 1) LAN 2) wand reader 3) TCP/IF 4) scanner 5) modem 8. A combination of hardware and software that allows communication and electronic transfer of information between computers is a- 1) Network 2) Backup system 3) Server 4) Peripheral 5) Modem 9. Which of the following represents the fastest data transmission speed? 1) bandwidth 2) bps 3) gbps 4) kbps 5) mbps 10. What is the major feature of the World Wide Web that makes it simple to learn and use? 1) Database interface 2) Graphical text interface 3) Graphical user interface 4) Point-to-Point Protocol 5) None of these 11. Which of the following is an input device that, when moved by the user on a flat surface, causes a pointer on the screen to move accordingly? 1) wand reader 2) mouse 3) keyboard 4) bar-code-reader 5) scanner 12. A bar code reader is an example of a(n)- 1) processing device 2) storage device 3) input device 4) output device 5) printer 13. If you regularly send letters to people, what type of program would you use to create the letters? 1) Hardware 2) System software 3) Utility software 4) Application software 5) None of these 14. A computer gets--- with the help of mouse or keyboard- 1) insert 2) instructions 3) guidance 4) input 5) None of these 15. Dot-matrix is a type of- 1) tape 2) printer 3) disk 4) bus 5) None of these 16. Which of the following represents one billion characters? 1) Byte 2) Gigabyte 3) Kilobyte 4) Megabyte 5) Terabyte 17. Which of the following menu types is also called a drop down menu? 1) Fly-down 2) Pop-down 3) Pop-up 4) Pull-up 5) Pull-Down 18. What is an onscreen picture that represents objects, such as a program or file? 1) Spool 2) NOS 3) Page 4) Pointer 5) Icon 19. Which of the following is not necessary to be considered computer-literate? 1) The ability to write the instructions that direct a computer 2) An awareness of the computer's importance, versatility, and pervasiveness in society 3) Knowledge of what computers are and how they work 4) The ability to interact with computers using simple applications 5) None of these 20. An area of the microprocessor chip used to temporarily store instructions and data that the processor is likely to use frequently is termed a(n)- 1) ALU 2) Bus 3) Cache 4) CPU 5) Flash SBI Question-Paper SBI PO Preliminary Exam Sample Question Paper Quantitative Aptitude Directions (Qs 1 - 5): What should come in place of the question mark (?) in the following questions? 1.(- 251 x 21 x - 12) ÷ ? = 158. 13 (1) 250 (2) 400 (3) 300 (4) 150 (5) None of these 2.25.6% of 250 + ?? (1) 4225 (2) 3025 (3) 2025 (4) 5625 (5) None of these 3.36865 + 12473 + 21045 - 44102 = ? (1) 114485 (2) 28081 (3) 26281 (4) 114845 (5) None of these 4.(15.20)2 - 103.04 ÷ ? = 8 (1) 12 (2) 6.5 (3) 8.2 (4) 16 (5) None of these 5.7428 x (3/4) x (2/9) x ? = 619 (1) 0.5 (2) 1.5 (3) 0.2 (4) 2.4 (5) None of these Directions (6-10): In the following questions, two equations numbered I and II are given. You have to solve both the equations and give answers. (1) If x > y (2) If x > y (3) If x < y (4) if x < y (5) If x = y or the relationship cannot be established. 6. I. 12x2 + 11x + 12 = 10x2 + 22x II. 13y2 - 18y + 3 = 9y2 - 10 7. I. 18/x2 + 6/x - 12/x2 = 8/x2 II. y3 + 9.68 + 5.64 = 16.95 8. I. ?1225 x + ?4900 = 0 II. (81)1/4 y + (343)1/3 = 0 9. I. (2)5 + (11)3 / 6 = x3 II. 4y3 = - (589 ÷4) + 5y3 10. I. (x7/5 ÷ 9) = 169 ÷ x3/5 II. y1/4 X y 1/4 X 7 = 273 ÷y1/2 Directions (11-15): In the following number series, a wrong number is given. Find out the wrong number. 11. 29, 37, 21, 43, 13, 53, 5 (1) 37 (2) 53 (3) 13 (4) 21 (5) 43 12. 600, 125, 30, 13, 7.2, 6.44, 6.288 (1) 125 (2) 30 (3) 13 (4) 6.44 (5) 7.2 13. 80, 42, 24, 13.5, 8.75, 6.375, 5.1875 (1) 8.75 (2) 13.5 (3) 24 (4) 6.375 (5) 42 14. 10, 8, 13, 15, 35, 135, 671, 4007 (1) 8 (2) 671 (3) 135 (4) 13 (5) 35 15. 150, 290, 560, 1120, 2140, 4230, 8400 (1) 2140 (2) 4230 (3) 560 (4) 290 (5) 1120 16. A certain amount was to be distributed among A, B and C in the ratio 2 : 3 : 4 respectively but was incorrectly distributed in the ratio 7 : 2 : 5 respectively. As a result of this, B got Rs. 40 less. What is the amount? (1) Rs. 210 (2) Rs. 270 (3) Rs. 230 (4) Rs. 280 (5) None of these 17. Rachita enters a shop to buy ice - creams, cookies and pastries. She has to buy at least 9 units of each. She buys more cookies than ice - creams and more pastries than cookies. She picks up a total of 32 items. How many cookies does she buy? (1) Either 12 or 13 (2) Either 11 or 12 (3) Either 10 or 11 (4) Either 9 or 11 (5) Either 9 or 10 18. The fare of a bus is Rs. X for the first five km and Rs.13 per km thereafter. If a passenger pays Rs. 2402 for a journey of 187 km, what is the value of X? (1) Rs. 29 (2) Rs. 39 (3) Rs. 36 (4) Rs. 31 (5) None of these 19. The product of three consecutive even numbers is 4032. The product of the first and the third is 252. What is the five times the second number? (1)80 (2) 100 (3) 60 (4) 70 (5) 90 20. The sum of the ages of 4 members of a family 5 yrs ago was 94 yr. Today, when the daughter has been married off and replaced by a daughter - in - law, the sum of their ages is 92. Assuming that there has been no other change in the family structure and all the people are alive, what is the difference in the age of the daughter and daughter - in - law? (1) 22yr (2) 11 yr (3) 25 yr (4) 19yr (5) 15 yr 21. A bag contains 13 white and 7 black balls. Two balls are drawn at random. What is the probability that they are of the same colour? (1) 41/190 (2) 21/190 (3) 59/190 (4)99/190 (5) 77/190 22. Akash scored 73 marks in subject A. He scored 56% marks in subject B and X marks in subject C. Maximum marks in each subject were 150. The overall percentage marks obtained by Akash in all the three subjects together were 54%. How many marks did he score in subject C? (1) 84 (2) 86 (3) 79 (4) 73 (5) None of these 23. The area of a square is 1444 sq m. The breadth of a rectangle is 1/4 th of the side of the square and the length of the rectangle is thrice its breadth. What is the difference between the area of the square and the area of the rectangle? (1) 1152.38 sq m (2) 1169.33 sq m (3) 1181.21 sq m (4) 1173.25 sq m (5) None of these 24. Rs. 73689 are divided between A and B in the ratio 4:7. What is the difference between thrice the share of A and twice the share of B? (1) Rs. 36699 (2) Rs. 46893 (3) Rs. 20097 (4) Rs. 26796 (5) Rs. 13398 25. A and B together can complete a task in 20 days. B and C can complete a task in 30 days. A and C can complete a task in 40 days. What is the respective ratio of the number of days taken by A when completing the same task alone to the number of days taken by C when completing the same task alone? (1) 2 : 5 (2) 2 : 7 (3) 3 : 7 (4) 1: 5 (5) 3 : 5 STATE BANK OF INDIA PAPER ON 7th JANUARY 2007 Reasoning Directions (1-5): In the questions given below, certain symbols are used with the following meaning: A @ B means A is greater than B. A * B means A is either greater than or equal to B. A # B means A is equal to B A $ B means A is either smaller than or equal to B A + B means A is smaller than B Now in each of the following questions assuming the given statements to be true, find which of the two conclusions I and II given below them is/are definitely True? Give answer (a) if only conclusion I is true. Give answer (b) if only conclusion II is true. Give answer (c) if either conclusion I and II are true. Give answer (d) if neither conclusion I nor II is true. Give answer (e) if both conclusions I and II are true. 1. Statements: D + T ; E $ V ; F * T ; E @ D Conclusions: I. D $ V II. D + F Ans: (b) 2. Statements: B + D ; E $ T ; T * P ; P @ B Conclusions: I. P $ D II. P @ D Ans: (c) 3.Statements: T * U ; U $ W ; V @ L ; W + V Conclusions: 1. V @ T II. L # W Ans: (d) 4.Statements: P $ Q ; N # M ; M @ R ; R * P Conclusions: I. P + N II. Q $ M Ans: (a) 5.Statements: E * F ; G $ H ; H # E ; G @ K Conclusions: I. H @ K II H * F Ans: (e) Directions(6-11): Study the following letter-number-symbol sequence carefully and answer the questions given below: 3 D 5 F E 3 8 $ M 2 1 K * P T @ U 9 A 7 1 £ H J 4 Q 6 6. What should come in place of the question mark (?) in the following Sequence? 5ES, MIP, ?, 1HQ (a) TUA (b) TU7 (c) @ 91 (d) T91 (e) None of these Ans: (b) 7.which of the following is exactly in the midway between the eleventh from the left end and the 7th from the right end? (a) P (b) @ (c) T (d) U (e) None of these Ans: (b) 8. Which of the following is the sixth to the right of the twentieth from the right end? (a) 5 (b) F (c) P (d) K (e) None of these Ans: (c) 9. How many such digits are there in the above sequence which are immediately proceded as well as followed by digits ? (a) None (b) One (c) Two (d) Three (e) more than three Ans: (a) 10. If the first fifteen elements are written in the reverse order then which of the following will be eighth to the left of the thirteenth element from right end ? (a) M (b) 8 (c) $ (d) * (e) None of these. Ans: (a) 11.If all the consonants Starting from B are given sequentially the value of even numbers such as B=2, C=4 and so on and all the vowels are given the value of 5 each, then what will be the value of the letters of the word CUSTOM ? (a) 92 (b) 86 (c) 82 (d) 96 (e) None of these Ans: (d) 12. How many such 5s are there in the following sequence that the sum of the two immediately following digits is greater than the sum of the two immediately preceding digits ? 3 7 6 5 8 3 2 4 5 5 4 8 7 9 1 5 3 4 8 7 5 9 8 7 6 4 (a) One (b) Two (c) Three (d) Four (e) None of these Ans: (c) 13. If A + B means "A is the sister of B", A × B means "A is the wife of B", A ÷ B means "A is the father of B" and A-B means "A is the brother of B"' then which of following expresses the relationship that "t is the daughter of p"? (a) P × Q ÷ R + S - T (b) P × Q ÷ R - T + S (c) P × Q ÷ R + T - S (d) P × Q ÷ R + S + T (e) None of these Ans: (b) 14.If the position of the first letter of English alphabet is interchanged with the position of the fourteenth letter, second letter with the fifteenth letter in such a way that M is interchanged with Z, then which of the following letters will be 9th to the right of 17th letter from the right? (a) F (b) E (c) R (d) T (e) None of these Ans: (a) Directions(15-18) : Read the following information and answer the questions given below:- (i) Seven friends P, Q, R, S, T, U and W have gathered at the Mumbai airport. Five of them are scheduled to go to five different places-Delhi, Chennai, Lucknow, Bangalore and Calcutta. (ii) Five of them are executives, each specialising in viz.Administration (admn.), Human Resource Management (HRM), Marketing ,Systems and Finance. (iii) T, an executive is going to Chennai and is neither from finance nor Marketing. (iv) W is a system specialist and is leaving for Delhi. U is an executive but is not going to one of the five places. (v) Q is an executive from HRM but has come at the airport to see his friends. (vi) P is an executive but not from Marketing and is flying to one of the destinations but not to Bangalore or Calcutta. 15. Who is going to fly to Bangalore? (a) Data inadequate (b) R (c) S (d) P (e) None of these Ans: (a) 16. Who among the following specialises in Marketing? (a) S (b) P (c) U (d) Data inadequate (e) None of these Ans: (c) 17. R has specialisation in which of the following fields? (a) Finance (b) Marketing (c) Either Marketing or Finance (d) None (e) All of these Ans: (d) 18.The one who is going to fly to Chennai is (a) Not an executive (b) From administration (c) S (d) From Finace (e) None of these Ans:(b) 19. How many pairs of letters are their in the word 'NURSING' which have as many letters between them as in the alphabet? (a) One (b) Three (c) Five (d) Six (e) None of these Ans: (b) Directions(20-26): Read the following information carefully and answer the questions given below. A famous museum issues entry passes to all its visitors for security reasons.Visitors are allowed in batches after every one hour. In a day there are six batches. A code is printed on entry pass which keeps on changing for every batch. Following is an illustration of pass-codes issued for each batch. Batch I: clothes neat and clean liked are all by Batch II: by clothes neat all are and clean liked Batch III: liked by clothes clean and neat all are and so on.... 20. If pass-Code for the third batch is 'night succeed day and hard work to for ', what will be the pass code for the sixth batch? (a) Work hard to for succeed night and day (b) Hard work for and succeed night to day (c) Work hard for to succeed night and day (d) Hard work for to succeed night and day (e) None of these Ans: (c) 21. If 'visit in zoo should the we time day' is the pass code for the fifth batch, 'zoo we the should visit day time in ' will be the pass code for which of the following batches? (a) II (b) IV (c) I (d) III (e) VI Ans: (d) 22. Sanjay visited the museum in the fourth batch and was issued a pass-code 'to fast rush avoid not do very run'. What would have been the pass-code for him had he visited the museum in the second batch? (a) rush do not avoid to run very fast (b) rush not do avoid to run very fast (c) avoid rush not do to run very fast (d) Data inadequate (e) None of these Ans: (a) 23. Subodh went to visit the museum in the second batch. He was issued a pass-code 'length the day equal of and night are'. However, he could not visit the museum in the second batch as he was little late. He then preferred to visit in the fifth batch. What will be the new pass -code issued to him? (a) and of are night the length equal day (b) and are of night the length equal day (c) and of are night the equal (d) and of are the night length day equal (e) None of these Ans: (a) 24. If pass-code for the second batch is 'to confidence hard you leads work and success', what will be the pass -code for the fourth batch? (a) leads success to you hard confidence and work (b) leads success you to hard confidence and work (c) leads success to you hard confidence work and (d) leads to success you hard confidence and work (e) None of these Ans: (a) 25. If the pass-code isued for the last (sixth) batch is 'and pencil by all boys used are pen'. What will be the pass-code for the first batch? (a) pencil and pen are used by all boys (b) pen and pencil used are by all boys (c) pen and pencil are used by all boys (d) pencil and pen are used all by boys (e) None of these Ans: (c) 26. If the pass-code for the sixth batch is 'not go the way to of out do'. What will be the pass-code for the third batch? (a) of do to out go not way the (b) of to do out not go way the (c) of to go out do not way the (d) Data inadequate (e) None of these Ans: (e) Directions(27-32): in each question below are given three statements followed by four conclusions-I, II, III and IV. You have to take the given statements to be true even if they seem to be at variance with commonly known facts. Read the all conclusions and then decide which of the given conclusions logically follow (s) from the given statements disregarding commonly known facts. 27.Statements Some books are pens All pens are chairs. Some chairs are Tables. Conclusions I. Some books are chairs. II . Some chairs are books. III. All tables are chairs. IV. Some tables are chairs. (a) All follow ( b) Only I, II, III follow (c) Only I, II, IV follow (d) Only II, III and IV follow (e) none of these Ans:(c) 28. Statements All cars are jeeps. All jeeps are buses. All buses are trucks. Conclusions I. All trucks are buses. II. All buses are jeeps. III. All jeeps are cars. IV. All cars are trucks. (a). None follows (b) All follow (c) Only III and IV follow (d) Only IV follows (e) None of these Ans:(d) 29.Statements Some trees are flowers. Some flowers are pencils. Some pencils are tables. Conclusions I. Some tables are flowers. II. Some pencils are trees. III. Some tables are trees. IV. Some trees are pencils. (a) All follow (b) None follows (c) Only I and III follow (d) Only II and IV follow (e) None of these Ans: (b) 30.Statements All rods are bricks. Some bricks are ropes. All ropes are doors. Conclusions I. Some rods are doors. II. Some doors are bricks. III. Some rods are not doors. IV. All doors are ropes. (a) Only I and II follow (b) Only I, II and III follow (c) Only Only either I or III and II follow (d) Only either I or III and IV follow. (e) None of these Ans.(c) 31. Statements Some books are pens. Some pens are watches. Some watches are books . Some books are watches. Conclusions I. Some radios are watches. II. Some radios are pens. III. Some watches are books IV. Some books are watches. (a) All follow (b) Only I and III follow (c) Only II and IV follow (d) Only I and IV follow (e) None of these Ans:(e) 32. Statements All towns are villages No village is forest Some forests are rivers. Conclusions I. Some forests are villages. II. Some forests are not villages. III.Some rivers are not villages. IV.All villages are towns. (a) All follow (b) Only either I or II follows (c) Only either I or II and III follow (d) None of these (e) None of these Ans: (e) 33. In a row of boys facing north, Sudhanshu is twelfth from his left. When shifted to his right by four places, he becomes eighteenth from the right end of the row? (a) 32 (b) 33 (c) 34 (d) Data inadequate (e) None of these Ans: (b) 34.In a certain code language PROBLEM is written as MPERLOB. How will NUMBERS be Written in that code? (a) SNUREMB (b) SNRUBME (c) SNRUEMB (d) SNRUMEB (e) None of these Ans:(c) Directions(35-40):In each question below is given a statement followed by two assumptions numbered I and II. An assumption is something supposed or taken for granted. You have to consider the statement and the following assumptions and decide which of the assumptions is implicit in the statement. Give Answer (a) if only assumption I is implicit. Give Answer (b) if only assumption II, is implicit. Give Answer (c) if either I or II is implicit. Give Answer (d) if neither I nor II is implicit. Give Answer (e) if both I and II are implicit. 35.Statement: The Government has recently hiked the prices of diesel and petrol to reduce the oil pool deficit. Assumptions: I. The amount earned by this increase may be substantial enough to reduce the deficit. II. There may be wide spread protests against the price hike. Ans:(e) 36. Statement: The X passenger car manufacturing company announced a sharp reduction in the prices of their luxury cars. Assumptions: I. There may be an increase in the sale of their luxury cars. II. The other, such car manufacturers may also reduce their prices. Ans:(e) 37. Statement : A foreign film producer rendered his apology before Indian society for misinterpreting a part of Indian epic. Assumptions : I. Indians are very sensitive to the misinterpretation of their epic. II. It is possible to derive wrong meaning from the epic. Ans:(d) 38.Statement : Aswin's mother instructed him to return home by train if it rains heavily. Assumptions I. Aswin may not be to decide himself if it rains heavily. II. The trains may ply even if it rains heavily. Ans:(b) 39. Statements : The Government of India decided to start a track II dialogue with its neighbour to reduce tension in the area. Assumption: I. The neighbouring country may agree to participate in the track II dialogue. II. The people involved in track II dialogue may be able to persuade their respective Governments. Ans:(e) 40. Statements :The host in one of the popular T.V programmes announced that the channel will contact the viewers between 9.00 a.m. to 6.00 p.m on weekdays and the lucky ones will be given fabulous prizes. Assumptions: I. The people may remain indoors to receive the phone call. II. More people may start watching the programme. Ans:(e) Directions(41-45): In making decisions about important questions, it is desirable to be able to distinguish between "Strong" arguments and "Weak" arguments are those which are both important and directly related to the question. "Weak" arguments are those which are of minor importance and also may not be directly related to the question or may be related to a trivial aspect of the questions. Instructions: Each question below is followed by a statement and two arguments numbered I & II. You have to decide which of the argument is a "Strong" argument and which is a "Weak" argument. Give answer (a) if only argument I is strong. Give answer (b) if only argument II is strong. Give answer (c) if their I or II is strong. Give answer (d) if neither I nor II is strong. Give answer (e) if both I and II are strong. 41.Statement: should the habit of late coming in educational institutions be checked? Arguments: I. No. Until it affects the work. II. Yes. Discipline must be maintained. Ans:(b) 42.Statement: Should seniority be the only criterion for the promotion? Arguments: I. No. All the senior employees are not interested in promotion. II. Yes. Otherwise senior employees do feel humiliated. Ans:(d) 43. Statement: Should children be prevented completely from watching television? Arguments: I. No. We get vital information regarding education through television. II. Yes. It hampers the study of children. Ans:(e) 44. Statement: Should trade unions be banned completely? Arguments: I. No. This is the only way through which employees can put their demands before management. II. Yes. Employees get their illegal demands fulfilled through these unions. Ans:(a) 45. Statement: should women be given equal opportunity in matter of employment in every field? Argument: I. Yes. They are equally capable. II. They have to shoulder household responsibilities too. Ans:(a) Directions(46-50): In each question below is given a statement followed by two conclusions numbered I and II.You have to assume everything in the statement to be true, then consider the two conclusions together and decide which of them logically follows beyond a reasonable doubt from the information given in the statement. Give answer (a) if only conclusion I follows. Give answer (b) if only conclusion II follows. Give answer (c) if either I or II follows. Give answer (d) if neither I nor II follows, and Give answer (e) if both I and II follow. 46. Statement: The cabinet of State 'X' took certain steps to tackle the milk glut in the state as the cooperatives and Government diaries failed to use the available milk-A news report Conclusion: I. The milk production of State 'X' is more than its need. II. The Government and co-operative diaries in State 'X' are not equipped in terms of resources and technology to handle such excess milk. Ans:(e) 47. Statement: It has been decided or the Government to withdraw 33% of the subsidy on cooking gas from the beginning of next month-A spokesman of the Government. Conclusions: I. People now no more desire of need such subsidy from Government as they can afford increased price of the cooking gas. II. The price of the cooking gas will increase at least by 33% from the next month. Ans:(d) 48. Statement: "The Government will review the present policy of the diesel price in view of further spurt in the international oil prices"-A spokesman of the Government. Conclusions: I. The Government will increase the price of the diesel after the imminent spurt in the international oil [prices. II. The Government will not increase the price of the diesel even after the imminent spurt in the international oil prices. Ans:(c) 49.Statement: My first and foremost task is to beautify this city-if city 'X' and Y can do it- why can't we do it-statement of Municipal Commissioner of City 'Z' after taking over charge. Conclusions: I. The people of city 'Z' are not aware about the present state of their city. II. The present commissioner has worked in city 'X' and Y and has good experience of beautifying cities. Ans:(d) 50.Statement: Women's Organisations in India have welcomed the amendment of the Industrial Employment Rules 1946 to curb sexual harassment at the work place. Conclusions: I. Sexual harassment of women at work place is more prevalent in India as compared to other developed countries. II. Many organisations in India will stop recruiting women to avoid such problems. Ans:(d) Directions (51-55): Each of the questions below consists of a question and two statements numbered I and II given below it. You have to decide whether the data provided in the statements numbered I and II given below it. You have to decide whether the data provided in the statements are sufficient to answer the questions. Read both the statements and - Give Answer (a) if the data in statement I alone are sufficient to answer the question, while the data in statement II alone are not sufficient to answer the question. Give answer (b) if the data in statement II alone are sufficient to answer the question, while, the data in statement I alone are not sufficient to answer the question. Give answer (c) if the data either in the statement I alone or in statement II alone are sufficient to answer the question. Give answer (d) if the data even in both statement I and II together are not sufficient to answer the question. Give answer (e) if the data in both statements I and II together are necessary to answer the question. 51. In which direction is Ravi facing? I. Asok is to the right of Ravi II. Samir is sitting opposite of Asok facing north. Ans:(d) 52. How M is related to I. The sister of K is the mother of N who is daughter of M . II. P is the sister of M. Ans: (e) 53. Is D brother of T? I. T is the sister of M and K. II. K is the brother of D. Ans: (d) 54. How many sons does P have? I. S and T are brothers of M. II. The mother of T is P who has Only One daughter. Ans: (e) 55. Who is the tallest among P, Q, R, S and T? I. R is taller than Q and T. II. T is taller than S and P and S is taller than Q and R. Ans: (b) Directions (56-60) : Below is given a passage followed by several possible inferences which can be drawn from the fact stated in the passage. You have to examine each inference separately in the context of the passage and decide up on its degree of truth or falsity. Mark answer (a) if the inference is "definitely true" i.e., it properly follows from the statement of facts given. Mark answer (b) if the inference is "probably true" though not "definitely true" in the light of the facts given. Mark answer (c) if the data are inadequate i.e, from the facts given you cannot say whether the inference is likely to be true or false. Mark answer (d) if the inference is "probably false" in the light of the facts given. Mark answer (e) if the inference is "definitely false" i.e, it cannot possibly be drawn from the facts given or it contradicts the given facts. With the purpose of upliftment of Gonda district in Uttar Pradesh, a new formula way evolved for practical success in several fields, such as, irrigation, animal husbandry, dairy farming, moral uplift and creation of financial resources. Small farms were clustered for irrigation by one diesel pump which could irrigate about 20 acres of land. Youth were prompted to take loans from the banks for purchase of engine pumps to be supplied to the farmers on rent. This formula worked so well that the villages in Gonda district were saturated with irrigation facilities. Cattle rearing was linked with multiple cropping . Most of the targets fixed for different areas were achieved, which was an unusual phenomenon. This could be possible only because of right motivation, participation and initiative of the people. Imagination and creativity combined together helped in finding out workable solutions to the problems of the community. 56. There was no problem and complaint of people residing in entire Gonda district, before the beginning of the project. Ans: (e) 57. Purchasing of engine pumps by individual farmers may be beyond their affordable limits. Ans: (a) 58. Earlier farming was not basically one of the professions of people in Gonda district. Ans: (e) 59.By using the same formula upliftment of any other district is possible. Ans: (a) 60.There are very few people who can motivate others in the right direction. Ans: (c) Quantitative Aptitude 61. The difference between a two digit number and the number obtained by interchanging the positions of its digits is 36. What is the difference between the two digits of that number? (a) 4 (b) 9 (c) 3 (d) Cannot be determined (e) None of these Ans: (a) 62. By how much is two-fifth of 200 greater than three-fifth of 125? Technical - Other SBI Reasoning ability solved question papers SBI stands for State Bank of India, SBI Central Recruitment and promotion of Department recruit clerical cadre in Associate Banks of State Bank of India SBI latest bank profile, SBI, free solved sample placement papers of clerk, SBI Associate Banks Clerks Recruitment 2012 notification and job details, SBI Dates of Written Examination : 07.10.2012 and 14.10.2012,SBI previous years solved question papers, SBI 2010,2011,2012 Clerks and po officer job written test examination syllabus and selection procedure, SBI PO and Associate banks clerks examination questions papers, SBIAssociate Banks Clerks Recruitment 2012 notification and detailed explanation with answers and solutions, How to crack Sbi written test examination tips and trick,SBI, General Awareness, General English, Quantitative Aptitude, Reasoning Ability, Marketing Aptitude / Computer Knowledge whole solved questions paper, Latest general awareness SBI Associate Bank clerical cadre question papers and selection procedure and test pattern 1. How many options does a binary choice offer ? (A) None (B) One (C) Two (D) It depends on the amount of memory in the computer (E) It depends on the speed of the computer’s processor 2. Data going into the computer is called— (A) Output (B) Algorithm (C) Input (D) Calculations (E) Flowchart 3. How many values can be represented by a single byte ? (A) 4 (B) 16 (C) 64 (D) 256 (E) 512 4. Transformation of input into output is performed by— (A) Peripherals (B) Memory (C) Storage (D) The Input-Output unit (E) The CPU 5. Device drivers are— (A) Tiny power cords for external storage devices (B) Experts who know how to maximize the performance of devices (C) Small, special-purpose programs (D) The innermost part of the operating system (E) Substitutes for operating system 6. A collection of programs that controls how your computer system runs and processes information called— (A) Operating system (B) Computer (C) Office (D) Compiler (E) Interpreter 7. Which of the following refers to a small, single-site network ? (A) LAN (B) DSL (C) RAM (D) USB (E) CPU 8. A set of instructions telling the computer what to do is called— (A) Mentor(B) Instructor (C) Compiler(D) Program (E) Debugger 9. If you receive an e-mail from someone you don’t know, what should you do ? (A) Forward it to the police immediately (B) Delete it without opening it (C) Open it and respond to them saying you don’t know them (D) Reply and ask them for their personal information (E) Reply and tell them you want to keep in touch with them 10. Which of the following can handle most system functions that aren’t handled directly by the operating system ? (A) Vertical-market applications (B) Utilities (C) Algorithms (D) Integrated software (E) Compilers 11. Microsoft Office is— (A) Shareware (B) Public-domain software (C) Open-source software (D) A vertical-market application (E) An application suite 12. Computers connected to a LAN (Local Area Network) can— (A) Run faster (B) Go on line (C) Share information and/or share peripheral equipment (D) E-mail (E) None of these 13. Which of the following refers to the memory in your computer ? (A) RAM (B) DSL (C) USB (D) LAN (E) CPU 14. Information travels between components on the motherboard through— (A) Flash memory (B) CMOS (C) Bays (D) Buses (E) Peripherals 15. One megabyte equals approximately— (A) 1,000 bits (B) 1,000 bytes (C) 1 million bytes (D) 1 million bits (E) 2,000 bytes 16. When you are working on a document on a PC, where is the document temporarily stored ? (A) RAM (B) ROM (C) The CPU (D) Flash memory (E) The CD-ROM 17. How are data organized in a spreadsheet ? (A) Lines and spaces (B) Layers and planes (C) Height and width (D) Rows and columns (E) None of these 18. Magnetic tape is not practical for applications where data must be quickly recalled because tape (A) A random-access medium (B) A sequential-access medium (C) A read-only medium (D) Fragile and easily damaged (E) An expensive storage medium 19. The blinking symbol on the computer screen is called the— (A) Mouse (B) Logo (C) Hand (D) Palm (E) Cursor 20. When cutting and pasting, the item cut is temporarily stored in— (A) ROM (B) Hard drive (C) Diskette (D) Dashboard (E) Clipboard Answers: 1.(C) 2. (C) 3. (E) 4. (E)5.(D) 6. (B) 7. (A) 8. (D)9.(B) 10.(A) 11. (E) 12. (C)13.(A) 14. (D) 15. (C) 16. (A)17.(D) 18. (E) 19. (E) 20. (E) Technical - Other SBI stands for State Bank of India, SBI latest bank profile, SBI,free solved sample placement papers of clerk, SBI Associate Banks Clerks Recruitment 2012 notification and job details, SBI previous years solved question papers, SBI 2010,2011,2012 Clerks and po officer job written test examination syllabus and selection procedure, SBI PO and Associate banks clerks examination questions papers, SBIAssociate Banks Clerks Recruitment 2012 notification and detailed explanation with answers and solutions, How to crack Sbi written test examination tips and trick, SBI, General Awareness, General English, Quantitative Aptitude, Reasoning Ability, Marketing Aptitude / Computer Knowledge whole solved questions paper, Latest general awareness Practice test for for Bank PO, Bank Clerk, MBA 1. A ____ is a named location on a disk where files are stored. A folder B pod C version D none of the above Ans (A) 2. The ____ allows you to choose where to go and is located below the Standards Buttons toolbar. A System menu B Address bar C Menu bar D none of the above Ans (B) 3. A ____ is a set of computer instructions that carry out a task on the computer. A program B database C memory file D none of the above Ans (A) 4. An operating system version designed for home use is Microsoft Windows XP ____. A Home Edition B Media Center Edition C Tablet PC Edition D none of the above Ans (A) 5. A user-interface that is easy to use is considered to be ____. A user-happy B user-simple C user-friendly D none of the above Ans (C) 6. When you press and release the secondary mouse button you are ____. A Right-clicking B Left-clicking C either a. or b. D neither a. nor b. Ans (A) 7. Another term for hyperlink is ____. A link B source C bar D none of the above Ans (A) 8. A ____ is a flash memory storage device that plugs into a USB port. A USB snap drive B USB flash drive C USB memory maker drive D none of the above Ans (B) 9. An operating system version designed for use with a Media Center PC is Microsoft Windows XP ____. A Home Edition B Media Center Edition C Tablet PC Edition D none of the above Ans (B) 10. A graphical user interface displays ____. A graphics B text C both (A) and (B). D neither (A) nor (B). Ans (C) 11. When you quickly press and release the left mouse button twice, you are ____. A Primary-clicking B Double-clicking C pointing D none of the above Ans(B) 12. Underlined text, such as text and folder names is referred to as a ____. A hyperlink B menu C source drive D none of the above Ans (A) 13. The ____ program compresses larger files into a smaller file. A WinZip B WinShrink C WinStyle D none of the above Ans (A) 14. An operating system version designed for use with a tablet PC is Microsoft Windows XP ____. A Home Edition B Media Center Edition C Tablet PC Edition D none of the above Ans (C) 15. The ____ displays the name of every computer user on the computer. A Wish list screen B Command screen C Welcome screen D none of the above Ans (C) 16. The ____ contains commands associated with the My computer window. A Standards menu B Start menu C System menu D none of the above Ans (C) 17. Press the ____ button to have the window fill the entire screen. A Close B Maximize C Minimize D none of the above Ans (B) 18. The ____ is the drive containing the files to be copied. A source drive B destination drive C USB drive D none of the above Ans (A) 19. A ____ allows you to write on screen with a digital pen and convert that writing into characters that the PC can process. A monitor RS B tablet PC C database manager D media center Ans (B) 20. A ____ is a pointing device. A monitor B mouse C keyboard D none of the above Ans (B) SBI Question Papers,SBI previous years solved question papers,SBI Aptitude Reasoning questions with answers and detailed explnations Question 1 In Agency agreement, a) the Principal should be a major b) the Agent should be a major c) anyone can be a minor d) both should be major Answer : a) the Principal should be a major Question 2 In Sale of Goods Act “quiet possession” means a) Buyer will get protection b) Seller will not make noise c) Buyer will remain quiet d) Buyer will possess and enjoy. Answer : d) Buyer will possess and enjoy. Question 3 The income earned on a Debenture is a) dividend b) interest c) discount d) incentive Answer : a) dividend Question 4 The bank receives the following four cheques through clearing. Which one of them is deemed to be a Crossed Cheque? a) On the top of the cheque the words, “Crossed Cheque” are written without any other marking. b) Two parallel transverse lines are drawn on the face of the cheque. c) On the back of the cheque two parallel transverse lines are drawn. d) A big “X” mark is made on the top of the cheque without any other marks. Answer : b) Two parallel transverse lines are drawn on the face of the cheque. Questions 5 Among the options which is open source software ? a) Java b)windows c) unix d) linux Answer : d) Linux Question 6 Spreadsheet is used for ________ a) To perform numerical and statistical calculation b) To maintain records c) To create documents d) All the above Answer : a) To perform numerical and statistical calculation Question 7 Vanya Mishra was awarded in 2012 with __________. a) Kalpana Chawla award b) Arjuna award c) C.V.Raman award d) J.C.Bose award Answer : a) Kalpana Chawla award Question 8 Orhan Pamuk wrote the book _________ a) The Famous Five b) The Enchanted Wood c) Snow d) The Jungle Book Answer : c) Snow Question 9 The author of the children book "Where the Wild things are" is _________ a) Rudyard Kipling b) Maurice Sendak c) George Orwell d) Charles Perrault Answer : b) Maurice Sendak Question 10 Mario Vargas Llosa got Nobel prize in Literature for the novel ________ . a) The Dream of the Celt b) The Feast of the Goat c) Death in the Andes d) The Time of Hero Answer : a) The Dream of the Celt Question 11 The number of medals won by India in Olympics 2012 is _____ a) 2 b) 5 c) 6 d) 1 Answer : c) 6 Question 12 Which of the following London Olympic medalist has been honoured as “Rajiv Gandhi Khel Ratna” ? a) Vijay Kumar b) Sushil Kumar c) Yogeshwar Dutt d) Saina Answer : a) Vijay Kumar , c) Yogeshwar Dutt Question 13 In London Olympics 2012, the player qualified for wrestling is __________. a) Abhinav Bindra b) Geeta Poghat c) Gagan Narang d) Aslam Sher Khan Answer : b) Geeta Poghat Question 14 Rajyavardhan Singh Rathore was head of ___________ committee. a) Arjuna Award b) Dronacharya Award c) Rajiv Gandhi Khel Ratna Award d) Dhyan Chand Award Answer : c) Rajiv Gandhi Khel Ratna Award Question 15 Bala buys an equal number of pens, pencils, sketch pens of Rs.2.50, Rs.3, Rs.5 respectively. How many of each did he buy if he spends Rs.168 ? a)12 b)16 c)18 d)20 Answer: b)16 Solution: Let us assume that he buy X number of pens, pencils and sketch pens. Then 2.50 x X + 3 x X + 5 x X = 168 10.50X = 168 X = 168 / 10.5 = 16 Hence he buys 16 numbers of pencils, pens, sketch pens respectively. Question 16 Prabha and Aarthi paid a total of Rs.600 for some good. If Prabha paid 120 more than that of Aarthi, then how much is already paid? a)Rs.250 b)Rs.150 c)RS.550 d)Rs.240 Answer : d)Rs.240 Solution : Let the amount paid by Aarthi be Rs.X Then, the amount paid by Prabha = Rs. X + 120 The total amount paid = Rs.600 = X + X + 120 = 2X + 120 = X = 240 Thus the amount paid by Aarthi is Rs.240 Question 17 A fruit seller had some apples. He sells 30% apples at the rate of Rs.30 per apple and the remaining 315 apples at the rate of Rs.20 per apple. The total selling price is : a)Rs.15900 b)Rs.10350 c)RS.11550 d)Rs.12420 Answer :b)Rs.10350 Solution : Let the total number of apples be X. Then X = 30X / 100 + 315 70X / 100 = 315 X=3150 / 7 = 450 Now 30% of X = 30 x 450 / 100 = 135 He sells 135 apples at the rate of Rs.30 per apple and 315 at the rate of Rs.20 per apple Then the total selling price = 135 x 30 + 315 x 20 = 4050 + 6300 =Rs.10350 Hence the answer is Rs.10350 Question 18 Devi buys some material worth Rs.X.She gets a deduction of Rs.650 on it. After getting deduction she pays tax of Rs.169. Then find X the value if she pays Rs.6000 for that material. a)Rs.6481 b)Rs.5689 c)RS.6819 d)Rs.5181 Answer :Rs.6481 Solution : Let the total amount be Rs.X After getting deduction the amount will be X - 650 And after paying the tax Rs.169, Rs.6000 = X - 650 + 169 = X - 481. X =6481 Hence she buys material worth Rs.6481 Question 19 If two-third of two-seventh of a number is 60, then two-fifteenth of that number is a)60 b)42 c)315 d)21 Answer : b)42 Solution : Let the number be X. Then 2/3 x 2/7 x X =60 X = 60 x 7 x 3 / 4 = 315 Then, 2/15 of 315 = 2/15 x 315 = 42. Hence the required number is 42 Question 20 Two times of the third of three consecutive even integers is three times of the first. Then the second integer is a)60 b)42 c)10 d)12 Answer : c)10 Solution : Let the consecutive three integers be X, X + 2 and X + 4 Then, 2(X + 4) = 3X X = 8 Now X + 2 = 10 The required second integer is 10 Question 21 Find the integer which when decreased by 2 is 80 times the reciprocal of the same number a)60 b)42 c)10 d)12 Answer : c)10 Solution : Let the number be X Decreasing 2 from X X - 2 = 80 x (1/X) X^2 - 2X = 80 X^2 -2x -80 = 0 (X-10)(X+8) = 0 x=10,-8 X has two values, but among the given options X value should be 10 Questions 22 . Four of the following five are alike in a certain way and so form a group. Which is the one that does not belong to the group? (1) 115 (2) 145 (3) 95 (4) 155 (5) 75 (ans) Solutions : Only 75 is divisible by 3. Questions 23. How many meaningful words can be made from the letters ADEL using each letter only once? (1) None (2) One (3) Two (4) Three (ans) (5) More than three Solutions : Three meaningful words DEAL, LEAD, LADE Questions 24 How many such pairs of letters are there in the word "ADVERTISE', each of which has as many letters between them in the word, as they have in the English alphabet ? (1) None (2) One (ans) (3) Two (4) Three (5) More than three Questions 25 63.5% of 8924.2 + ?% of 5324.4 = 6827.5862 (1) 36 (2) 22 (Ans) (3) 17 (4) 31 (5) 9 Ans : 63.5/100 of 8924.2 + ?/100 of 5324.4 = 6827.5862 8924 x 64/100 + 5325 x ?/100 = 6828 ? 5711 + 5325 x ? / 100 = 6828 ? ? = (6828 - 5711) x 100 5325 = 111700 5325 = 22 (approx) 9546324 ÷ 4584 = ? (1) 2149 (2) 1986 (3) 2083 (Ans) (4) 2247 (5) 1805 Ans : ? = 9546324 ÷ 4584 = 2082.53 = 2083 (approx) Questions 26 4 men, 5 women and 3 children together can complete a piece of work in 16 days. In how many days can 10 women alone complete the piece of work in 16 days. In how many days can 10 women alone complete the piece of work if 10 men alone complete the work in 24 days ? (1) 18 (2) 15 (3) 12 (4) Cannot be determined (Ans) (5) None of the above Ans : Cannot be determined Questions 27 In how many different ways can the letters of the word 'FLEECED' be arranged ? (1) 840 (Ans) (2) 2520 (3) 1680 (4) 49 (5) None of these Ans : Required number = 7! = 7 x 6 x 5 x 4 x 3 ! 3! 3! = 7 x 6 x 5 x 4 = 840 Question 28 Arul was going to market from his home. He walked 4 Km to the north, turned right and walked 3 Km. He then turned left and walked 4 Km. Then he took a left and walked till he reached the market which was exactly to the east of his house. Find the distance of the market from his home along north. Answer : 8km Question 29 Starting from his house, Rahul walked 5 Km to the north east to reach a shop. From there he took a right and walked another 5 Km to reach his friends house. How far is his friend's house from his house (along eastern direction)? Answer : sqrt(50) Question 30 Once economically sound city introduced a series of reform measures to _____ the economy. a. improve b. revive c. hurt d. encourage Answer : b. Revive Explanation : Reviving economy means to recover economy to a (good) state as it was before. Question 31 Wounds go away but deep _____ may turn permanent. a. Worries b. Pain c. Cuts d. Scars Answer : d. Scars Explanation : Worries, Pain and Cuts often heal with time while deep scars can be permanent. Some may argue that worries and pain may not go away in some cases. But looking at the context of the sentence we can see that it starts with a reference to 'Wounds'. Hence adding scars to the next half of the sentence seems appropriate. Question 32 ______ packets of a popular shampoo are being distributed free to people in this apartment as a part of advertisement/promotion exercise. a. Large b. Sample c. Fortified d. Extra Answer : b. Sample Explanation : Though Large, Fortified (strengthened by some ingredient), Extra seem correct by meaning, but 'Sample' fits the blank well. Usually, many companies distribute sample packets (in small quantities) as part of advertisements/promotions. Question 33 A train of length 150 meters can cross a bridge in 40 seconds when travelling at a speed of 40km/hr. Then what is the length of the bridge? a)180m b)182 c)183 d)185 Answer : c)183 Solution: Let the length of the bridge be X. The speed of the train = 40km/hr = 40 x 5/18 m/sec = 100/9 m/sec. The time taken to cross the bridge = 40 seconds. Then, [150 + X ] / 30 = 100/9 150 + X = 3000 /9 X = 3000/9 - 150 = 1650/9 = 183.33 Hence the answer is 183 m. Question 34 A train crosses a bridge and a bike standing on the bridge in 40 seconds, 25 seconds respectively. What is the length of the bridge if the speed of the train is 50.4km/hr? a)180m b)210 c)183 d)185 Answer : b)210 Solution: Given that, The speed of the train = 50.4km/hr = 50.4 x 5/18 m/sec = 14 m/sec The train crosses a bike(standing object) in 25 seconds.Then, Length of the train = (14 x 25)m = 350 m. Now, let the length of the bridge is X m. the train crosses the bridge in 40 seconds. Then, (X + 350)/40 = 14 X + 350 = 14 x 40 = 560 X = 210 Hence the bridge is 210m long. Question 35 In what time a train 120 meters long travelling at a speed of 70km/hr crosses a cyclist who is at the speed 5km/hr in the direction opposite to the train? a)4.76sec b)5.76sec c)8.92sec d)6.14sec Answer : b)5.76sec Solution: Given that the speed of the train and a cyclist is 70km/hr and 5km/hr respectively. Then, the Speed of train relative to cyclist = (70 + 5) km/hr = 75 km/hr = 75 x 5/18 = 125/6 m/sec. The time taken to cross the cyclist = 120 / (125/6) = 120 x 6/125 =5.76 sec hence the answer is 5.76 seconds. Placement Paper SBI Professional Knowledge Model Questions for technical officers recruitment, SBI specialists officers model questions with answers, SBI specialists officers professional knowledge practice questions with answers 1. Which is Computer Memory that does not forget ? Ans: ROM 2. The computer memory holds data and ? Ans: program 3. What is means by term RAM ? Ans: Memory which can be both read and written to 4. Which computer memory is esentially empty ? Ans: RAM 5. The bubbles in a bubble memory pack are created with the help of ? Ans: magnetic field 6. Virtual memory is - Ans: an illusion of an extremely large memory 7. Special locality refers to the problem that once a location is referenced Ans: a nearby location will be referenced soon 8. An example of a SPOOLED device Ans: A line printer used to print the output of a number of jobs 9. Page faults occurs when Ans: one tries to divide a number by 0 10. Overlay is Ans: a single contiguous memory that was used in the olden days for running large programs by swapping Operating System Question Answer 11. Concurrent processes are processes that - Ans: Overlap in time 12. The page replacement policy that sometimes leads to more page faults when the size of the memory is increased is - Ans: FIFO 13. The only state transition that is initiated by the user process itself is - Ans: Block 14. Fragmentation is - Ans: fragments of memory words unused in a page 15. Give Example of real time systems Ans: Aircraft control system, A process control system 16. Dijkstra’s banking algorithm in an operating system solves the problem of - Ans: Deadlock Avoidance 17. In a paged memory system, if the page size is increased, then the internal fragmentation generally - Ans: Becomes more 18. An operating system contains 3 user processes each requiring 2 units of resources R. The minimum number of units of R such that no deadlock will ever occur is - Ans: 4 19. Critical region is - Ans: A set of instructions that access common shared resources which exclude one another in time 20. Kernel is - Ans: The set of primitive functions upon which the rest of operating system functions are built up 21. Necessary conditions for deadlock are - Ans: Non-preemption and circular wait, Mutual exclusion and partial allocation 22. In a time sharing operating system, when the time slot given to a process is completed, the process goes from the RUNNING state to the - Ans: READY state 23. Supervisor call - Ans: Are privileged calls that are used to perform resource management functions, which are controlled by the operating system 24. Semaphores are used to solve the problem of - Ans: Mutual exclusion, Process synchronization 25. If the property of locality of reference is well pronounced in a program- Ans: The number of page faults will be less Operating System Question Answer for Competitive Exams 26. Pre-emptive scheduling, is the strategy of temporarily suspending a running process- Ans: before the CPU time slice expires 27. Mutual exclusion problem occurs - Ans: among processes that share resources 27. Sector interleaving in disks is done by - Ans: the operating system 28. Disk scheduling involves deciding- Ans: the order in which disk access requests must be serviced 29. Dirty bit is used to show the - Ans: page that is modified after being loaded into cache memory 30. Fence register is used for- Ans: memory protection. 31. The first-fit, best-fit and worst-fit algorithm can be used for- Ans: contiguous allocation of memory 32. Give example of single-user operating systems- Ans: MS-DOS, XENIX 33. In Round Robin CPU Scheduling, as the time quantum is increased, the average turn around time- Ans: varies irregulary 34. In a multiprogramming environment- Ans: more than one process resides in the memory 35. The size of the virtual memory depends on the size of the - Ans: Address Bus 36. Give example of Scheduling Policies in which context switching never take place- Ans: Shortest Job First, First-cum-first-served 37. Suppose that a process is in ‘BLOCKED’ state waiting for some I/O service. When the service is completed, it goes to the- Ans: READY State 38. One of the following is not included in the 7 P’s of Marketing. Find the same_____________ (A). Placement (B). Price (C). Production (D). Promotion (E). Product Ans. (C). Production 39 Analysis of marketing problem helps in ______________ (A). Evaluating marketing opportunities (B). Reducing marketing staff (C). Reducing profits (E). Motivation Ans. (A). Evaluation Marketing Opportunities 40. Innovation is marketing is same as _________________ (A). Motivation (B). Perspiration (C). Aspiration (D). Creativity (E). Team work Ans. (D). Creativity 41.Market Segmentation is required for _________________ (A). preferential market (B). OTC Marketing (C). Internal Marketing (D). Identifying sales persons (E). Identifying prospects Ans. (E). Identifying prospects 42 Bank ATMs are__________________ (A). Delivery outlets (B). market plans (C). Personalized Products (D). Tools for overcoming buyers resistance (E). Motivating tools Ans. (C). Personalized Products 43. EMI can be marketing tool when________________ a. EMI is very low b. EMI is very high c. EMI is fluctuating d. EMI is constant e. EMI is ballooning 44. Advertisements are necessary for ______________ a. only old products b. launching new products c. only costly products d. only obsolete products e. advertisements are wasteful expenses 45. Publicity is required for ____________________ a. Generating more number of leads b. better training of sales persons c. market survey d. Product designing e. OTC Marketing 46. NAV is the price of __________________ a. Entire fund value b. one unit of a fund c. surrender value d. average value of shares e. dividends paid in a year 47.A Master policy in the case of Life insurance indicates___________________ a. policy is sale b. policy is in the name of servant c. only one life is assured d. there are several beneficiaries e. life assured should be a male 48. Customer database is useful for ________ a. advertisements b. word-of-mouth publicity c. CRM Functions d. PR functions e. Sales persons training 49. CRM(Customer Relationship Management) is ______________ a. A pre-sales activity b. A tool for lead generation c. an ongoing daily activity d. the task of a DSA e. back office duty 50 Find the incorrect answer_________________ a. cross-selling is an expensive way of marketing b. market segmentation can boost lead generations c. customer lifetime value is a marketing tool d. surrogate marketing is a type of viral marketing e. internet banking can replace ATMs 51. Financial inclusion needs canvassing the accounts of ______________ a. Financial Institutions b. NRIs c.HNIs d. Housewives e. Persons below a specified income level 52. Effective retail banking presupposes____________ a. Large Premises b. Huge kiosks c. big sales force d. coordination between marketing and front office staff e. More products SBI and SBT associate banks English,Verbal ability General Awareness,marketing aptitude questions for practice,SBT and SBI previous years solved question papers,SBT and SBI solved question papers of aptitude,data intrepretation,data analysis,reasoning questions with answers,SBT and SBI whole questions for practice SBI Associates Bank P.O Exams Directions (1 -15): Read the following passage carefully and answer the questions given below it. Certain words/expressions are given in bold in the passage to help you locate them while answering some of the questions. Radically changing monsoon patterns, reduction in the winter rice harvest and a quantum increase in respiratory diseasesall part of the environmental doomsday scenario which is reportedly playing out in South Asia. According to a United Nations Environment Programme report, a deadly threekm deep blanket of pollution comprising a fearsome cocktail of ash. acids, aerosols and other particles has enveloped this region. For India, already struggling to cope with a drought^the implications of this are devastating and further crop failure will amount to a life and death question for many Indians. The increase in premature deaths will have adverse social and economic consequences and a rise in morbidities will place an unbearable burden on our crumbling health system. And there is no one to blame but ourselves. Both official and corporate India has always been allergic to any mention of clean technology. Most mechanical two wheelers roll off the assembly line without proper pollution control system. little effort is made for R & D on simple technologies, which could make a vital difference to people's lives and the en-vironment . However, while there is no denying that South Asia must clean up its act, skeptics might question the timing of the haze report. The Kyoto meet on climate change is just two weeks away and the Stage is set for the usual battle between the developing world and the West, particularly the U.S. President Mr. Bush has adamantly refused to sign any protocol, which would mean a change in American consumption level. U.N. environment report will likely find a place in the U.S. arsenal as it plants an accusing finger towards controls like India and China. Yet the U.S. can hardly deny its own dubious role in the matter of erasing trading quotas. Richer countries can simply buy up excess credits from poorer countries and continue to pollute. Rather than try to get the better of developing countries, who undoubtedly have taken up environmental shortcuts in their bid to catch up with the West, the U.S. should take a look at the environmental profligacy, which is going on within. From opening up virgin territories for oil exploration to relaxing the standards for drinking water, Mr. Bush's policies are not exactly beneficial, not even to America's interests. We realize that we are all in this together and that pollution anywhere should be a global concern otherwise there will only be more tunnels at the end of the tunnel. 1. Both official and corporate India is allergic to: (1) failure of monsoon (2) poverty and inequality (3) slowdown in industrial production (4) mention of clean technology (Ans) (5) crop failure 2. Which, according to the passage. Is a life and death question to many Indians? (1) Increase in respiratory diseases (2) Use of clean technology (3) Thick blanket of pollution over the region (4) Failure in crops (Ans) (5) Dwindling agricultural yield 3. If the rate of premature deaths increases it will: (1) exert added burden on our crumbling economy (2) have adverse social and economic consequences (Ans) (3) make positive effect on our efforts to control population (4) have less job aspirants in the ' society (5) have healthy effect on our economy 4. Choose the word which is SIMILAR in meaning to the word 'profligacy' as used in the passage. (1) wastefulness (Ans) (2) conservation (3) upliftmeht (4) criticality (5) denouncement Explanations : The meaning of the word Profligacy (Noun) is using money, time, materials etc in a careless way; wastefulness. Hence, profligacy and wastefulness are synonymous, 5. According to the passage, India cannot tolerate any further: (1) crop failure (Ans) (2) deterioration of health care system (3) increase in respiratory diseases (4) proliferation of nuclear devices (5) social and economic consequences 6. According to the passage, two wheeler industry is not adequately concerned about (1) passenger safety on the roads (2) life cover insurance of the vehicle owners (3) pollution control system In the vehicles (Ans) (4) rising cost of the two wheelers (5) rising cost of the petrol in the country 7. What could be the reason behind timing of the haze report just before the Kyoto meet, as indicated in the passage ? (1) United Nations is working handinglove with U.S. (2) Organizers of the forthcoming meet to teach a lesson to the U.S. (3) Drawing attentions of the world towards devastating effects of environment degradation. (4) U.S. wants to use it as a handle against the developing countries in the forthcoming meet (Ans) (5) The meet is a part of political agenda of the U.N. 8. Choose the word which is SIMILAR in meaning to the word 'allergic' as used in the passage. (1) liking (2) passionate (3) possessive (4) crumbling (5) repugnant Explanations : The word Allergic (Adjective) means having an allergy to some thing: having strong dislike of something/ somebody; a medical condition that causes you to react badly or feel ill or sick when you eat or touch a particular substance. Look at the sentence : I like cats but unfortunately I am allergic to them. You could see he was allergic to housework. Out of the given alternatives repugnant (Adjective) means making you feel strong dislike or disgust. Look at the sentence : The idea of eating meat was repugnant to her. We found his suggestion absolutely repugnant. Hence, repugnant and allergic are synonymous. 9. Which of the following is the indication of environmental degradation in South Asia ? (1) Social and economic inequality (2) Crumbling health care system (3) Inadequate pollution control system (4) Overemphasis on technology (5) Radically changing monsoon pattern (Ans) 10. What must we realize, according to the passage ? (1) No country should show superiority over other countries (2) U.N. is putting in hard efforts in the direction of pollution control (3) All countries must join hands in fighting pollution (Ans) (4) Nobody should travel through tunnel to avoid health hazards (5) We all must strive hard to increase agricultural production 11. Which of the following finds place in the United Nations Environment Programme Report? (1) Changing monsoon patterns (2) Substantial increase in respiratory diseases (3) A serious cover of pollution over the region (Ans) (4) Reduction in winter rice harvest (5) None of these 12. Which of the following statements is NOT TRUE in the context of the passage ? (1) U.N. environment report blames countries like India and China (2) Developing countries have taken environment shortcuts in their bid to catch up with the West. (3) U.S. is also to be blamed for environmental degradation and pollution (4) Indians cannot afford to have any further crop failure (5) U.S. has tightened safety standards for drinking water (Ans) 13. According to the passage, Kyoto meet is going to witness (1) calm and dispassionate thinking on the issue of pollution control (2) a blaming game between developed and developing countries (Ans) (3) refusal of U.N. to work as arbitrator (4) U.S. agreeing to look at the issue of lowering its consumption (5) countries agreeing for higher monetary allocation to R & D. 14. Choose the word which is MOST OPPOSITE in meaning to the word 'dubious' as used in the passage ? (1) unquestionable (Ans) (2) dissimilar (3) illegal (4) antisocial (5) innovative Explanations : The word Dubious (Adjective) means not certain and slightly suspiclous about something: suspiclous, doubtful. Look at the sentence : I was rather dubious about the whole idea. They indulged in some highly dubious business practices to obtain their current position in the market. Hence, dubious and unquestionable are antonymous. 15. Choose the word which is the MOST OPPOSITE in meaning to the word 'morbidity' as used in the passage. (1) powerfulness (2) healthiness (Ans) (3) softness (4) acuteness (5) purposeful Explanations : The word Morbidity (Noun) means a strong interest in sad or unpleasant things, especially disease or death. Hence, morbidity and healthiness are antonymous. Directions (1625): Which of the phrases (1), (2), (3) and (4) given below should replace the phrase given in bold in the following sentence to make the sentence grammatically cor; rect. if the sentence is correct as it is and 'No correction is required', mark (5) as the answer. 16. The executive had received several warnings before been sus¬pended finally for his lack of punctuality. (1) after suspension (2) after suspending (3) before suspended (4) before being suspended (Ans) (5) No correction required Explanations : .......'Before being suspended' (Participle) should be used in place of 'before been suspended'. Here, 'been' has been wrongly used. 17. Accordingly to the senior partner's instructions they have remitted the amount to your bankers. (1) According on (2) On accord of (3) In accordance with (Ans) (4) Accordingly as (5) No correction required Explanations : We should use 'according to' or 'in accordance' with in place of 'according to' 'According to' means as stated or reported by somebody/something: following, agreeing with or depending on something Look at its use in the sentence: You have been six times according to out records. The salary will be fixed according to qualifications and experience 18. The museum has planned for a ten day exhibition showcasing the ' rich culture of the South. (1) plan for (2) has been planning (3) planning on (4) have a plan (5) No correction required (Ans) 19. Since the collapse of his business he has become frequent depressed and addicted to alcohol. (1) frequent depression and addicted for (2) frequently depress and addict to (3) frequently depressing and addicted on (4) frequently depressed and addicted .to (Ans) (5) No correction required Explanations : The word 'frequent' is an advective where as depressed is a Verb. Hence, the word 'frequently' (Adverb) should be used. 20. The government will refrain from intervening in the dispute except the company requests it to do so. (1) with the dispute except (2) in the dispute unless (Ans) (3) to the dispute excepting (4) in the dispute without (5) No correction required Explanations : .........In the dispute unless' should be used in place of in the dispute except' as the sense of sentence is conditional. 21. The issue of employee pension schemes will come to the Governing Board meeting next week. (1) shall come about in (2) will come before (3) will come up at (4> shall come to (Ans) (5) No correction required Explanations : '....... will come up at' should be used. 22. The steep rise in oil prices is the reason on account of which we must conserve energy. (1) the reason (Ans) (2) the reason for (3) the reason because (4) the reason to (5) No correction required Explanations : The words 'reason' and 'on account of bear the some meaning. Hence, both can be used independently. not together. 23. Several customers have requested that the branch timings on weekdays should be changed to reduce inconvenience. (1) is changed (2) have changed (3) shall change (4) can change (5) No correction required (Ans) 24. Having failed to plan their political campaign in advance the party members got each other into a mess. (1) get one another (2) got themselves (Ans) (3) have got anyone (4) has got everyone (5) No correction required Explanations : It is to be noted that 'each other' is used between two persons. Hence, got themselves' should be used in place of 'got each other'. 25. Despite his youth he has the reputation of being one of the most efficient administrators'in the organization. (1) from the efficient (2) off the more efficient (3) of the efficient in (4) among the most efficiently of (5) No correction required (Ans) Directions (26 - 30) : Read each sentence to find out whether there is any grammatical error in it. The error if any, will be in one part of the sentence, the number of that part is the answer. If there is no error, mark (5). (Ignore errors of punctuation, if any) 26. Villagers want to build (l)/a bridge crossed the river (2) /to connect their village (3) /to the highway. (4)/ No error (5) Explanations : The use of the word 'crossed' is incorrect. Hence Use 'across in place of 'crossed'. (Ans : 2 ) 27. He has invited one (1)/ of his biggest business rivals (2)/ to his office (3) / to discuss the merger.(4)/ No error (5) (Ans : 5) 28. To avoid any interruption (1)/ during the presentation (2)/ he checked all the system (3)/ the previous day. (4)/ No error (5) (Ans : 4) 29. It is the government (1)/ responsibility to provide (2)/ athletes with the necessary facilities (3)/ for their training. (4)/ No error (5) Explanations : It is the government's ....... is the correct usage. (Ans : 1) 30. The management is (1)/ not willing to (2)/ make no concession (3)/ to the employee's demands. (4)/ No error (5) Explanations : 'Make anyconcess ion' is the correct usage. (Ans : 3) Directions (31 - 35) : In each of the following sentences there are two blank spaces. Below each sentence, there are five pairs of words denoted by numbers (1), (2),(3). (4) and (5). Find out which pair of words can be filled up in the blanks In the sentence in the same sequence to make the sentence meaningfully complete. (36-40) ANS : The meaningful paragraph should be: To day there is growing concern about globle warming energy and water crises. To address these issues .... 31. With the merger, the newly formed company has come to _______the country's _______into a modern economy. (1) lead — acceptance (2) exemplify — transformation (3) promote — development (Ans) (4) reflect — transfer (5) stall — exchange 32. _____governance does not encourage respect for authority and ______ the condition of the exploited. (1) Frail — enhance (2) Lack — recognizes (3) Inadequate — heeds. . (4) Weak — accentuates (5) Effectual — alleviate (Ans) 33. Although businesses are less ________ than they were before liberalization some parts of the economy remain_to restrictions. (1) fettered — subject (Ans) (2) shunned — accessible (3) ignored — vulnerable (4) restrict — expose (5) defunct — resistant 34. Today the city ________ free housing and hospitals and clean streets has become the ______ of the entire country. (1) offers — example (2) known — pride (3) with — envy (4) providing — challenge (Ans) (5) supplies — dream 35. Since its launch, the computer programme has_______for two thirds of all software sales_______ (1) allowed — legally (2) plans — globally (3) provided — finally (4) competed — demand (5) accounted — domestically (Ans) Directions (3640) : Rearrange the following six sentences (A), (B), (C), (D), (E) and (F) in the proper sequence to form a meaningful paragraph; then answer the questions given below them. (A) To address these issues Indian corporates are increasingly tuning ecofriendly. (B) At present however there are only a dozen green buildings in the private sector. (C) However though an ecofriendly building may cost more upfront however it is cost effective because of lower operating costs in the long run. (D) Today there is growing concern about global warming, energy and water crises. (E) The reason is the construction cost of an ecofriendly building is 15% to 20% more than putting up a conventional building. (F) Planting trees, using energy saving lighting systems and constructing ecofriendly green buildings are some of the measures they are taking. 36. Which of the following will be the FIRST sentence after rearrangement? (D A (2) B (3) C (4) D (Ans) (5) E 37. Which of the following will be the SECOND sentence after rearrangement? (1) A (Ans) (2) C (3) D (4) E (5)F 38. Which of the following will be the THIRD sentence after rearrangement? (DB ' (2)D (3) C (4) E (5) F (Ans) 39. Which of the following will be the FDTTH sentence after rearrangement ? (DC (2)D (3) E (Ans) (4) F (5) A 40. Which of the following will be the SIXTH (LAST) sentence after rearrangement ? (D B (2) C (Ans) (3) D (4) E (5) F Directions (4150) : In the following passage, there are blanks, each of which has been numbered. These numbers are printed below the passage and against each, five words are suggested, one of which fits the blank appropriately. Find out the appropriate word in each case. Mass migration has produced a huge world wide economy of its own which has (4_D so fast during the past few years that the figures have (42) experts. Last year remittances sent home by migrants were expected to (431 $232 billion according to the World Bank which (44) these figures. (45) though the flow of remittances is to alleviate the plight of the migrant's family it cannot on its own lift entire nations out of poverty. Those who study the (46) of remittances argue that the money allows poor countries to put off basic decisions of economic management like (47) their tax collection systems and building schools. Remittances to poor countries can also (48) the fact that they do not produce much at home. The challenge is now to find programmes that (49) the benefits of remitted cash while (50) some of its downside. 41. (1) accelerated (Ans) (2) grew (3) expand (4) increase (5) escalating 42. (1) (2)encouraged (3) astonished (Ans) (4)convinced (5) disturb 43. (1) rise (2) represent (3) project (4) exceed (Ans) (5)recover 44. (1) record (2) tracks (Ans) (3) estimat (4) reporte (5) surveys 45. (1) detrimental (2) minor (3) profuse (4) benefited (5) vital (Ans) 46. (1) circumstance (2) profit (3) impact (Ans) (4) status (5) quality 47. (1) declaring (2) established (3) measuring (4) reforming (Ans) (5) govern 48. (l) mask (Ans) (2) hid (3) review (4) display (5) supported 49. (1) launch (2) predict (3) optimum (4) appreciate (5) maximize (Ans) 50. (1) augmenting (2) avoiding (Ans) (3) suspend (4) projects (5) detracting 50. (1) augmenting (2) avoiding (Ans) (3) suspend (4) projects (5) detracting GENERAL AWARENESS, MARKETING & COMPUTER 51.The Criminal Procedure Code (Amendment) Act, 2008 came into effect on December 31, 2009. It incorporates the recommendations of_to prevent overcrowding of jails with undertrials. a. The Law Commission b. The Justice Mallmath Committee's report c. The guidelines issued by the Supreme Court Select the correct answer using the codes given below: (1) All of the above (Ans) (2) Only a (3) Only b (4) Both b and c (5) Both a and b 52.__on December 31, 2009, became the first Indian woman to ski to the South Pole when she crossed a 900 km Antarctic ice trek to reach the South Pole to mark the 60th anniversary of the founding of the Commonwealth. (1) AvaniMathur (2) Deepika Sharma (3) Amita Chauhan (4) Reena Kaushal (Ans) (5) MadhumitaSolanki 53. The Indian Science Award, instituted by the Department of Science and Technology of the Union government, was given by Prime Minister Manmohan Singh to C. R Rao, statistician, at the 97th Indian Science Congress in_on January 3, 2010. (1) Kochi (2) Chennai (3) Thiruvananthapuram (Ans) (4) Pune (5) Kolkata 54. New Year began on a bloody note in Pakistan as seventyfive people were killed and over 50 injured in Shah Hasan Khan, near Lakki Marwat in the _ district of the NorthWestern Frontier Province. (1) Chitral (2) Dera Ismail (3) Bannu (Ans) (4) Peshawar (5) Hamirpur 55. Russia and the United States, on _, 2009, missed the deadline to sign a replacement to the Strategic Arms Reduction Treaty (START) which expired at midnight. (1) December 1 (2) December 5 (Ans) (3) December 25 (4) December 31 (5) December 14 56. The Mojave Desert, on December 7,2009, played host to a very modern spectacle when Sir Richard Branson unveiled Virgin Galactic's Space Ship Two for carrying passengers to the brink of space. The Mojave Desert is cov-ered the most in_. (1) Utah (2) California (Ans) (3) Nevada (4) Arizona (5) None of the above 57. The Supreme Court of which of the following countries, on January 3. 2010, ratified a 25year prison sentence for the former President, Alberto Fujimori? (1) Peru (Ans) (2) Chile (3) South Korea (4) Taiwan ^ (5) Cambodia 58. The Rajya Sabha on December 1, 2009, unanimously approved, by voice vote, the Workmen's Compensation (Amendment) Bill, 2009. Which of the following is/ I are correct regarding the Bill? a. It seeks to give higher compensation to workers and their families in the event of injury or death. b. It empowers the Centre to enhance the compensation and funeral expenses, by notification, from time to time. c. It empowers the government to specify, by notification, monthly wages for an employee. Select the correct answer using the codes given below. (1) Only a (2) Only b (3) Only c (4) All of the above (Ans) (5) None of these 59. Multilateral funding agency of the Asian Development Bank, on December 3, 2009, approved 200million dollar loan to finance a road project in_. (1) Orissa (2) Jharkhand (Ans) (3) Bihar (4) West Bengal (5) Chhattisgarh 60. Consider the following statements related to the selection of Vihaan Networks Ltd., a Shyam Group company, as "Technology Pioneers2010." a. It was selected by the World Economic Forum as a pioneer. b. It was chosen for its innovative and pioneering work in developing solarpowered mobile phone base stations WorldGSM — foi use in rural areas. c. Vihaan Networks is the only company to have found a way ol building sustainable phone networks for 3 billion people in rural areas. Which of the statements giver above is/are not correct? Selecl the correct answer using th< codes given below: (1) Only a (2) Both b and c (3) Both a and b (4) All of the above (5) None of these (Ans) 61. The Union Cabinet on Decembei 10, 2009, approved amendments to the Energy Conservation Act _______, to introduce the system o issuing energy saving certificate! to be traded in the domestic mar ket. (1) 2008 (2) 2006 (3) 2004 (4) 2002 (5) 2001 (Ans) 62. The Haryana Power Generation Corporation, on December 24, 2009, became the first State sector power generation utility in the country to get certified for_for its power stations at Yamunanagar and Panipat and corporate office at Panchkula. a. ISO: 9001 b. ISO: 14001 c. OHSAS: 18001 Select the correct answer using the codes given below: (1) All of the above (Ans) (2) Both a and c (3) Both a and b (4) Both b and c (5) None of these 63. ONGC. on December 1, 2009, signed agreements to pick up____per cent interest in Phase 12 of the gigantic South Pars gas field and get_per cent of Iran LNG's project that will convert the gas into liquefied natural gas for exports. (1) 20,40 (2) 30,30 (3) 40. 20 (Ans) (4) 50. JP (5) 15. 45 64. Reliance Exploration and Production DMCC a wholly owned subsidiary of Reliance Ind SBI Whole Test paper SBI Bank PO Assistant clerks exam solved questions for practice,SBI Reasoning aptitude computer awareness general awareness marketing aptitude, free solved sample placement papers,SBI and SBT Associate Banks practice question papers,All Bank IBPS PO CWE clerical Gramin Bank RRBs IBPS Specialists officers recruitment test pattern syllabus previous years solved question papers 1. Statement: Should all the profit making public sector units be sold to private companies? Arguments: Yes. This will help the government to augment its resources for implementing the development programmes. No. The private companies will not be able to run these units effectively. Yes. There will be a significant improvement in the quality of services. No. There would not be job security for the employees at all the levels. A. Only II and III are strong B. All are strong C. Only III and IV are strong D. Only I, II and III are strong E. Only II, III and IV are strong Answer: Option C Explanation: The government cannot sell off public sector units just to pool up funds for development. Besides, if it does so, these units shall be handed over to private companies which are fully equipped to run these units effectively. So, neither I nor II holds strong. Privatization shall surely ensure better services, but private companies adopt hire and fire policy and they are free to terminate the services of any employee as and when they wish to. Thus, both III and IV hold strong. Direction (for Q.No. 2): In each question below is given a statement followed by three assumptions numbered I, II and III. You have to consider the statement and the following assumptions, decide which of the assumptions is implicit in the statement and choose your answer accordingly. 2. Statement: The company has decided to increase the price of all its products to tackle the precarious financial position. Assumptions: The company may be able to wipe out the entire losses incurred earlier by this decision. The buyers may continue to buy its products even after the increase. The company has adequate resources to continue production for few more months. A. Only I and III are implicit B. Only II is implicit C. Only II and III are implicit D. None is implicit E. None of these Answer: Option A Explanation: It is mentioned that the company has taken the decision to make up for the financial deficit. So, I is implicit. The response of the buyers to the increased prices cannot be deduced from the statement. So, II is not implicit. Since the company seeks to improve its financial position by increasing the prices of its products, so III is also implicit. Learn more problems on : Statement and Assumption Direction (for Q.No. 3): Translate from an imaginary language into English. Then, look for the word elements that appear both on the list and in the answer choices. 3. Here are some words translated from an artificial language. gemolinea means fair warning gerimitu means report card gilageri means weather report Which word could mean "fair weather"? A. gemogila B. gerigeme C. gemomitu D. gerimita Answer: Option A Explanation: Gemo means fair; linea means warning; geri means report;mitumeans card; and gilameans weather. Thus, gemogila is the correct choice. Learn more problems on : Artificial Language Direction (for Q.No. 4): Choose the picture that would go in the empty box so that the two bottom pictures are related in the same way as the top two are related. Answer: Option C Explanation: Camera is to photograph as teakettle is to a cup of tea. The camera is used to make the photo; the teakettle is used to make the tea. Direction (for Q.No. 5): The logic problems in this set present you with three true statements: Fact 1, Fact 2, and Fact 3. Then, you are given three more statements (labeled I, II, and III), and you must determine which of these, if any, is also a fact. One or two of the statements could be true; all of the statements could be true; or none of the statements could be true. Choose your answer based solely on the information given in the first three facts. 5. Fact 1: Most stuffed toys are stuffed with beans. Fact 2: There are stuffed bears and stuffed tigers. Fact 3: Some chairs are stuffed with beans. If the first three statements are facts, which of the following statements must also be a fact? I: Only children's chairs are stuffed with beans. II: All stuffed tigers are stuffed with beans. III: Stuffed monkeys are not stuffed with beans. A. I only B. II only C. II and III only D. None of the statements is a known fact. Answer: Option D Explanation: None of the three statements is supported by the known facts. Direction (for Q.No. 6): In each question below is given a statement followed by two conclusions numbered I and II. You have to assume everything in the statement to be true, then consider the two conclusions together and decide which of them logically follows beyond a reasonable doubt from the information given in the statement. Give answer: (A) If only conclusion I follows (B) If only conclusion II follows (C) If either I or II follows (D) If neither I nor II follows and (E) If both I and II follow. 6. Statements: Use "Kraft" colours. They add colour to our life. - An advertisement. Conclusions: Catchy slogans do not attract people. People like dark colours. A. Only conclusion I follows B. Only conclusion II follows C. Either I or II follows D. Neither I nor II follows E. Both I and II follow Answer: Option D Explanation: The slogan given in the statement is definitely a catchy one which indicates that catchy slogans do attract people. So, I does not follow. Nothing about people's preference for colours can be deduced from the statement. Thus, II also does not follow. Learn more problems on : Statement and Conclusion Discuss about this problem : Discuss in Forum Direction (for Q.No. 7): Choose the statement that is best supported by the information given in the question passage. 7. Close-up images of Mars by the Mariner 9 probe indicated networks of valleys that looked like the stream beds on Earth. These images also implied that Mars once had an atmosphere that was thick enough to trap the sun's heat. If this were true, something happened to Mars billions of years ago that stripped away the planet's atmosphere. This paragraph best supports the statement that A. Mars now has little or no atmosphere. B. Mars once had a thicker atmosphere than Earth does. C. the Mariner 9 probe took the first pictures of Mars. D. Mars is closer to the sun than Earth is. E. Mars is more mountainous than Earth is. Answer: Option A Explanation: The paragraph states that Mars once had a thick atmosphere, but that it was stripped away. The other choices, true or not, cannot be found in the passage. Learn more problems on : Analyzing Arguments Discuss about this problem : Discuss in Forum Direction (for Q.No. 8): In each of the following questions, three statements are given followed by four conclusions numbered I, II, III and TV. You have to take the given statements to be true even if they seem to be at variance with commonly known facts and then decide which of the given conclusions logically follows from the given statements disregarding commonly known facts. 8. Statements: All rods are bricks. Some bricks are ropes. All ropes are doors. Conclusions: Some rods are doors. Some doors are bricks. Some rods are not doors. All doors are ropes. A. Only I and II follow B. Only I, II and III follow C. Only either I or III, and II follow D. Only either I or III, and IV follow E. None of these Answer: Option E Explanation: All rods are bricks. Some bricks are ropes. Since the middle term 'bricks' is not distributed even once in the premises, no definite conclusion follows. Some bricks are ropes. All ropes are doors. Since one premise is particular, the conclusion must be particular and should not contain the middle term. So, it follows that 'Some bricks are doors'. II is the converse of this conclusion and so it holds. All rods are bricks. Some bricks are doors. Since the middle term 'bricks' is not distributed even once in the premises, no definite conclusion follows. However, I and III involve the extreme terms. But, since they are not contradictory, they do not form a complementary pair. Hence, only II follows. Direction (for Q.No. 9): Three of the words will be in the same classification, the remaining one will not be. Your answer will be the one word that does NOT belong in the same classification as the others. 9. Which word does NOT belong with the others? A. branch B. dirt C. leaf D. root Answer: Option B Explanation: A branch, leaf, and root are all parts of a tree. The dirt underneath is not a part of the tree. Learn more problems on : Verbal Classification Direction (for Q.No. 10): In each question below is given a statement followed by three courses of action numbered I, II and III. You have to assume everything in the statement to be true, then decide which of the three given suggested courses of action logically follows for pursuing. 10. Statement: The Company X has rejected first lot of values supplied by Company A and has cancelled its entire huge order quoting use of inferior quality material and poor craftsmanship. Courses of Action: The Company A needs to investigate functioning of its purchase, production and quality control departments. The Company A should inspect all the valves rejected by Company X. The Company A should inform Company X that steps have been taken for improvement and renegotiate schedule of supply. A. Only I and II follow B. Only II follows C. II, and either I or III follow D. All I, II and III follow E. None of these Answer: Option A Explanation: First of all, company A should inspect the rejected valves to ensure if they are really sub-standard. If yes, it should scrutinise its working thoroughly and remove its lackenings, be it in the quality of raw material or craftsmanship. So, both I and II follow. III seems to be a far-off action which can be implemented only after the first two steps are put into practice. Thus, III does not follow, Learn more problems on : Course of Action Discuss about this problem : Discuss in Forum Direction (for Q.No. 11): In each question below is given a statement followed by two assumptions numbered I and II. You have to consider the statement and the following assumptions and decide which of the assumptions is implicit in the statement. Give answer (A) If only assumption I is implicit (B) If only assumption II is implicit (C) If either I or II is implicit (D) If neither I nor II is implicit (E) If both I and II are implicit. 11. Statement: "Avon Cycles - Fast, easy to ride, impressive, reliable, crafted and up-to-date automation." - An advertisement. Assumptions: There is no other cycle with any of these features. People do not bother about the cost. A. Only assumption I is implicit B. Only assumption II is implicit C. Either I or II is implicit D. Neither I nor II is implicit E. Both I and II are implicit Answer: Option D Explanation: The advertisement is for Avon cycles and nothing about the cost or the features of other brands of cycles, is mentioned. So, neither I nor II is implicit. Learn more problems on : Statement and Assumption Discuss about this problem : Discuss in Forum Direction (for Q.Nos. 12 - 14): In each question below is given a statement followed by three assumptions numbered I, II and III. You have to consider the statement and the following assumptions, decide which of the assumptions is implicit in the statement and choose your answer accordingly. 12. Statement: The successful man has the ability to judge himself correctly. Assumptions: Inability to judge correctly causes failure. To judge others is of no use to a successful man. The successful man cannot make a wrong judgement. A. None is implicit B. All are implicit C. Only I and II are implicit D. Only II and III are implicit E. Only I and III are implicit Answer: Option B Explanation: Assumptions I and III directly follow from the statement and so both are implicit. Also, the basic quality of a successful man is that he can judge himself. This means that he need not judge others. So, II is also implicit. Learn more problems on : Statement and Assumption Discuss about this problem : Discuss in Forum 13. Statement: Quality of life of a person is not dependent only on his wealth. Assumptions: The aim of most people is just to acquire more wealth. There are some factors other than wealth which contribute to the quality of life. Wealth does not contribute to the quality of life at all. A. Only I is implicit B. Only I and II are implicit C. Only II is implicit D. Only II and III are implicit E. Only I and III are implicit Answer: Option C Explanation: Clearly, I does not follow from the statement. So, it is not implicit. The statement mentions that quality of life does not depend only on wealth. This means that there are some other factors as well, which govern the quality of life. But this does not imply that wealth does not contribute at all. So, II is implicit while III is not. Learn more problems on : Statement and Assumption Discuss about this problem : Discuss in Forum 14. Statement: Ravi decided to leave office at 4.00 p.m. to catch a flight to Bangalore departing at 6.00 p.m. Assumptions: The flight to Bangalore may be delayed. He may be able to reach airport well before 6.00 p.m. He may get adequate time to search for a vehicle to go to the airport. A. None is implicit B. Only II is implicit C. Only II and III are implicit D. All are implicit E. None of these Answer: Option C Explanation: I cannot be assumed from the given statement and so it is not implicit. Also, knowing that his flight is scheduled to depart at 6 p.m., Ravi would leave accordingly, keeping enough time to search for a vehicle and to reach the airport well before time. So, both II and III are implicit. Learn more problems on : Statement and Assumption Discuss about this problem : Discuss in Forum Direction (for Q.No. 15): In each question below is given a statement followed by two conclusions numbered I and II. You have to assume everything in the statement to be true, then consider the two conclusions together and decide which of them logically follows beyond a reasonable doubt from the information given in the statement. Give answer: (A) If only conclusion I follows (B) If only conclusion II follows (C) If either I or II follows (D) If neither I nor II follows and (E) If both I and II follow. 15. Statements: The XYZ Medical College has started a cell which will conduct counselling workshops in the field of stress management to patients and general public. Conclusions: The hospital has needed resources to start such activity. Patients and general public feel a need to have such cell in the hospital. A. Only conclusion I follows B. Only conclusion II follows C. Either I or II follows D. Neither I nor II follows E. Both I and II follow Answer: Option E Explanation: Since the hospital has started the activity, it must have been well-equipped for the same. So, I follows. Also, any new activity is started keeping in mind the need for it. So, II also follows. Discuss about this problem : Discuss in Forum Direction (for Q.No. 16): In each of the following questions, a statement/group of statements is given followed by some conclusions. Without resolving anything yourself choose the conclusion which logically follows from the given statements). 16. Soldiers serve their country. A. Men generally serve their country. B. Those who serve their country are soldiers. C. Some men who are soldiers serve their country. D. Women do not serve their country because they are not soldiers. Answer: Option C Direction (for Q.No. 17): Each question given below consists of a statement, followed by two arguments numbered I and II. You have to decide which of the arguments is a 'strong' argument and which is a 'weak' argument. Give answer: (A) If only argument I is strong (B) If only argument II is strong (C) If either I or II is strong (D) If neither I nor II is strong and (E) If both I and II are strong. 17. Statement: Should higher education be restricted to only those who can bear the expenditure? Arguments: Yes. Higher education is very costly; hence it should not be given free. No. There are a large number of brilliant students who cannot afford to pay and they should be given higher education. A. Only argument I is strong B. Only argument II is strong C. Either I or II is strong D. Neither I nor II is strong E. Both I and II are strong Answer: Option B Explanation: For the all-round progress of the nation, all the students, especially the talented and intelligent ones, must avail of higher education, even if the government has to pay for it. So, only argument II holds. Direction (for Q.No. 18): A good way to figure out the relationship in a given question is to make up a sentence that describes the relationship between the first two words. Then, try to use the same sentence to find out which of the answer choices completes the same relationship with the third word. 18. Exercise is to gym as eating is to A. food B. dieting C. fitness D. restaurant Answer: Option D Explanation: A gym is a place where people exercise. A restaurant is a place where people eat. Food (choice a) is not the answer because it is something people eat, not a place or location where they eat. The answer is not choice b or c because neither represents a place where people eat. Direction (for Q.No. 19): The words in the bottom row are related in the same way as the words in the top row. For each item, find the word that completes the bottom row of words. 19. apples fruit supermarket novel book ? A. bookstore B. magazine C. vegetable D. shopping Answer: Option A Explanation: The relationship above the line is as follows; apples are a kind of fruit; fruit is sold in a supermarket. Below the line, the relationship is: a novel is a kind of book; books are sold in a bookstore. Direction (for Q.No. 20): Read the question carefully and choose the correct answer. 20. At the baseball game, Henry was sitting in seat 253. Marla was sitting to the right of Henry in seat 254. In the seat to the left of Henry was George. Inez was sitting to the left of George. Which seat is Inez sitting in? A. 251 B. 254 C. 255 D. 256 Answer: Option A Explanation: If George is sitting at Henry's left, George's seat is 252. The next seat to the left, then, is 251. 1. Many a times we read in newspapers about a term ?dematerialization?. In context with the markets, what is dematerialization of securities? Ans. Electronic holding of the shares replacing the paper securities 2. Name the identity cards which are being distributed to all persons above the age of 18 residing in the nine maritime states and four Union Territories under the national Population Register (NPR) scheme. Ans. Resident Identity Card 3. What is the new disinvestment target set by the Union Budget for the fiscal year 2012-13 (in crores)? Ans. 30,000 4. How much percentage of fund allocated to the priority sector lending by the commercial banks in India, as stipulated by the Reserve Bank of India (RBI), goes to Agricultural sector? Ans. 18% 5. The book titled ?The World Beneath His Feet? is the biography of which sportsman? Ans. Pulela Gopichand 6. The Reserve Bank of India has introduced a new facility recently, via which the banks can dip below 1% of their statutory liquidity ratio to avail cash from this window. What is the name of this facility ? Ans. Marginal Standing Facility 7. Name India?s first cruise missile that will travel at a sub-sonic speed developed by the Defence Research & Development Organisation (DRDO)? Ans. Nirbhay 8. Which is the first private sector employer of India to introduce the National Pension Scheme for its employees? Ans. Wipro 9. Which bank won the prestigious Asian Banker Achievement Award for being the strongest bank in Asia-Pacific region? Ans. State Bank of India 10. The 5th edition of the World Energy Summit was held recently at: Ans. Abudhabi 11. Daniel Shechtman from Israel is the Nobel Prize winner of 2011. Which field does he relate to? Ans. Chemistry 12. Who is the author of the famous novel ?War and Peace?? Ans. Leo Tolstoy 13. The 35th National Games will be held in: Ans. Kerala 14. Who is known as ?The Grand Old Man of India?? Ans. Dhada Bhai Naoroji 15. The currency of Myanmar is: Ans. Kyat 16. Which of the following countries is not a member of South Asian Association for Regional Cooperation (SAARC)? a)China b)India c)Nepal d)Bangladesh Ans. China 17. How many public sector banks are currently there in India? Ans. 27 18. Dhada Saheb Phalke awards are given by the Ministry of: Ans. Information and Broadcasting 19. When milk is churned the cream from it is seperated by: Ans. Centifugal force 20. Who is the cabinet minister of civil aviation department of the Union Government? Ans. Ajith Singh Computer General Awareness for Bank PO and Clerk Computer General Awareness Sample Questions for Bank PO and Clerk 1. This ......... tier processes HTTP protocol, scripting tasks, performs calculations, and provides access to data. (a)Client (b)Applications/Web server (c)Enterprise server (d)DBA (e)None of these Ans. Client 2. A polymorphic virus: (a)modifies its program code each time it attaches itself to another program or file (b)is a malicious-logic program that copies itself repeatedly in memory or on a disk drive until no memory or disk space remains (c)a malicious logic program that hides within or looks like a legitimate program (d)infects a program file, but still reports the size and creation date of the original, uninfected program (e)None of these Ans. modifies its program code each time it attaches itself to another program or file 3. The Secure Electronic Transaction (SET) specification : (a)is a notice, issued and verified by a certificate authority, that guarantees a user or Website is legitimate (b)provides private key encryption of all data that passes between a client and a server (c)allows users to choose an encryption scheme for the data that passes between a client and a server (d)uses a public key encryption to secure credit-card transaction systems (e)None of these Ans. Uses a public-key encryption to secure credit-card transaction systems 4. ......... allows wireless mobile devices to access the Internet and its services such as the Web and e-mail: (a)TCP/IP (b)Ethernet (cWAP (d)Token ring (e)None of these Ans. WAP 5. 'DOS' floppy disk does not have: (a)A Boot Record (b)A File Allocation Table (c)A Root Directory (d)Virtual Memory (e)BIOS Ans. Virtual Memory 6. A passive threat to computer security is: (a)Malicious Intent (b)Sabotage (c)Accident Errors (d)Espionage Agents (e)None of these Ans. Malicious Intent 7. All of the following are basic principles of net works, except: (a)each computer must have a network card (b)there must be communications media connecting the network hardware devices (c)there must be at least one connecting device (d)each computer must have software that supports the movement of information (e)None of these Ans. there must be communications media connecting the network hardware devices 8. 'MICR' technology used for clearance of cheques by banks refers to: (a)Magnetic Ink Character Recognition (b)Magnetic Intelligence Character Recognition (c)Magnetic Information Cable Recognition (d)Magnetic Insurance Cases Recognition (e)None of these Ans. Magnetic Ink Character Recognition 9. All the information collected during database development is stored in a: (a)repository (b)data warehouse (c)RAD (d)CASE (e)None of these Ans. data warehouse 10. The ......... component is used by the data analyst to create the queries that access the database. (a)data extraction (b)end-user query tool (c)end-user presentation tool (d)data store (e)None of these Ans. end-user query tool 1. First Governor of RBI was: (a)Hilton Young (b)Paul Samuelson (c)C.D.Deskmukh (d)O.A Smith Ans. (d) O.A Smith 2. At the time of nationalization who was the Governor of RBI: (a)O.A Smith (b)J.B Taylor (c)C.D. Deshmukh (d)K.C.Neogy Ans. (c) C.D. Deshmukh 3. The RBI was nationalized in the year: (a)1949 (b)1956 (c)1959 (d)1947 Ans. (a) 1949 4. The general superintendence and director of the bank is entrusted to central board of directors of: (a)10 members (b)20 members (c)25 members (d)30 members Ans. (b) 20 members 5. Paper currencies of our country are issued by RBI under: (a)Section 22 of the RBI act 1934 (b)Section 24 of the RBI act 1934 (c)Section 28 of the RBI act 1934 (d)None of these Ans. (a) Section 22 of the RBI act 1934 6. One rupee currency notes bear the signature of: (a)PM (b)President of India (c)Governor of RBI (d)Finance Secretary of India Ans. (d) Finance Secretary of India 7. Ten rupees notes bear the signature of: (a)President (b)Finance Minister (c)Secretary of Ministry of finance (d)Governor of RBI Ans. (d) Governor of RBI 8. Which of the following is the banker of the banks: (a)IDBI (b)SBI (c)RBI (d)UTI Ans. (c) RBI 9. In which of the following banks one can?t open a personal account: (a)Co-Operative Banks (b)Commercial banks (c)Regional Rural Banks (d)RBI Ans. (d) RBI 10. Which of the following banks is the banker to the government: (a)SBI (b)SEBI (c)RBI (d)IRDA Ans. (c) RBI 1. The term of office of an elected member shall be ......... years from the date of which he enters upon office. Ans. 5 2. In a circle co-operative union reservation for women is ......... Ans. 1 3. The authority competent to reject a ballot paper shall be the ......... Ans. Returning Officer 4. Any dispute relating to the election of the circle co-operative union shall be referred to the ......... Ans. Registrar 5. The management of the affairs of the circle co-operative union shall vest in the body constituted under section ......... Ans. 88 6. Section ......... of KCS Act deals about the establishment of State Co-operative Union. Ans. 89 7. The Audit of a State Co-operative Union is done by: Ans. Director of Co-operative Audit 8. Approval of annual report, budget and audited accounts of the union is the duty of: Ans. General body 9. The managing committee of the State Co-operative Union consist of ......... members. Ans. 23 10. The number of Government nominees in the State Co-operative Union: Ans. 2 1. Name the Co-operative institution which bagged ?Coffey International Award 2011? for providing unique mobile service empowering the farmers through timely information delivery: (a) IFFCO Kissan Sanchar Ltd (b) Krishak Bharati Cooperative Limited (KRIBHCO) (c) MILMA (d) None of these Ans. IFFCO Kissan Sanchar Ltd 2. Who is considered to be the father of Co- operative movement in India? (a) Robert Owen (b) Frederic Nicholson (c) Edward Law (d) Dupernix Ans. Frederic Nicholson 3. The Planning and co- ordination function of Co- operative education is conducted by: (a) NCCF (b) NCCT (c) NCUI (d) None of these Ans. NCCT 4. One Hundred and Eleventh Constitution Amendment Bill is notable because: (a) Which made right to property (b) Which made right to freedom of religion (c) Which made right to form cooperative societies as a fundamental right (d) None of these Ans. Which made right to form cooperative societies as a fundamental right 5. National Agricultural Cooperative Marketing Federation of India Ltd (NAFED), a farmer?s co-operative was established in: (a) 1990 (b) 1960 (c) 1979 (d) 1958 Ans. 1958 6. The first diary co-operative society formed was: (a) Kaira (b) Anand (c) Gujarat (d) Katra Ans. Katra 7. The book ?Co- operative Movement in India? written by: (a) C.R Fay (b) E.M Hough (c) G.D.H Cole (d) S.S Puri Ans. E.M Hough 8. Which Committee recommended the formation of State Bank of India? (a) National Commission of Agriculture (b) All India Rural Credit Survey Committee (c) Gorwala Committee (d) Maclagan Committee Ans. All India Rural Credit Survey Committee 9. NCCF stands for: (a) National Co operative Consumer Federation (b) National Consumer Co operative Forum (c) National Co operative Federation (d) None of these Ans. National Co operative Consumer Federation 10. The largest milk producing nation in 2010-11 was: (a) India (b) Japan (c) Denmark (d) Ireland Ans. India 1. Which day is observed as World House Sparrow Day? Ans. March 20 2. Which is the International Monetary Fund?s (IMF) unit of account? Ans. SDR 3. The khandelwal committee to recommendations are related to the: Ans. Banking sector 4. The 123rd birth anniversary of Muzzaffar Ahmed was observed in August 2011. Muzzaffar Ahmed was one of the founders of the: Ans. Communist party 5. What is the India?s rank in the last National Security Index (NSI), 2011? Ans. 5th 6. Who is the author of the book ?India on the Growth Tumpike?? Ans. Vijay Kelkar 7. Which of the following prizes is given to Mathematicians? (a) Kluge Prize (b) Kalinga Prize (c) Abel Prize (d) Pulitzer prize Ans. Abel Prize 8. In August 2011 NASA launched the billion dollar solar powered spacecraft JUNO which is on a five year journey to: Ans. Jupiter 1. PDF means: Ans. Portable Document Format 2. ISDN stands for: Ans. Integrated Services Digital Network 3. Which is a computer printing device for printing vector graphics? Ans. Plotter 4. Which is the default mode of printing the documents? Ans. Portrait 5. Which is the disk subsystem that increases performance or provides fault tolerance or both? Ans. RAID RAID stands for Redundant Array of Inexpensive (or sometimes "Independent") Disks. 6. Who is the person responsible for the design, implementation, maintenance and repair of an organization?s database? Ans. DBA (Data Base Administrator) 7. Tab key is an abbreviation for: Ans. Tabulator key 8. Which is the computer shortcut for undo the previous operation? Ans. Ctrl+Z 9. Which is the lossy compressed music format that is very popular for digitizing music? Ans. MP3 10. Which is the best method to move text from one page to a different page? Ans. Cut and Paste Marketing and Computer 1. Zero based Budgeting (ZBB) means: Ans. Each year, budgeting starts from a scratch 2. The abbreviation ISP stands for : Ans. Internet Service Provider 3. Acid-test of a brand is ?????.. Ans. Brand loyalty 4. In Banks ROA means ?????.. Ans. Return on assets 5. In the context of globalization, BPO means: Ans. Business Process Outsourcing 6. SWOT analysis refers to: Ans. Strategic planning for selling product 7. In banks, loans and advances are considered as ?????. Ans. Assets 8. In the era of globalization of business, M & A means??????? Ans. Mergers & Acquisitions Marketing and Computer 1. DOS and Windows 3.x support file names upto ????characters length. Ans. Eight 2. In DOS the ?Label? command is used to Ans. Display the label of the disk. 3. The subsystem of the kernel and hardware that cooperates to translate virtual to physical address comprises ????. Ans. Memory Management Subsystem 4. A device operating at the physical layer is called a Ans. Repeater 5. For separating channels in FDM, it is necessary to use Ans. Time slots 6. Encryption and decryption are functions of Ans. Session layer 7. Which type of switching uses the entire capacity of a dedicated link? Ans. Datagram Packet switching 8. In ????? each packet of a message follows the same path from sender to receiver. Ans. The datagram approach to packet switching Marketing and Consumer Co-operatives 1. Which is the apex institution of marketing co-operatives? Ans. Marketfed 2. Which is the national Apex institution in the field of Marketing Co-operatives Ans. NAFED 3. Marketfed was established in ????.. Ans. 1960 4. To meet the losses due to outright purchase, marketing co-operatives keep a Fund called: Ans. Price fluctuation fund 5. Committee on Co-operative Marketing was appointed in: Ans. 1966 6. NAFED means????? Ans. National Agricultural Federation 7. The headquarters of NAFED: Ans. New Delhi 8. The Bio-fertilizer plant of NAFED is situated at: Ans. Indore 9. National Horticulture Research and Development Foundation (NHRDF) was established in: Ans. 1977 10. First Co-operative Marketing society in India is: Ans. Hubli 1. The first European bank in India which was established in 1771. a)Bank of England b)State Bank of India c)Bank of Bengal d)None of these 2. With the approval of the parliament in 1806 the directors of the East India Company established the first presidency bank known as: a)Bank of Bengal b)Bank of Madras c)Bank of Bombay d)Bank of Maharashtra 3. Bank of Bombay was the second presidency bank established in ???. With a capital of Rs. 52,25,000. a)1840 b)1854 c)1862 d)1880 4. Which one of the following is not a presidency bank? a)Bank of Bengal b)Bank of Madras c)Bank of Sourastra d)Bank of Bombay 5. Which one of the following joint stock banks were established during the period of swadeshi movement. a)Allahabad bank in 1870 b)Oudh Commercial Banks in 1881 c)Punjab National Bank in 1895 d)All of these 6. Imperial Bank of India Act was passed in: a)1919 b)1920 c)1921 d)1922 7. ????.. are the private banks organized on a joint stock system, run basically with profit motive and specialize in financing short and long term needs of trade, industry and commerce. a)Schedule Bank b)Non Schedule Banks c)Commercial Banks d)Industrial Banks 8. Which one of the following is not a primary function of commercial banks? a)Accepting deposit b)Advancing loan c)Discounting bills of exchange d)Agency services 9.The greatest advantage of ????? deposits are that the customers can remit any amount to the bank and withdraw unlimited quality of money without any notice to the banker. a)Savings b)Current c)Fixed d)Recurring 10. Which of the following is an agency service of Commercial banks? a)Buying and selling of securities for customers b)Making periodical payments of insurance premium, collection of interest and dividends c)Collection of cheques, draft, bill of exchange d)All of these 11. Which of the following is not a secondary function of commercial bank? a)Accept current deposit b)Accept fixed deposit c)Advancing loans d)General utility services 12. Time deposits are otherwise called: a)Fixed deposit b)Savings deposit c)Current deposit d)Recurring deposit 13. ?A commercial bank is not merely purveyors of money, but also in an important sense, manufacturers of money.?- who said this statement? a)J.M Keynes b)J.B Say c)Sayers d)Crowther 14. ?If loans are the children of deposits, deposits are also the children of loan? said : a)J.M Keynes b)J.B Say c)John Paget d)Crowther 15. Bank Money is created by ????. a)RBI b)Government c)Monetary authority d)Commercial bank Whole-Testpaper SBI- Self-Evaluation-Practice Papers SBI Written Test Reasoning Ability Questions 1.How many pairs of letters are there in the word SPONTANEOUS which have number of letters between them in English alphabet? Five One Four Two Three Ans. Five In a certain code MISTAKEN is written as SRHLOFLB. How is GROUNDED written in that code? CDCMTNQF TNQFCDCM EFEOTNQF TNQFEFEQ None of these Ans. TNQFEFEQ 3. If ‘tree’ means ‘mountain’; ‘mountain’ means ‘watre’; ‘water’ means ‘jungle’; ‘jungle’ means ‘bus’; ‘bus’ means ‘truck’; and ‘truck’ means ‘house’; then where do the fish live? Water Jungle Mountain Bus Truck Ans. Jungle 4. In a certain code language ‘nik ka pa’ means ‘who are you’; ‘ka na ta da’ means you may come here’; and ‘ho ta sa’ means ‘ come and go’; what does ‘nik’ means in that code language? Who Are Who or are Data inadequate None of these Ans. Who or are 5. In certain code BEND is written as 5%7@ and DREAM is written as @2%48. How is MADE written in that code? · 84@% · 8@4% · 85@% · 84%5 · None of these Ans. 84@% 6. Four of the following five are alike in a certain way and so form a group. Which is the one that does not belong to that group? · 18 · 24 · 36 · 34 · 26 Ans. 36 7. Four of the following five are alike in a certain way and so form a group. Which is the one that does not belong to that group? · Coconut · Lotus · Lilly · Rose · Marigold Ans. Coconut Directions (Q 8-12): Read the following information carefully and answer the questions given below: A, B, D, F, G, H and K are seven members of a family. They belong to three generations. There are two married couples belonging to two different generations. D is son of H and is married to K. F is granddaughter of B. G’s father is grandfather of A. B’s husband is father-in-law of K. H has only one son 8. How is F related to G · Son · Nephew · Niece · Data inadequate · None of these Ans. Son 9. How is H related to B? · Father · Father in law · Uncle · Data inadequate · None of these Ans. None of these 10. How is K related to G? · Sister-in-law · Sister · Niece · Data inadequate · None of these Ans. Niece 11. Which of the following is the pair of married ladies? · HK · HD · KF · BF · None of these Ans. BF 12. How many female members are there among them? · Two · Three · Four · Data inadequate · None of these Ans. None of these 13. If in a certain code “DELGATE” is written as “82627342” and “DEMON” is written as “82915” , how is “DENOTE” written as in that code? · 825413 · 825412 · 824152 · 825142 · none of these Ans :825412 14. Four of the following five are alike in a certain way and so form a group, which is the one that does not belong to that group? · Helium · Argon · Chlorine · Crypton · Neon Ans: Crypton 15. If “AM=13”, “DAM=52”, then what will be the value of “SAM”? · 32 · 263 · 247 · 208 · 228 Ans: 208 16. Pointing to a photograph , a boy said ,”She is the daughter of my grandfather’s only son”, how is the boy related to the girl in the photograph? · Cousin · Father · Brother · Uncle · None of these Ans: Cousin 17. Akhil ranks 22nd from the top in a class of 42 students, what is his rank from the last? · 20 · 19 · 21 · 22 · None of these Ans: None of these 18. If ‘green’ means ‘yellow’, yellow’ means’ white’; ‘white’ means ‘red’; ‘red’ means ‘violet’ ; ‘violet’ means ‘black’; then which of the following will be the colour of human body. · Red · Black · Green · White · None of these Ans: None of these 19. Four of the following five are alike in a certain way and so form a group. Which is the one that does not belong to that group? · Mica · Zinc · Iron · Chlorine · Aluminium Ans: Mica 20. Which of the following relates to FLOWER in the same way as RTERBN relates to SECTOR? · RWLGPF · EOFKUQ · EOFMXS · RWLEPD · RWLEND Ans: RWLEND 21. What should come next in the following sequence of letters? B B C B C D B C D E B C D E F B C D E F G B C D E F · B · G · H · D · None of these Ans: G 22. EARN is related to RANE and BOND is related to NODB in the same way as TEAR is related to · AERT · ATRE · ARET · REAT · None of these Ans: AERT Directions (23-25) i) ‘PxQ’ means ‘P, is brother of Q’ ii) ‘P-Q’ means ‘P is sister of Q’ iii) ‘P+Q’ means ‘P is sister of Q’ iv) ‘P÷Q’ means ‘P is mother of Q’ 23. Which of the following represents ‘M is nephew of N’? · N-K+M · NxK÷M · N÷KxM · N-K+MxT · None of these Ans: N-K+MxT 24. How is T related to D in the expression: H+T÷R-D · Nephew · Niece · Nephew or Niece · Data inadequate · None of these Ans: None of these 25. Which of the following represents F is daughter of W? · W÷R+F · WxRxF · W+RxF-T · W+R-F+T · None of these Ans: W+RxF-T Quantitative Aptitude 1. If the numerator of a fraction is increased by 2 and the denominator is increased by 1, the fraction becomes 5/8 and if the numerator of the same fraction is increased by 3 and the denominator is increased by 1 the fraction becomes ¾. What is the original fraction? · Data inadequate · 2/7 · 4/7 · 3/7 · None of these Ans: 3/7 2. Seema invested an amount of Rs.16000 for two years at compound interest and received an amount of Rs.17640 on maturity. What is the rate of interest? · 8pcpa · 5pcpa · 4pcpa · Data inadequate · None of these Ans: 5pcpa 3. What should come in place of question mark (?) in the following equation? 27 3 +118 2 - 32 5 = 11 6 + ? A sum of money is to be distributed among A, B, C and D in the respective proportion of 5:2:4:3.If C gets rs.1000 more than D, what is B’s share? · Rs.2000 · Rs.1500 · Rs.500 · Cannot be determined · None of these Ans: Rs.2000 10. By how much is 3/5th of 80 smaller than 4/5th of 65? · 15 · 2 · 3 · 4 · None of these Ans: 4 11. When the price of a product was decreased by 10%, the number sold was increased by 30%. What was the effect on the revenue? · 17% decrease · No change · 17% increase · Cannot be determined · None of these Ans: 17% increase 12. Jyoti took a loan of Rs. 1200 with simple interest for as many years as the rate of interest. If she paid Rs. 432 as interest at the end of the loan period, what was the rate of interest? · 6% · 3.6% · 18% · Cannot be determined · None of these Ans: 6% By how much are two-fifth of 200 greater than three-fifth of 125? · 15 · 3 · 5 · 30 · None of these Ans: 5 19. The area of a rectangle field is 460 square meters. If the length is 15 per cent more than breadth, what is breadth of the rectangular field? · 15 meters · 26 meters · 34.5 meters · Cannot be determined · None of these Ans: None of these 20. Deepa bought a calculator with 30% discount on the listed price. Had she not got the discount, she would have paid Rs.82.50 extra. At what price did she buy the calculator? · Rs.192.50 · Rs.275 · Rs.117.85 · Cannot be determined · None of these Ans: Rs.192.50 21. A motor starts with the speed of 70 kmph with its speed increasing every two hours by 10kmph. In how many hours will it cover 345 kms? · 2 ¼ hours · 4 ½ hours · 4 hours 5 minutes · Cannot be determined · None of these Ans: 4 ½ hours 22. Seats for Maths, Physics and Biology are in the ratio of 5:7:8 respectively. There is a proposal to increase these seats by 40%, 50% and 75% respectively. What will be the respective ratio of increased seats? · 2:3:4 · 6:7:8 · 6:8:9 · Cannot be determined · None of these Ans: 2:3:4 23. A grocer purchased 2kg of rice at the rate of Rs. 15 per kg and 3 kg of rice at the rate of Rs. 13 per kg. At what price per kg should he sell the mixture to earn 33 1/3 % profit on the cost price? · Rs.28.00 · Rs.20.00 · Rs.18.40 · Rs.17.40 · None of these Ans: Rs.18.40 24. A boat takes 6 hours to travel from place M to N downstream and back from N to M upstream. If the speed of the boat in still water is 4 km/hr, what is the distance between the two places? · 8kms · 12kms · 6kms · Data inadequate · None of these Ans: Data inadequate 25. If on selling 12 notebooks any seller makes a profit equal to the selling price of 4 notebooks, what is his percent profit? · 50 · 25 · 16 2/3 · Data inadequate · None of these Ans: 50 General Awareness/ Computer Knowledge 1. The RBI is the apex body of Indian Financial System, which of the following functions are performed by the RBI? I. Monitoring money supply in economy II. Issues currency other than coins and one rupee note III. Functions as Bankers bank IV. Works as banker to the Government Select the Code: · I & II are correct · I, II & III are correct · II, III & IV are correct · All the above are correct · None of these Ans: All the above are correct 2. Which among the following bodies recommends MSp (Minimum Support Price) for the e crops in India? · FCI (Food Corporation of India) · RBI (Reserve Bank of India) · Ministry of agriculture · CACP (Commission for Agricultural Costs and Prices · None of these Ans: CACP (Commission for Agricultural costs and Prices) 3. Ports are key components of infrastructure. How many major and minor ports does India have? · 12 Major & 184 Minor · 10 major & 180 Minor · 12 Major & 175 Minor · 20 Major & 170 Minor · 14 Major & 185 Minor Ans: 12 major & 184 Minor 4. Which of the following makes a network insecure? a) Encryption b) Network Interface Card (NIC) c) Static keys d) Both (b) & (c) e) None of these Ans: Static Key 5. Project management will be made a separate division on the basis of a) Interdependencies among departments b) Sharing of resources c) Size of the project d) Either (b) or (c) e) All the above Ans: Either (b) or (c) 6. The Steering Committee is made for a) Reviewing user requirements and ensuring that all controls are considered b) Strategic planning for computer installation c) Evaluating specific project plans for systems d) Both (a) & (b) e) None of these Ans: Strategic planning for computer installation 7. Efficient system requirement analysis is required for: a) Reverse engineering b) Joint Application Design (JAD) c) Traditional System Development Life Cycle d) Either (a) or (c) e) All of these Ans: Traditional System Development Life Cycle 8. Asia’s largest Tulip garden recently opened in Kashmir is named · Sirag bagh · Zabarwan bagh · Khas bagh · None of these · All of these Ans: Siraj Bagh 9. Which of the following public sector organizations is listed on the New York stick exchange · SAIL · MTNL · GAIL · None of these · All of these Ans: None of these 10. “Vedanta” the Aluminium major company of India was earlier known as · Nalco · Hindalco · Sail · Balco · None of these Ans: Balco 11. Which one of the following estimates the national Income of India? · RBI · Finance Commission · CSO · Planning Commission · None of these Ans: CSO 12. Telnet helps in a) Remote login b) Connecting to television c) Transferring files across net d) Either (b) or (c) e) All of these Ans: Remote Login 13. What is NOT true about FTP? a) It is protocol b) It is a program or command c) It transfers files over net d) Both (a) & (c) e) All are true Ans: Both (a) & (b) 14. The strategy for testing a function when developing a software is made at a) Test strategy b) Test plans c) Test case design d) Both (a) & (b) e) None of these Ans: test plans 15. Network layers function is to a) Select paths for data transfer b) Break the Network in to subnet c) Determine the IP d) Either (b) or (c) e) All of these Ans: Select paths for data transfer 16. Which of the following states has declared 2008 as the “year of education”? · Haryana · Bihar · Gujarat · Uttar Pradesh · None of these Ans: Haryana 17. Which of the following is the name of India’s one of the major micro finance institution? · Grameen · Compartamos · Brac · Bantra · Spandana Ans: Grameen 18. Who amongst the following is selected as the “Banker of the year 2007” by the Business Standard? · M.V Kamath · K.C Chakrabarty · Anil Khandelwal · O.P bhatt · None of these Ans: O.P Bhatt 19. What is “H5N1”? · It is new category of visa provided by the USA for students · It is the name of the virus which causes Bird flu · It is the code name of India’s Space Mission 2010 · It was the code name of the military operation launched by Sri Lanka against LTTE · None of these Ans: It is the name of the virus, which causes Bird flu 20. Which of the following countries has landed into deep turmoil and the Government of the country has announced the formal and of the 2002 Norwegian-Mediated ceasefire Agreement with the rebel group of the country? · Nepal · Pakistan · Afghanistan · Sri Lanka · None of these Ans: Sri Lanka 21. The fifth National Winter Games were organized in · Jaipur · Gulmarg · Hyderabad · Chennai · Delhi Ans: Gulmarg 22. Something which has easily-understood instructions is said to be · User friendly · Information · Word processing · Icon · None of these Ans: User friendly 23. An example of non-numeric data is · Bank Balance · Examination marks · Real numbers · Employee Address · None of these Ans: None of these 24. The word processing task associated with changing the appearance of documents is · Writing · Formatting · Editing · Storing · None of these Ans: Formatting 25. The best reason that a computer needs to have a hard disk is because · It can then use the same program as other computers · It would not work without one · It can store information when it is switched off · It can store information while it is working · None of these Ans: It would not work without one Marketing Knowledge 1. Suryachakra Power on August 8,2007 said that it would set up a 1200 mega watt thermal power project in… · Maharashtra · Rajasthan · Orissa · West Bengal · None of these Ans: Orissa 2. US-based laboratory equipment major Thermo Fisher Scientific inc. on July 27, 2007 announced that it has entered into an agreement to acquire Qualigens Fire Chemicals. It is a division of · Ranbaxy · Glaxo Smith Kline · Cipla · Dr. Reddy;s · None of these Ans: Glaxo Smith kline 3. J.K Lakshmi Cement Director Shailendra Chouksey on August 1,2007 declared that the company plans to invest about… in the next four years for expansion plans. · Rs.500 crore · Rs.700 crore · Rs.900 crore · Rs.1000 crore · None of these Ans: Rs.1000 crore 4. Software solutions developer Infosys Technologies in August, 2007 has announced to invest rs.30 crore on expansion plans in….. · Chandigarh · Jabalpur · Shimla · Ahmedabad · None of these Ans: Chandigarh 5. India’s largest wine maker Champagne Indage is in talks with… to acquire land to setup a brewery there. · Karnataka · Tamil Nadu · West Bengal · Rajasthan · None of these Ans: Karnataka 6. Which among the following in August 2007 bags Rs.415 crore-tunnel contract from Brihan Mumbai Municipal Corporation (BMC)? · DLF · Unitech · HCC · DLIF · None of these Ans: HCC 7. Which country has lifted the ban on Indian rice and has started issuing quarantine import certificates from July20, 2007? · Russia · United States of America · Japan · South Korea · None of these Ans: Russia 8. Which country on July 27, 2007 has expressed support for a proposed change in its laws to raise the cap on H-1 B visas for highly skilled workers from India? · Britain · Russia · USA · China · None of these Ans: USA 9. According to Inter nation Comparisonal programme in Asia report, India is ranked… in economic well-being. · 10th · 11th · 17th · 23rd · None of these Ans: 17th 10. On August 1, 2007 the Reserve Bank of India announced a hike in the cash reserve ratio (CRR) by half-a-percentage point to….. · 5 percent · 6 percent · 6.5 percent · 7 percent · None of these Ans: 7 percent 11. A decision is taken by the income tax department in August , 2007 to scrutinize the tax returns of · Real estate players · Bank employees · Coaching institutes · Small institutes · Small entrepreneurs · None of these Ans: Real estate players 12. The Public Investment Board (PIB), on August 13, 2007 has given a nod to 1000 Mega Watt coat based thermal power station with.. · Karnataka · Tamil nadu · Chennai · Maharashtra · None of these Ans: Tamil Nadu 13. ‘Slump’ period is characterized by… · Low output · Low employment · Low output and low employment · High output and low employment · None of these Ans: Low output and low employment 14. A rise in the market price of an asset producing a given income stream is equipment to: · An increase in the rate of interest · A decrease in the rate of interest · The bond’s future value · The bond’s present value · None of these Ans: A decrease in the rate of interest 15. The draft outline of the Second Five Year Plan was prepared by.. · A.C Mahalanobis · P.C Mahalanobis · Y,K Reddy · Arjuna Swami · None of these Ans: P.C Mahalanobis 16. Indirect taxes must be subtracted from the subsidies added to…. To reach net domestic product at factor cost · Net domestic product at market price · Gross domestic product at market price · Gross domestic product at factor price · Net domestic product abroad · None of these Ans: net domestic product at market price 17. Market indices are expressed in · Absolute terms · Relative terms · Both the above · None of these Ans: relative terms 18. A computer base system that works in conjunction with revising information system to support the firms, management in solving problems that relate to marketing of the firms product is known as · MKS · MIS · MTS · MLS · None of these Ans: MIS 19. In family life cycle, full nest 1 stage refers to · Young couple without children · The birth of the first child · Teenaged or college going children · Death of a life partner · None of these Ans: The birth of the first child 20. In the present context, management is · An art · A science · Both Art & Science · None of these Ans: both Art & science 21. On November 19, 2007, Merill Lynch has picked 49% in seven residential projects of DLF for Rs: · 1481 crore · 1575 crore · 1660 crore · 1890 crore · None of these Ans: 1481 crore 22. The Terrorism Insurance Pool, begun by the Finance Ministry in the year 2002 from Rs.200 crore has…. in size since then · Remain same · Doubled · Tripled · Halved · None of these Ans: Remain same 23. In a bid to prevent encroachment of its lands, Western railway in December, 2007 has decided to: · Employ guards · Make fences · Grow vegetables · None of these Ans: Grow vegetables 24. The Nehru Rozgar Yojana (NRY) was launched in the year: · 1970 · 1997 · 1989 · 1951 · None of these Ans: 1989 25. Money can serve as a : · Medium of exchange · Store of value · Unit of value · All of the above · None of these Ans: All of the above SBI Written Test Questions SBI Bank exam written test Quantitative Aptitude ,Reasoning, General Awareness Questions with answers Model Test paper of SBI Clerk Exam Questions (1-5)What should come in Place of ( ? ) in the following Questions ? 1) 58% of 842 + ? = 1200 1) 874.54 2) 711.64 3) 674.74 4) 543.84 5) None of these Ans 2 2) 52% of ? = 3387.28 1) 6980 2) 6342 3) 6718 4) 6539 5) None of these Ans 5 3) Sq.root of 12321 1) 111 2) 121 3) 81 4) 91 5) None of these Ans 1 4) 72% of 654 - 41% of 312 = ? 1) 276.60 2) 296.72 3) 336.84 4) 342.96 5) None of these Ans 4 5) 48 % of 840 = 36% of ? 1) 1080 2) 1320 3) 1240 4) 720 5) None of these Ans 5 6) If the Product of two successive positive integers is 6162, which is the smallest integer ? 1) 78 2) 72 3) 76 4) 74 5) None of these Ans 1 7) The Simple interest accured on an amount of Rs 10,530/- at the end of 5 years is 6318/- what is the rate of interest of p.c.p.a ? 1) 8 2) 14 3) 10 4) 12 5) None of these Ans 4 8) A plot of 640 sq.ft is available at the rate of Rs 4600/- per sq.ft. If 30% of the total cost of the plot is to be paid for booking the plot, how much is the booking amount ? 1) Rs 938100 /- 2) Rs 883200 /- 3) Rs 983200 /- 4) Rs 838100 /- 5) None of these Ans 2 9) A single person take 6 minutes to fill a bottle from 11 am to 12:30 pm. 1845 bottles are to be filled. How many persons should be employed on this Job ? 1) 121 2) 127 3) 135 4) 139 5) None of these Ans 5 10) The Cost of 12 Notebook and 16 pens is Rs 852/-. What is the Cost of 9 Note Books and 12 Pens ? 1) Rs 743 2) Rs 639 3) Rs 567 4) Cannot be determined 5) None of these Ans 4 11) If (93) Square is to be added to the Square of a number the answer so obtained is 16441.What is the Number ?1) 67 2) 73 3) 89 4) 91 5) None of these Ans 5 12) The Average Age of a Man and his Son is 18 Years.The ratio of their Ages is 5:1 resp. What will be the ratio of their ages after 6 years? 1) 10:3 2) 5:2 3) 4:3 4) 3:1 5) None of these Ans 4 13) 42 Percent of first no is the 56 Percent of the second no.What is the Respective ratio of the first no to the Second no ? 1) 4:5 2) 31:42 3) 4:3 4) Cannot be determined 5) None of these Ans 3 14) one-fifth of a number is 48.What is the 62% of that no? 1) 164.66 2) 148.8 3) 178.22 4) 182.44 5) None of these Ans 2 15) If an Amount of Rs 85,470 /- is distribut SBI BANK PO (PT) SOLVED PAPER (Held on July 6 , 2008) General English Directions—(Q. 41–50) Read the following passage carefully and answer the questions given below it. Certain words are printed in bold to help you to locate them while answering some of the questions. The yearly festival was close at hand. The store room was packed with silk fabrics. gold ornaments, clay bowls full of sweet curd and platefuls of sweetmeats. The orders had been placed with shops well in advance. The mother was sending out gifts to everyone. The eldest son, a government servant, lived with his wife and children in far off lands. The second son had left home at an early age. As a merchant he travelled all over the world. The other sons had split up over petty squabbles, and they now lived in homes of their own. The relatives were spread all across the world. They rarely visited. The youngest son, left in the company of a servant, was soon bored and stood at the door all day long, waiting and watching. His mother, thrilled and excited, loaded the presents on trays and plates, covered them with colourful kerchiefs, and sent them off with maids and servants. The neighbours looked on. The day came to an end. All the presents had been sent off. The child came back into the house and dejectedly said to his mother, “Maa, you gave a present to everyone, but you didn’t give me anything !” His mother laughed, “I have given all the gifts away to everyone, now see what’s left for you.” She kissed him on the forehead. The child said in a tearful voice, “Don’t I get a gift ?” “You’ll get it when you go far away.” “But when I am close to you, don’t I get something from your own hands ?” His mother reached out her arms and drew him to her. “This is all I have in my own hands. It is the most precious of all.” 41. Why did the woman's second son travel ? (A) He was restless by nature (B) He did not want to stay at home (C) He was rich and could afford to travel (D) His job was such that he had to travel (E) None of these 42. Why did the woman’s eldest son not attend the festival ? (A) He was not on good terms with his youngest brother who lived at home (B) He had quarrelled with his mother (C) His wife did not allow him to return home (D) His job prevented him from taking leave (E) None of these 43. How did the woman prepare for the festival ? 1. She bought expensive gifts for her children and neighbours. 2. She ordered her servants to prepare sweets and food well in advance. 3. She made sure that her youngest child was looked after so that he wouldn’t be bored. (A) None (B) Only 1 (C) Only 2 (D) Both 1 and 2 (E) All 1, 2 and 3 44. What did the youngest child do while his mother was busy ? 1. He waited for a chance to steal some sweetmeats. 2. He pestered his mother to give him a present. 3. He stood at the door with servants. (A) Only 1 (B) Only 2 (C) Both 1 and 3 (D) Only 3 (E) None of these 45. Which of the following can be said about the woman ? (A) She was a widow who had brought up her children single handedly (B) She was not a good mother since her children had left home at an early age (C) She enjoyed sending her family gifts at festival time (D) She gave expensive presents to show that she was wealthy (E) She rarely visited her grand-children because they all lived abroad 46. What did the boy receive from his mother ? (A) She taught him the value of patience (B) She encouraged him to grow up and live independently like his brothers (C) She showed him the importance of giving expensive gifts (D) She gave him a hug to express her love (E) None of these 47. Which of the following is TRUE in the context of the passage ? (A) The woman usually ignored her youngest son (B) The woman’s eldest son lived abroad (C) The members of the woman’s family did not care about her (D) The woman made all the preparations herself since she did not want to burden the servants (E) The woman sent gifts to her children to ensure that they visited her Directions—(Q. 48–49) Choose the word which is most nearly the SAME in meaning as the word printed in bold as used in the passage. 48. Left (A) Gone (B) Quit (C) Remaining (D) Disappeared (E) Forgot 49. Packed A) Filled (B) Squeezed (C) Crowd (D) Collected (E) Untidy 50. Choose the word which is most OPPOSITE in meaning of the word dejectedly as used in the passage. (A) Calmly (B) Happily (C) Willingly (D) Fortunately (E) Softly Directions—(Q. 51–55) Read each sentence to find out whether there is any error in it. The error, if any, will be in one part of the sentence. The letter of that part is the answer. If there is no error, the answer is (E). (Ignore errors of punctuation, if any) 51. Many multinational companies (A) / have not been as (B) /successful in India (C) /than we expected. (D) No error (E) 52. He has ruined (A) /his eyesight (B) /by not using (C) /his spectacles regularly. (D) No error (E) 53. Mostly of the (A) /newly recruited officers (B) /have no experience (C) /in the banking sector. (D) No error (E) 54. The resignation of (A) /one of our directors (B) /have caused the price (C) / of shares to fall. (D) No error (E) 55. There are many (A) /ways of which (B) /inflation can (C) /be measured. (D) No error (E) Directions—(Q. 56–60) Which of the phrases (A), (B), (C) and (D) given below should replace the phrase given in bold in the following sentence to make the sentence grammatically meaningful and correct. If the sentence is correct as it is and ‘No correction is required.’ mark (E) as the answer. 56. Each of the loan must be approved by the Branch Manager— (A) Every loan (B) Each one of the loan (C) Any of the loan (D) All of the loan (E) No correction required 57. The issue was taken before the Municipal Corporation meeting last week— (A) Taking place at (B) Taken after (C) Being taken in (D) Taken up at (E) No correction required 58. He has asked for the names of those employees involved in the project. (A) had asked (B) having asked about (C) was asked that (D) is asking (E) no correction required 59. Considerate the traffic, it is better to leave for the airport an hour early— A) While considering (B) Consideration of (C) Considering (D) Being considerate to (E) No correction required SBI BANK PO (PT) SOLVED PAPER (Held on July 6 , 2008) General English 60. He is a good leader, knowing that to motivate his employees to achieve— (A) That known when (B) Who knows how (C) Which knows how (D) Knowing what (E) No correction required Directions—(Q. 61–65) Rearrange the following six sentences (1), (2), (3), (4), (5) and (6) in the proper sequence to form a meaningful paragraph; then answer the questions given below them. (1) The able bodied men of the tribe gathered to discuss how to climb the mountain. (2) As part of their plundering they kidnapped a baby of one of the families. (3) One day the mountain tribe invaded those living in the valley. (4) “We couldn’t climb the mountain. How could you?”, they asked, “It wasn’t your baby !” she replied. (5) There were two tribes in the Andes –one lived in the valley and the other high up in the mountains. (6) Two days later they noticed the child’s mother coming down the mountain that they hadn’t yet figured out how to climb. 61. Which of the following should be the SECOND sentence after rearrangement ? (A) (1) (B) (2) (C) (3) (D) (4) (E) (5) 62. Which of the following should be the FIFTH sentence after rearrangement ? (A) (6) (B) (5) (C) (4) (D) (3) (E) (2) 63. Which of the following should be the FIRST sentence after rearrangement ? (A) (1) (B) (2) (C) (3) (D) (4) (E) (5) 64. Which of the following should be the SIXTH (LAST) sentence after rearrangement ? (A) (1) (B) (2) (C) (3) (D) (4) (E) (5) 65. Which of the following should be the THIRD sentence after rearrangement ? (A) (1) (B) (2) (C) (3) (D) (4) (E) (5) Directions—(Q. 66–70): In each question below a sentence with four words printed in bold type is given. These are lettered (A), (B), (C) and (D). One of these four words printed in bold may be either wrongly spelt or inappropriate in the context of the sentence. Find out the word, which is wrongly spelt or inappropriate, if any. The letter of that word is your answer. If all the words printed in bold are correctly spelt and also appropriate in the context of the sentence mark (E) i.e., all correct as your answer. 66. The income (A) of many people in rural (B) India is not adequate (C) to satisfy (D) their basic needs. All correct (E) 67. He is always (A) prompt (B) in caring (C) out instructions. (D) All correct (E) 68. The revized (A) rates (B) of interest will be effective (C) immediately. (D) All correct (E) 69. Such transactions (A) are quiet (B) expensive (C) and time consuming (D) for customers. All correct (E) 70. The guidelines (A) of the new scheme (B) are expected (C) to be finally (D) soon. All correct (E) Directions—(Q. 71–80) In the following passage there are blanks each of which has been numbered. These numbers are printed below the passage and against each five words/ phrases are suggested one of which fits the blank appropriately. Find out the appropriate word in each case. I used to look …(71)… to the holidays. I was usually …(72)… to my uncle’s house where I …(73)… his children. I did not get paid a salary for …(74)… What I received in return however, was far more …(75)… My uncle was an avid reader. During the time I spent with his family I had an …(76)… to read the vast amount of books and magazines that he possessed. This improved my English to some …(77)… Reading became my new …(78–79)… spending my pocket money on a ticket to the cinema I began to …(80)… books. This has benefited me greatly. 71. (A) forward (B) towards (C) backward (D) up (E) around 72. (A) went (B) sent (C) visited (D) travelled (E) gone 73. (A) cared (B) occupy (C) guarded (D) taught (E) played 74. (A) them (B) whom (C) this (D) now (E) which 75. (A) expensive (B) deserving (C) helping (D) demanding (E) valuable 76. (A) opportunity (B) ability (C) use (D) encouragement (E) achievement 77. (A) distance (B) extent (C) time (D) limits (E) degrees 78. (A) activity (B) hope (C) hobby (D) duty (E) worship 79. (A) despite (B) though (C) by (D) instead of (E) while 80. (A) sell (B) read (C) exchange (D) invest (E) buy Answers: 41. (D) 42. (E) 43. (A) 44. (D) 45. (C) 46. (D) 47. (B) 48. (C) 49. (A) 50. (C) 51. (D) Replace ‘than’ with ‘as’. 52. (E) 53. (A) Change ‘Mostly’ to ‘Most’. 54. (C) Change ‘have’ to ‘has’. 55. (B) Replace ‘of’ with ‘in’. 56. (A) 57. (D) 58. (E) 59. (C) 60. (B) 61. (C) 62. (A) 63. (E) 64. (D) 65. (B) 66. (E) 67. (C) 68. (A) 69. (E) 70. (D) 71. (A) 72. (B) 73. (D) 74. (C) 75. (E) 76. (A) 77. (B) 78. (C) 79. (D) 80. (E) Whole-Testpaper SBI Clerk Exam (Held on 13-07-2008) General Awareness 1. Oscar Awards are given for the excellence in the field of - (A) films (B) Literature (C) Sports (D) Politics (E) Status ANS (A) 2. Which of the following is not a food crop ? (A) Wheat (B) Barley (C) Maize (D) Gram (E) Cotton ANS (E) 3. Union Budget always presented first in - (A) The Loksabha (B) The Rajyasabha (C) Joint session of Parliament (D) Meeting of the Union cabinet (E) The State Assemblies ANS (A) 4. The Government of India has earmarked about Rs. 70,000 crore in Union Budget to help which of the following sections of the society ? (A) Children who are bonded labour (B) Persons working in hazardous industries (C) Farmers (D) People living below poverty line (E) None of these ANS (E) 5. Jose Louise has taken over as the prime minister of - (A) France (B) Argentina (C) Spain (D) New zealand (E) Italy ANS (C) 6. The conference of Economic/ Finance Ministers of ASEAN was held recently in - (A) Jakarta (B) Bali (C) New Delhi (D) Tokyo (E) Islamabad ANS (B) 7. Which of the following International forums/ organizations has made a decision of not to go for reckless lending ? A) G-7 (B) G-8 (C) SAARC (D) World Bank (E) IMF ANS (E) 8.Mahmood Ahmadinejad who was in India on an official visit recently is the - (A) Prime Minister of Iraq (B) President of Iran (C) Prime Minister of Iran (D) President of Iraq (E) None of these ANS (B) 9. India recently started "Maitree Express" to which of the following destinations ? (A) Islamabad (B) Karanchi (C) Dhaka (D) Kathmandu (E) None of these ANS (C) 10. The first ever General elction took place in which of the following countries in Indian sub-continent ? (A) Bhutan (B) China (C) Pakistan (D) Bangladesh (E) Sri Lanka ANS (A) 11. As per the latest figures published in the newspapers the growth of economy in India for the fiscal year ended March 2008 has been at which of the following levels ? About - (A) 6 % (B) 6.5 % (C) 7 % (D) 7.5 % (E) 9 % ANS (E) 12. Name of B.S.Yeddyurappa was recently in news as he has taken over as the - (A) Governor of Karnataka (B) Chief Minister of Karnataka (C) Governor of Andhra Pradesh (D) Chief Minister of Andhra Pradesh (E) None of these ANS (B) 13. The Govt. of India Planning to put up a bill in the parliament to ensure free and compulsory education for all those who are of the age of - (A) 3 Years (B) 5 Years (C) 6-14 Years (D) 12-20 Years (E) Upto 21 Years ANS (C) 14. "World No Tobacco Day" is observed on which of the following days ? (A) Ist May (B) 10th May (C) 21st May (D) 31 st May (E) 1st June ANS (D) 15. Late Vijay Tendulkar who died recently was a famous - (A) Social Work (B) Politician (C) Sportsman (D) Play Wright (E) None of these ANS (D) 16. Mnay times we erad in newspapers about the GM Crops.What is the full form of GM ? (A) Genetically Marketed (B) Genetically Modified (C) Green & Moisturious (D) Globally Marketed (E) None of these ANS (B) 17. Barack Obama whose name was in news recently is from which of the following countries ? (A) UK (B) USA (C) France (D) Italy (E) None of these ANS (B) 18. Which of the following is an Oscar winning documentary on climate change in which former US Vice President Al Gore has featured as a narrator ? A) An inconvenient truth (B) The Sea (C) Road to prediction (D) Remember My Name (E) None of these ANS (A) 19. Defence minister from 27 nations recently gathered at which of the following places to discuss security amongst the countries of Asia Pacific Region ? (A) Beijing (B) New Delhi (C) Singapore (D) Kuala Lumpur (E) None of these ANS (C) 20. Who amongst the following is NOT a Lawn Tennis Player ? (A) Serena Williams (B) Katarina Srebotnik (C) Andy Murray (D) Nichlas Almagro (E) Ricky Ponting ANS (E) 21. The head office of the Stae Bank Of India is located in - A) Kolkat (B) New Delhi (C) Pune (D) Ahmedabad (E) None of these ANS (E) Mumbai 22. Which of the following is NOT the name of the sensitive index of any global stock exchange ? (A) Nasdaq (B) Nikkei (C) Kospi (D) Dow (E) Combix ANS (E) 23. Mnay times we read the term 'ECB'in financial newspapers.What is the full form of 'ECB'? (A) Essential Commercial Borrowing (B) Essential Credit & Borrowing (C) External Credit & Business (D) External Commercial Borrowing (E) None Of These ANS (D) 24. Who amongst the following is the new face in Union Cabinet after it was reshuffled recently ? A) Mnai Shankar Aiyer (B) Rahul Gandhi (C) Priyanka (D) Jyotiraditya Scindia (E) None of these ANS (D) 25. Who amongst the following has taken over as the chief minister of Nagaland ? (A) Manik Sarkar (B) Dinesh Nandan Sahay (C) P.Longon (D) K.Shankaranarayanan (E) None of these ANS (E) SBI BANK CLERK MODEL QUESTIONS General English 1. Read each sentence to find out whether there is any error in it. If there is no error, the answer is 'd' (Ignore errors of punctuation, if any) (a) You would not have used (b) such an insulting language (c) if you would have been accompanied (d) by your elder brother. (e) No error Answer - c 2.Read each sentence to find out whether there is any error in it. If there is no error, the answer is 'd' (Ignore errors of punctuation, if any) All the furnitures have been (a)/ replaced by the owner of the house (b)/ before shifting. (c)/ No error. (d) Answer - a 3.Read each sentence to find out whether there is any error in it. If there is no error, the answer is 'd' (Ignore errors of punctuation, if any) As soon as I shall reach New Delhi (a)/ I shall send you the file (b)/ you have asked for. (c)/ No error. (d) Answer - a 4.In each of these questions, the given sentence has a blank space. Four alternative words (a), (b), (c) and (d) are given after the sentence. Select from the alternatives that word as your answer which you consider most appropriate for the blank space. He does not suffer from any ....... disease at all. a. Acute b. Chronic c. Temporary d. Irregular Answer - b 5.In each of these questions, the given sentence has a blank space. Four alternative words (a), (b), (c) and (d) are given after the sentence. Select from the alternatives that word as your answer which you consider most appropriate for the blank space. We are not sure if their business is strictly ......... a. Prudent b. Honest c. Judicious d. Legitimate Answer - a 6.In each of these questions, the given sentence has a blank space. Four alternative words (a), (b), (c) and (d) are given after the sentence. Select from the alternatives that word as your answer which you consider most appropriate for the blank space. The claims made by the manufacturer of the product are so extravagant that only the ...... will accept them on face value. a. Cynical b. Gullible c. Rich d. Indifferent Answer - b 7.In each of these questions, the given sentence has a blank space. Four alternative words (a), (b), (c) and (d) are given after the sentence. Select from the alternatives that word as your answer which you consider most appropriate for the blank space. Justice should be ..... even-handedly. a. Discharged b. Performed c. Declared d. Administered Answer - d 8.In each of these questions, the given sentence has a blank space. Four alternative words (a), (b), (c) and (d) are given after the sentence. Select from the alternatives that word as your answer which you consider most appropriate for the blank space. The leader of the opposition party condemned the communal riots in ... terms. a. Unambiguous b. Unparliamentary c. Unequivocal d. Uncontrollable Answer - c 9.In each of these questions, the given sentence has a blank space. Four alternative words (a), (b), (c) and (d) are given after the sentence. Select from the alternatives that word as your answer which you consider most appropriate for the blank space. They should spend less time ... about and more with their children. a. Gallivanting b. Gravitating c. Frisking d. Flirting Answer - c 10.In each of these questions, the given sentence has a blank space. Four alternative words (a), (b), (c) and (d) are given after the sentence. Select from the alternatives that word as your answer which you consider most appropriate for the blank space. The Government's handling of the situation led to ...... violations of human rights. a. Tumultuous b. Obvious c. Ostensible d. Flagrant Answer - d 11.In each of these questions, the given sentence has a blank space. Four alternative words (a), (b), (c) and (d) are given after the sentence. Select from the alternatives that word as your answer which you consider most appropriate for the blank space. A bullet ........ his cheek. a. Grazed b. Glazed c. Grasped d. Screamed Answer - a 12.In each of these questions, the given sentence has a blank space. Four alternative words (a), (b), (c) and (d) are given after the sentence. Select from the alternatives that word as your answer which you consider most appropriate for the blank space. The injured man was still groggy and could only give a ..... account of the accident. a. Garish b. Gangling c. Garbled d. Garrulous Answer - c 13. In each of these questions, the given sentence has a blank space. Four alternative words (a), (b), (c) and (d) are given after the sentence. Select from the alternatives that word as your answer which you consider most appropriate for the blank space. The teacher's counsel had a ........ effect on the mischievous student's conduct. a. Memorable b. Salutary c. Sudden d. Forceful Answer - b 14.In these questions, pick out the most effective word from the given words to fill in the blank to make the sentence meaningfully complete. The majority of village folk in India are illiterate and .... superstitious. (a) Overtly (b) Delicately (c) Stubbornly (d) Covertly (e) Critically Answer - c 15.In these questions, pick out the most effective word from the given words to fill in the blank to make the sentence meaningfully complete. I must admit, my parents stood by me in my times of ...... . (a) Passion (b) Duress (c) Involvement (d) Development (e) Criticism Answer - b BANK CLERK MODEL QUESTION PAPER 2 NUMERICAL ABILITY TEST 1. Sum of smallest six digit no. and greatest five digit no. is: a. 199999 b. 201110 c. 211110 d. 1099999 e. None of these 2. Value of 112 * 54. is : a. 6700 b. 70000 c. 76500 d. 77200 e. None of these 3. 1399*1399 a. 1687401 b. 1901541 c. 1943211 d. 1957201 e. None of these 4. When a no. is multiplied by 13 product consist of all 5’s. The smallest such no. is a. 41625 b. 42135 c. 42515 d. 42735 e. None of these 5. If n is –ve no. then which of the following is least. a. 0 b. –n c. 2n d. n2 e. None of these 6. If -1<=x <=2 and 1<= y <= 3 then least value of 2y-3x is : a. 0 b. -3 c. -4 d. -5 e. None of these 7. The least prime no. is : a. 0 b. 1 c. 2 d. 3 e. None of these 8. The sum of prime no.’s b/w 60 and 75 is: a. 199 b. 201 c. 211 d. 272 e. None of these 9. Total no. of even prime no.’s is : a. 0 b. 1 c. 2 d. None e. None of these 10. How many No’s B/w 400 and 600 begin with or end with digit 5. a. 40 b. 100 c. 110 d. 120 e. None of these 11. The digit in unit’s place of product 81*82……..*89 is: a. 0 b. 2 c. 6 d. 8 e. None of these 12. The sum of first 45 natural no’s is : a. 1035 b. 1280 c. 2070 d. 2140 e. None of these 13. The unit’s digit in the product of 771 * 659 * 3 65 is : a. 1 b. 2 c. 4 d. 6 e. None of these 14. Which no. is exactly divisible by 11 a. 235641 b. 245642 c. 315624 d. 415624 e. None of these 15. The largest natural no. which exactly divides the product of any 4 consecutive natural no’s is: a. 6 b. 12 c. 24 d. 120 e. None of these 16. The diff. b/w squares of 2 consecutive odd integers is always divisible by: a. 3 b. 6 c. 7 d. 8 e. None of these 17. The smallest no. to be added to 1000 so that 45 divides the sum exactly is : a. 10 b. 20 c. 35 d. 80 e. None of these 18. The least no. which must be subtracted from 6709 to make it exactly divisible by 9 is : a. 2 b. 3 c. 4 d. 5 e. None of these 19. Find the no. nearest to 99547 and exactly divisible by 687: a. 98928 b. 99479 c. 99615 d. 100166 e. None of these 20. The least no. by which 72 must be multiplied in order to produce a multiple of 112 is a. 6 b. 12 c. 14 d. 18 e. None of these 21. Which largest no. of 5 digits is divisible by 99: a. 99909 b. 99981 c. 99990 d. 99999 e. None of these 22. A no. when divided by 114 leaves the remainder 21 if same no. is divided by 19 the remainder will be a. 1 b. 2 c. 7 d. 21 e. None of these 23. The diff. b/w 2 no’s is 1365 when larger no. is divided by smaller the quotient is 6 and the remainder is 15. The smaller no. is : a. 240 b. 270 c. 295 d. 360 e. None of these 24. The divisor is 10 times the quotient and 5 times the remainder if remainder is 46 then dividend is : a. 4236 b. 4306 c. 4336 d. 5336 e. None of these 25. A four digit no. divisible by 7 becomes divisible by 3 when 10 is added to it the largest such no. is : a. 9947 b. 9987 c. 9989 d. 9996 e. None of these ANSWERS WITH EXPLANATION : 1) a 2) 1120000/16 = b 3) (1400-1)*(1400-1) = d 4) 555555/13 = d 5)c 6)c 7)c 8)d 9)b 10)c 11)a 12)a 13)c 14)d 15)c 16)d 17)a 18)c 19)c 20)c 21)c 22)d 23)b 24)d 25)b SBI BANK CLERK MODEL TEST PAPER NUMERICAL ABILITY 1.Let R = qs—4. When s=8, R=16. When s=10, R is equal to a. 11 b. 14c. 20 d. 21 Answer-d 2. If 272/3 × 81-1/2 = 3x, the value of x is a. -1 b. 0 c. 1 d. 2 Answer -b 3. 0.333 .... × 0.444 .... is equal to a. 0.148148148 .... b. 0.777 ....c. 0.121212 .... d. 1.333 .... Answer-a 4. Answer-d 5. On converting the following base-2 numeral in base ten, 1101101, we get a. 96 b. 104 c. 108 d. 109 Answer-d 6. The number of prime factors of (6)10×(7)17×(55)27 a. 54 b. 64 c. 81 d. 91 Answer-d 7. A train crosses a pole in 15 seconds, while it crosses 100 meter long platform in 25 seconds. The length of the train is - a. 125m b. 135 m c. 159 m d. 175 m Answer-c 8.Two taps A and B can fill a tank in 12 minutes and 15 minutes respectively. If both the taps are opened simultaneously and the tap A is closed after 3 minutes, then how much more time will it take to fill the tank by tap B? a. 7 min & 15 sec b. 7 min & 45 sec c. 8 min & 5 sec d. 8 min & 15 sec Answer-d 9. The milk and water in two vessels A and B are in the ratio 4:3 and 2:3 respectively. In what ratio, the liquids be mixed in both the vessels so that the new liquid contains half milk and half water? a. 7:5 b. 1:2 c. 2:1 d. 6:5 Answer-a 10.A car covers a distance of 715 km at a constant speed. If the speed of the car would have been 10 km/hr more, then it would have taken 2 hours less to cover the same distance. What is the original speed of the car? a. 45 km/hr b. 50 km/hr c. 55 km/hr d. 65 km/hr Answer-c 11. A person covered some distance in 12 hours. He covered half the distance by rail @ 75 km per hour and the rest by car @ 45 km/hr. The total distance covered by him was a. 450 km b. 675 km c. 337.5 km d. 1350 km Answer-b 12. A sum of Rs. 427 is to be divided among A, B and C in such a way that 3 times A's share, 4 times B's share and 7 times C's share are all equal. The share of C is a. Rs. 84 b. Rs. 147 c. Rs. 196 d. Rs. 240 Answer-a 13. A and B entered into a partnership investing Rs. 12000 and Rs. 9000 respectively. After 3 months C also joined them with a capital of Rs. 15000. The share of C in the half yearly profit of Rs. 9500 is a. Rs. 3500 b. Rs. 3000 c. Rs. 2500 d. Rs. 4000 Answer-c 14.The ratio of income of A and B is 5:4 and their expenditure is as 3:2. If at the end of the year, each saves Rs. 800, then the income of A is a. Rs. 1700 b. Rs. 1800 c. Rs. 2000 d. Rs. 2200 Answer-c 15.A and B can together finish a work in 30 days. They worked at it for 10 days together and then B left. The remaining work was done by A alone in 30 more days. B alone can finish the work in a. 48 days b. 60 days c. 75 days d. 90 days Answer-d 16.The ratio between the curved surface area and the total surface area of a right circular cylinder is 1:2. If the total surface is 616 sq. cm, the volume of the cylinder is a. 1848 cm3 b. 1232 cm3 c. 1078 cm3 d. 980 cm3 Answer-c 17.A circular wire of radius 42 cm is cut and bent in the form of a rectangle whose sides are in the ratio of 6 : 5. The smaller side of the rectangle is a. 30 cm b. 60 cm c. 72 cm d. 108 cm Answer-b 18.A man walking at the rate of 6km per hour crosses a square field diagonally in 9 seconds. The area of the field is- a. 125 sq. cm b. 112.5 sq. cm c. 110 sq. cm d. 100v2 sq. m Answer-b 19.A rectangular carpet has an area of 240 sq. cm. If its diagonal and the longer side are together equal to five times the shorter side, the length of the carpet is - a. 10 cm b. 24 cm c. 26 cm d. 27.5 cm Answer-b 20.The ratio of 435 : 25 is same as a. 4 : 1 b. 2 :1 c. 7 : 5 d. 7 :10 Answer-a 21. A sphere and a cube have equal surface areas. The ratio of the volume of the sphere to that of the cube is a. vp : v6 b. vp : v6 c. vp : v3 d. v6 : vp Answer-d 22.The marked price of a table is Rs. 3000 and is available at successive discounts of 20% and 10% respectively. If there is an additional discount of 5% on cash payment, then what is the cash price of the table? a. Rs. 2400 b. Rs. 2160 c. Rs. 2100 d. Rs. 2052 Answer-d 23. A certain sum becomes Rs. 5290 in 2 years and Rs. 6083.50 in 3 years at C.I. The rate of interest per annum is a. 10% b. 12% c. 15% d. 16? % Answer-c 24.A person borrowed Rs. 500@ 3% per annum S.I and Rs. 600 @4½% per annum on the agreement that the whole amount will be returned only when the total interest becomes Rs. 126. The number of years, after which the borrowed sum is to be returned, is a. 2 b. 3 c. 4 d. 5 Answer-b 25.A sum of Rs. 12,000 doubles in 5 years at C.I. What will be the amount after 20 years? a. Rs. 1,20,000 b. Rs. 1,92,000 c. Rs. 1,24,000 d. Rs. 96,000 Answer-b 26. A person sold 320 mangoes for the C.P. of 400 mangoes. His gain percent is a. 10% b. 15% c. 12½% d. 25% Answer-d 27. A person bought some oranges @ Rs. 10 per dozen and bought the same amount of oranges @ Rs. 8 per dozen. He sold these oranges @ Rs. 11 per dozen and gained Rs. 120. The total number of oranges bought by him was a. 30 dozen b. 40 dozen c. 50 dozen d. 60 dozen Answer-d 28.On selling a certain commodity for Rs. 425, there is as much gain as loss on selling it for Rs. 355. The C.P. of the commodity is a. Rs. 370 b. Rs. 385 c. Rs. 390 d. Rs. 400 Answer-c 29.A sum of Rs. 800 amounts to Rs. 920 in three years at S.I. If the rate of interest is increased by 5% then the amount will increase to a. Rs. 950 b. Rs. 980 c. Rs. 1010 d. Rs. 1040 Answer-d 30.Of the three numbers, second is twice the first and is also thrice the third. If the average of the three numbers is 44, the largest number is a. 24 b. 36 c. 17 d. 72 Answer-d 31.A house owner wants to get his house painted. He is told that this would require 25 kg of paint. Allowing for 15% wastage and assuming that the paint is available in 2kg tins, the number of tins required for painting the house is - a. 15 b. 12 c. 10 d. 20 Answer-a 32.Price of food grains have risen by 10% and of other items of consumption by 15%. If the ratio of an employee's expenditure on food grains and other items is 2:5, by how much should his salary be increased so that he may maintain the same level of consumption as before, assuming that his present salary is Rs. 3500? a. Rs. 300 b. Rs. 350 c. Rs. 375 d. Rs. 475 Answer-d   STATE BANK OF INDIA PAPER ON 27th APRL 2008 Reasoning Ability 1. In a certain code BOARD is written as 53169 and NEAR is written as 2416. How is NODE written in that code? (a) 2394 (b) 2894 (c) 2934 (d) 2694 (e) None of these Ans. (a) 2. What should come next in the following sequence of letters? B B C B C D B C D E B C D E F B C D E F G B C D E F (a) B (b) G (c) H (d) D (e) None of these Ans. (b) 3. In a certain code SPORADIC is written as QNORDJEB. How is TROUBLES written in that code? (a) SQTNTFMC (b) TNQSRDKA (c) TNQSTFMC (d) TFQSCMFT (e) None of these Ans. (c) 4. Four of the following five are alike in a certain way and so form a group. Which is the one that does not belong to that group? (a) Mica (b) Zinc (c) Iron (d) Chlorine (e) Aluminium Ans. (a) 5. if it is possible to make only one meaningful word with the first, the third the fifth and the eighth letters of the word SHAREHOLDING, which of the following will be the second letter of that word? If no such word can be made, give 'X' as the answer and if more than one such word can be made, give 'Y' as the answer (a) L (b) E (c) S (d) X (e) Y Ans. Y 6. Four of the following five are alike in a certain way and so form a group. Which is the one that does not belong to that group? (a) 18 (b) 24 (c) 36 (d) 34 (e) 26 Ans. (c) 7. If 'green' means 'yellow' , 'yellow' means 'white' , ' white' means 'red' , 'red' means 'violet' , 'violet' means 'black' , then which of the following will be the colour of human blood? (a) Red (b) Black (c) Green (d) White (e) None of these Ans. (e) 8. How many such pairs of letters are there in the word: GLIMPSE each of which has as many letters between them in the word as in the English alphabet? (a) None (b) One (c) Two (d) Three (e) More than three Ans. (c) 9. EARN is related to RANE and BOND is related to NODB in the same way as TEAR is related to (a) AERT (b) ATRE (c) ARET (d) REAT (e) None of these Ans. (a) 10. How many such digits are there in the number 5813649 each of which is as far away from the beginning of the number as when the digits are rearranged in the ascending order? (a) None (b) One (c) Two (d) Three (e) More than three Ans. (c) 11. If ‘´’ means 'added to ' ¸’ means 'multiplied by' ; '+' means 'subtracted from' and '-' means 'divided by' then 24+36-12´8¸4=? (a) 36 (b) 53 (c) 5 (d) 20 (e) None of these Ans. (b) 12. Four of the following five are alike in a certain way and so form a group. Which is the one that does not belong to that group? (a) Coconut (b) Lotus (c) Lilly (d) Rose (e) Marigold Ans. (a) Directions (13 - 15) : i) ‘P ´ Q’ means ‘P, is brother of Q’ ii) ‘P - Q’ means “P is sister of Q’ iii) ‘P + Q’ means ‘P is sister of Q’ iv) ‘P ¸ Q’ means ‘P is mother of Q’ 13. Which of the following represents 'M is nephew of N'? (a) N - K + M (b) N ´ K¸ M (c) N ¸ K ´ M (d) N - K + M ´ T (e) None of these Ans. (d) 14. How is T related to D in the expression: H + T ¸ R- D? (a) Nephew (b) Niece (c) Nephew or Niece (d) Data inadequate (e) None of these Ans. (e) 15. Which of the following represents F is daughter of W? (a) W ¸ R+ F (b) W ´ R ´ F (c) W + R ´ F -Y (d) W + R - F + T (e) None of these Ans. (c) Directions (16 - 20) : In each of the questions below are given four statements followed by four conclusions numbered I, II, III and IV. You have to take the given statements to be true even if they seem to be at variance from commonly known facts. Read all the conclusions and then decide which of the given conclusions logically follows from the given statements disregarding commonly known facts 16. Statements : Some bags are trunks All trunks are shirts Some shirts are books All books are shops Conclusions : I. Some shops are bags II. Some books are bags III. Some shops are shirts IV. Some shirts are bags (a) Only I and II follow (b) Only I and III follow (c) Only III and IV follow (d) Only II and IV follow (e) None of these Ans. (c) 17. Statements : All pens are chairs All flowers are chairs All chairs are trucks All trees are trucks Conclusions : I. Some trucks are pens II. Some trucks are chairs III. Some trees are pens IV. Some trees are chairs (a) Only I and III follow (b) Only I and II follow (c) Only III and IV follow (d) Only II and IV follow (e) None of these Ans. (b) 18. Statements : All desks are pillars Some pillars are towns All towns are benches Some benches are cars Conclusions: I. Some cars are towns II. Some benches are desks III. Some benches are pillars IV. Some cars are pillars (a) None follows (b) Only I follows (c) Only II follows (d) Only III follows (e) Only IV follows Ans. (d) 19. Statements : All stations are houses No house is garden Some gardens are rivers All rivers are ponds Conclusions : I. Some ponds are gardens II. Some ponds are stations III. Some ponds are houses IV. No pond is Station (a) Only I follows (b) Only either II or IV follows (c) Only I and II follow (d) Only I and IV follow (e) None of these Ans. (e) 20. Statements : Some towers are lanes Some lanes are roads Some roads are rivers Some rivers are jungles Conclusions : I. Some jungles are roads II. Some roads are lanes III. Some jungles are towers IV. No jungle is road (a) Only I follows (b) Only II follows (c) Only either I or IV follows (d) Only IV follows (e) Only either I or IV and II follow Ans. (e) Directions (21-25) : Study the following arrangement carefully and answer the questions given below- B­A M 3 # D 2 E K 9 $ F @ N I T 4 1 U W Ó H 8% V J 5 Y 6 * 7 R 21. How many such symbols are there in the above arrangement each of which is either immediately preceded by a letter or immediately followed by a letter but not both? (a) None (b) One (c) Two (d) Three (e) More than three Ans. (d) 22. If all the symbols in the above arrangement are dropped which of the following will be the twelfth from the left end? (a) 9 (b) U (c) I (d) 1 (e) None of these Ans. (c) 23. How many such numbers are there in the above arrangement each of which is immediately followed by a consonant but not immediately preceded by a letter? (a) None (b) One (c) Two (d) Three (e) More than three Ans. (b) 24. Four of the following five are alike in a certain way based on their positions in the above arrangement and so form a group. Which is the one that does not belong to that group? (a) D K 3 (b) 4 N U (c) 8 J Ó (d) F K N (e) 5 * % Ans. (e) 25. Which of the following is the seventh to the right of the eighteenth from the right end of the above arrangement? (a) H (b) 9 (c) % (d) E (e) None of these Ans. (a) Directions (26-30) : In each question below is given a group of digits/ symbols followed by four combinations of letters numbered (a), (b), (c) and (d). You have to find out which of the combination correctly represents the group of digits/ symbols based on the following ceding system and the conditions those follow. If digits, give (e) i.e. 'None of these' as the.answer. Digit/ Symbol 1 # $ 9 8 6 % @ Ó 7 2 4 * 3 5 Letter Code B D E N I V R G H K T J P F A Conditions (i) If both the first and the last elements in the group are odd digits then both are to be coded as Y. (ii) If the first element is a symbol and the last element is an even digit, the codes for the first and the last elements are to be interchanged (iii) If the first element is an odd digit, and the last element is a symbol, both are to be coded as 'Z' (iv) If the first element is an even digit and the last element ' is odd digit, both are to be coded as the code for the odd digit 26. 3$95#1 (a) FENADB (b) BENADF (c) ZENADZ (d) FENADF (e) None of these Ans. (e) 27. 8%Ó3#5 (a) YRHFDY (b) ARHFDI (c) ARHFDA (d) YRHFDA (e) None of these Ans. (c) 28. Ó8143*$ (a) EIBJFPH (b) VIBJFPY (c) EIBJFP# (d) ZRGNDZ (e) None of these Ans. (d) 29. 6%@9#3 (a) VRGNDF (b) FRGNDF (c) YRGNDF (d) ZRGNDZ (e) None of these Ans. (b) 30. *$6724 (a) JEVKTP (b) PEVKTJ (c) YEVKTY (d) ZEVKTZ (e) None of these Ans. (a) Directions (31-35) : Each of the questions below consists of a question and two statements numbered I and II given below it. You have to decide whether the data provided in the statements are sufficient to answer the question. Read both the statements and Give answer (a) if the data in statement I alone are sufficient to answer the question, while the data in statement II alone are not sufficient to answer the question. Give answer (b) if the data in statement II alone are sufficient to answer the question, while the data in statement I alone are not sufficient to answer the question. Give answer (c) if the data either in statement I alone or in statement II alone are sufficient to answer the question, Give answer (d) if the data in both the statements I and II together are not sufficient to answer the question. Give answer (e) if the data in both the statements I and II together are necessary to answer the question. 31. How is M related to F? Statements : I. F is sister of N who is mother of R II. M has two brothers of which one is R. Ans. (d) 32. On which date in March was Pravin's father's birthday? Statements : I. Pravin correctly remembers that his father's birthday is after 14th but before 19th March II. Pravin's sister correctly remembers that their father's birthday is after 17th but before 21st March Ans. (e) 33. Among M, N, R and D each having different age who is the youngest? Statements : I. N is younger than only D among them II. T is older than R and younger than M Ans. (e) 34. Village D in which direction of village H? Statements : I. Village H is to the South of village A which is to the South - East of village D II. Village M is to the East of village D and to the North - East of village H Ans. (c) 35. How is 'food' written in a code language? Statements : I. 'Always eat good food' is written as 'ha na pa ta' in that code language II. 'Enjoy eating good food' is written as 'ni ha ja pa' in that code language Ans. (c) Quantitative Aptitude Directions (36-55) : What should come in place of the question mark (?) in the following questions? 36. (-251´ 21´ - 12) + ? = 158.13 (a) 250 (b) 400 (c) 300 (d) 150 (e) None of these Ans. (b) 37. 25.6% of 250 + Ö? = 119 (a) 4225 (b) 3025 (c) 2025 (d) 5625 (e) None of these Ans. (b) 38. 36865 + 12473 + 21045 - 44102 = ? (a) 114485 (b) 28081 (c) 26281 (d) 114845 (e) None of these Ans. (c) 39. [(15.20)2 - 103.04] ¸ ? = 8 (a) 12 (b) 6.5 (c) 8.2 (d) 16 (e) None of these Ans. (d) 40. 0.4 ´ 1.5 0.2 = ? (a) 2 (b) 3 (c) 1.2 (d) 0.3 (e) None of these Ans. (b) 41. 7428 ´ 3/4 ´ 2/9 ´ ? = 619 (a) 0.5 (b) 1.5 (c) 0.2 (d) 2.4 (e) None of these Ans. (a) 42. (560 ¸ 32) ´ (720 ¸ 48) = ? (a) 262.5 (b) 255 (c) 263.5 (d) 271.25 (e) None of these Ans. (a) 43. 748 ´ ? ´ 9 = 861696 (a) 122 (b) 132 (c) 128 (d) 124 (e) None of these Ans. (c) 44. 3.2% of 500 ´ 2.4% of ? = 288 (a) 650 (b) 700 (c) 600 (d) 750 (e) None of these Ans. (d) 45. 333 ´ 33 ´ 3 = ? (a) 32697 (b) 36297 (c) 32679 (d) 39267 (e) None of these Ans. (e) 46. [(36)2 + (47)2] ¸ ? = 28.04 (a) 55 (b) 75 (c) 105 (d) 125 (e) None of these Ans. (d) 47. 3Ö? = (28 ´ 24) + 14 (a) 85184 (b) 140608 (c) 97336 (d) 117649 (e) None of these Ans. (e) 48. 14785 - 358 - 4158 - 9514 =? (a) 755 (b) 825 (c) 721 (d) 785 (e) None of these Ans. (a) 49. 156 + 328 ´ 0.8 =? (a) 287.2 (b) 352.8 (c) 418.4 (d) 451.2 (e) None of these Ans. (c) 50. 1148 ¸ 28 ´ 1408 ¸ 32 =? (a) 1800 (b) 1804 (c) 1814 (d) 1822 (e) None of these Ans. (b) 51. 3Ö148877 = ? (a) 51 (b) 59 (c) 53 (d) 49 (e) None of these Ans. (c) 52. 1425 + 8560 + 1680 ¸ 200 = ? (a) 58.325 (b) 9973.4 (c) 56.425 (d) 9939.4 (e) None of these Ans. (e) 53. [(12)2 ´ (14)2 ¸ (16)2 = ? (a) 282.24 (b) 1764 (c) 126 (d) 104.25 (e) None of these Ans. (e) 54. 2 4/25 + 3 1/5 ? (a) 1 7/15 (b) 6 4/25 (c) 6 4/15 (d) 5 9/25 (e) None of these Ans. (d) SBI question papers SBI and SBT specialists officers recruitment examination questions and answers SBT and SBI previous years solved question papers, SBI current affairs questions and answers, SBI aptitude questions and answers, SBI reasoning questions and answers, SBT and SBI specialists officers recruitment procedure, Test pattern Question bank SBI Associate Bank clerical cadre, specialists officers cadre, probationay officers solved question papers 1 Four of the following five are alike in a certain way and so form a group. Which is the one that does not belong to that group ? (A) 19(B) 17(C) 23(D) 29(E) 27 Ans: . (E) Exp All the rest are prime numbers 2 How many meaningful English words can be made with the letters TEBI using each letter only once in each word ? (A) None(B) One(C) Two(D) Three(E) More than three Ans: . (B) Exp B I T E 3 In a certain code LONG is written as 5123 and GEAR is written as 3748. How is LANE written in that code ? (A) 5427(B) 5247(C) 5847(D) 5237(E) None of these Ans:(A) Exp LONG GEAR L A N E 5 1 2 3 3 7 4 8 SBI Whole-Testpaper SBI PO Exam - Solved Previous Year Question Paper Reasoning Ability: Directions (Q. 1-5): Read each statement carefully and answer the following questions: 1. Which of the following expressions will be true if the expression R > O = A > S < T is definitely true? 1) O > T 2) S < R 3) T > A 4) S = O 5) T < R 2. Which of the following symbols should replace the question mark (?) in the given expression in order to make the expressions P > A as well as T ? L definitely true? P > L ? A? N = T 1) ? 2) > 3) < 4) ? 5) Either ? or < 3. Which of the following symbols should be placed in the blank spaces respectively (in the same order from left to right) in order to complete the given expression in such a manner that makes the expressions B > N as well as D ? L definitely true? B _ L _ O _ N _ D 1) =, =, ?, ? 2) >, ?, =, > 3) >, <, =, ? 4) >, =, =, ? 5) >, =, ?, > 4. Which of the following should be placed in the blank spaces respectively (in the same order from left to right) in order to complete the given expression in such a manner that makes the expression A < P definitely false? ___ ? ___ < ___ > ___ 1) L, N, P, A 2) L, A, P, N 3) A, L, P, N 4) N, A, P, L 5) P, N, A, L 5. Which of the following symbols should be placed in the blank spaces respectively (in the same order from left to right) in order to complete the given expression in such a manner that makes the expression F > N and U > D definitely false? F __ O __ U __ N __ D 1) <, <, >, = 2) <, =, =, > 3) <, =, =, < 4) ?, =, =, ? 5) >, >, =, < Directions (Q. 6 - 10): Study the following information and answer the following questions: A, B, C, D, E, G, and I are seven friends who study in three different standards, namely 5th, 6th, and 7th, such that not less than two friends study in the same standard. Each friend has a different favourite subject, namely History, Civics, English, Marathi, Hindi, Maths and Economics also but not necessarily in the same order. A likes Maths and studies in the 5th standard with only one other friend who likes Marathi. I studies with two other friends. Both the friends who study with I like languages (here languages include only Hindi, Marathi and English). D studies in the 6th standard with only one person and does not like Civics. E studies with only one friend. The one who likes History does not study in the 5th or 6th standard. E does not like languages. C does not like English, Hindi or Civics. 6. Which combination represents E's favourite subject and the standard in which he studies? 1) Civics and 7th 2) Economics and 5th 3) Civics and 6th 4) History and 7th 5) Economics and 7th 7. Which of the following is I's favourite subject? 1) History 2) Civics 3) Marathi 4) Either English or Marathi 5) Either English or Hindi 8. Who among the following studies in the 7th standard? 1) G 2) C 3) E 4) D 5) Either D or B 9. Which of the following combinations is definitely correct? 1) I and Hindi 2) G and English 3) C and Marathi 4) B and Hindi 5) E and Economics 10. Which of the following subjects does G like? 1) Either Maths or Marathi 2) Either Hindi or English 3) Either Hindi or Civics 4) Either Hindi or Marathi 5) Either Civics or Economics Directions (Q. 11 - 15): Study the information and answer the following questions. In a certain code language 'economics is not money' is written as 'ka la ho ga','demand and supply economics' is written as 'mo ta pa ka', 'money makes only part' is written as 'zi la ne ki' and 'demand makes supply economics' is written as 'zi mo ka ta'. 11. What is the code for 'money' in the given code language? 1) ga 2) mo 3) pa 4) ta 5) la 12. What is the code for 'supply' in the given code language? 1) Only ta 2) Only mo 3) Either pa or mo 4) Only pa 5) Either mo or ta 13. What may be the possible code for 'demand only more' in the given code language? 1) xi ne mo 2) mo zi ne 3) ki ne mo 4) mo zi ki 5) xi ka ta 14. What may be the possible code for 'work and money' in the given code language? 1) pa ga la 2) pa la tu 3) mo la pa 4) tu la ga 5) pa la ne 15. What is the code for 'makes' in the given code language? 1) mo 2) pa 3) ne 4) zi 5) ho Directions (Q. 16 - 20): Study the given information and answer the following questions: When a word and number arrangement machine is given an input line of wordsand numbers, it arranges them following a particular rule. The following is an illustration of input and rearrangement (All the numbers are two-digit numbers). Input: 40 made butter 23 37 cookies salt extra 52 86 92 fell now 19 Step I: butter 19 40 made 23 37 cookies salt extra 52 86 92 fell now Step II: cookies 23 butter 19 40 made 37 salt extra 52 86 92 fell now Step III: extra 37 cookies 23 butter 19 40 made salt 52 86 92 fell now. Step IV: fell 40 extra 37 cookies 23 butter 19 made salt 52 86 92 now. Step V: made 52 fell 40 extra 37 cookies 23 butter 19 salt 86 92 now. Step VI: now 86 made 52 fell 40 extra 37 cookies 23 butter 19 salt 92 Step VII: Salt 92 now 86 made 52 fell 40 extra 37 cookies 23 butter 19 Step VII is the last step of the above arragement as the intended arrangement is obtained. As per the rules followed in the given steps, find out the appropriate steps for the given input. Input: 32 proud girl beautiful 48 55 97 rich family 61 72 17 nice life. 16. How many steps will be required to complete the given input? 1) Five 2) Six 3) Seven 4) Eight 5) Nine 17. Which of the following is the third element from the left end of step VI? 1) beautiful 2) life 3) 61 4) nice 5) 17 18. Which of the following is step III of the given input? 1) proud 72 girl 48 family 32 beautiful 17 55 97 rich 61 nice life. 2) Life 55 girl 48 family 32 beautiful 17 proud 97 rich 61 72 nice 3) girl 48 family 32 beautiful 17 proud 55 97 rich 61 72 nice life 4) family 32 beautiful 17 proud girl 48 55 97 rich 61 72 nice life 5) girl 48 life 55 family 32 beautiful 17 proud 97 rich 61 72 nice 19. What is the position of 'nice' from the left end in the final step? 1) Fifth 2) Sixth 3) Seventh 4) Eighth 5) Ninth 20. Which element is third to the right of 'family' in Step V? 1) beautiful 2) 17 3) proud 4) 97 5) 32 General Awareness: 1. Who was the Captain of Australian Cricket Team which currently (March 2013) visited India? 1) Michael Clarke 2) Shane Watson 3) Shane Warne 4) Michael Hussey 5) Ricky Ponting 2. Government, as part of the recapitalisation plan, infused capital in State Bank of India recently. Indicate the approximate capital infusion. 1) Rs.500 Cr. 2) Rs.1,000 Cr. 3) Rs.1,500 Cr. 4) Rs.2,000 cr. 5) Rs.3,000 Cr. 3. The NRIs while investing in the equity of a company cannot buy more than a prescribed percentage of the paid-up capital of that company. What is the prescribed percentage? 1) 2% 2) 3% 3) 4% 4) 5% 5) 6% 4. Every year March 20 is celebrated as what day? 1) World Sparrow Day 2) International Women's Day 3) World Cuckoo Day 4) International Children's Day 5) International Mother's Day 5. One of the famous Indian sportspersons released his/her autobiography 'Playing to Win' in November 2012. Name the sportsperson correctly. 1) Saina Nehwal 2) Mary Kom 3) Yuvraj Singh 4) Sachin Tendulkar 5) Sushil Kumar 6. Which of the following terms is associated with the game of Cricket? 1) Bouncer 2) Love 3) Goal 4) Mid Court 5) Collision 7. Who is the author of the book Women of Vision? 1) Ravinder Singh 2) Preeti Shenoy 3) Amish Tripathi 4) Durjoy Dutta 5) Alam Srinivas 8. Prof. Muhammad Yunus, the recipient of the Nobel Peace Prize 2006, is the exponent of which of the following concepts in the filed of banking? 1) Core Banking 2) Micro Credit 3) Retail Banking 4) Real Time Gross Settlement 5) Internet Banking 9. Invisible export means export of 1) Services 2) Prohibited goods 3) Restricted goods 4) Good as per OGL list 5) Other than those given as options 10. The European Union has adopted which of the following as a common currency? 1) Dollar 2) Dinar 3) Yen 4) Euro 5) Peso 11. Banks is India are required to maintain a portion on of their demand and time liabilities with the Reserve Bank of India. This portion is called 1) Reverse Repo 2) Cash Reserve Ratio 3) Bank Deposit 4) Statutory Liquidity Ratio 5) Government Securities 12. Pre-shipment finance is provided by the banks only to 1) Credit Card Holders 2) Students aspiring for further studies 3) Brokers in equity market 4) Village Artisans 5) Exporters 13. Banking Ombudsman is appointed by 1) Government of India 2) State Governments 3) RBI 4) ECGC 5) Exim Bank 14. The Holidays for the Banks are declared as per 1) Reserve Bank Act 2) Banking Regulation Act 3) Negotiable Instruments Act 4) Securities and Exchange Board of India Act 5) Companies Act 15. Interest on Savings deposit nowadays is 1) Fixed by the RBI 2) Fixed by the respective Banks 3) Fixed by the depositors 4) Fixed as per the contract between Bank and the Consumer Court 5) Not paid by he Bank 16. Interest below which a bank is not expected to lend to customers is known as 1) Deposit Rate 2) Base Rate 3) Bank Rate 4) Prime Lending Rate 5) Discount Rate 17. The customers by opening and investing in the Tax Saver Deposit Account Scheme in a Bank would get benefit under 1) Sales Tax 2) Customs Duty 3) Excise Duty 4) Professional Tax 5) Income Tax 18. In Banking business, when the borrowers avail a Term Loan, initially they are given a repayment holiday and this is referred as 1) Subsidy 2) Interest waiver 3) Re-phasing 4) Interest concession 5) Moratorium 19. One of the IT companies from India has become the first Indian Company to trade on NYSE Euronext London and Paris markets from February 2013 onwards. Which is this company? 1) Wipro Infotech Ltd. 2) L&T Infotech 3) HCL Technologies Ltd. 4) Infosys Technologies Ltd. 5) Polaris Financial Technology Ltd. 20. BSE (Bombay Stock Exchange), the oldest stock exchange in Asia, has joined hands with one more international index in February 2013. This association has resulted in change of name of BSE index. What is the change of name effected? 1) Dow Jones BSE Index 2) NASDAQ BSE Index 3) S&P BSE Index 4) Euronext BSE Index 5) Other than those given as options 21. A non-performing asset is 1) Money at call and short notice 2) An asset at cease to generate income 3) Cash balance in till 4) Cash balance with the RBI 5) Balance with other banks 22. RBI released its guidelines for entry of new banks in the private sector in the month of Febraury 2013. One of the norms is at least a prescribed percentage of branches of a new bank should be set up in unbanked rural centres with a population upto 9,999. What is the percentage of such branches prescribed in the norms? 1) 10% 2) 15% 3) 18% 4) 25% 5) Other than those given as options 23. The Union Budget for 2013-14 proposed by the Finance Minister on 28th February 2013 announced introduction of a new variety of bonds by the Government. What is the name of these bonds? 1) Deep Discount Bonds 2) Zero Coupon bonds 3) Bullet Bonds 4) Inflation Indexed Bonds 5) Inflation Variable Bonds 24. Government usually classifies its expenditure in terms of planned and non-planned expenditure. Identify which is the correct definition of planned expenditure. 1) It represent the expenditure of all the State Governments. 2) It represents the total expenditure or the Central Government. 3) It is the expenditure which is spent through centrally sponsored programmes and flagship schemes of the Government. 4) It reprensents the expenditure incurred on Defence. 5) Other than those given as options 25. Which of the following organisations is made specifically responsible for empowering Micro, Small and Medium enterprises in India? 1) NABARD 2) RBI 3) SIDBI 4) ECGC 5) SEBI Directions (Q. 1-10): Read the following passage carefully and answer the questions given below it. Certain words have been printed in bold to help you locate them while answering some of the questions. Financial Inclusion (FI) is an emerging priority for banks that have nowhere else to go to achieve business growth. The viability of FI Business is under Question, because while banks and their delivery partners continue to make investments, they haven't seen commensurate returns. In markets like India, most programmes are focussed on customer on boarding, an expensive process which people often find difficult to afford, involving issuance of smart cards to the customers. However, largescale customer acquisition hasn't translated into large-scale business, with many accounts lying dormant and therefore yielding no return on the bank's investment. For the same reason. Business Correspondent Agents, who constitute the primary channel for financial inclusion, are unable to pursue their activity as a full-time job. One major reason for this state of events is that the customer on-boarding process is often delayed after the submission of documents (required to validate the details of the concerned applicant) by the applicant and might take as long as two weeks. By this time initial enthusiasm of applicants fades away. Moreover, the delivery partners don't have the knowledge and skill to propose anything other than the most basic financial products to the customer and hence do not serve their banks' goal to expanding the offering in unbanked markets. Contrary to popular perception, the inclusion segment is not a singular impoverished, undifferentiated mass and it is important to navigate its diversity to identify the right target customers for various programmes. Rural markets do have their share of rich people who do not use banking services simply because they are inconvenient to access or have low perceived value. At the same time, urban markets, despite a high branch density, have multitude of low wage earners outside the financial net. Moreover, the branch timings of banks rarely coincide with the off-work horus of the labour class. Creating affordability is crucial in tapping the unbanked market. No doubt pricing is a tool, but banks also need to be innovative in right-sizing their proposition to convince customers that they can derive big value even from small amounts. One way 'of' doing this is to show the target audience that a bank account is actually a lifestyle enabler, a convenient and safe means to send money to family or make a variety of purchases. Once banks succeed in hooking customers with this value proposition they must sustain their interest by introducing a simple and intuitive user application, ubiquitous access over mobile and other touch points, and adopting a banking mechanism which is not only secure but also reassuring to the customer. Technology is the most important element of financial inclusion strategy and an enabler of all others. The choice of technology is therefore a crucial decision, which could make or mar the agenda. Of the various selection criteria, cost is perhaps the most important. This certainly does not mean buying the cheapest package, but rather choosing that solution which by scaling transactions to huge volumes reduces per unit operating cost. An optimal mix of these strategies would no doubt offer an innovative means of expansion in the unbanked market. 1. Which of the following facts is true as per the passage? 1) People from rural areas have high perceived value of banking services. 2) Cost is not a valid Criterion for technological package selection for financial inclusion initiatives. 3) The inclusion segment is a singular impoverieshed, undifferentiated mass. 4) The branch timings of banks generally do not coincide with the off-work hours of the labour class in urban markets. 5) All the given statements are true. 2. According to the passage, for which of the following reasons do the delivery partners fail to serve their bank's goal to expand in the unbanked markets? (A) They do not have adequate client base to sell their financial products. (B) They do not have adequate knowledge and skills to explain anything beyond basic financial products to the customers. (C) They do not have the skills to operate advanced technological aids that are a prerequisite to tap the unbanked market. 1) Only (B) 2) Only (C) 3) All (A), (B) & (C) 4) Only (A) 5) Both (B) and (C) 3. According to the passage, for which of the following reasons is the viability of financial inclusion under question? 1) Banks always prefer the cheapest package (to cust cost) while making a choice of technology to be used. 2) The Business Correspondent Agents are highly demotiviated to pursue their activity as a full-time job. 3) The investments made by banks and their delivery partners are not yielding equal amounts of returns. 4) Banks do not have adequate number of delivery partners required to tap the unbanked market. 5) Banks do not have adequate manpower to explore the diversity of the unbanked market and thereby identify the right target customers for various programmes. 4. In the passage, the author has specified which of the following characteristics of the customer on-boarding process? 1) It involves collection of documents from the applicants in order to validate their details. 2) It involves issuance of smart cards to the customers. 3) It suffers from latency as it takes a long time after submission of documents by the customer. 4) It is an expensive process which people find difficult to afford. 5) All of the given characteristics have been specified 5. What did the author try to highlight in the passage? (A) The ailing condition of financial inclusion business at present (B) Strategies that may help banks expand in the unbanked market (C) Role of government in modifying the existing financial-inclusion policies 1) Both (A) and (B) 2) All (A), (B) and (C) 3) Only (C) 4) Only (A) 5) Only (B) 6. According to the passage, which of the following ways may help banks sustain the interest of their customers after hooking them? (A) Adoption of a banking mechanism which is not only secure but reassuring to the customers (B) Increasing the number of delivery partners in rural market. (C) Introduction of a simple and intuitive user application 1) Only (A) 2) Only (C) 3) Only (B) 4) All (A), (B) and (C) 5) Both (A) and (C) Directions (Q. 7-8): Choose the word which is MOST SIMILAR in meaning to the word printed in bold as used in the passage. 7. Multitude 1) Impoverished 2) Handful 3) Acknowledged 4) Plenty 5) Solitued 8. Ubiquitous 1) Quintessential 2) Popular 3) Omnipresent 4) Simplified 5) Abnormal Directions (Q. 9-10) Choose the word which is MOST OPPOSITE in meaning to the word printed in bold as used in the passage. 9. Dormant 1) Emaciated 2) Pertinent 3) Cornered 4) Rejected 5) Active 10. Delayed 1) Perturbed 2) Popularised 3) Expedited 4) Stabilised 5) Repressed Directions (Q. 11-20): Read the following passage carefully and answer the questions given below it. Certain words have been printed in bold to help you locate them while answering some of the questions. The evolution of Bring your Own Device (BYOD) trend has been as profound as it has been rapid. It represents the more visible sign that the boundaries between personal life and work life are blurring. The 9 am - 5 pm model of working solely from office has become archaic and increasingly people are working extended hours from a range of locations. At the very heart of this evolution is the ability to access enterprise networks from anywhere and anytime. The concept of cloud computing serves effectively to extend the office out of office. The much heralded benefit of BYOD is greater productivity. However, recent research has suggested that this is the greatest myth of BYOD and the reality is that BYOD in practice poses new challenges that may outweigh the benefits. A worldwide survey commissioned by Fortinet chose to look at attitudes towards BYOD and security from the user's point of view instead of the IT Managers'. Specifically the survey was conducted in 15 territories on a group of graduate employees in their early twenties because they represent the fist generation to enter the workplace with an expectation of own device use. Moreover, they also represent tomorrow's influencers and decision makers. The survey findings reveal that for financial organisations, the decision to embrace BYOD is extremely dangerous.Larger organisations will have mature IT strategies and policies in place. But what about smaller financial business? They might not have such well developed strategies to protect confidential data. Crucially, within younger employee group, 55% of the people share an expectation that they should be allowed to use their own devices in the workplace or for work purposes. With this expectation comes the very real risk that employees may consider contravening company policy banning the use of own devices. The threats posed by this level of subversion cannot be overstated. The survey casts doubt on the idea of BYOD leading to greater productivity by revealing the real reason people want to use their own devices. Only 26% of people in this age group cite efficiency as the reason they want to use their own devices, while 63% admit that the main reason is so they have access to their favourite applications. But with personal applications so close to hand the risks to the business must surely include distraction and time wasting. To support this assumption 46% of people poled acknowledged time wasting as the greatest threat to the organisation, while 42% citing greater exposure to theft or loss of confidential data. Clearly, from a user perspective there is great deal of contradiction surroundnig BYOD and there exists an undercurrent of selfishness where users expect to use their own devices, but mostly for personal interest. They recognise the risks to the organisation but are adamant that those risks are worth taking. 11. According to the passage, for which of the following reasons did Fortinet conduct the survey on a group of graduate employees in their early twenties? (A) As this group represents the future decision makers. (B) As this group represents the first generation who entered the workforce with a better understanding of sophisticated gadgets. (C) As this group represents the first generation to enter the workplace expecting that they can use their own devices for work purpose. 1) All (A), (B) and (c) 2) Only (C) 3) Both (A) and (C) 4) Only (A) 5) Only (B) 12. Which of the following is not true BYOD? 1) BYOD enables employees to access enterprise network from anywhere and anytime. 2) Due to evolution of BYOD trend the 9 am- 5 pm model of working solely from office has become outdated. 3) Recent research has confirmed that BYOD boosts organisational productivity. 4) The concept of cloud computing facilitates the BYOD trend. 5) All the given facts are true 13. According to the passage, why would the decision to embrace BYOD prove dangerous to smaller financial businesses? 1) Their employers have poor knowledge about their devices, which in turn poses a threat the confidential data of the organisation. 2) Their employees are more vulnerable to misplacement of devices. 3) They may lack mature IT strategies and policies required to protect confidential data. 4) They cannot afford to deal with damage liability issues of employee-owned devices. 5) Their employees have a tendency to change jobs frequently. 14. According to the passage, the expectation of Younger employees that they should be allowed to use their own devices in the workplace, entails which of the following risks? A) Younger employees may deliberately transfer confidential data of their companies to rivals if they are not allowed to use their own devices for work purpose. B) Younger employees may strongly feel like leaving the company if it prevents usage of their own device and join some other company that does not have such stringent policies. C) Younger employees may consider flouting company policy prohibiting usage of their own devices in the workplace or for work purposes. 1) Only (C) 2) Only (B) 3) Both (A), (C) 4) only (A) 5) All (A), (B) and (C) 15. According to the findings of the survey conducted by Fortinet, why do majority of employees prefer using their own devices for work purpose? 1) As they often find that the devices provided by the company lack quality. 2) As they have access to their favourite applications while working 3) As majority of them believe that output doubles when they use their own devices for work purpose 4) As handling data from their own devices reinforces their sence of responsibility 5) As it helps them create a brand of their own 16. What is/are the author's main objective(s) in writing the passage? (A) To break the myth that BYOD promotes employee efficiency and organisational productivity (B) To suggest ways to control subversion across levels of corporate chain of command (C) To throw light upon the fact that employees, even after knowing the risks involved, prefer to use their own devices for work purpose, mostly for personal benefits 1) Both (A) and (C) 2) All (A), (B) and (C) 3) Only (C) 4) Only (A) 5) Only (B) Directions (Q. 17-18): Choose the word which is most SIMILAR in meaning to the word printed in bold as used in the passage. 17. Heralded 1) Suspected 2) Publicised 3) Dragged 4) Objective 5) Benefit 18. Outweigh 1) Control 2) Venture 3) Perish 4) Determine 5) Surpass Directions (Q. 19-20): Choose the word which is most OPPOSITE in meaning to the word printed in bold as used in the passage. 19. Embrace 1) Contradict 2) Disobey 3) Curtail 4) Reject 5) Obscure 20. Subversion 1) Compliance 2) Sanity 3) Popularity 4) Destabilisation 5) Clarity Directions (Q. 21-25): Read each sentence to find out whether there is any grammatical mistake/ error in it. The error, if any, will be in one part of the sentence. Mark the number of that part with error as your answer. If there is 'No error', mark 5) 21. 1) There cannot be any situation where/ 2) somebody makes money in an asset / 3) located in India and does not pay tax/ 4) either to India or to the country of his origin. / 5) No error 22. 1) India has entered a downward spiral / 2) Where the organised, productive /3) and law abide sectors are subject to / 4) savage amounts of multiple taxes. /5) No error 23. 1) The bank may have followed / 2) an aggressive monetary tightening policy /3) but its stated aim of / 4) curbing inflation have not been achieved. / 5)No error 24. 1) Equal Opportunities for advancement / 2) across the length and breadth /3) of an organisation will / 4) keep many problems away. / 5) No error 25. 1) A customised data science degree / 2) is yet to become / 3) a standard programme/ 4) to India's Premier educational institutes. / 5) No error Directions (Q. 26-30): Each question below has two blanks, each blank indicating that something has been omitted. Choose the set of words for each blank that best fits the meaning of the sentence as a whole. 26. When you want to digitalise a city .......... with millions, you don't bet ...............the odds. 1) proceeding, into 2) teeming, against 3) undergoing, adhere 4) dangling, for 5). falling, above 27. The numbers ............. by the legitimate online music service providers indicate that a growing number of users are ......... to buy music. 1) morphed, ignoring 2) labelled, thriving 3) figured, fanatic 4) painted, interested 5) touted, willing 28. If India is ........... on protecting its resources, international business appears equally .............. to safeguard its profit. 1) dreaded, fragile 2) stubborn, weak 3) bent, determined 4) approaching, settled 5) obsessed, prepared 29. Brands .......... decision-simplicity strategies make full use of available information to ......... where consumers are on the path of decision making and direct them to the best market offers. 1) diluting, divulge 2) tempting, maintain 3) imputing, overdrive 4) pursuing, assess 5) employing, trust 30. Lack of financing options, ........... with HR and technological ........... make small and medium enterprises sector the most vulnerable component of our economy. 1) except, loophole 2) coupled, challenges 3) armed, benefits 4) registered, strategies 5) strengthened, facilities SBI Current-Affairs SBI latest 2013 current affairs questions related bank based questions and answers SBI PO Exam based current affirs 2013 covering in January,February and March SBI IBPS PSU and all competitive examination current affairs and General Awareness & Banking Awareness questions and answers for practice.for bank,IBPS,SBI written test conists bank based current affairs, banking terms, finance news, finance terms, computer knowledge questions latest economy, finance, national international current affairs and general awareness 2013.For improving your timing spend in bank,sbi ,psu lic,giic,nic insurance companies all practice more model question papers with latest test pattern Nagesh Pydah is Oriental Bank's CMD - Oriental Bank of Commerce (OBC) has a new Chairman and Managing Director in Mr Nagesh Pydah.Prior to this appointment, Mr Pydah was an Executive Director at Punjab National Bank (PNB). He succeeds T.Y.Prabhu. ONGC, GAIL sign pact for marketing gas - State-run Oil and Natural Gas Corporation (ONGC) and GAIL India (GAIL) on Monday announced that they had reached a landmark agreement to ensure mutual business growth in the areas of natural gas as well as petrochemicals.The two PSU giants signed a number of agreements and exchanged documents in this regard in the presence of the ONGC Chairman and Managing Director R. S. Sharma and GAIL Chairman and Managing Director B. C. Tripathi. India to replace US as 2nd largest economy by 2050 PwC: India is poised to overtake the USA and emerge as the World's second largest economy on purchasing power parity basis by 2050 and has the potential to supersede China to the top spot, says a report published by PwC. Birla Corporation plans to set up cement plant in Assam: Birla Corporation, flagship company of the M. P. Birla Group, is planning to set up a one-million tonne cement plant in Assam at an investment of around Rs.450 crore. The company signed a memorandum of understanding with the Assam Mineral Development Corporation. SAIL signs MoU with Hindustan Prefab: Steel Authority of India Ltd. (SAIL) on Wednesday signed a memorandum of understanding (MoU) with Hindustan Prefab Ltd. (HPL) for jointly exploring the techno-economic viability of carrying out the business of prefabricated structures in steel and cement GDP growth for 2009-10 revised up to 8%: India's GDP (gross domestic product) growth stands revised upwards to 8 per cent for 2009-10 from the 7.4 per cent expansion estimated earlier for the fiscal, mainly on the strength of better showing by sectors such as manufacturing and services. Unlimited ATM service from Federal Bank: Federal Bank will allow its customers unlimited withdrawal of cash from ATMs (automated teller machines) of other banks from January 26.Now, a bank customer can, using his/her bank's ATM card, make up to five transactions from other banks' ATMs. For every transaction beyond five, the customer has to pay Rs. 20 as fee. This is in accordance with an RBI's directive. However, Federal Bank would allow its customers to make any number of transactions from other banks' ATMs and the Rs.20 fee was being waived effective from Republic Day. 1. Who has been appointed as Director General, National Human Rights Commission? (A) Dr. Ashok Sahu (B) Kanwalijit Deol (C) Krishna Kumar Shrivastava (D) Madhulata Sansi (E) None of these ANS. (B) 2. Which state government has recently announced to set up Rural Administrative Service? (A) Bihar (B) West Bengal (C) Assam (D) Uttar Pradesh (E) None of these ANS. (C) 3. Who is the Prime Minister of Belarus? (A) Ahmed Shafiq (B) David Cameron (C) Dr. Fakhruddin Ahmed (D) Mikhail Myasnikovich (E) None of these ANS. (D) 4. The vice president M. Hamid Ansari presented the ??Moortidevi Award for 2010?? to eminent Urdu scholar ?? for his outstanding work ?? Urdu Ghazal aur Hindustani Zehan wa Tehzeeb??. (A) Prof. Shahnaz Nabi (B) Prof. Gopi Chand Narag (C) Prof. Abdul Aleem (D) Prof. P.K. Abdul (E) Prof. S.R. Kidwai ANS. (B) 5. National integration day is celebrated on??? to mark the birth anniversary of late Indira Gandhi, the 1st woman Prime Minister of India. (A) 09th November (B) 19th November (C) 29th November (D) 19th December (E) 29th December ANS. (B) 6. Private final consumption Expenditure at current prices is estimated at Rs. ?? crore in Q2 of 2012-13. (A) 13,31,582 (B) 14,31,582 (C) 15,31,582 (D) 16,31,582 (E) 17,31,582 ANS. (A) 7. Government final consumption expenditure at current prices is estimated at Rs. ?? crore in Q2 of 2012-13. (A) 1,68,801 (B) 2,68,801 (C) 3,68,801 (D) 4,68,801 (E) 5,68,801 ANS. (B) 8. Who has been appointed as the BBC?s new Director General? (A) Karan Thapar (B) Jimmy Savile (C) George Entwistle (D) Tony Hall (E) None of these ANS. (D) 9. In which city the world travel market-2012 was held? (A) Paris (B) London (C) New York (D) Berlin (E) None of these ANS. (B) 10. Pranay Sahay has been appointed Director General, ??. (A) Assam Rifles (B) RPF (C) NCC (D) ITBP (E) CRPF ANS. (E) Placement Paper State bank of India SBI computer-professional knowledge questions with answers,SBI free solved sample placement papers of professional knowledge questions with answers,SBI Aptitude,Reasoning English questions with answers 1. UNIVAC is a. Universal Automatic Computer b. Universal Array Computer c. Unique Automatic Computer d. Unvalued Automatic Computer a. Universal Automatic Computer Explanation: There are no computers with the name as in other options. UNIVAC was the first general purpose electronic digital computer designed for commercial use, produced by Universal Accounting Company of John Mauchly and J.P.Eckert in 1951. 2. CD-ROM stands for a. Compactable Read Only Memory b. Compact Data Read Only Memory c. Compactable Disk Read Only Memory d. Compact Disk Read Only Memory Correct Answer: d. Compact Disk Read Only Memory Explanation: There are no objects with the name as in other options. CD-ROM is a non-volatile optical data storage medium using the same physical format as audio compact disk, readable by a computer with a CD-ROM drive. The standard 12 cm diameter CD-ROM store about 660 megabytes. 3. ALU is a. Arithmetic Logic Unit b. Array Logic Unit c. Application Logic Unit d. None of above Explanation: ALU is a unit in Central Processing Unit in a computer system that is responsible for arithmetic calculations and logical operations. Apart from ALU, the CPU contains MU (Memory Unit) and CU (Control Unit). 4. VGA is a. Video Graphics Array b. Visual Graphics Array c. Volatile Graphics Array d. Video Graphics Adapter Explanation: VGA is a type of Graphics Adapter. Graphic Adapter is an electronic board that controls the display of a monitor. This device helps the motherboard to work with the monitor and in VGA and SVGA the last letter ?A? stands for ?Array? whereas in MDA, CGA, MCGA the last letter ?A? stands for ?Adapter?. 5. IBM 1401 is a. First Generation Computer b. Second Generation Computer c. Third Generation Computer d. Fourth Generation Computer Explanation: IBM 1401 is a Second Generation Computer and is the first computer to enter Nepal in 2028 BS for census. Government of Nepal had brought this computer on rent and later purchased for data processing in Bureau of Statistics. After this computer, another ICL 2950/10, a British computer, was purchased by the fund of UNDP and UNFPA for the census of 2038 BS is second computer in Nepal. 6. MSI stands for a. Medium Scale Integrated Circuits b. Medium System Integrated Circuits c. Medium Scale Intelligent Circuit d. Medium System Intelligent Circuit Explanation: After the invention of IC chips the development of computers plunged into next phase. Small Scale Integration and Medium Scale Integration (SSI and MSI) were used in third generation of computers and Large Scale Integration and Very Large Scale Integration (LSI and VLSI) are being used in fourth generation of computers. People are now expecting ULSI (Ultra Large Scale Integration) Circuits to be used for fifth generation computers. 7. The capacity of 3.5 inch floppy disk is a. 1.40 MB b. 1.44 GB c. 1.40 GB d. 1.44 MB Explanation: Microfloppy disks (3.5 inch) if it is high density (MF2HD) can store 1.44 MB and if it is low density (MF2DD), it can store 720 KB. Mini Floppy disks (5.25 inch) if it is high density (MD2HD) can store 1.2 MB and low density (MD2DD) stores 360 KB of data. 8. The first computer introduced in Nepal was a. IBM 1400 b. IBM 1401 c. IBM 1402 d. IBM1402 Explanation: IBM 1401, a second generation computer was brought in Nepal by the Government of Nepal paying One Lakh and twenty five thousands per month to use in the census in 2028 B.S. Before this computer, Nepal was using a calculating device called Facit for statistical tasks. 9. WAN stands for a. Wap Area Network b. Wide Area Network c. Wide Array Net d. Wireless Area Network Explanation: There are three different classes of computer network namely, Local Area Network (LAN) that covers a small geographical area such as a room, a building or a compound; Metropolitan Area Network (MAN) that has a citywide coverage; and Wide Area Network (WAN) that covers the whole globe or beyond the globe. 10. MICR stands for a. Magnetic Ink Character Reader b. Magnetic Ink Code Reader c. Magnetic Ink Cases Reader d. None Explanation: MICR (Magnetic Ink Character Reader) is kind of scanner that can scan and identify the writing of magnetic ink. This device is used in banks to verify signatures in Checks. 11. EBCDIC stands for a. Extended Binary Coded Decimal Interchange Code b. Extended Bit Code Decimal Interchange Code c. Extended Bit Case Decimal Interchange Code d. Extended Binary Case Decimal Interchange Code Extended Binary Coded Decimal Interchange Code Explanation: EBCDIC is an 8-bit binary code for larger IBMs primarily mainframes in which each byte represent one alphanumeric character or two decimal digits. 256 characters can be coded using EBCDIC. 12. BCD is a. Binary Coded Decimal b. Bit Coded Decimal c. Binary Coded Digit d. Bit Coded Digit Explanation: BCD is a binary coded notation in which each of the decimal digits is expressed as a 8-bit binary numeral. For example in binary coded decimal notation 12 is 0001 0010 as opposed to 1100 in pure binary. 13. ASCII stands for a. American Stable Code for International Interchange b. American Standard Case for Institutional Interchange c. American Standard Code for Information Interchange d. American Standard Code for Interchange Information c. American Standard Code for Information Interchange Explanation: ASCII is a code which converts characters ? letters, digits, punctuations and control characters such as Alt, Tab etc ? into numeral form. ASCII code is used to represent data internally in micro-computers. ASCII codes are 7 bits and can represent 0 to 127 and extended ASCII are 8 bits that represents 0 to 255. 14. Which of the following is first generation of computer a. EDSAC b. IBM-1401 c. CDC-1604 d. ICL-2900 a. EDSAC Explanation: IBM-1401, CDC-1604 is second generation computer. ICL-2900 is a fourth generation computer. EDSAC is important in the development of computer since it was the first computer to use John von. Neumann?s Stored Program Concept. It used 3000 vacuum tubes and computers with vacuum tubes are of first generation computers. 15. Chief component of first generation computer was a. Transistors b. Vacuum Tubes and Valves c. Integrated Circuits d. None of above b. Vacuum Tubes and Valves Explanation: Transistors were used for second generation computers and integrated circuits in third generation. First generation computers used vacuum tubes and valves as their main electronic component. Vacuum Tubes were invented by Lee DeForest in 1908. 16. FORTRAN is a. File Translation b. Format Translation c. Formula Translation d. Floppy Translation c. Formula Translation Explanation: FORTRAN (Formula Translation) is one of the earlier High Level programming languages used to write scientific applications. It was developed by IBM in 1956. 17. EEPROM stand for a. Electrically Erasable Programmable Read Only Memory b. Easily Erasable Programmable Read Only Memory c. Electronic Erasable Programmable Read Only Memory d. None of the above a. Electrically Erasable Programmable Read Only Memory Explanation: There are three types of ROM namely, PROM, EPROM and EEPROM. PROM can?t be reprogrammed, EPROM can be erased by exposing it in high intensity ultraviolet light and EEPROM can be erased and reprogrammed electrically. It is not needed to be removed from the computer to be modified. 18. Second Generation computers were developed during a. 1949 to 1955 b. 1956 to 1965 c. 1965 to 1970 d. 1970 to 1990 Correct Answer: 1956 to 1965 Explanation: Second generation computers used transistors as their main electronic component. Transistor was invented by Bell Lab Scientists John Burdeen, Walter Brattain and William Shockley in 1947 and won the Nobel Prize in 1956 but it was not used in computers till 1956. The second generation continued until the implementation of IC chips invented by Jack Kilby in Texas Instruments in 1958. 19. The computer size was very large in a. First Generation b. Second Generation c. Third Generation d. Fourth Generation a. First Generation Explanation: It is obvious that computers developed with more power, reliability, speed and smaller sizes due to the enhancement of technology. First generation computers used 1000s of vacuum tubes that required lot of space made them gigantic in size. Single transistor could replace 1000 vacuum tubes and a single IC chip replaced 1000s of transistors made computers smaller and more speedy. 20. Microprocessors as switching devices are for which generation computers a. First Generation b. Second Generation c. Third Generation d. Fourth Generation Fourth Generation Explanation: Microprocessors further revolutionized the development of computers. Personal microcomputers were possible due to the microprocessors. The first microprocessor called Intel 4004 was developed by American Intel Corporation in 1971. Microprocessors are used in the computers of fourth generation computers. What do you call the programs that are used to find out possible faults and their causes? 21. Who invented the high level language C? A) Dennis M. Ritchie B) Niklaus Writh C) Seymour Papert -Answer D) Donald Kunth 22. Which of the following is not a type of Software A) System Software B) Application Software C) Utility Software D) Entertainment Software -Answer 23. Which of the following is not the classification of computers based on application? A) Electronic Computers B) Analog Computers C) Digital Computers -Answer D) Hybrid Computers 24 Which of the following registers is used to keep track of address of the memory location where the next instruction is located? A) Memory address register B) Memory data register C) Instruction register D) Program counter -Answer 25 Networking such as LAN, MAN started from A) First generation B) Second generation -Answer C) Third generation D) Fourth generation 26. Which characteristic of computer distinguishes it from electronic calculators? A) Accuracy B) Storage -Answer C) Versatility D) Automatic 27. Which of the following is the largest manufacturer of Hard Disk Drives? A) IBM B) Seagate -Answer C) Microsoft D) 3M 28 LSI, VLSI & ULSI chips were used in which generation? A) First B) Second C) Third -Answer D) Fourth 29. Which is the type of memory for information that does not change on your computer? A) RAM B) ROM C) ERAM D) RW / RAM -Answer 30Which generation of computer is still under development a.Fourth Generation b.Fifth Generation c.Sixth Generation d.Seventh Generation b. Fifth Generation Explanation: Today?s computer fall under the fourth generation computers that uses the LSI and VLSI Integrated Circuits. For fifth generation computers it is expected for Ultra Large Scale Integrated Circuits (ULSI). The study for using Gallium Arsenide instead of silicon chips is underway. Electrons can travel 5 times faster in Gallium Arsenide compared to the silicon. Similarly, genetically engineered protein molecules are being tried for processors. Fifth generation computers are expected to have artificial intelligence, ability to use natural language. 31Artificial Intelligence is associated with which generation? a.First Generation b.Second Generation c.Fifth Generation d.Sixth Generation c. Fifth Generation Explanation: ?A computer is a slave in a box?. This is a saying due to the lack of intelligence in computer in spite of their speed, accuracy, diligence and reliability. A computer can only perform instructed task in the way already programmed. Thus, scientists are trying to develop artificial intelligence in computers so that they can think and make decisions themselves. This is expected for fifth generation computers. 32Which operation is not performed by computer a.Inputting b.Processing c.Controlling d.Understanding d. Understanding Explanation: Computers can?t understand since they don?t have intelligence. Let?s hope in fifth generation they will achieve artificial intelligence and be able to understand. 33Fifth generation computer is also known as a.Knowledge information processing system b.Very large scale integration (VLSI) c.Both of above d.None of above a. Knowledge information processing system Explanation: Fifth Generation computers will have artificial intelligence and will be able to understand making knowledge information processing system possible, 34Central Processing Unit is combination of a.Control and storage b.Control and output unit c.Arithmetic logic and input unit d.Arithmetic logic and control unit d. Arithmetic logic and control unit Explanation: Arithmetic logic and control unit along with memory unit forms a complete central processing unit and central processing unit along with input/output unit forms a computer system. 35The brain of any computer system is a.Control Unit b.Arithmetic Logic Unit c.Central Processing Unit d.Storage Unit c. Central Processing Unit Explanation: Control unit or arithmetic logical unit alone can not represent the function of brain to compare with neither does storage unit only. Central Processing Unit is the combination of control, arithmetic logic and memory units thus can be compared with brain that controls the body, processes its functions and remembers. 36Pick the one that is used for logical operations or comparisons such as less than equal to or greater than. a.Arithmetic and Logic Unit b.Control Unit c.Both of above d.None of above a. Arithmetic and Logic Unit Explanation: Logical sub-unit of ALU performs all the comparisons such as less than equal to or greater than. Arithmetic sub-unit of ALU performs calculations such as addition, subtraction and so on. 37Analog computer works on the supply of a.Continuous electrical pulses b.Electrical pulses but not continuous c.Magnetic strength d.None of the above a. Continuous electrical pulses Explanation: Analog computers works on the continuous electrical pulses and digital computers work on the discrete electrical pulses. Analog computers need to process physical quantities such as temperature, pressure, speed etc. 38Digital devices are a.Digital Clock b.Automobile speed meter c.Clock with a dial and two hands d.All of them a. Digital Clock. Explanation: Automobile speed meter and clock with a dial and two hands are analog devices. Speed meter works on speed of wheel and the later works on tension of dialed spring. 39The computer that process both analog and digital is called a.Analog computer b.Digital computer c.Hybrid computer d.Mainframe computer c. Hybrid Computer Explanation: Analog, Digital and Hybrid are the three classes of computers based on the work or principle on which computer work. Hybrid computers can perform both the tasks of analog computers as well as hybrid computers. These computers are used in aircraft, hospitals where measuring physical quantities and converting them into digital data to analyze is required. 40Offline device is a. A device which is not connected to CPU b. A device which is connected to CPU c. A direct access storage device d. An I/O device Offline device is a. A device which is not connected to CPU SBI computer awareness questions SBI Associate Banks Clerks Exam Exam Held On : 16-01-2011 Marketing Aptitude/ Computer Knowledge, SBI Associate Banks Clerks Exam Exam Held On : 16-01-2011 ,State Bank of India Clerical Cadre, Probationary officers, Associate clerks questions for practice, SBI and SBT Examination solved question papers SBT and SBI largest collection of free solved placement papers SBI Reasoning Ability solved question paper for practice, SBI Model question papers for practice SBI Marketing and Computer Awareness questions and answers SBI Computer Awareness questions with answers 1. Assembly language is? 1.Low-level programming language 2.High level programming language 3.Machine language 4.All of the above 5.None of these 2. In Assembly language? 1.Mnemonics are used to code operations 2.Alphanumeric symbols are used for addresses 3.Language lies between high-level languageand machine 4.All of the above -Answer 5.None of these 3. The following computer?s memory is characterized by low cost per bit stored? 1.Primary 2.Secondary -Answer 3.Hard disk 4.All of the above 5.None of these 4. The following is true for Auxiliary Storage? 1.It has an operating speed far slower than that of the primary storage. -Answer 2.It has an operating speed faster than that of the primary storage. 3.It has an operating speed equivalent than that of the primary storage. 4.All of the above 5.None of these 5. Following is true for Bandwidth? 1.The narrow the bandwidth of a communications system the less data it can transmit in a given period of time. -Answer 2.The narrow then bandwidth of a communications system the more data it can transmit in a given period of time. 3.The wider the bandwidth of a communications system the less data it can transmit in a given period of time. 4.All of the above 5.None is true 6. Acknowledgement from a computer that a packet of data has been received and verified is known as? 1.ACK -Answer 2.BCK 3.ECK 4.All of the above 5.None of these 7. The following is a communications device (modem) which allows an ordinary telephone to be used with a computer device for data transmission? 1.Keyboard 2.Acoustic coupler -Answer 3.Mobile phone 4.All of the above 5.None of these 8. ALGOL is the 1.High-level language -Answer 2.Low level language 3.Machine language 4.All of the above 5.None of these 9. A high level programming language named after Ada Augusta, coworker with Charles Babbage? 1.Augustan 2.Babbage 3.Ada -Answer 4.Charlie 5.All of the above 10. Following is a logic circuit capable of forming the sum of two or more quantities? 1.Adder -Answer 2.Multiplier 3.Address 4.Access 5.None of these 11. To identification particular location in storage area one have a? 1.Address -Answer 2.Password 3.Logic 4.Mouse 5.None of these 12. A local storage register in the CPU which contains the address of the next instruction to be executed is referred as? 1.Key register 2.Address register -Answer 3.Password 4.All of the above 5.None of these 13 .A sequence of precise and unambiguous instructions for solving a problem in a finite number of operations is referred as? 1.Address 2.Algorithm -Answer 3.Advice 4.All of the above 5.None of these 14 .A character set that contains letters, digits, and other special characters such as $, @, +? % etc.is referred as? 1.Numeric 2.Alphanumeric -Answer 3.Alphabetic 4.All of the above 5.None of these 15. One of the components of central Processing Unit (CPU) of the computer that performs mathematical and logical operations is? 1.ALU (Arithmetic Logic Unit) -Answer 2.Address register 3.Analog 4.All of the above 5.None of these 16. A standard intended to connect relatively low­speed devices such as keyboards, mouse, modems and printers is? 1.Access bus -Answer 2.Access time 3.Connector 4.All of the above 5.None of these 17. The concept that one can disconnect peripherals and plug them in without letting computer shut down or reconfigure the system is referred as? 1.Hot plugging -Answer 2.Cold plugging 3.Access bus 4.All of the above 5.None of these 18. The time interval between the instant at which data is called from a storage device and the rust delivery begins in? 1.Access time -Answer 2.Delivery time 3.Service time 4.All of the above 5.None of these 19. The automatic execution of lower-priority (background) computer programs when higher­priority (foreground) programs are not using the system resources is referred as? 1.Background processing -Answer 2.Foreground processing 3.Null processing 4.All of the above 5.None of these 20. A form of access to a computer or network with specified user name and password is referred as? 1.Account -Answer 2.Address 3.Algorithm 4.All of the above 5.None of these 21 .A local storage area also called a register, inwhich the result of an arithmetic or logic operation is formed, is? 1.Accumulator -Answer 2.Address 3.Algorithm 4.All of the above 5.None of these 22. Computer that operates on data which is in the form of continuously variable physical quantities? 1.Digital computer 2.Analog computer -Answer 3.Mechanical computer 4.All of the above 5.None of these 23. Anonymous FTP is the? 1.Internet file transfer protocol -Answer 2.Protocol that requires password 3.None access files 4.None of these 24 .EFF sites allows anonymous FTP that? 1.Do not require a password or access -Answer 2.Requires password or access 3.Is a none access file 4.All of the above 5.None of these 25. ANSI is? 1.American National Standards Institute 2.A USA based national organization that establishes uniform standards in several fields of computers. 3.1. and 2. both are true -Answer 4.All of the above 5.None is true 26. APL is? 1.A high level language for specifying complex algorithms. 2.A real-time language primarily for scientificapplications. 3.Only 1. is true 4.Both 1. and 2. are true -Answer 5.None of these 27. The overall design, construction, organization and interconnecting of the various components of a computer system is referred as? 1.Computer Architecture -Answer 2.Computer Flow chart 3.Computer Algorithm 4.All of the above 5.None of these 28. Asynchronous communication is? 1.Communication between indepen-dentlyoperating units -Answer 2.Communication between dependent operating units 3.Communication between independent and dependent operating units 4.All of the above 5.None of these 29. Audio response is? 1.Output medium 2.Produces verbal responses from the computer system 3.Both 1. and 2. are true -Answer 4.Only A is true 5.None is true 30. Automated Office refers to the merger of ............... in an office environment. 1.Computers 2.Office 3.Telecommunications 4.All of the above -Answer 5.None is true 31. Auxiliary storage is 1.Secondary storage -Answer 2.Primary storage 3.Processing device 4.All of the above 5.None of these 32. Archive is? 1.Backup storage 2.Forward operation -Answer 3.Primary storage 4.All of the above 5.None of these 33. A branch of computer science that deals with computers that possess reasoning, learning and thinking capabilities that resemble those of human beings is recognized as? 1.Software engineering 2.Artificial intelligence -Answer 3.Hardware engineering 4.All of the above 5.None of these 34. ASCII is? 1.A standard coding system for computers -Answer 2.Hardware device 3.Software 4.All of the above 5.None of these 35. Following is true for Bandwidth? 1.The wider the bandwidth of a communication system the more data it can transmit in a given period of time. -Answer 2.The narrow the bandwidth of a communication system the more data it can transmit in a given period of time. 3.The wider the bandwidth of a communication system the less data it can transmit in a given period of time. 4.All of the above 5.None is true 36. ASCII-8 is? 1.An extended version of ASCII-7 -Answer 2.Is a 8-bit code 3.Both 1. and 2. are true 4.Both 1. and 2. are false 5.None of these 37. ASCII File is? 1.Document file in the universally recognized text format -Answer 2.Word processor 3.System file 4.All of the above 5.None of these 38. A program that translates mnemonic statements into executable instructions is referred as? 1.Software 2.Assembler -Answer 3.Translator 4.All of the above 5.None of these 39 .An assembler is? 1.Translator -Answer 2.Hardware 3.Assembly language 4.All of the above 5.None of these 40. Following is not true for Backup files? 1.These are the files which are generated automatically in when one save a document. 2.These files help in protecting the document due to out of order of the computer or power failure 3.These files delete as soon computer is off -Answer 4.All of the above 5.None of these 41. An algebra that deals with logical propositions which are either true or false is referred as? 1.Boolean algebra -Answer 2.Modern Algebra 3.Abstract Algebra 4.All of the above 5.None of these 42. Bandwidth is? 1.The range of frequencies available for data transmission -Answer 2.Data transmission rate 3.Alternate for rubber band 4.All of the above 5.None is true 43. A coding structure in which characters are represented by means of a series of parallel bars is? 1.Bar code -Answer 2.Menu bar 3.Numeric bar 4.All of the above 5.None of these 44. n Broadband system or a network system 1.Several analog signals share the same physical network channel -Answer 2.Only digital signals share the same physical network channel 3.Single analog signals share the same physical network channel 4.All of the above 5.None of these 45. Bit stands for? 1.Binary digit 2.One binary piece of information 3.Both 1. and 2. are true -Answer 4.All of the above 5.None is true 46. Broadband channel is the? 1.The fastest carriers where data transfer rates is of 1 million baud (bits/second) or more. -Answer 2.The slower carriers where data transfer rates is of 56k baud 3.Musical channel 4.All of the above 5.None of these 47. BLOB is? 1.Binary Large Object 2.A long bit string representing complex data 3.Object oriented language 4.Only 1. and 2. are true -Answer 5.None of these 48. A group of related items/section of program oding treated as a unit is referred as? 1.Block -Answer 2.Duplex 3.Street 1 4.All of the above 5.None of these 49. An operation in which data is moved to a different location is referred as? 1.Block move -Answer 2.Street move 3.Delete 4.All of the above 5.None of these 50. The following is responsible for number of logical records in a physical record? 1.Blocking factor -Answer 2.Block 3.Boolean algebra 4.All of the above 5.None of these TEST ? V: COMPUTER 51. A digital communication system capable of interconnecting, a large number of computers, terminals and other peripheral devices within a limited geographical area is called? 1.LAN -Answer 2.WAN 3.Internet 4.All of the above 5.None of these 52. A secondary storage device that uses a long plastic strip coated with a magnetic material as a recording medium is? 1.Magnetic tape -Answer 2.Compact disk 3.Hard disk 4.All of the above 5.None of these 53. An E-mail protocol for storage, manage-ment and change, especially in corporate offices? 1.Message handling service-Answer 2.Postal service?s 3.Data storage 4.All of the above -Answer 5.None of these 54. Which of the following is an advantage of stored programs? 1.Reliability-Answer 2.Reduction in operation costs 3.The computers becoming general-purpose 4.All of the above 5.None of these 55. Which of the following is not true for primary storage? 1.It is a part of the CPU-Answer 2.It allows very fast access of data 3.It is relatively more expensive then other storage devices. 4.Can be removed and transported. 5.All of the above 56. Function of control unit in the CPU is? 1.To decode program instructions- 2.To transfer data to primary storage 3.To perform logical operations 4.To store program instructionsAnswer 5.All of the above 57. Electronic spreadsheets are useful in situation where relatively .......... data must be input 1.Small -Answer 2.Large 3.No 4.All of the above 5.None is true 58. A list of options from which a program user can select anyone in order to perform a desired action is referred as? 1.Menu 2.Index-Answer 3.Directory 4.All of the above 5.None of these 59. A method whereby messages to be transmitted between computers are all sent to a central computer, which gathers them and routes them to the appropriate destination(s) is known as? 1.Message switching-Answer 2.Message delete 3.Message accept -Answer 4.All of the above 5.None of these 60. Which of the following describe one or more characteristics of a modem computer? 1.An electronic deviceAnswer 2.A mechanical device 3.A electro-mechanical device 4.All of the above 5.None of these 61. Combining records from two or more ordered files into a single ordered file is called? 1.Menu -Answer 2.Merging 3.Taking 4.All of the above 5.None of these 62. The ascending order of a data hierarchy is 1.Bit-byte-record-field-file-database- 2.Byte-bit-field-record-file-databaseAnswer 3.Byte-bit-record-file-field-database 4.Bit-byte-field-record-file-database 5.All of the above 63. A LSI chip contains the entire CPU of computer except main memory is called? 1.Microprocessor 2.Hard disk 3.Input device 4.Output deviceAnswer 5.All of the above 64. A storage medium that loses its contents in the event of power failure is called? 1.Volatile storage-Answer 2.Non volatile storage 3.Permanent storage 4.All of the above 5.None of these 65. One-millionth of a second is referred as? 1.Microsecond -Answer 2.Millisecond 3.Hour 4.Minute 5.All of the above 66. Alternate facilities of programs, data files,hardware equipments, etc. used in case the original one is destroyed, lost, or fail to operate is? 1.Backup -Answer 2.Forehead 3.Delete 4.All of the above 5.None of these 67 One-thousandth of a second is called? 1.Microsecond -Answer 2.Millisecond 3.Hour 4.Minute-Answer 5.All of the above 68. The CPU (central processing unit) consists of 1.Input, output, and processing 2.Control unit, primary storage, andsecondary storage-Answer 3.Control unit, arithmetic-logic unit, andprimary storage 4.All of the above 5.None of these 69. The product of data processing is? 1.Data 2.Information 3.Software -Answer 4.Computer 5.All of the above 70. The input device used by computer is? 1.Motherboard 2.Central processing unit-Answer 3.Keyboard 4.System unit 5.All of the above 71. The structure of interconnecting of nodes of a computer network is referred as? 1.Network topology 2.Internet 3.Local Area Network-Answer 4.Wide Area Network 5.All of the above 72. Nible is? 1.Double of byte 2.Half of byte 3.Ten times of byte 4.Five times of byte-Answer 5.All of the above 73. Multiprogramming is the? 1.The inter-leaved execution of two or more different and independent programs by the same computer 2.The execution of single by the different computers 3.The inter-leaved execution of two or more different and independent programs by the different computers-Answer 4.All of the above 5.None of these 74. A facility available in an operating system that allows multiple functions from the same application package is referred as? 1.Multi-threading-Answer 2.Multi-programming 3.Multi-processing 4.All of the above 5.None of these 75. Narrowband channel is a communication channels? 1.That handle low volumes of data 2.Used mainly for telegraph lines and low speed terminals-Answer 3.Both 1. and 2. 4.All of the above 5.None of these 76. Following is the example of Network? 1.LAN-Answer 2.WAN 3.Internet 4.All the above 5.None of these 77. Peer-to-peer is a description? 1.Of communications between two dissimilar devices-Answer 2.Of communications between two equal devices 3.Of communications between two different devices 4.All of the above 5.None of theses 78. Non-document files are? 1.Pure ASCII files that can be listed for any word processing software-Answer 2.Non ASCII files that can be listed for any word processing software 3.Pure ASCII files that can be listed for specific word processing software 4.Non ASCII files that can be listed for pecific word processing software 5.All of the above 79. The method of dividing a physical channel into many logical channels so that a number of independent signals may be simultaneously transmitted on it is referred as? 1.Multithreading -Answer 2.Multiprocessing 3.Multiplexing 4.All of the above 5.None of these 80. An interconnected computer configura-tions or computers with two or more independent CPUs that have the ability to simultaneously execute several programs is called? 1.Multithreading -Answer 2.Multiprocessing 3.Multiplexing 4.All of the above 5.None of these 81. Following is not a non-impact printer? 1.Thermal printer -Answer 2.Ink-jet printer 3.Laser printer 4.Dot-matrix printer 5.All of the above 82. A number system with a base of 8 is referred as? 1.Binary -Answer 2.Decimal 3.Octal 4.All of the above 5.None of these 83. Devices converts digital signals to Sine waves at the sending end and back to digital signals at the receiving end is called? 1.Modem -Answer 2.Telephone 3.Mobile phone 4.All of the above 5.None of these 84. Dividing a project into segments and smaller units in order to simplify the analysis, design and programming effort is referred as? 1.Monitoring 2.Modular approach 3.Microprocessing 4.All of the above-Answer 5.None of these 85. The technique where digital signals are converted to its analog form for transmission over an analog facility is called? 1.Modulation -Answer 2.Digitization 3.Multiprocessing 4.All of the above 5.None of these 86. Multiprocessor is a computer system? 1.Where many CPU under a common control 2.Consisting of more than one input devices under common control 3.Where many output devices under-Answer 4.All of the above 5.None of these 87. The point at which the flow of text in a document moves to the top of a new page? 1.Page break -Answer 2.Page insert 3.Page format 4.All of the above 5.None of these 88. Technique to implement virtual memory where memory is divided into units of fixed size memory is? 1.Paging -Answer 2.De-fragments 3.Segmentation 4.All of the above 5.None of these 89. An adder where all the bits of the two operands are added simultaneously? 1.Parallel adder-Answer 2.Half adder 3.Full adder 4.All of the above 5.None of these 90. An extra bit added to a string of bits that enables the computer to detect internal errors in the transmission of binary data? 1.Parity bit 2.Byte 3.Nibble 4.All of the above 5.None of these 91. Password is? 1.Code by which a user gains access to a computer system 2.Pass for viewing movie 3.Entrance without paying something 4.All of the above 5.None of these 92. PL 1 (Programming Language One) is? 1.High level programming language 2.Low level programming language 3.Machine language 4.Assembly language 5.All of the above 93. Pascal is? 1.A high-level programming language-Answer 2.Named after Blaise Pascal 3.Named after mathematician that facilitates structured programming techniques 4.All of the above 5.None of these 94. S torage device where time to retrieve stored information is independent of address where it is stored is called? 1.Random access memory (RAM) 2.Primary memory 3.Secondary memory 4.All of the above -Answer 5.None of these 95. Octal number system has a base? 1.Two -Answer 2.Four 3.Eight 4.Ten 5.All of the above 96. Since input and output devices are on the edge of the main processing therefore they are referred as? 1.Peripheral devices 2.Processing devices 3.Personal devices-Answer 4.All of the above 5.None of these 97. A system changeover method where complete changeover to the new usa burmanica (E)None of these 26.What was the estimated production of foodgrains in India for the year 2011-12? (A)230 million tonnes (B)210 million tonnes (C)257 million tonnes (D)280 million tonnes (E)305 million tonnes Ans (c) 27.Which of the following crops in India was losing area under cultivation since 1980-81 to 2011-12? (A)Wheat (B)Rice (C)Pulses (D)Coarse Cereals (E)None of these 28.Which crops of the following are covered under the National Food Security Mission ? (A)Wheat, Rice and Bajra (B)Rice, Sugarcane and Maize (C)Wheat and Coarse Cereals (D)Wheat, Rice and Pulses (E)Wheat, Maize and Jowar Ans (d) 29.During which Five Year Plan, India has achieved the maximum per-capita growth rate ? (A)Eleventh Plan (B)Tenth Plan (C)Eighth Plan (D)Fifth Plan (E)Sixth Plan 30.At present, where from the maximum resources are mobilised for India?s public expenditure under plans (A)From current recejpjs. (B)From abroad (C)From public enterprises (D)From borrowings (E)From deficit